A Matter of Some Gravity

Guest Post by Willis Eschenbach

A couple of apparently related theories have been making the rounds lately. One is by Nikolov and Zeller (N&Z), expounded here and replied to here on WUWT. The other is by Hans Jelbring, discussed at Tallblokes Talkshop. As I understand their theories, they say that the combination of gravity plus an atmosphere without greenhouse gases (GHGs) is capable of doing what the greenhouse effect does—raise the earth at least 30°C above what we might call the “theoretical Stefan-Boltzmann (S-B) temperature.”

So what is the S-B temperature, theoretical or otherwise?

A curious fact is that almost everything around us is continually radiating energy in the infrared frequencies. You, me, the trees, the ocean, clouds, ice, all the common stuff gives off infrared radiation. That’s how night-vision goggles work, they let you see in the infrared. Here’s another oddity. Ice, despite being brilliant white because it reflects slmost all visible light, absorbs infrared very well (absorptivity > 0.90). It turns out that most things absorb (and thus emit) infrared quite well, including the ocean, and plants (see Note 3 below). Because of this, the planet is often treated as a “blackbody” for IR, a perfect absorber and a perfect emitter of infrared radiation. The error introduced in that way is small for first-cut calculations.

The Stefan-Boltzmann equation specifies how much radiation is emitted at a given temperature. It states that the radiation increases much faster than the temperature. It turns out that radiation is proportional to absolute temperature to the fourth power. The equation, for those math inclined, is

Radiation = Emissivity times SBconstant times Temperature^4

where the Stefan-Boltzmann constant is a tiny number, 0.0000000567 (5.67E-8). For a blackbody, emissivity = 1.

This “fourth-power” dependence means that if you double the absolute temperature (measured in kelvins), you get sixteen (2^4) times the radiation (measured in watts per square metre, “W/m2”). We can also look at it the other way, that temperature varies as the fourth root of radiation. That means if we double the radiation, the temperature only goes up by about 20% (2^0.25)

Let me call the “theoretical S-B temperature” the temperature that an evenly heated stationary blackbody planet in outer space would have for a given level of incoming radiation in W/m2. It is “theoretical”, because a real, revolving airless planet getting heated by a sun  with the same average radiation will be cooler than that theoretical S-B temperature. We might imagine that there are thousands of mini-suns in a sphere around the planet, so the surface heating is perfectly even.

Figure 1. Planet lit by multiple suns. Image Source.

On average day and night over the planetary surface, the Earth receives about 240 W/m2 of energy from the sun. The theoretical S-B temperature for this amount of radiation (if it were evenly distributed) is about -18°C, well below freezing. But instead of being frozen, the planet is at about +14°C or so. That’s about thirty degrees above the theoretical S-B temperature. So why isn’t the planet a block of ice?

Let me take a short detour on the way to answering that question in order to introduce the concept of the “elevator speech” to those unfamiliar with the idea.

The “elevator speech” is simply a distillation of an idea down to its very basics. It is how I would explain my idea to you if I only had the length of an elevator ride to explain it. As such it has two extremely important functions:

1. It forces me to clarify my own ideas on whatever I’m discussing. I can’t get into handwaving and hyperbole, I can’t be unclear about what I’m claiming, if I only have a few sentences to work with.

2. It allows me to clearly communicate those ideas to others.

In recent discussions on the subject, I have been asking for that kind of “elevator speech” distillation of Jelbring’s or Nikolov’s ideas, so that a) I can see if whoever is explaining the theory really understands what they are saying and, if so, then b) so that I can gain an understanding of the ideas of Jelbring or Nikolov to see if I am missing something important.

Let me give you an example to show what I mean. Here’s an elevator speech about the greenhouse effect:

The poorly-named “greenhouse effect” works as follows:

• The surface of the earth emits energy in the form of thermal longwave radiation.

• Some of that energy is absorbed by greenhouse gases (GHGs) in the atmosphere.

• In turn, some of that absorbed energy is radiated by the atmosphere back to the surface.

• As a result of absorbing that energy from the atmosphere, the surface is warmer than it would be in the absence of the GHGs.

 OK, that’s my elevator speech about why the Earth is not a block of ice. Note that it is not just saying what is happening. It is saying how it is happening as well.

I have asked, over and over, on various threads, for people who understand either the N&Z theory or the Jelbring theory, to give me the equivalent elevator speech regarding either or both of those theories. I have gotten nothing scientific so far. Oh, there’s the usual handwaving, vague claims of things like ‘the extra heat at the surface, is just borrowed by the work due to gravity, from the higher up regions of the atmosphere‘ with no mechanism for the “borrowing”, that kind of empty statement. But nothing with any meat, nothing with any substance, nothing with any explanatory value or scientific content.

So to begin with, let me renew my call for the elevator speech on either theory. Both of them make my head hurt, I can’t really follow their vague descriptions. So … is anyone who understands either theory willing to step forward and explain it in four or five sentences?

But that’s not really why I’m writing this. I’m writing this because of the claims of the promoters of the two theories. They say that somehow a combination of gravity and a transparent, GHG-free atmosphere can conspire to push the temperature of a planet well above the theoretical S-B temperature, to a condition similar to that of the Earth.

I hold that with a transparent GHG-free atmosphere, neither the hypothetical “N&Z effect” nor the “Jelbring effect” can possibly raise the planetary temperature above the theoretical S-B temperature. But I also make a much more general claim. I hold it can be proven that there is no possible mechanism involving gravity and the atmosphere that can raise the temperature of a planet with a transparent GHG-free atmosphere above the theoretical S-B temperature.

The proof is by contradiction. This is a proof where you assume that the theorem is right, and then show that if it is right it leads to an impossible situation, so it cannot possibly be right.

So let us assume that we have the airless perfectly evenly heated blackbody planet that I spoke of above, evenly surrounded by a sphere of mini-suns. The temperature of this theoretical planet is, of course, the theoretical S-B temperature.

Now suppose we add an atmosphere to the planet, a transparent GHG-free atmosphere. If the theories of N&K and Jelbring are correct, the temperature of the planet will rise.

But when the temperature of a perfect blackbody planet rises … the surface radiation of that planet must rise as well.

And because the atmosphere is transparent, this means that the planet is radiating to space more energy than it receives. This is an obvious violation of conservation of energy, so any theories proposing such a warming must be incorrect.

Q.E.D.

Now, I’m happy for folks to comment on this proof, or to give us their elevator speech about the Jelbring or the N&Z hypothesis. I’m not happy to be abused for my supposed stupidity, nor attacked for my views, nor pilloried for claimed errors of commission and omission. People are already way too passionate about this stuff. Roger Tattersall, the author of the blog “Tallbloke’s Talkshop”, has banned Joel Shore for saying that the N&Z hypothesis violates conservation of energy. Roger’s exact words to Joel were:

… you’re not posting here unless and until you apologise to Nikolov and Zeller for spreading misinformation about conservation of energy in their theory all over the blogosphere and failing to correct it.

Now, I have done the very same thing that Joel did. I’ve said around the web that the N&Z theory violates conservation of energy. So I went to the Talkshop and asked, even implored, Roger not to do such a foolish and anti-scientific thing as banning someone for their scientific views. Since I hold the same views and I committed the same thought-crimes, it was more than theoretical to me. Roger has remained obdurate, however, so I am no longer able to post there in good conscience. Roger Tallbloke has been a gentleman throughout, as is his style, and I hated to leave. But I did what Joel did, I too said N&Z violated conservation of energy, so in solidarity and fairness I’m not posting at the Talkshop anymore.

And more to the point, even if I hadn’t done what Joel did, my practice is to never post at or even visit sites like RealClimate, Tamino’s, and now Tallbloke’s Talkshop, places that ban and censor scientific views. I don’t want to be responsible for their page views counter to go up by even one. Banning and censorship are anathema to me, and I protest them in the only way I can. I leave them behind to discuss their ideas in their now cleansed, peaceful, sanitized, and intellectually sterile echo chamber, free from those pesky contrary views … and I invite others to vote with their feet as well.

But I digress, my point is that passions are running high on this topic, so let’s see if we can keep the discussion at least relatively chill …

TO CONCLUDE: I’m interested in people who can either show that my proof is wrong, or who will give us your elevator speech about the science underlying either N&K or Jelbring’s theory. No new theories need apply, we have enough for this post. And no long complicated explanations, please. I have boiled the greenhouse effect down to four sentences. See if you can match that regarding the N&K or the Jelbring effect.

w.

NOTE 1: Here’s the thing about a planet with a transparent atmosphere. There is only one object that can radiate to space, the surface. As a result, it is constrained to emit the exact amount of radiation it absorbs. So there are no gravity/atmospheric phenomena that can change that. It cannot emit more or less than what it absorbs while staying at the same temperature, conservation of energy ensures that. This means that while the temperature can be lower than the theoretical S-B temperature, as is the case with the moon, it cannot be more than the theoretical S-B temperature. To do that it would have to radiate more than it is receiving, and that breaks the conservation of energy.

Once you have GHGs in the atmosphere, of course, some of the surface radiation can get absorbed in the atmosphere. In that case, the surface radiation is no longer constrained, and the surface is free to take up a higher temperature while the system as a whole emits the same amount of radiation to space that it absorbs.

NOTE 2: An atmosphere, even a GHG-free atmosphere, can reduce the cooling due to uneven insolation. The hottest possible average temperature for a given average level of radiation (W/m2) occurs when the heating is uniform in both time and space. If the total surface radiation remains the same (as it must with a transparent atmosphere), any variations in temperature from that uniform state will lower the average temperature. Variations include day/night temperature differences, and equator/polar differences. Since any atmosphere can reduce the size of e.g. day/night temperature swings, even a transparent GHG-free atmosphere will reduce the amount of cooling caused by the temperature swings. See here for further discussion.

But what such an atmosphere cannot do is raise the temperature beyond the theoretical maximum average temperature for that given level of incoming radiation. That’s against the law … of conservation of energy.

NOTE 3: My bible for many things climatish, including the emissivity (which is equal to the absorptivity) of common substances, is Geiger’s The Climate Near The Ground, first published sometime around the fifties when people still measured things instead of modeling them. He gives the following figures for IR emissivity at 9 to 12 microns:

Water, 0.96

Fresh snow, 0.99

Dry sand, 0.95

Wet sand, 0.96

Forest, deciduous, 0.95

Forest, conifer, 0.97

Leaves Corn, Beans, 0.94

and so on down to things like:

Mouse fur, 0.94

Glass, 0.94

You can see why the error from considering the earth as a blackbody in the IR is quite small.

I must admit, though, that I do greatly enjoy the idea of some boffin at midnight in his laboratory measuring the emissivity of common substances when he hears the snap of the mousetrap he set earlier, and he thinks, hmmm …

0 0 votes
Article Rating
1.2K Comments
Oldest
Newest Most Voted
Inline Feedbacks
View all comments
michael hammer
January 13, 2012 9:45 pm

Willis, I agree with amonst everything you said with one exception. I question the emissivity of fresh snow. Yes cetainly water and ice have IR emissivities very close to 1 but fresh snow is a mixture of ice and air with each “layer” or crystal a few to a few 10’s of microns thick. each transition of refractive index gives rise to reflections and the repeated and rapid change of refractive index results in a highly reflective surface. That is also what makes crazed paint surfaces appear whitish. It means absorptivity and hence also emissivity should be low – well below 1. I know how at least some of these absorptivity measurements are made (using an integrating sphere) and integrating spheres give the wrong answer for translucent materials – they suggest a reflectivity lower than it actually is (I know I design them). Consider how the interior of an igloo could be at +18C with minimal heating if the interior walls had an absorptivity of about 1. A high absorptivity would make an igloo more or less the equivalent of a sub zero cool store. Much more significantly, consider how the temperature in the antarctic as measured from satellites is around 180K when the coldest point even on the high plateau is more like 220K. The only way that I know for a low temperature to be recorded spectroscopically is if the surface emissivity is low.
Its an important issue because of the claim that snow gives rise to positive feedback. Supposedly the snow reflects incoming energy whilst radiating outgoing energy so it signifiucantly cools the surface yet the ground underneath would absorb incoming energy and thus would be more warming. If the emissivity of snow is far less than 1 then that theory is disproven.

January 13, 2012 9:57 pm

“Now, I’m happy for folks to comment on this proof”
Seems to me to be sound.
The notion of emissivity for S-B as usually expressed is a surface emissivity. For a partly transparent medium like a gas, it’s a bit more complicated.

Stephen Singer
January 13, 2012 9:58 pm

I’ll take a shot at this task though I’m a bit fuzzy still about it. I’ll give my current wimpy understanding.
1. Planet with mass and GHG free atmosphere.
2. Planets gravity pulls down on the gas compressing it causing heat near the surface.
3. Convection sets in and gas rises. cools then back to step 2.
It’s just to simple to be believable.

General P.Malaise
January 13, 2012 10:16 pm

great article.
….my tomatoes still die when the temperature goes below freezing.

Mydogsgotnonose
January 13, 2012 10:17 pm

Willis, if i may call you that, you have in my view made the same fundamental ,mistake as most ‘climate scientists’, and it comes from not understanding heat transfer.
First a correction to the IPCC thought experiment: if you remove the atmosphere you have no water precipitation so no clouds or ice, but you still have the seas [which by magic can’t evaporate!]. Emissivity falls from 0.3 to 0.07 so equilibrium radiative temperature falls to ~0°C meaning maximum GHG warming is 15 K.
However, if you the put back the atmosphere sans H2O and CO2, you still have aerosols and convection. Real present GHG warming is ~9 K.
The key is that you must always have lapse rate warming and it’s controlled by convection. The real radiative temperature with an atmosphere is always less by that convective temperature drop.

E.M.Smith
Editor
January 13, 2012 10:22 pm

The only problem I see with your proof is the assumptions. A “GHG Free” atmosphere. On the one hand you say everything on the surface emits in the IR and give a long list of emissivities near one. On the other you say “except the air”…. It just causes me to think that you are defining the air as the functional equivalent of a vacuum… If it can absorb surface heat and convect, but NEVER emit, how can it ever then cool to return to the surface? Does not the assumption force another paradox? High altitude hot air that reaches equilibrium at the highest possible temperature that ever rises from any spot?
It just seems to me like it is a bit of a tautology with all the reality squeezed out of it…
Or, as one wag once said of ‘how an economist stranded on an island would use his strongest skills to open a food tin without a can opener: ‘First, assume the can is open…’..”

Anteros
January 13, 2012 10:22 pm

Hi Willis,
I assume when you say the earth receives 240 W/m2, that’s a typo for 340?
If not, I’m going to have to go back to school 🙂

January 13, 2012 10:27 pm

“And because the atmosphere is transparent, this means that the planet is radiating to space more energy than it receives. ”
I fail to see how this is a given with an IR transparent atmosphere. It would behave like a black body and radiate like one, except for one thing: the atmosphere would be heated by conduction and distribute heat by convection. As the nonGHG atmosphere cannot radiate IR, it would have to heat up and that’s the source of elevated temperature. Once heated, the atmosphere would go to equilibrium with the surface and would distribute heat back to the surface where IR would carry it away.
I would say that CO2 and water vapor are energy leaks, or rather small holes in the glass of the greenhouse, allowing heat to move both in and out during the day and out only at night.

January 13, 2012 10:34 pm

I think I see the problem you have with conservation of energy, but let’s do a little story.
You have a black body that is emitting exactly as much as it absorbs, but when the Sun first hit it, this was not the case. There was a period when more heat was going in than out. Once it had warmed, then the surface went to equilibrium with the input and output.
The same would be true when the IR-clear atmosphere is added. There would be a time during which the surface/atmosphere would be getting up to temperature and then go to equilibrium as above. Thus, the planet is warmer than a gasless black body.
That’s my read—it just appears that you ignore conduction between the surface and the atmosphere.

alex
January 13, 2012 10:36 pm

OMG.
Did not expect, Eschenbach has such grave problems with physics.
He writes
“the Stefan-Boltzmann constant is a tiny number, 0.0000000567 (5.67E-8).”
Sorry, the the Stefan-Boltzmann constant is a DIMENSIONAL constant.
You never can say a dimensional constant is “tiny”, because its numerical value depends on the units,
For example
σ = 5.6704)×10−8 W m−2 K−4 in SI
or
σ = 5.6704×10−5 erg s-1 cm−2 K−4 in CGS
or you can also write it
σ = 0.56704 erg s-1 m−2 K−4
and it looks already not that tiny!
.
.

Anything is possible
January 13, 2012 10:39 pm

I’ve been reading a lot, but saying very little, ever since first becoming acquainted with Nikolov and Zeller’s theory.
FWIW, my “elevator speech” can be summed up thus :
On a planet with no atmosphere, radiation is king, no other processes occur and it can explain surface temperatures almost perfectly (witness the Moon).
Introduce an atmosphere, regardless of composition, and things start to get complicated, because you introduce the additional processes of convection and conduction.
Gravity is a key, (but unchanging) element because, along with the mass, it defines the vertical structure of any atmosphere, and hence the surface pressure.
The greater the surface pressure on a planet, the more influence conduction and convection have on surface temperatures, and the less influence radiation has (witness the unchanging temperatures on the surface of Venus).
Higher surface pressures elevate surface temperatures by suppressing convection because the greater the weight of the overlying atmosphere, the more energy (heat) is required to enable the process to get under way.
Nobody is ever going to disprove N + Z by obsessing about the details of all the complex processes that occur in the atmosphere, you have to look at the bigger picture.
If N+Z are correct (reserving judgement on that one) what they are is essentially saying is that the average surface temperature of any planetary body is pre-determined by insolation, surface pressure and albedo, and that all the processes that occur in any atmosphere – regardless of structure or composition will adjust, and act in such a way essentially by definition, as to preserve the average surface temperature of that body. Holy moley!!!
Ultimately, there will be only one way to settle this argument. There are literally billions of planetary bodies out there waiting to be discovered. One day (which maybe sooner than anyone anticipates) we will develop the technology to acquire the relevant data from a large enough number of those bodies that the theory will be proved, or disproved, once and for all. Until then, there’s going to be one heck of an argument. Try and Enjoy it! (:-

crosspatch
January 13, 2012 10:40 pm

We aren’t talking about heat. We are talking about temperature. In a perfectly transparent atmosphere all of the heat being radiated would radiate out. But radiation would be the smallest of available heat transfer mechanisms. There would also be conduction because the atmosphere touches the planet. So the actual physical surface interface of the planet would be slightly cooler and the atmosphere will be slightly warmer. We would have a warmer temperature at 5 feet above the ground (but a cooler temperature AT the ground). The problem is all this talk about radiative heat loss at the surface. The main loss of heat from the surface of the planet isn’t radiation, it is and the loss from evaporation of surface water which absorbs a huge amount of heat when simply changing state from liquid to vapor without any change in temperature. [SNIP– way off topic. I’m talking about a GHG free atmosphere, as are Jelbring and N&K. Sorry, w.]

beaker
January 13, 2012 10:42 pm

Just a small correction. Night vision uses a silicon detector and amplifies the available ambient light. Silicon is not responsive in the MW or LWIR, 3-5um and 8-12um, the naturally occuring atmospheric IR windows where us radiometric guys make our measurments. A FLIR or Forward Looking Infrared “see’s” thermal differentials without visible light. I believe you are referring to a FLIR, not night vision.

January 13, 2012 10:50 pm

@Willis Eschenbach
Not being familiar with the N&Z or Jelbrick theorems, you do ignore the far reduced but still present conduction and subsequent convection present in the system. This would cause a drop in the energy available to be radiated due to loss to kinetic energy. I don’t know how much of an effect this would have on allowing increased warming above and beyond the S-B calculations because I don’t want to do the math right now but it is a place for energy to go since you added a liquid to the system.

January 13, 2012 10:52 pm

How would Earth’s surface temperature change if atmospheric pressure were doubled, that is, increased to 2 atm by adding more N₂ (and nothing else)?

crosspatch
January 13, 2012 10:56 pm

In other words, on a planet without an atmosphere surrounded by a sphere of tiny suns, the surface is some temperature and all photons of LWIR radiate from the surface. Add an atmosphere and the temperature of the SURFACE drops because that atmosphere conducts some of the heat away, the temperature of the atmosphere rises and where each photon radiates from is variable due to convection. Some will radiate from the surface, some will radiate from 1 foot above the surface, some will radiate from 1000 feet above the surface. Even if the air is completely transparent to IR it will still be warmed by conduction by contact with the surface of the planet, cool the surface, warm the atmosphere. The atmospheric temperature will still vary by the adiabatic lapse rate.
You seem to have been assuming a warming atmosphere without any associated cooling of the surface. Don’t confuse surface temperature with “surface temperature”. What we call surface temperature in most cases is the temperature of the atmosphere some distance above the surface. To my knowledge, nobody actually records the surface temperature of the planet itself except possibly the oceans. [NOT SO. Repeat after me, “GHG-free atmosphere”. The surface is the only thing that can radiate. w.]

NoIdea
January 13, 2012 11:09 pm

Having made only a superficial reading of N&Z, can someone explain the difference between that and Harry Huffman’s explanation? They seem largely the same although Huffman argues N&Z are not quite correct. As for Willis’ wish for a clear explanation, Huffmans several posts on his blog appear to do so quite well. For me as a layperson looking from the outside, Huffmans explanation makes sense but more usefully meets the test of Occam’s razor…

Genghis
January 13, 2012 11:14 pm

1. The non greenhouse gas atmosphere is a perfect conduction insulator to space, it can’t radiate its heat out.
2. The radiation transparent atmosphere is heated from the surface via conduction/convection until the atmospheric average temperature is at the S-B average. (Almost exactly the same way GHG’s heat the atmosphere, except that it is the planets surface molecules directly heating the atmosphere.)
3. According to the ideal gas law the average temperature will be at the midpoint of the atmosphere with the upper half lower than the S-B average temperature and the lower half higher than the S-B average temperature. I think with our earth the temp at 5k altitude is something like -18˚.
4. The thicker and denser the atmosphere, the higher the near surface atmospheric temperature will be.
5. Once the atmosphere is heated, the net radiation from the surface will be exactly the same as if there is no atmosphere, but the near surface atmosphere temperature will be warmer than the S-B average.
I can give a more detailed explanation tomorrow, but I just wanted to get this in tonight.

Pops
January 13, 2012 11:17 pm

One tiny nit, Willis.
That’s how night-vision goggles work, they let you see in the infrared.
Night vision goggles amplify visible and near-visible photons reflecting off objects and impinging on the device, such as are produced by starlight. You’re probably thinking of infrared sensors (e.g. FLIR) that rely on the emissivity of the observed scene in LWIR, MWIR, or SWIR bands – which “curiously” happen to align with gaps in CO2 and H2O absorption of infrared radiation.
In military applications, night-vision goggles are typically helmet-mounted devices used by everyone from infantry to pilots. IR sensors are usually larger devices with relatively massive optics and cooling systems, and are found mounted on airborne platforms and connected to MFDs in the cockpit.

crosspatch
January 13, 2012 11:24 pm

I have used light amplification night vision devices that also had IR capability along with an IR search lamp built into the device for use in completely dark conditions (inside of something where there is no ambient light to amplify).

jorgekafkazar
January 13, 2012 11:27 pm

It’s late, so I’ll just maunder. Hopefully, I’ll maunder minimally.
☻Wouldn’t a planet lit by thousands of suns eventually achieve solar temperature, say, 5000°?
☻The problem with Stephen Singer’s comment is that the compressed atmosphere would heat over its entire depth, not merely at the surface.
☻Meinhundthatkeinenase is looking at a more complicated model. Convection heating would occur, but the transparent atmosphere would not radiate the extra heat.
☻But is there really such a thing as a transparent atmosphere?
☻How were the absorption spectra of gases determined? Did they shine light down a tube 100 miles long? Or did they use a shorter tube? How long? Did they control the temperature of the tube foot by foot? Is that short a tube accurate enough for the levels of energy and accuracy we’re dealing with?
☻What about diurnal atmospheric thermal tides? Wouldn’t the daily increase and decrease in atmospheric thickness translate to work? Wouldn’t work heat the imaginary column of air? Wouldn’t it heat more at the bottom than at the top, creating a “lapse rate”?
☻Could it be that other factors will cancel out, leaving the apparent lapse rate as an artifact dependent only on atmospheric mass or density?
☻Night, all.

January 13, 2012 11:33 pm

amazing willis how few people can follow instructions.
nick stokes is correct. Nice proof.
Sorry to see TB act that way. Tamino once banned Lucia for asking a question of the 2nd law.

wayne
January 13, 2012 11:35 pm

“How would Earth’s surface temperature change if atmospheric pressure were doubled, that is, increased to 2 atm by adding more N2 (and nothing else)?”
Well, by Stephen Wilde, the increased pressure and density would bear down on the surface increasing the conduction of energy from the surface cooling it, which would therefore reduce the amount of radiation also cooling the surface. In other words the conduction / radiation ratio would shift in the conduction direction. More conduction, less radiation.
Since conduction is slow, the air at the surface would be warmer and that would also cause more convection to constantly bring more cooler upper air in contact with the surface. All in all, the lower air would be warmer, probably in the doubling case much warmer. But that is just way we get our environmental lapse rate right now in our atmosphere.
Also, if there is any absorption of radiation by the air, then more radiation would be absorbed lower due to the increased density and that can be from both solar radiation and surface infrared radiation. That too causes more warming lower.
There’s more. High density gases hold onto their internal energy better radiatively, there are more molecules in a given volume so the distance that radiation travels between absorptions decreases. This too keeps the temperature higher for it is harder for that energy to radiatively escape from a given volume of air.
Each one of those statements seems true, so it appears just that simple, more mass causes more warming of the low atmosphere and this likewise causes more warming of the air all of the way up to the top.

Peter Czerna
January 13, 2012 11:36 pm

@willis
You are absolutely right that the N&Z speculations are junk. Almost every sentence in their contributions is either incomprehensible or plain nonsense.
Brave of you to give an ‘elevator speech’ version of GHG theory. Given the demands for concision, though, I think you go wrong or oversimplify on a number of points.
• The surface of the earth emits energy in the form of thermal longwave radiation.
– Get the nitpicking out of the way: ‘thermal’ is not necessary.
– The surface of the Earth also loses energy by conduction to the atmosphere and the subsequent convection.
The convected air, both before and after mixing, is at a lower temperature than the surface and so radiates less.
Unlike the 2D surface, the 3D lower atmosphere acts as a thermal accumulator during periods of low insolation (e.g. nightime). In electronic terms it would be the equivalent of a capacitor.
– I appreciate that I have gone beyond strict GHG ideas here, but you cannot discuss planetary temperatures just in terms of GHGs.
• Some of that energy is absorbed by greenhouse gases (GHGs) in the atmosphere.
– OK, of course.
• In turn, some of that absorbed energy is radiated by the atmosphere back to the surface.
– I get tetchy whenever I read about this ‘backwelling’ radiation.
In radiative terms, photons are emitted randomly in all directions: forwards, sideways and backwards. A proportion will head back towards the surface, most will not.
A GHG molecule that has absorbed radiation from the surface will also have its kinetic energy increased and pass on some energy by colisions with neighbouring atoms.
A proportion of any radiated energy will also be absorbed by other gas molcules.
What counts here is not the warming of the surface, but the warming of the atmosphere.
• As a result of absorbing that energy from the atmosphere, the surface is warmer than it would be in the absence of the GHGs.
– Again, the only temperature you are considering is that of the ‘surface’ and neglecting the heat stored in the atmosphere.
– At the moment, where I live, we have snow and ice on the ground but a pleasantly warm south wind and bright sun. I don’t care much about the surface temperature today.
Sorry – we’ve gone up three escalators by now and got to the roof.

crosspatch
January 13, 2012 11:40 pm

[NOT SO. Repeat after me, “GHG-free atmosphere”. The surface is the only thing that can radiate. w.]

NOT SO! The atmosphere TOUCHES the surface. It will warm by conduction. That will COOL the surface and WARM the atmosphere. Radiation has nothing to do with that. The atmosphere will then convect … GHG or no GHG. Now the surface has cooled and the atmosphere has warmed. Add a little wind and the conductive loss increases at the surface and the atmosphere warms even more. The efficiency of heat transfer by conduction is much greater than efficiency by radiation. That warmed air (with NO greenhouse gas at all of any sort, make it pure nitrogen if you like) will then begin to convect. Some of it will radiate at the lower altitudes, some of it will radiate at higher altitudes. Earth will radiate all the heat that it receives but that heat will not be even because of the chaos of turbulence caused by that convection. Add hills, mountains, random trees, etc. and that turbulence can become quite chaotic and result in very uneven temperatures.
Model the atmosphere as a bunch of concentric surfaces that are transparent to IR but still radiate. The surface at 1 foot altitude has a given amount of radiation and a given amount of surface area. The surface at 1000 feet has a larger surface area so with a given amount of total heat content it’s “surface” is cooler. At 10000 feet, the “surface” is even cooler and at the very tipppy top of the atmosphere you have the greatest “surface area” of all but the temperature at any given point of it is lower.
Even an atmosphere with no GHG will still exchange heat by conduction and will still convect and will still radiate heat as it rises. It will still have an adiabatic lapse rate.

crosspatch
January 13, 2012 11:50 pm

Take for example a radiator on a car (bad name, actually, because it doesn’t lose heat by radiation, it looses heat by conduction to the air, more air flow, more transfer of heat. Air flow doesn’t change radiation one iota. It is a heat exchanger to air). Rule of thumb is you lose 10% efficiency with every 1,000 feet of altitude because the air becomes less dense. At over 10,000 feet they basically become useless unless they are significantly over sized. About the only place on the planet that radiation would play a significant part in heat loss would be at the top of Mt. Everest.
Part of my life experience is designing flight qualified electronics that were required to be convection cooled to 10,000 feet altitude. That is hard to do because at 10,000 feet, heatsinks start relying on radiation as the major heat transport and radiation is the LEAST efficient heat transfer mechanism there is. The result was that heatsinks had to be much larger to work at 10,000 feet because conductive loss is so much less.
Earth’s surface acts as a conductive heat exchanger to the atmosphere. Radiation at the Earth’s surface is a minority route of heat transfer. Conductive losses are much greater.

Bill Hunter
January 13, 2012 11:53 pm

“Now suppose we add an atmosphere to the planet, a transparent GHG-free atmosphere. If the theories of N&K and Jelbring are correct, the temperature of the planet will rise.
But when the temperature of a perfect blackbody planet rises … the surface radiation of that planet must rise as well.
And because the atmosphere is transparent, this means that the planet is radiating to space more energy than it receives. This is an obvious violation of conservation of energy, so any theories proposing such a warming must be incorrect.”
I am not sure this is correct. If the GHG-free atmosphere is cooling the thinnest surface layer (via conduction and convection) what it has absorbed it is not radiating. Like a water-cooled engine block if you remove the water its going to heat up to equilibrium to radiate as much energy as it absorbing from the combustion. But with the water cooling system the block is cooler.
OK so the heat goes into the GHG-free atmosphere and convects upwards. Its not going to radiate rapidly because a substance is as efficient at radiating as it is at absorbing radiation. And a GHG-free atmosphere is considered to absorb zero by AGW scientists. I think that is incorrect also. Argon has a emissivity about .0005, meaning its quite transparent but not completely transparent. CO2 has an emissivity around .08 meaning its about 160 times more efficient at radiating than the IR inert Argon. I presume this relationship holds with other monoatomic gases as well. Supposedly the diatomic gases have higher emissivities than the monoatomic gases and both have far less emissivity as triatomic gases like CO2 and water vapor.
But the problem is an inefficient emitting substance has to be hotter to emit the same level of radiation as a substance which is a theoretical black body. So this heat that is being absorbed into the GHG-free atmosphere is trapped there and the surface film is being cooled in the process and also not emitting its budget to the sky. . . .so the system heats until the budget is balanced again.
Keep in mind when thinking about surface temperatures that we don’t measure the actual surface temperature but instead the warmth of the air 6 feet above the surface so a little irrationality is possible here.
Gravity’s role is to run the convection engine that separates the heat from the surface and warms the entire atmosphere.
Now I am not sure thats how it works but it does seem logical.
And I think the interesting part is even if thats not how it works the idea of a GHG-free atmosphere is still an impossibility. The term GHG-free atmosphere is in essence an oxymoron as the only way to have a GHG-free area above a surface is to have nothing there.
Finally, it may be up to 160 times less efficient at absorbing GHG but when there is 2500 times more of it than CO2 it may belittle the effect of CO2 so this should still work even if the mechanism is back radiation.
So this is when somebody pops in with the radiation curve and the CO2 bight. Well maybe that bight is created in a thin area of the atmosphere where water vapor is missing.
With that I am going to put my shield over my head.

January 13, 2012 11:54 pm

The basic issue is that there is no such thing as an equlibrium temperature, or equlibrium flux. The solar flux is zero at night and about 1000 W.m-2 around summer noon. The sun heats the ground during the day. 1000 W.m-2 corresponds to an equlibrium temperature of about 93 C. Dry ground may reach 50 or 60 C. At 50 C, the surface can only dissipate an excess of ~ 200 W.m-2 by IR emission. There rest is dissipated by convection. Some heat is stored below the surface by thermal conduction and gets radiated later in the day. The peak surface temperature is not reached until about 2 hours after the peak solar flux. This is the classic ‘signature’ of a therma storage oscillator. It means no thermal equilibrium.
The air is heated mainly by convection and the first 2 km or so above the surface acts as a ‘thermal blanket’ that cools quite slowly by radiation at night when the convection stops. Typical ‘average’ numbers are 50 W.m-2 lost from the surface and about 50 W.m-2 lost at the 2 km boundary. This is the ‘dynamic balance’ of the so called greenhouse effect. The heat capacity of a 2 km x 1 m^2 column of air is about 2 MJ.K-1. 12 hours of night time cooling at 100 W.m-2 is 4.3 MJ.m-2, so the night time ‘thermal blanket’ cooling is around 4 or 5 K. Usually, the air is moved on by the weather system before this cooling is observed.
As the air rises through the atmosphere it cools by expansion. Most of the IR cooling radiation to space originates from the water bands at around 5 km altitude. The ‘average’ lapse rate is about -6.5 K.km-1. To get to 5 km the air has to cool by 5 x 6.5 = 32.5 K. This is where most of the 250 W.m-2 ‘thermal equilbrium’ to space emission originates. The cooling occurs as the linewidths of the water band lines start to narrow and the water vapor concentration drops becasue of condensation. The IR flux ‘escapes’ beteen the water lines and does not get reabsorbed.
Almost all of LWIR flux reaching the surface comes from the first 2 km ‘blanket’. The air above 2 km up to the tropopause acts as a second independent thermal reservior that cools to space 24/7. It just keeps on cooling until it gets heated again by the next convective ‘pulse’ the following day. The daily temperature fluctuation of this upper reservoir is about 2 K or less. There is no equilibrium and the tropopause just moves up and down in altitude with latitude and seasons to maintain the thermal balance.
The Earth’s surface temperature can only be explained by the dynamic coupling of heat into thermal storage reservoirs. There are at least four thermal reservoirs and 6 energy transfer processes needed to explain even the basics of the surface temperature.
It is time to stop thinking in terms of an equlibrium flux.
(A freight elevator ride?)
This is a complicated subject. There are more details on my website at http://www.venturaphotonics.com. There is also a book ‘The Dynamic Greenhouse Effect and the Climate Averaging Paradox’ available on Amazon.

crosspatch
January 13, 2012 11:55 pm

Take a rock in a still air in full sun and take its temperature.
Put a fan on that rock for 30 minutes and take its temperature again.
The rock will be cooler, the air will be warmer. No greenhouse gasses needed.

wayne
January 13, 2012 11:59 pm

“which “curiously” happen to align with gaps in CO2 and H2O absorption of infrared radiation”
Glad you said that Pops, that is so curious isn’t it. So where is this -20 degC worth of LW radiation energy coming from when pointing a thermal radiation thermometer at the sky if the instrument is tuned to AVOID all GHG lines, right in the radiative “window” frequencies? That definitely perked my mind up. Hmm…
(I did earlier read of a report directly from the FLIR manufacturer on that very subject)

January 14, 2012 12:07 am

Verifying Willis’ numbers

On average day and night over the planetary surface, the Earth receives about 240 W/m2 of energy from the sun. The theoretical S-B temperature for this amount of radiation (if it were evenly distributed) is about -18°C, well below freezing.

Radiation = Emissivity times SBconstant times Temperature^4
240 = 1 x 5.67 x 10^(-8) x Temperature^4
Temperature^4 = (240 x 10^8)/5.67 = 42.32804233 x 10^8
Temperture = (42.32804233 x 10^8)^0.25 = 42.32804233^0.25 x 10^2 = 2.55 x 100 = 255 K
Deg C = K – 273 = 255 – 273 = – 18

But instead of being frozen, the planet is at about +14°C or so. That’s about thirty degrees above the theoretical S-B temperature.

Difference between actual and theoretical global mean temperature = 18 + 14 = 32 deg C
Willis, it is all right!

Lew Skannen
January 14, 2012 12:08 am

If the atmosphere is GHG free I cannot see how the temperature at the surface can change.
If there are GHGs present I believe that the Stefan-Boltzman equilibrium surface of the planet is raised above the surface and is therefore a larger sphere. This can then allow for radiative equilibrium with a higher surface temperature without violation conservation of energy.

January 14, 2012 12:11 am

@Willis
Again you are narrowing yourself into only considering a single form of heat transfer, Incoming radiation heats the blackbody, this in turn through conduction loses some of that heat through conduction to the atmosphere, all of the energy transfered by conduction cannot radiate as IR. You are now ignoring an entire realm of thermo dynamics, yes any radiative energy absorbed by the blackbody has to go somewhere once it is in S-B equilibrium but it is not restricted to a single energy form, heat is both EM radiation and kinetic energy.
Adding the atmosphere would add a second source of radiation once it has also reached an equilibrium point because even if the atmosphere is IR transparent it will still radiate IR of which a bit less than half will radiate back into the planet. This in turn will begin to raise the energy going into the planet by radiance, which will then raise its temperature under the S-B. The only way the atmosphere does not conduct heat energy away from the planet is if there is no way for them to physically interact, though the only way I see gravity having anything to do with this is as a driver of convection which just speeds the rate of conduction into the atmosphere.
Energy from the sun must equal the energy leaving the blackbody in ALL FORMS not just radiation.

Paul Martin
January 14, 2012 12:14 am

Willis: consider the difference between ground frost (radiative cooling) and air frost (radiative cooling + convection).

David
January 14, 2012 12:23 am

I have a problem with your proof by contradiction. Let me quote it here so that it is fresh in the mind:
So let us assume that we have the airless perfectly evenly heated blackbody planet that I spoke of above, evenly surrounded by a sphere of mini-suns. The temperature of this theoretical planet is, of course, the theoretical S-B temperature.
Now suppose we add an atmosphere to the planet, a transparent GHG-free atmosphere. If the theories of N&K and Jelbring are correct, the temperature of the planet will rise.
But when the temperature of a perfect blackbody planet rises … the surface radiation of that planet must rise as well.
And because the atmosphere is transparent, this means that the planet is radiating to space more energy than it receives. This is an obvious violation of conservation of energy, so any theories proposing such a warming must be incorrect.

Let me propose a thought experiment. We have a black box floating in space evenly surrounded by a sphere of mini-suns, as at the start of your proof by contradiction. The black box radiates exactly the same energy as it receives and is in temperature equilibrium. The temperature of the black box is given by the S-B law.
We don’t know what is inside the black box. At its simplest, it could be a single solid mass. Or perhaps there could be two solid masses inside the black box, one at temperature T1 and one at temperature T2. Each mass radiates towards each other as well as the universe generally. Each mass within the black box is continuously exchanging energy with the other, and their temperatures vary over time as well. We don’t care because overall the black box as a whole is radiating as much energy as it receives. We can use the S-B law to calculate an equivalent black body temperature, but this is an exercise in maths: the temperature we derive from such a calculation is relevant only to the black box as a whole. It is not an “average temperature” of the two bodies within the black box; in fact we cannot deduce any information about the masses within the black box using only the S-B law. We can only use the S-B law to describe the black box itself, and everything it contains.
Of course the black box could contain many, many masses. Some could be solid (earth) some liquid (oceans) and some gaseous (atmosphere). Each mass could vary in temperature in complex ways, continually transferring energy between each other. But overall the entire system can be described by the S-B law, and the entire system overall is in equilibrium with the universe.
Getting back to your proof by contradiction. When you add the atmosphere you now have two masses inside the black box. You then assume (this is the assumption that you are going to come back to and say is false, once the proof by contradiction is completed) that the atmosphere heats the ground. You then say that because the ground is hotter, it must radiate more energy (by S-B law) which is false therefore the assumption is false.
But this is not correct. You have not taken into account any change in temperature of the atmosphere.
Take your black box which is radiating according to S-B law. Put inside the black box some solid mass (ground). Take your measurements and deduce the black body equivalent temperature. Now add a second mass to the black box, without introducing any extra energy into the system. This second mass is a gas (atmosphere). Now assume that the presence of the atmosphere increases the temperature of the ground. The two masses exchange energy inside the black box, the ground heating and the atmosphere cooling until they reach equilibrium with each other. The entire system (the black box) will still be in equilibrium with the universe if one of the masses inside (the atmosphere) is at a different temperature compared to the temperature of the other mass inside (the ground).
Your proof by contradiction is flawed. You cannot deduce the temperature of any item within the black box according to the S-B law. You can only look at the system as a whole. You said: …I’m interested in people who can either show that my proof is wrong… and I believe I have done so.
I cannot say anything about the Jelbring or the N&Z hypothesis. As I understand it, the presence of an atmosphere in gravity will lead to a higher pressure and higher temperature at the surface. This can only come about because the temperature reduces at altitude – conservation of energy as well as various Gas Laws apply. This is exactly how our own atmosphere behaves. Whether or not this entirely explains the temperature rise of the Earth over the black body temperature, I cannot say. I am not trying to prove their theorem; I am only pointing out the mistake in your own criticism of that theorem.
I trust that you shall receive this in good faith. I am not impugning your character nor taking personal attacks; I just don’t see how your proof by contradiction is correct. Perhaps there is something I’ve missed; please correct me if so. Kindest regards,
David

Julian Braggins
January 14, 2012 12:26 am

By introducing an unnecessary condition in your thought experiment, i.e. the multiple suns, you have made it impossible to gain any insight to the real working of the atmosphere.
Without that condition, the air would heat up to the surface temperature.
Differential heating , night / day, polar / equatorial, would induce circulation.
Lapse rate heating, determined by the depth of the atmosphere would ensue.
A new higher temperature equilibrium would be established.
But what would I know, I left school at 15, 64 years ago 😉

wayne
January 14, 2012 12:33 am

Willis: “Michael, the energy radiated by the surface is fixed. It must emit what it recieves, no more and no less. ”
Absolutely false. Conduction carrying away energy from that same surface does reduce the radiation leaving from that surface when in equilibrium. Willis, you can call it a reduction in the emissivity of that surface if your mind prefers. Well… that is really what does happens, the emissivity decreases. A surface receiving all of its energy by radiation and part of that energy leaving by conduction, then, the amount leaving by radiation WILL decrease under the amount received. WILLIS…. do you happen to remember the first law of thermodynamics???

Jerker Andersson
January 14, 2012 12:34 am

“NOTE 1: Here’s the thing about a planet with a transparent atmosphere. There is only one object that can radiate to space, the surface.”
Unless your point is to exclude conduction and convection in this thought experiment the GHG free atmosphere will also remove heat from the surface by conduction and convection. Each molecule in the atmosphere can emit radiation both upward to space and downward to the surface just as a GHG does. But it would not be able to absorb any outgoing radiation from the surface or reabsorb any IR emitted from the atmosphere toward space, only a GHG could do that. For the part that goes downward it will retransfer energy to the surface again.
In this case the more air molecules you add the more IR can be transfered via the atmosphere back to space until you get a balance where the energy transfer via conduction between surface and atmosphere is the same as the IR radiated from the atmosphere back out to space.
The atmosphere in this case works as a heat sink for the surface.
But since in this case both surface and the atmosphere would emit IR to the space then the temperature of the surface must be lower in order to split some of the radiation back to space between the atmosphere and the surface. As you said, the energy must be conserved.
To sum it up for a GHG free 100% transparent atmosphere.
– The surface gets heated by the incomming radiation.
– Some energy gets transfered from the surface to the atmosphere through conduction.
– Backward radiation out to space is now split up between atmosphere and surface thus the surface has to be cooler since it is emitting less.

Editor
January 14, 2012 12:34 am

Willis
Nice article.
I enjoy the input from both R Gates and Joel Shore over here and think Blogs such as this one would be diminished if everyone were to sing from the same song sheet. Both are knowlegable and argue their corner well-although I disagree with most of what they say and they do have a blind spot on Climate History.
Banning Joel from Talk Shop for the reasons stated seems extreme and makes me uncomfortable as it has echoes of Real Climate.
Come on Roger, you’ve always seemed a resaonable person. How about showing that you accept all scientific viewpoints even if you disagree with them? Banning should only be applied to especially irksome or abusive trolls and neither Joel or R Gates fit into that category by any stretch of the imagination.
tonyb

David McKeever
January 14, 2012 12:37 am

The reason your thought experiment could radiate more heat than it gets (from the sun) is that you are adding thermal energy int he form of moving molecules. Boyle’s gas laws tell us if you pressurize a gas it will increase heat. So assuming that when you say you add an atmosphere you are adding low density gas molecules and allowing gravity to be your pressurizer then you would have a one time spike of temp. until it convects into the land/ocean/air and radiates out. Then it would return to the temp before you added the atmosphere. If the atmosphere was already compressed to 1 atm. at the sea level and lower pressures at higher altitudes before you added it, then it wouldn’t have any effect. (I think.)

Roger Clague
January 14, 2012 12:38 am

crosspatch 13 Jan 10.40pm.
This comment was snipped by the Willis, the guest poster.
crosspatch is debating and disagreeing with the post. He is not off topic. He has been censored. Please restore this comment.
Moderators should moderate not the poster.
The theory of gravitational enhancement claims there cannot be a Greenhouse gas free atmosphere. The mass of all gases in the atmosphere cause the so-called Greenhouse Effect.

crosspatch
January 14, 2012 12:39 am

“1. The non greenhouse gas atmosphere is a perfect conduction insulator to space, it can’t radiate its heat out.”
What? Everything with a temperature radiates. I don’t care WHAT a substance is, it will cool. It will cool until it reaches equilibrium with its background. Greenhouse has absolutely nothing to do with radiation, it has to do with absorption. Something can be completely transparent to LWIR and still cool down by radiation. A greenhouse gas is something that absorbs LWIR. It has nothing to do with what it radiates. So I can have a non greenhouse gas that heats via direct contact … conduction. It will STILL radiate that heat away. It just didn’t heat by absorbing LWIR.
Are you guys sure you know what greenhouse is?

kiwistonewall
January 14, 2012 12:39 am

A slight correction to make your statements work within the laws of Thermodynamics:
The poorly-named “greenhouse effect” works as follows:
• The surface of the earth emits energy in the form of thermal longwave radiation.
• Some of that energy is absorbed by greenhouse gases (GHGs) in the atmosphere.
At Night there is a net radiation by the atmosphere back to the surface.
(During the day, there is no heating, only a slowing of heat loss, as net radiation is outward)
• As a result of absorbing that energy from the atmosphere, the surface is warmer at night than it would be in the absence of the GHGs.
So the effect is that the night time temperature is higher and the day time temperature is cooler (due to the thermal mass of the air & oceans)
However, there is the same contradiction with the greenhouse effect.
Because there is ALWAYS more energy radiating from GHG out to space, (since emissions go equally in all directions, and the geometry of a sphere means just more than 50% is outward) this means that IF the planet gets MORE energy to the surface from GHG then the planet is radiating to space more energy than it receives. This is an obvious violation of conservation of energy, so any theories proposing such a warming must be incorrect.
It matters not how many “layers” of GHG you have, Once equilibrium is established, more than half is always heading to space. The real world atmosphere is turbulent, but that standard explanation is always worked out on a static atmosphere.
It seems to me that there is a maximum effect, and once this is reached, there can be no further warming. Everyone seems to forget that classic climate theory (from textbooks prior to 1988) was that the atmosphere warms by convection and cools by radiation.

Hoser
January 14, 2012 12:44 am

A curious fact is that almost everything around us is continually radiating energy in the infrared frequencies. You, me, the trees, the ocean, clouds, ice, all the common stuff gives off infrared radiation.
And the air.
GHGs just absorb in certain bands of outgoing IR and make the air warmer. That makes the entire atmosphere radiate more, half the time right back down to the surface. An atmosphere sans GHG would still radiate, just less.
How would the atmosphere get warm without GHGs? Conduction and convection as others have mentioned.
The assumption made in the post is ALL of the energy absorbed during the day is immediately radiated back to space, and so we seem to have trouble explaining a higher than expected temperature.
If a significant fraction of the solar energy is absorbed, it may take time for it to be radiated away. Consequently, the temperature will rise until balance occurs. When the Earth is significantly warmer the total output matches the total input, accounting for delayed release of input energy. One mechanism for absorption of solar energy that cannot be immediately re-radiated is ocean mixing. Only the surface will radiate to space. The solid surface of Earth can act in the same way – as an energy sponge. The delay in reradiation must raise the temperature of the Earth to achieve balance.
To illustrate: Try walking barefoot in a freshly paved parking lot in Florida some summer.

Steve C
January 14, 2012 12:45 am

Willis, surely your transparent atmosphere is, like the standard GHE model, an unphysical and unhelpful concept, since in reality all gases will have some absorptive / emissive characteristics. If that were not so then, by definition, adding as much of this ‘atmosphere’ as you like cannot have any influence on the purely radiative balance between your planet’s surface and its shell of mini-suns, as you say. As soon as you concede that the added atmosphere can absorb and emit on its own account, though, you have an ‘atmosphere effect’, temperature gradients in that atmosphere, convection and so on, whereupon yes, the planet’s surface will warm as the effective radiating level moves upwards and away from the planet’s surface.
It seems to me that the basic error (in the Earth case, at anyrate) is in the assumption that the planetary surface receives a quarter of the available power coming in from the sun, continually. It doesn’t. I’m much more impressed by Postma’s thermodynamic analysis (‘Understanding the Atmosphere Effect’, ‘The Model Atmosphere’ etc.) in which he shows that consideration of a dynamic system, with a rotating planet orbiting a single sun, renders the introduction of a ‘greenhouse effect’ unnecessary, the (considerably) higher equilibrium temperature at the subsolar point plus the insulation of the atmosphere giving quite enough increase in average energy input and surface temperature to account for what we observe.
If I’m making a cup of tea, I need one cup of boiling water. Two cups of lukewarm water – even if the amount of heat energy is identical – just don’t do the work! It’s not only how much energy there is in the system, it’s also how it’s distributed.
As ever, a thought-provoking post. And now I know to keep an eye open for a copy of that Geiger book.

Jerker Andersson
January 14, 2012 12:51 am

“If there are no GHGs in the atmosphere, the atmosphere will not and cannot radiate energy.”
As far as I know all atoms above 0K raidate IR, even molecules in our atmosphere. Otherwise the air can be heated to any temperature and it will never cool until it gets into physical contact with cooler molecules i.e heat transfer via conduction.

Peter Czerna
January 14, 2012 12:53 am

@willis/crosspatch
Of course a gas can radiate energy, whether it has GHGs or not. If conduction/convection increases its temperature, it radiates more.
Why should gases not obey thermodynamic principles?

January 14, 2012 12:55 am

Why everybody here imitates Mr. Eschenbach, using hyphen in “Stefan-Boltzmann temperature”, “Stefan-Boltzmann equation”, and “Stefan-Boltzmann constant” expressions, as if “Stefan” and “Boltzmann” were two people who developed this formula?
Stefan Boltzmann was one man, his last name was Boltzmann, his first man was Stefan, and the correct way to use his name is “Boltzmann’s equation” or “Stefan Boltzmann’s equation” (if you insist for some reason on repeating his first name all the time) but not “Stefan-Boltzmann equation.” The “S-B” abbreviation, in this context, is incorrect.
Boltzmann killed himself, because the “consensus” among most of the respected, published and peer-reviewed “scientists” of his time was that Boltzmann was nuts. Since then, however, his classic formula has become a mandatory part of any high-school course of physics, and it is not clear to me, why discussing it here is of any interest, and why one has to be “mathematically inclined” to read one of the simplest formulas ever.
P.S. By the way, all gases and gas mixtures radiate heat, not only those containing the ubiquitous “green-house gases.” An atmosphere containing no water vapor or other “GHG” gases would warm up under sunlight and radiate heat, because light dissipates, to some extent, in an atmosphere consisting of any gas, however transparent.

Mike Edwards
January 14, 2012 1:00 am

Willis,
Thank you for showing the incorrectness of the Nikolov and Zeller idea in such a simple and straightforward way.
Meanwhile Robert G Brown’s article on WUWT showed some of the true complexity of calculating the surface temperature distribution of the Earth – no N&Z in sight!

Edim
January 14, 2012 1:01 am

“How would Earth’s surface temperature change if atmospheric pressure were doubled, that is, increased to 2 atm by adding more N2 (and nothing else)?”
Well, according to the GHG hypothesis, that would reduce the mixing ratio of atmospheric CO2 and less heat would be trapped. The surface would cool.
In reality 2 atm would change a lot of things, evaporation, convection, density of the atmosphere…

Alan Wilkinson
January 14, 2012 1:08 am

Since both N2 and O2 absorb and radiate in the UV region there is no such thing as a transparent non-GHC atmosphere. This is a good account of gas radiation: http://www.heliosat3.de/e-learning/remote-sensing/Lec7.pdf

daniel kaplan
January 14, 2012 1:10 am

1) Earth/Atmosphere is a thermal machine (details unimportant)
2) Mechanical Energies are Earth rotational energy + Earth/Moon Potential Energy (Tides)+…….+Atmospheric energies (small)
3) Energy imbalance goes into tiny change of earth rotation speed or ……

Julian Braggins
January 14, 2012 1:14 am

Gravity, plus the Sun, water vapor, rain, runs all the hydro power. The atmosphere is a little more diffuse, and harder to extract work from, but it does heat up as it falls down.
By the way, O2 and N2 seem to have significant IR absorption bands, considering their proportion of the atmosphere, was your hypothetical atmosphere also hypothetical Willis?

January 14, 2012 1:21 am

Alexander Feht says: January 14, 2012 at 12:55 am
“Stefan Boltzmann was one man, his last name was Boltzmann, his first man was Stefan”

No, Boltzmann’s name was Ludwig. Joseph Stefan was his predecessor as Professor of Theoretical Physics at the University of Vienna.
He was a highly respected scientist in his lifetime, and had been appointed to the most eminent scientific chair in the Austrian Empire. He had been received by the Emperor, shortly before his death.

kiwistonewall
January 14, 2012 1:24 am

To correct some misunderstanding:
Yes, all gases will emit and absorb energy, but for non GH gases the effect is small over thermal ranges (what we associate with earth temperature ranges). For Oxygen and Nitrogen, only collisions provide the dipoles needed for thermal absorption and emission. This is significant only at high pressure and temperature (where there are more collisions). Since the atmosphere is mostly non GH gases, even though this effect is small, it is by no means zero, but it is maybe less than 1% of that of the GH gases. Thermodynamics means that heated gases must cool to that of their surroundings. They do this by mainly by expanding, which is the cause of convection.

January 14, 2012 1:26 am

Hello again Willis
We didn’t quite finish our discussion at the Moon/Mistress thread so I’d like to continue it here if you’re willing.
The key concept to keep in mind is that warming by conduction (between a solid surface and a gas) is always more efficient than cooling by conduction.
This is because a warming parcel of air rises and expands, is replaced by a cooler parcel of air which is also warmed by the surface and the cycles goes on so long as the surface is warmed by insolation.

First floor, giftware

Cooling by conduction is not quite the exact opposite of warming by conduction due to the phenomenon of temperature inversion.
When a warm parcel of air conducts with a cooler surface, the air cools, the surface radiates away the newly attained warmth. However, the parcel of air higher up cannot make its way down because it is blocked by the now cooler parcel of air underneath.
Though conduction WITHIN the molecules of air will move some warmth to the adjacent lower and cooler molecules, this process is very slow and inefficient, otherwise temperature inversions wouldn’t happen.

Second floor, ladies wear

Temperature inversions on Earth are most common near coastal upwelling zones (note the famous Californian smogs of the 70’s and 80’s) where cold upwelling water cools the air immediately above which prevents the air higher up from descending because it is now warmer than the air below.
They are also very common at the poles especially in winter where warmer air from lower lattitudes conducts with the frigid surface forming a barrier which stops the warmer air above from descending.

third floor, menswear

Therefore, with the above in mind, a previously bare rock planet, injected with a non-GHG atmosphere will first and foremost have its outgoing longwave reduced because of conduction.
Due to the fact that the equator of a sphere is the warmest, this is where the most conduction will occur. The atmosphere will transport this equatorial heat towards the higher lattitudes.
This conduction cannot cease until the atmosphere attains the same temperature as the surface at the equator.
In all this time, the outgoing longwave from the surface will be lower than that of a blackbody, i.e. the system as a whole (surface plus atmosphere) is accumulating heat.

fourth floor, manchester

The process will come to equilibrium when the atmosphere (at all lattitudes) reaches the same temperature as the surface AT THE EQUATOR (or near enough so that convection is a tiny trickle)
Therefore, the AVERAGE temperature of the ATMOSPHERE will be higher than the AVERAGE temperature of the SURFACE, i.e. higher than the SB temperature.
Theoretically, because of the limited conduction from a warm atmosphere to the cold polar surface will warm the surface by a small amount (temperature inversion prevents continual heat transfer by conduction) the polar surface will be warmer than it was when without an atmosphere.
By the same token, the equator will be cooler than they were when without an atmosphere.
Put simply, a planet with a non-GHG atmosphere would be dominated by temperature inversions.

fifth floor, home made pies

This is where I humbly get off
p.s. my last response at the Moon/Mistress thread is at http://wattsupwiththat.com/2012/01/08/the-moon-is-a-cold-mistress/#comment-860181
my best regards as always.
We need more Willisses in Science. He makes us think

Hail Core
January 14, 2012 1:40 am

I have talked with one engineer who is working infrared warmer manufacturer company. They have tried to warm air with infrared warmer and they have never managed to do so. They have even set tens of radiators to work at same time (hundreds kilowatts), no measurable results in air temperature. So the claims that infrared radiation warms atmosphere can’t be correct, so the main question is, what is absorption when you deal with gases (or air). How gases can backradiate anything if you can’t increase it’s temperature with radiation? If you have dust, water or other tiny particles in air, then radiation can warm these a little bit. They don’t add energy only transfers the original energy, so these can’t warm (add energy that increases temperature) anything cause they are not energy sources.

January 14, 2012 1:45 am

Alexander Feht says:
January 14, 2012 at 12:55 am
Why everybody here imitates Mr. Eschenbach, using hyphen in “Stefan-Boltzmann temperature”, “Stefan-Boltzmann equation”, and “Stefan-Boltzmann constant” expressions, as if “Stefan” and “Boltzmann” were two people who developed this formula?
Stefan Boltzmann was one man, his last name was Boltzmann, his first man was Stefan

Come on. Jožef Štefan was a full professor at the University of Vienna, he has discovered (and published) that energy flux of black body radiation is proportional to T⁴ in 1879 (based on measurements of French physicists Dulong and Petit). Ludwig Boltzmann was his student, he earned his PhD under his supervision in 1866 (the same year prof. Štefan became Director of the Physical Institute) and has extended Štefan’s results in 1884 to grey body radiation (at that time he was professor of Experimental Physics at the University of Graz). Also, Boltzmann’s beard was much thicker, therefore we are most definitely talking about two separate persons.
It is not a shame to be ignorant, but it is to pretend to know.

Patrick Davis
January 14, 2012 1:55 am

I asked an alarmist a very simple question recently. What emites/gives off IR, a block of ice, a bottle of water or a pot of boiling water? He (I assume so based on name) answered water vapour. He could not grasp the fact that ice emits IR. And then accused me of asking him a “trick” question!! This is the level of scientific stupidity anyone equiped with even a basic knowledge physics has to deal with in this “debate”.

tallbloke
January 14, 2012 1:57 am

Apologies for the long comment, there’s quite a lot to deal with here.
Nikolov and Zeller’s extended conference poster ‘The Unified Theory of Climate’ was originally posted at the Talkshop a day before it was posted here at WUWT. On my website Willis says of it:
” I find the work of Nikolov and Jelbring to be laughable. I cannot even understand Nikolov. I invited him to state the core of his theory in a few sentences, since his writing is unintelligible….. He talks about atmospheric sponges and bowls, I can’t make sense of it…..As a result, I can’t tell if Nikolov’s theory violates the laws of thermodynamics.”
I ran a word search on the text but could not find any references to “atmospheric sponges and bowls”. Willis often complains that people argue against what they think he said rather than directly quoting him. I think he should follow his own advice and extend the same courtesy to others.
I also republished Hans Jelbring’s 2003 E&E paper ‘The Greenhouse Effect as a function of atmospheric Mass’ on which Hans Jelbring was kind enough to engage with Talkshop contributors.
Willis didn’t place any comment on that thread but said elsewhere on my website:
“Here’s the short proof, by contradiction. Jelbring proposes that a perfectly transparent, GHG-free atmosphere will raise the temperature of a planet’s surface well above the S-B temperature obtained from the average impinging solar radiation. (This is the situation of the Earth, for example.)
But if that is so, and the surface is somehow warmed above the S-B temperature, and the atmosphere is transparent, then the surface must radiate more energy to space than it is receiving, which is clearly impossible. Q. E. D…..I said the same thing to his face—his theory breaks the laws of thermodynamics.”

Once again Wiillis is doing what he tells other people not to do; arguing about what he thinks Dr Jelbring said instead of quoting him: Here is a brief excerpt from the beginning of section 2.1 where Hans Jelbring sets up the definitions for the model Earth in his 2003 paper. It is sufficient to dismiss Willis’ ‘proof’:
“A simplified model of Earth will be considered. The model planet does not rotate. It
neither receives solar radiation nor emits infrared radiation into space.”

Hans Jelbring tells me he has never met Willis face to face. To help resolve this apparent contradiction Willis could tell us when and where he
“said the same thing to his face—his theory breaks the laws of thermodynamics.”
Perhaps Willis is speaking figuratively, and is referring to an interaction with Hans Jelbring on the old CS email list? Before banning himself from the Talkshop because I ‘banned Joel Shore’ Willis should have engaged directly with Hans Jelbring on the same page where the entire E&E paper is published. That would have been the more scientific (and courteous) thing to do in my opinion.
Willis complains that I am preventing Joel Shore from expressing his scientific beliefs. This is incorrect. I won’t let Joel engage in the various very active and nicely undisrupted threads we have running at the moment but instead offered him a guest post where he could set out his scientific position formally on a thread of his own. It’s a strange kind of censorship which offers the ‘victim’ the microphone and points the way to the stage. According to Willis:
“Joel may have a hundred reasons not to want to invest the time and effort in a guest post.”
Joel himself says:
“I’m not particularly interested in doing a guest post. I have my hands full just trying to respond to all the misguided people over at WUWT and, with classes starting again tomorrow, I won’t be able to allow this time sink to continue for too much longer.”
Considering the much bigger loudhailer Joel has here at the biggest climate site in the world, it’s understandable why he would think it more important to spend his time here rather than on a website which gets around 1/20 of the traffic WUWT does.
The benefit to the Talkshop is that it enables its contributors to continue calmly discussing the merits and demerits of the properly set out scientific positions in the papers kindly provided by
Nikolov and Zeller
Hans Jelbring
Gerlich and Tscheuschner
R.P Sheehan
Johann Josef Loschmidt
Coombs and Laue
Roman et al.
Velasco et al.
William Gilbert
and
Dean Brooks
Cheers
TB.
[SNIP: No, there’s not a lot to deal with here, Roger, you have your blog for that. As I clearly requested above, we are dealing with two things:
• Your contribution of an elevator speech explaining Nikolov/Jelbrings work. Since you have not given one, I assume you don’t understand them.
• Your demonstration that my proof is wrong.
As a result, I have snipped your complaints about me and what I have done and what I understand or don’t understand. You can whine about that with your friends on your own website. Here, we’re discussing a couple very particular scientific questions. Come back if and when you are willing to give us your elevator speech or show my proof is wrong.
Sadly, but firmly,
w.]

[NOTE: Tallbloke’s post has been restored in its full glory. -w.]

January 14, 2012 1:57 am

No gas is transparent to all radiation – even if it is transparent to infrared, it will absorb ultraviolet, x-rays, something, and be heated by them..
So the true receiving area, even for a planet with a perfect GHG free atmosphere, is the disk section of surface + atmosphere.
And I put it to you that the resulting lapse rate could raise the surface temperature at least a little above the SB theoretical limit.

Rob R
January 14, 2012 2:07 am

Alexander Feht and others,
If all gasses are capable of absorbing radiant energy and if all gasses are capable of emitting at least a small amount of energy by radiation, then presumably all gasses are, to at least a small degree, greenhouse gasses. So any mixed atmospheric gas, for instance one composed exclusively of N2, O2 and Ar, will absorb some component of shortwave radiation and emit longwave radiation. Therefore there will be a greenhouse effect even if it is not large.
The question then would be whether these gasses are or are not capable of sublimation from a liquid or solid state at the Earths blackbody temperature. If the “gasses” are capable of existing in a gaseous phase at this temperature then there will be an atmospheric greenhouse effect even though it may be a small one. So logically the temperature of the gas at the earths surface must be greater than the blackbody temperature.
It would therefore seem that the greehouse effect attributable to the usual suspects (primarily H2O, CO2, O3, CH4) is less than 100% of the full greenhouse effect.
Could someone please explain where the above is incorrect?

kwik
January 14, 2012 2:08 am

Willis Eschenbach says:
January 14, 2012 at 12:37 am
“I will repeat it again. If there are no GHGs in the atmosphere, the atmosphere will not and cannot radiate energy. That’s the whole point. The only thing that can radiate is the surface. You keep claiming the atmosphere will radiate. It will not.”
I dont understand this . Are you saying I cannot heat Nitrogen?
If I put Nitrogen in a bottle. I lower the bottle in water that is constantly at 293 K. I leave it there for 50 years. Now the Nitrogen should be at 293K too.
I build a rocket with an insulation chamber.. I put the Nitrogen bottle inside this chamber. I launch the rocket into space. Via radiocontrol I manage to open the insulation chamber and release the nitrogen bottle into empty space.
The nitrogen bottle is now at 293K ? Empty space is…2-3K? Wouldnt the Nitrogen bottle ratiate energy?

January 14, 2012 2:14 am

Willis,
Thank you for your intellectual honesty: your critiques of the warmist theory is not blindfolding you when physical reality and valid physical laws have to be taken into account. So, for two reasons I appreciate the title that you gave to this post: a game of words and a call for seriousness.
Warming is not only due to GHG. Some of the Incoming solar radiation is absorbed by nitrogen, oxygen, CO2 and water in the UV/visible range, contributing to a temperature increase of the air mass.
In the atmosphere, there are enough components to ensure a long wave radiation according to SB law.: GHG (CO2, water, methane, nitrogen oxide), liquid water in form of clouds (cloudiness is about 60% of the Earth surface), and aerosols (liquids and solids).
Please have a look at this simple two layers model which is just assuming the existence of an atmosphere, regardless of its composition: http://climate.mr-int.ch/TwoLayersClimateModel.html
After having taken various feedback mechanisms the calculated surface temperature increase due to GHGs since the beginning of the industrial era is approx. 0.4°C, quite below values cited in most papers about warming, in particular by the IPCC. But some warming takes place thanks (or because of) GHGs. For any doubling of the atmospheric CO2 concentration a temperature increase of 0.5-0.6 °C can be expected.
A simple (simplistic?) model giving reasonable orders of magnitude, and satisfying the golbal heat balance requirement.

January 14, 2012 2:18 am

Alexander Feht:
I don’t usually quote Wikipedia, But Ludwig Boltzmann’s superviser/tutor was Josef Stefan.
And I agree with other, all gases radiate EM if above 0K. It can’t be ignored if a “greenhouse gas”-free atmosphere is being hypothesised.

kwik
January 14, 2012 2:23 am

I forgot to mention; This is my Elevator speech. Only the Elevator was very fast.

Roger Knights
January 14, 2012 2:25 am

michael hammer says:
January 13, 2012 at 9:45 pm
Willis, I agree with amonst everything you said with one exception. I question the emissivity of fresh snow. Yes cetainly water and ice have IR emissivities very close to 1 but fresh snow is a mixture of ice and air with each “layer” or crystal a few to a few 10′s of microns thick. each transition of refractive index gives rise to reflections and the repeated and rapid change of refractive index results in a highly reflective surface. That is also what makes crazed paint surfaces appear whitish. It means absorptivity and hence also emissivity should be low – well below 1. I know how at least some of these absorptivity measurements are made (using an integrating sphere) and integrating spheres give the wrong answer for translucent materials – they suggest a reflectivity lower than it actually is (I know I design them). Consider how the interior of an igloo could be at +18C with minimal heating if the interior walls had an absorptivity of about 1. A high absorptivity would make an igloo more or less the equivalent of a sub zero cool store. Much more significantly, consider how the temperature in the antarctic as measured from satellites is around 180K when the coldest point even on the high plateau is more like 220K. The only way that I know for a low temperature to be recorded spectroscopically is if the surface emissivity is low.
Its an important issue because of the claim that snow gives rise to positive feedback. Supposedly the snow reflects incoming energy whilst radiating outgoing energy so it signifiucantly cools the surface yet the ground underneath would absorb incoming energy and thus would be more warming. If the emissivity of snow is far less than 1 then that theory is disproven.

Wow–this is something that might be put to the test–maybe even by a college-class or a high-school science fair project! (If the kid can get a grant.)
[COMMENT: Roger, I use the MODIS IR emission library for this question. It gives the same value as Geiger, .98 or so – w.]

Archonix
January 14, 2012 2:27 am

I found this thread somewhat interesting until I read tallbloke’s rebuttal just above. Willis is being disingenuous, setting up a strawman to knock down and thus not engaging with the actual theoretical model he claims to be taking apart.
[SNIP: If you wish to accuse me of being disingenuous, quote my words or go home. I won’t stand for this kind of vague nasty accusation, that’s a slimy tactic. And if you believe a man like Tallbloke, who censors scientific opinion that he disagrees with, you are an idiot. -w.]

Robin Hewitt
January 14, 2012 2:33 am

Okay I obviously know very little compared to you lot, but surely planet Earth has to radiate more than it receives to maintain a crust… ?

Gary Hladik
January 14, 2012 2:38 am

Steve C says (January 14, 2012 at 12:45 am): “Willis, surely your transparent atmosphere is, like the standard GHE model, an unphysical and unhelpful concept, since in reality all gases will have some absorptive / emissive characteristics.”
I assume W’s “transparent atmosphere” is a conceptually ideal experimental tool much like the frictionless planes and weightless pulleys I remember (vaguely) from physics class. Granted it may not apply to any real atmosphere, but we’re talking about a narrow thought experiment, not real life.

January 14, 2012 2:40 am

Somehow, I have difficulty with the concept of a perfectly transparent atmosphere, one that does not absorb radiation. As has been pointed out, neither could it emit radiation. An interesting concept, but not available in this particular galaxy.
Genghis is closest to my thoughts on this. The atmosphere is going to sort itself out to the adiabatic lapse rate temps, and the radiative “surface” is going to rise, warmer below, cooler above.
The dry adiabatic lapse rate is g / Cp, where g is gravity and Cp is the specific heat of the atmosphere. The lapse rate does not vary with elevation, which means that Cp doesn’t vary with density, so I don’t see how a denser atmosphere would perforce be warmer.
Lapse rate indeed has nothing to do with elevation, but density does. A denser atmosphere will be thicker, for any gas. We think of elevation in terms of “up,” but a better term for our purposes would be “depth.” Starting at the top of the atmosphere, zero depth, zero density, the deeper you go, more more distance that lapse rate has to work, thus higher temperature.

Geoff Sherrington
January 14, 2012 2:45 am

Willis [NOT SO. Repeat after me, “GHG-free atmosphere”. The surface is the only thing that can radiate. w.]
Since both the surface and the atmosphere consist of atoms and molecules, it might help if you explain the fundamental difference between them that allows the surface to radiate, but not the atmosphere.
You must envision some solid materials that emit far infra red that passes unchanged through the non-GHG atmosphere. They are all around us. The commercial FLIR camera will detect temperature differences that show as lifelike images via a microbolometer detector, into the tens of microns wavelength range at least. The “image” can travel many kilometers through the atmosphere. The cameras perform better when the interior optical path, where the image is smaller and more concentrated after passing through lenses, is filled with dry nitrogen gas to remove absorbers CO2 and H2O (gas). The idealised FLIR camera would seem to be a working example of your preferred elevator in action.
For a model where an atmosphere is suddenly introduced, there would be an equilibration period. Depending on the precise method, there should be heating from gravitational compression. Then there would be cooling, because the introduction process is a once-off, not a continuous one. The cooling cannot reintroduce hot material by conduction or convection to lead to an overall gain because the external heat sorce has a constant input in the first approximation.
BTW, it would be interesting to know if there is a compilation of global surface temperatures taken a few meters BELOW the surface. We seems to be fixated on certain concepts because Stevenson screens with thermometers were a meter and a bit above the surface.

John Marshall
January 14, 2012 2:47 am

Willis I cannot believe that you do not understand adiabatic heat gains due to compression. [SNIP- I specifically asked you to stick to elevator speeches and disproving my proof. -w.]

joshua Corning
January 14, 2012 2:47 am

shot in the dark.
[SNIP … I specifically asked people not to shoot in the dark. -w]
Anyway i hope that helped.
[It didnt. -w]

wayne Job
January 14, 2012 2:47 am

I have only one question, why all the argument over a hyperthetic planet with an unreal atmosphere using equations that only apply to something that does not exist.
We have a real planet and a real atmosphere and a sun, the vagarities of which are real, there is a mystery to solve and the answers do not lay in hyperthetics.

Steve (Paris)
January 14, 2012 2:48 am

Is there a virtual bar where Willis, TB and JS can line up at the bar and sink a few whiskies and settle their differences? Such’he said/you said/they said’ squables are a distraction from the far more important AGW debate. Its good to disagree and debate, its not good to throw your toys out of the pram.

gbaikie
January 14, 2012 2:51 am

[SNIP: read the instructions. Elevator speeches and disproofs only. -w.]

Patrick Davis
January 14, 2012 3:04 am

And discussing the chemical composition of methane, CH4, with the same person in my previous post claimed CH4 had “4 carbons”…and was “interesting”. I think CH4 having “4 carbons” would be interesting to many.

January 14, 2012 3:08 am

[snip . . off topic . . kbmod]

Pete in Cumbria UK
January 14, 2012 3:13 am

[SNIP: read the instructions. Elevator speeches and disproofs only. -w.]

Sandy
January 14, 2012 3:17 am

The air above the south pole in winter has as little CO2 and H2O as you’ll get anywhere, so makes the nearest thing to a GHG-free atmosphere we have.
If calibrated beacons at various wavelengths were put there the Dr. Spencer’s satellites could get a direct reading of absorption from a GHG-free atmosphere??

Filbert Cobb
January 14, 2012 3:18 am

Steve (Paris) says:
January 14, 2012 at 2:48 am
“Is there a virtual bar where Willis, TB and JS can line up at the bar and sink a few whiskies and settle their differences? Such’he said/you said/they said’ squables are a distraction from the far more important AGW debate. Its good to disagree and debate, its not good to throw your toys out of the pram.”
Well said, Sir. However, responsible bar staff would eject – at least two of them – for being excessively drunk wih their own importance

gbaikie
January 14, 2012 3:24 am

[SNIP: read the instructions. Elevator speeches and disproofs only. -w.]

tallbloke
January 14, 2012 3:31 am

[SNIP: read the instructions. Elevator speeches and disproofs only. -w.]

tallbloke
January 14, 2012 3:35 am

[SNIP: read the instructions. Elevator speeches and disproofs only. -w.]

Jordan
January 14, 2012 3:41 am

The thought experiment is interesting, but its usefulness may be limited by its own constraints.
The issue is defined in terms of surface temperature, and not temperature of the gas.
This ideal transparent gas can be warmed from the surface, and this means gas temperature above the surface could rise above the surface temperature. All we need is a day/night temperature profile at the surface, and convection to selectively warm the gas at higher levels towards the higher daytime surface temperature. This could eventually settle with the almost the whole atmosphere at the highest surface temperature with a very slim Temperature profile near the surface due to conduction.
Energy is conserved as the gas is a perfect insulator (constrained to not radiate at any frequency).
I’m not sure whether this thought experiment adds much to the debate. As I understand things, all matter will emit photons at certain frequencies so long as electrons are excited and then drop to ground state. The ideal non-emitting atmosphere sounds unphysical.
There may be more productive thought experiments if we were to focus more on the kinetic theory of temperature and pressure.
Cheers – I’m now off to the football (soccer) to observe other examples of kinetics.

gbaikie
January 14, 2012 3:42 am

“Willis Eschenbach says:
January 14, 2012 at 12:19 am
Genghis says:
January 13, 2012 at 11:14 pm
1. The non greenhouse gas atmosphere is a perfect conduction insulator to space, it can’t radiate its heat out.
True
2. The radiation transparent atmosphere is heated from the surface via conduction/convection until the atmospheric average temperature is at the S-B average. (Almost exactly the same way GHG’s heat the atmosphere, except that it is the planets surface molecules directly heating the atmosphere.)
I don’t think so. As soon as the dry adiabatic lapse rate temperature profile is established, circulation will stop and the atmospheric temperature will stabilize. ”
Perhaps, but on earth we have a night and and day. Whereas night may in some degree may stablize. Day time with sunlight will heat the ground and air will rise- destabilizing the adiabatic lapse rate.

Roger Longstaff
January 14, 2012 3:50 am

I think that the planet has to be considered as a whole – including its atmosphere – and that it must obey the laws of thermodynamics regardless of energy transport mechanisms. I have tried the following as a mind clearing exercise:
“Considering the entropy of the planet as a whole, including the conductive effects of the surface – gas boundary and with calculations performed within a boundary high above the top of the atmosphere, the lapse rate acts to maximise entropy, and minimise enthalpy, in a near – equilibrium thermodynamic system that constantly seeks equilibrium as a consequence of rotation. the laws of thermodynamics subsume the effects of all energy transport mechanisms – conduction, convection and radiation.”
I think that he implication is that only insolation, gravity, rotation and the mass of the atmosphere are responsible for “climate”, and that doubling, tripling or whatever the mass of CO2 will have a negligible effect on the total mass of the atmosphere, and hence would not lead to any measurable effects on the “climate”.
I am not sure if the “total mass of the atmosphere” should read (the bulk specific heat of the atmosphere”

David Reeve
January 14, 2012 3:54 am

Willis, you have just done something no man has done before. In all the years of lurking on climate sites this is the first time I’ve seen peoples objections to the radiation balance model reduced to pretty much a single theme. Normally we all but drown in the extravagence of response.
My challenge to you is to now is to get just the right words to convey the fact that, if a gas doesn’t significantly absorb in a given wavelength range, then it follows that it doesn’t significantly emit in the same range. A challenge for the elevator speech maestro.

AusieDan
January 14, 2012 3:56 am

Willis – I need to think more about your explanation, so I may (or may not) come back again some time later.
As a first comment, it seems to me that your explanation of why N&Z are wrong can equally be applied to greenhouse theory.
If all energy leaving the earth is radiated from the surface, then, by your own explanation, greenhouses gasses cannot warm the surface, as all additional warming would be radiated out into space to preserve equilibrium at the top of the atmosphere and surface temperatures would quickly return to pre greenhouse levels.
Something may be wrong with your facts or with you analysis, me-thinks.
Never-the-less, I almost enjoy your posts.

wayne
January 14, 2012 3:57 am

[SNIP: read the instructions. Elevator speeches and disproofs only. -w.]

AusieDan
January 14, 2012 3:58 am

Please forgive me.
What I meant to say was …… that I almost ALWAYS enjoy your posts.

richard verney
January 14, 2012 4:03 am

[SNIP: read the instructions. Elevator speeches and disproofs only. -w.]

AusieDan
January 14, 2012 4:08 am

Willis – just a quick, second comment.
I suggest that you think some more about how heat is transferred around the planet and up and down in the atmosphere, before being radiated out into space.
And a request for clarification – is heat radiated out into space directly from the surface or from the top of the atmosphere?
How do these two factors mesh with temperature on the surface, the near surface where it is measured, and the top of the atmosphere?
How does the lapse rate fit in and how is that caused?

January 14, 2012 4:09 am

Willis is correct…if the atmosphere cannot absorb or emit IR, then the surface must emit thermal radiation at the same average rate as it absorbs solar radiation, which then (through the S-B relationship) yields a much lower average energy-equilibrium temperature than that observed.
Conduction into the ground or to the atmosphere does not alter this fact. Conduction might change the timeline if the ground or atmosphere are not at the same temperature as the surface, but eventually the average temperature of the surface must correspond to the average rate of solar absorption, since IR emission by the surface is the only way for the planet to cool in the face of solar heating (in the absence of a GHE).
As I have discussed on my blog, the observed temperature lapse rate of the atmosphere only describes how temperature CHANGES with height *IF* the atmosphere is convecting. It says nothing about what the temperature will be, in an absolute sense, which is an energy budget issue. If the atmosphere cannot absorb/emit IR, it would become isothermal, and all convection would cease. The greenhouse effect is what destabilizes the troposphere (by cooling the upper layers) and enables convective overturning. Without the GHE, we would not have weather as we know it.

Bomber_the_Cat
January 14, 2012 4:12 am

I think many commentators are confused because they have heard or read the somewhat loose statement ‘All substances above absolute zero emit electromagnetic radiation’. This is certainly true for solids and liquids but it is NOT true for low density gases in our atmosphere. At atmospheric pressures gases absorb and radiate in specific spectral lines. Let me clarify, this is not just my opinion, this is an empirical result based on measurement (no models involved). Gases such as oxygen and nitrogen do absorb (and emit) at short wavelengths but not perceptibly in the long wave infrared where the greenhouse effect is important.
Kwik says: January 14, 2012 at 2:08 am “Are you saying I cannot heat Nitrogen?”.
No, Kwick, this means that you cannot heat nitrogen with long wave infrared radiation. You can heat it, as you describe, by putting it in a bottle and immersing the bottle in hot water – but not by LWIR radiation.
As I say, these things are confirmed by actual measurements. The following graph of the spectral distribution of downwelling (‘back’) radiation at the earth’s surface is very informative in this respect. Note that there is no typical blackbody distribution, instead radiation comes back from the atmosphere in specific spectral bands. There is no contribution from oxygen or nitrogen. Notice the dominant input from CO2 – the clear ‘fingerprint’ of CO2 in ‘back radiation’.
http://scienceofdoom.files.wordpress.com/2010/04/longwave-downward-radiation-surface-evans.png
Again – these are measurements, not models.
So, when Willis talks of a transparent GHG-free atmosphere he postulates an atmosphere that is not going to absorb radiation – and if it doesn’t absorb then it doesn’t emit – Kirchoff’s Law,
Is that clear?

Peter Czerna
January 14, 2012 4:15 am

Marshall and all the other adiabatic compressors
Adiabatic compression cannot warm an atmosphere in any lasting way.
When a gas is compressed adiabatically its temperature is raised as long as no heat escapes from the gas.
In the real world, near-adiabatic compression usually only happens when a gas is compressed very quickly, as, for example, in a bike pump.
If you hold the gas compressed, heat will gradually escape by conduction/convection/radiation and the temperature will fall until it reaches ambient temperature again. A compressed atmosphere will just leak heat to its surroundings and return to ambient temperature.
A gas is not hotter just because it is denser. If it were, liquified gases would not be possible.
How can gravity apply continuous work compressing the atmosphere?
When does it decompress? (If a gas decompresses it loses heat (because of the work it is doing in expanding) and its temperature falls).
In other words a one-off compression will not permanently raise the temperature of an atmosphere.

Paul Potter
January 14, 2012 4:15 am

Solids absorb and radiate in a black body manner because of there electron structure gives near continuous energy levels. Gas however do not interact with radiation fields unless the energy of the radiation corresponds to an discrete energy level of the molecule most the vibration modes are in the IR most of the atomic levels are in the visible or higher energies. Green house gases have vibrations modes in the IR and absorb and radiate.
Now come on this is basic physics.
The adiabatic lapse rate which this thread is talking about only gives the temperate change between the top and bottom of atmosphere. If the earth temp is -18C then the top will be a lot lower.
It tells us nothing on its own about the surface temperature.
Paulus

gbaikie
January 14, 2012 4:26 am

[SNIP: read the instructions. Elevator speeches and disproofs only. Stick to the topic. -w.]

AusieDan
January 14, 2012 4:28 am

Willis,
As a final comment tonight, may I suggest that you put aside all that you have learnt about greenhouse theory and just read the N&Z paper and see what it actually says.
[SNIP: If you understand it, give us the elevator speech. If not, why are you posting? w.]

Bryan
January 14, 2012 4:34 am

[SNIP: read the instructions. Elevator speeches and disproofs only. -w.]

Viv Evans
January 14, 2012 4:35 am

subscribe

gbaikie
January 14, 2012 4:51 am

[SNIP: read the instructions. Elevator speeches and disproofs only. -w.]

coturnix
January 14, 2012 4:52 am

Mouse fur, 0.94
—–
Wow, thats soo cool, mice are soo cute.
Wanna add few remarks:
[SNIP: read the instructions. Elevator speeches and disproofs only. And mice. Mice are cute. -w.]

coturnix
January 14, 2012 4:57 am

[SNIP: read the instructions. Elevator speeches and disproofs only. And mice. Mice are cute. -w.]

Andy West
January 14, 2012 5:06 am

[SNIP: read the instructions. Elevator speeches and disproofs only. And mice. Mice are cute. -w.]

Bill Illis
January 14, 2012 5:10 am

What is the physics explanation for why temperatures of a gas/star/planet increase as it is gravitationally compressed?
Why does matter do this?
I think the answer to this question will point to the answer about an atmosphere in gravitational equilibrium.
Otherwise, in a non-GHG atmosphere, the molecules next to the surface will be colliding with the surface 8 billion times per second.
Energy will be transferred from the surface to the non-GHG molecules in translational energy. These molecules then appararently NEVER lose their energy without colliding with another non-GHG molecule or the surface again. I don’t see how the atmosphere does not continually increase in temperature then. At some point, millions of years worth of solar energy will be locked up in the inert atmosphere.
[COMMENT: This is total nonsense. I mean, there is not a valid scientific thought in it. I leave it in to give a flavor of the nonsense that I am snipping. Give me your elevator speech, disprove my proof, or stay schtumm. What in that is not clear? -w.]

William
January 14, 2012 5:23 am

Let’s take a step back.
[SNIP: No, let’s not. Read the instructions. Elevator speeches and disproofs only. -w.]

January 14, 2012 5:25 am

Like Willis, I found Nikolov & Zeller’s poster, well, inscrutable. And I confess to being appalled at how many arguments on this thread are so clearly based on violating conservation of energy.
But anyone who has even a nodding acquaintance with tallbloke’s evenhandedness on the one hand and Joel Shore’s often ill-mannered thread monopolizing on the other should be able to appreciate that, not blessed with the moderator manpower that Wattsupwiththat enjoys, tallbloke’s choice to segregate Joel Shore’s output in a separate thread was a reasonable solution to the problem, brought on by Joel Shore, of how to avoid intolerable disruption but preserve open discourse.
I personally have benefited greatly from tallbloke’s blog, and I commend it to everyone’s attention.

January 14, 2012 5:27 am

[NOT SO. Repeat after me, “GHG-free atmosphere”. The surface is the only thing that can radiate. w.]
You keep saying that, but it’s BS.
[SNIP: No, it’s not bs, it’s called science. -w]

January 14, 2012 5:53 am

[SNIP: read the instructions. Elevator speeches and disproofs only. -w.]

markus
January 14, 2012 6:02 am

[SNIP: read the instructions. Elevator speeches and disproofs only. -w.]

January 14, 2012 6:02 am

[snip – you seem to forget the comment where you told Willis and I to go F*** ourselves about a month ago. As a result of that, and continued threadjacking and references to your website to try to draw traffic, you’ve been banned, do the words get out and stay out have any meaning to you? Apparently not. – Anthony]

simpleseekeraftertruth
January 14, 2012 6:10 am

OK Willis, I’ll bite.
Daytime;
Consider a photon leaving the sun in the IR absorption band of CO2.
[SNIP- I’m talking about a GHG-free atmosphere, why are you discussing CO2? -.w]

NoIdea
January 14, 2012 6:14 am

Willis, my best shot at an elevator speech version of Huffman’s analysis. I am not saying that I believe him to be right, or that I even properly understand his claims, I merely observe that to me, a layperson, it makes sense and is simple. My original question was to seek a comparison between N&Z and Huffman’s ideas.
—————————————————————————————————-
All energy in the system is received from the sun.
The ‘US Standard Atmosphere’ defines the pressure/temperature gradient for the Earth atmosphere. For example, at sea level, the pressure is determined to be 1013 millibars and temperature 15C, while at 20000 metres the pressure is determined to be 55.29 Mb and the temperature -56.5C.
The atmosphere is largely warmed by direct solar infrared irradiation (the surface can and does warm a part of the atmosphere – but only transiently and locally).
‘Visible’ light passes through the atmosphere without warming it but is also reflected back by clouds, ice etc. The portion of the solar irradiance which warms the Earth atmosphere is the same portion as heats the Venusian atmosphere – the differing albedos play no part in that absorption.
The Venusian atmosphere at the same pressure levels as defined in the US Standard Atmosphere has a temperature that is 1.176 times that of earth (for example, at 1000Mb Earth has a temp of 288K and Venus 1.176 times that or 338K, while at 600Mb the figures are 260.8K and 302.1K respectively).
The 1.176:1.00 ratio is derived from the difference in distance from the sun – Venus receives 1.91 times the power from the sun when compared to Earth due to being closer, and applying SB to obtain the 4th root of the power gives 1.176.
Atmospheric pressure does not cause heating rather it enables the atmosphere to retain more heat energy per volume at higher pressures. ‘Greenhouse gasses’ via radiation facilitate increasingly rapid distribution of heat throughout the atmosphere rather than heating it per se (ie local temperature variations are more quickly dissipated by heat transfer, both vertically and around the planet).
The greenhouse effect does not warm the atmosphere, the sun does directly.

wayne
January 14, 2012 6:30 am

Bill Illis says:
January 14, 2012 at 5:10 am
[SNIP: PLease stick to either an elevator speech, or a formal showing that my proof above is incorrect. Thanks, -w.]

PhilC
January 14, 2012 6:30 am

Stefan-Bolzmann radiation is called black body radiation because it applies to bodies. S-B radiation may apply to the atmosphere, but at orders of magnitude less than the surface. Willis is right to ignore any atmospheric S-B effect.
“Greenhouse” gases – CO2, H2O, NH4, etc, are such because their asymmetric modes of molecular vibration interact with long-wave radiation, at quantised energy levels. O2, N2 don’t have asymmetric modes of vibration. They can’t absorb long wave radiation ON ANYTHING LIKE THE SAME ORDER OF MAGNITUDE, and can therefore be ignored as “greenhouse” gases. It’s basic physical chemistry.
[Thanks, Phil. Fighting this level of ignorance is like wading through molasses. I appreciate the assistance. -w.]

tim in vermont
January 14, 2012 6:33 am

I doubt that these novel theories are correct since they sound to me like perpetual motion machines, but there are two sources of energy that would have to be ruled out or at least quantified, the rotation of the planet and tidal effects, since these could perturb the atmosphere and keep the machine from running down. Instead, the orbit of the moon, and the speed of the earth’s rotation runs down.
That being said, it sounds like a theory people want to believe.

Steve Fitzpatrick
January 14, 2012 6:40 am

Willis,
Of course you are correct. Not sure how many you will help to understand this simple concept, but I applaude your efforts. I become too frustrated when I try to do this sort of thing.
Too bad about Tallbloke; he is supporting nonsense and behaving like Tamino…. a bad combination.

Rob B
January 14, 2012 6:42 am

Willis, what is the temperature of the atmosphere as due to difficulties in losing energy it doesn’t seem well constrained?

Bill Illis
January 14, 2012 6:45 am

wayne says:
January 14, 2012 at 6:30 am
Bill Illis says:
January 14, 2012 at 5:10 am
[SNIP: PLease stick to either an elevator speech, or a formal showing that my proof above is incorrect. Thanks, -w.]

Richard M
January 14, 2012 6:45 am

First of all I think this deserves a historical perspective. The reason the GHG-less atmosphere became a discussion point is because it highlights the claims of N&Z. If gravity/conduction/convection are to replace the GHE then they should do it when no GHGs are present.
Willis is absolutely correct that this simply can’t work. I initially tried to find some way to make N&Z work myself. I failed and I admit it.
However, there is something very important to take out of the N&Z paper. If one follows it to its obvious conclusion one finds that, yes, GHGs are required, but, it only takes a modest amount of GHGs to provide the MAXIMUM GHE. That’s why N&Z found so many planets temperature can be determined by a simple formula. They all have reached the maximum GHE. Hence, the addition of more GHGs will have no impact on the temperature.
One possible reason for this has to do with the turbulence of gases. The turbulence mixes the gases very well and drives the heavier gases like CO2 to greater altitudes. Since the GHE is essentially determined by the height of the effective radiating temperature, that height is almost immediately quite high. This also means that additional CO2 has almost no effect. The basic reason for this has to do with gravity placing bounds on the atmosphere.
Think of a basketball. I can fill it with a little air and the structure does not change much. Put in more air and it gets a little more firm but still stays pretty much the same. I have to put in a lot of air to get the ball to expand to any degree. Gravity puts the same general limits on an atmosphere. Clearly, not as much as my analogy but along the principle.
As a result the GHE becomes a function of pressure (gravity and mass) just as N&Z stated. However, some GHGs are required.
[Richard, thanks … but if you could boil that handwaving down to an elevator speech, I might be able to understand what you are saying. Leave out the basketballs, and explain the science of what you claim N&Z say is happening. -w.]

January 14, 2012 6:49 am

Now, I’m happy for folks to comment on this proof, or to give us their elevator speech about the Jelbring or the N&Z hypothesis
Wills. I can see a fairly easy way to prove or disprove the thesis. First a thought experiment and then a real one.
You and most folks only talk in terms of radiative theory when talking about temperature but that is not the only way that energy is exchanged.
Lets say that the sun is shining and the atmosphere is 100% transparent to infrared energy. We have to think about what is actually happening when the sun shines on a surface. Speaking in simple terms a photon in the visible spectrum (say 550 nanometers) has energy x. That photon strikes a surface that is capable of absorbing that energy x. This raises the temperature of a single molecule by y. Since this is happening en masse, you would do a summation of x and y and eventually you get the rise in temperature of the surface as described in the S-B equations.
There is a lag time between absorption and emission of energy, depending on many factors but between the absorption time T and the emission time T1, there is time for elastic collisions between the molecules of the surface, and the molecules of the non IR emitting atmosphere above it. These elastic collisions increase the energy of the molecules, and thus the temperature. Since the molecules cannot radiate in the infrared anymore than they can absorb energy in the infrared, the temperature of the atmosphere rises and via convection, the energy absorbed by the molecules is spread through the same elastic collisions between molecules. Thus we now have a mechanism whereby the temperature of the atmosphere rises without any resort to radiative theory at all.
This is very easy to test. Take a cylinder of air without any GHG’s in it. The cylinder needs to be to be at say 25c. At the bottom of the cylinder you have a radiative surface that you raise in temperature to say 75c, which is close to what the temperature of the ground would be with the sun shining directly down on it. Use multiple temperature probes at 50 cm intervals from the bottom of the cylinder to the top.
If this hypothesis is true you will se a temperature gradient between the first probe at 50cm and subsequent ones. As convection takes hold due to the gradient, eventually the temperature of the gas will rise to the temperature of the emitting surface, and this is without any resort to GHG’s.
The reverse would happen at night with the atmosphere transferring energy to the ground and with nothing more than mechanical energy equations I can describe an atmosphere above the S-B average temperature.
So the elevator pitch is that we can describe a atmosphere with no GHG’s that is above the S-B temperature by resorting to nothing more than equations of mechanical energy transfer.
What is wrong with this picture?
[Thanks, Dennis. If as you claim the atmosphere is above the S-B temperature and the surface is still at S-B temperature, heat will be constantly flowing 24/7 from the warmer atmosphere to the cooler surface … and that violates conservation of energy. -w.]

Leonard Weinstein
January 14, 2012 6:49 am

Atmospheric Greenhouse Effect
Both sides of the argument generally stated miss the basic point of an atmospheric greenhouse gas causing a higher ground temperature. In order to understand what is occurring, you first have to …
[SNIP: Read the instructions, stick to an elevator speech for Nikolov or Jelbring, or a falsification of my proof. Thanks, -w.]

January 14, 2012 6:53 am

The mechanics of W. Eschenbach’s thinking is interesting to observe. Temperature is a property of matter. Willis can’t grasp how gravity, which directly controls the gradient density of an atmosphere which is constantly stirred, agitated and wafted, can influence our surface temperature. Wow. Does gravity influence density? Is heating more efficient when you stir a dense or thin fluid?
[Again, I’ll leave this in, to give a flavor of the vague nonsense folks are posting. Please read the instructions, stick to an elevator speech for Nikolov or Jelbring, or a falsification of my proof. Thanks, -w.]

ShrNfr
January 14, 2012 6:54 am

[SNIP: Read the instructions, stick to an elevator speech for Nikolov or Jelbring, or a falsification of my proof. Thanks, -w.]

Michael Reed
January 14, 2012 7:01 am

This is my “elevator speech” version of the discussion so far.
Postulate. Gibberish-gibberish-gibberish.
You’re right. Gibberish-gibberish-gibberish.
You’re wrong. Gibberish-gibberish-gibberish.
You’re all wrong and the original postulate is beeswax. This is why. Gibberish-gibberish-gibberish.
What’s a liberal arts education to make of all this? Could someone please tell me if humans are causing the planet to burn up, or what?

Richard M
January 14, 2012 7:04 am

[SNIP: Read the instructions, stick to an elevator speech for Nikolov or Jelbring, or a falsification of my proof. Thanks, -w.]

Paul Bahlin
January 14, 2012 7:05 am

[SNIP: Read the instructions, stick to an elevator speech for Nikolov or Jelbring, or a falsification of my proof. Thanks, -w.]

beng
January 14, 2012 7:07 am

I’m convinced you’re right, Willis. The simple energy conservation is enough. Roy Spencer & many others agree.
Willis & others should continue to be patient. Everyone can’t be convinced. I feel fortunate for a good thermo education & work experience to follow most of this — at least in general. Coal-fired boiler design (especially in radiant boilers) has to take into account radiation effects of H2O & CO2 in the flue gas.

ferd berple
January 14, 2012 7:07 am

Why not test the data?
There is a recent statistical method that can be used to separate cause and effect from multiple interacting variables. Originally developed for economics, this technique looks like a possible way forward to resolve the cause and effect climate question.
http://en.wikipedia.org/wiki/Vector_autoregression
Vector autoregression (VAR) is a statistical model used to capture the linear interdependencies among multiple time series. VAR models generalize the univariate autoregression (AR) models. All the variables in a VAR are treated symmetrically; each variable has an equation explaining its evolution based on its own lags and the lags of all the other variables in the model. VAR modeling does not require expert knowledge, which previously had been used in structural models with simultaneous equations.
Sims advocated VAR models as providing a theory-free method to estimate economic relationships, thus being an alternative to the “incredible identification restrictions” in structural models.[1]. Sims was awarded the 2011 Nobel Prize in Economics for his work in applying VAR models to macroeconomic analysis.[2]
[Interesting, so I’ll leave it. But in general, read the instructions, stick to an elevator speech for Nikolov or Jelbring, or a falsification of my proof. Thanks, -w.]

January 14, 2012 7:08 am

I am enjoying the thought process here. Some aren’t paying attention to Willis’ constraints:
1. The perfectly smooth, perfectly round, single substance planet is receiving equal amounts of heat/energy uniformly from all directions.
2. The added surrounding atmosphere is perfectly transparent to the heat/energy sources.
In this scenario, it would logically seem that since the atmosphere by definition can have no effect, it is having no effect; the denseness of the atmosphere caused by gravity will not matter.
However, now replace that atmosphere with one of 100% of any other gas, also insuring that the parameters will not be such that the gas will cool or warm enough to change state. Now, the warmer surface of the planet will heat the portion of that atmosphere that touches it, and heat “rises” (moves away from the center of gravity) toward the upper levels of the atmosphere while the cooler gas replaces it and is being heated and it “rises”, etc. until it reaches equilibrium.
If I’ve got that correct, than here are my questions: If there was only a “ball” of that gas in that same theoretical position, what would the temperature of center of the gas be? Would it not be lower than the surface of the theoretical planet? If so, then with all the conduction and convection going on in the planet/gas configuration, wouldn’t the gas ultimately be a coolant for the planet? Wouldn’t gravity be the driving force causing the heat to be moved away from the surface of the planet by pulling the cooler, denser gas toward it?
In this regard, I have a serious doubt regarding “down welling” in this scenario, too. Even if the chosen 100% gas is CO2, wouldn’t warmer molecules always “rise” or move away from the surface? As has been pointed out many times, the radiation is “out welling” – moving out in all directions. Essentially, if all the molecules in a given area are “out welling” at essentially the same rate, wouldn’t they all move somewhat together, taking their heat/energy with them, “up” or away from the center of gravity?

Rob B
January 14, 2012 7:11 am

Willis, can you also confirm that your model has an atmosphere thst can not radiate any energy at all? It seems there are a lot of posters that question that assumption, if so.
I thought anything raisef in energy tends to radiate it away eventually.

January 14, 2012 7:14 am

I suggest you can test both theories with data at the South Pole during the six months of night. There is no direct energy input from the sun. Water vapor is the least of any place on earth and the accumulation of snow is a measure of energy conservation. The air is thinner at that altitude. Energy is being delivered by the atmosphere and lost to space by radiation. These input and output rates can tell us if atmospheric CO2 has had any significant “greenhouse” effect.

1DandyTroll
January 14, 2012 7:15 am

Essentially, more mass equals more gravity which results in a hotheaded person, radiating fury at a crazier rate the denser he be. :p

Peter Sørensen
January 14, 2012 7:21 am

I dont know the theories of N&Z but the physics in your example are quite easy and clear. An atmosphere will increase the temperature of the planet even if the atmosphere does not emit infrared. The reason is that the atmosphere will get heated by the ground by convective heat transfer. Because of this heating the air at the ground will raise up and create air circulation causing a redistribution of heat from hot parts of the planet to cold parts of the planet. In other words heat will be transfered from the ground to the atmosphere in hot places and heat will be transfered from the atmosphere to the ground in cold places. The result is that acording to the T^4 law the temperature has to be higher to emit the same amount of energy. All of this ofcause assumes that the planet is rotating. The confusion stems from the fact that there are more than one way that heat can be transfered or moved.
– Convective heat transfer only happens between a liquid/solid gas/solid og gas/liquid interface
– conductive heat transfer only happens inside solids
– radiation
– Mass transfer for eksample when air moves from one place to another carrying stored heat
– Phase transitions like evaporation and solidification.
In this case Willis has forgotten about the convective heat transfer. and the mass transfer modes of moving heat. If convective heat transfer and mass transfer does not happen Willis is right.

January 14, 2012 7:28 am

In an atmosphere basically IR-inert, there would still be an adiabatic lapse rate due to pressure differential with altitude.
Gas heated by conduction from the surface would expand and rise (convection), trading thermal kinetic energy for potential (gravitational) energy.
This thermal energy convected away from the surface as potential energy is no longer available for reradiation, and the potential energy acquired will be expended when the now-cooled body of gas eventually descends to equalize the pressure differential caused by the rising of another warmed body of gas.
This is why the “evenly-heated average” image has to be discarded for a more realistic picture of constantly-changing areas of maximal warmth (i.e. planetary rotation; noon is always moving).
So whether weather is climate or not, weather clearly regulates climate, and both received solar energy and gravity contribute to both weather and climate. The question (here again) is the relative magnitude of the “greenhouse” contribution, not whether it exists or not.

January 14, 2012 7:28 am

jorgekafkazar says:
January 13, 2012 at 11:27 pm
“Is that short a tube accurate enough for the levels of energy and accuracy we’re dealing with?”
If I remember my light physics correctly, light energy is lost by destructive interference laterally during transmission through the atmosphere. I do not know what proportion is lost but the sky is blue because of the lateral dispersion. This dispersion to the atmosphere might the atmosphere a bit but it would decrease the energy reaching the surface, making it difficult to hold the planet up to a gas less black body radiation budget. It would not be fair.
Also, I know the discussion involves surface temperature but the numbers we have for Earth are NOT surface temps, they are from 5–6 feet above the surface; thus, atmosphere is being measured not the surface. Measuring the surface would be much too circumstantial regarding conditions, materials, etc. I am coming to an apples and oranges position, albeit reluctantly.

ferd berple
January 14, 2012 7:31 am

In science there is only one test that has any meaning. The ability to predict. The method that gives the best predictions is the best theory.
N&Z have found a means to predict the temperature of planets based on atmospheric density. Repeat their exercise with radiation and see if it gives a better result. Not just on one of two planets and moons, but all of them.
Because in the end, a theory that matches the data in one of two cases, but fails in other cases, is a failed theory. So far, the climate models which are based on the radiation theory have done a pretty poor job of forecasting temperatures since 2000. This suggests there is more to the story that GHG.

jjthom
January 14, 2012 7:33 am

Willis
I admire your honesty and your knowledge. (perhaps a tiny bit less anger wouldn’t go amiss!)
II totally agree with your reasoning.
The only difficulty I have is with “bb radiation”:
a) it is said all matter radiates broadband according to its temperature
b) it is then said gases do not radiate in this manner but only radiate at discrete wavelengths (GGs at LWIR and O2 N2 in UV This is the reason why the sun does not heat the atmosphere little UV gets through to gnd level)
c) most people seem to have trouble with the concept of GHG free air not being able to cool / heat other than by conduction ( a N2 packet – uncontained – of gas in a vacuum would not cool – any containment would heat and then BB radiate to cool the gas by conduction).
Perhaps they also need to consider a roughened black anodised aluminium block radiating efficiently – change the surface – polish it to 100% reflectance and most would agree that radiation will be reduced. i.e. change its state and you change its level of radiation.
Can the same be said of a gas.
Does liquid nitrogen have a bb radiation? I would say yes.
Does changing to gas then change the only radiation to a few lines in UV? I am not sure but side with the possibility that non-GHGs do not have bb-radiation
Can anyone who can show proof that gasses to NOT have “BB-radiation” please?
scienceofdoom has promised to cover this!

beng
January 14, 2012 7:36 am

My elevator speech to fellow engineers is this: GH gasses are qualitatively like applying insulation on a hot pipe — it impedes heat-loss to the environment (outer space in earth’s case). Once applied, the pipe’s insulation surface (tropopause) is cooler than the bare pipe is (earth’s surface). So it’s losing less heat when insulated & the heated pipe surface (earth’s surface) is warmer than w/o the “insulation”.
That’s where the proposed 33K GHG rise (insulation effect) comes from — 288K (earth surface) – 255K (tropopause surface). Not a bad approximation, but valid IMO for an atmosphere w/only non-condensible GH gasses.

pochas
January 14, 2012 7:37 am

There, Willis… Is everything clear now?

snert
January 14, 2012 7:39 am

I recently watched a Horizon program on the BBC that mentioned the ‘South Atlantic Anomaly’. A strange depression in the earths magnetic field that affected the Hubble Space telescope. If this is the case then according to the Svensmark hypothesis then surely there should be an increase in cloudiness in this area. Does anybody now whether this is the case ? An interesting little study for some enterprising research student.

Steve Richards
January 14, 2012 7:40 am

Planet without atmosphere warms to S-B temperature. Energy out = energy in. Everyone happy.
Wrap planet in an atmosphere (GHG free if you wish), atmosphere provides a insulation effect, ‘slowing’ the release of the out going energy, temperature of planet rises to a sufficient level that the original energy out figure is reached.
Planet plus atmosphere give same energy out = energy in but with the planets near surface warmer than before. Everyone happy.
In both cases the energy in and out are the same, no rules broken.
Specifying which heat transport mechanism is in use and by how much does not help the basic explanation.

Alan D McIntire
January 14, 2012 7:40 am

Hunter raised questions regarding snow as a blackbody. Between temperatures of
250 and 273 K, reasonable for snow, the emissivity is about 0.98. You’ll notice that snow around trees- picking up the infrared radiation from the tree, melts a lot faster than snow further from the trees.

ferd berple
January 14, 2012 7:42 am

Nature is not in any way influenced by the logic of argument. What may seem completely rational and logical has been shown time and time again to be wrong. If your argument describes nature, then it will have predictive power. If there exists a single case where this prediction fails, this is evidence that the argument does not describe nature, simply coincidence.

David
January 14, 2012 7:46 am

Willis, you responded to one post as valid. Your response did not mae sense to me. here is the assertion you rejected…
4. The thicker and denser the atmosphere, the higher the near surface atmospheric temperature will be.
Willis responded…
I don’t think so. The dry adiabatic lapse rate is g / Cp, where g is gravity and Cp is the specific heat of the atmosphere. The lapse rate does not vary with elevation, which means that Cp doesn’t vary with density, so I don’t see how a denser atmosphere would perforce be warmer.
—————————————
I am not certain Willis’s response makes sense to me. The lapse rate may be constant, but it is a constant VARIATION,which appears to be predicated on g and Cp. (IE, the greater the gravity, the higher the specific heat of the atmosphere) What does “specific heat emanate from? If specific heat, which is the heat capacity per unit mass of a material, then the more materials there are per volume, then the greater heat per volume. Therefore an atmosphere of more (denser) material, will have a higher specific heat content then a thinner atmosphere. The lapse rate will be the same in both atmosphers, just the starting point or temperature will be different.
What am I missing here?

Zeke
January 14, 2012 7:47 am

More blue box replies please… Thank you.

Dolphinhead
January 14, 2012 7:53 am

Willis
interesting post as always.
There seems to be some confusion about the roles of conduction and convection in the GHG free atmosphere. I think the point is that when the GHG free atmosphere is ‘added’ it will take some time for the system to reach equilbrium. During this period the atmosphere will be heated by conduction and I think it therefore follows that during this period the outgoing radiation will be LESS than the incoming radiation simply because some of that energy is being used to heat the atmosphere by conduction. Once the system reaches equilibrium I think you are quite correct -incoming radiation will equal outgoing radiation. The outgoing radiation from the surface will pass through the atmosphere as if it was not there.
Roy Spencer says that in this model convection has no role to play. I would not argue with Roy. But even if there was convection all it would do is assist in raising the atmosphere to equilibrium temperature.

Alan D McIntire
January 14, 2012 7:56 am

Willis Eschenbach listed a book as a reference. More convenient would be online sites.
Years ago, I was searching the web trying to get at least a highschool science understanding of the greenhouse effect, as opposed to heuristic handwaving,. The firs numerical site I found was the late John Daly’s site: Here’s a good link describing the greenhouse effect.
http://www.john-daly.com/miniwarm.htm
From there, it was guest papers like this,
http://www.john-daly.com/artifact.htm,
other sites like this,
http://www.geo.utexas.edu/courses/387H/Lectures/chap2.pdf
and finally links to other sites, leading to “Climate Audit”, and finally to the promised land-
“Watts Up With That?”

wsbriggs
January 14, 2012 7:59 am

Willis, excellent elevator pitch!
I agree completely, as the gas, which can’t radiate or absorb, will gain energy from contact with the surface, causing the surface temperature to drop. Since the gas can’t radiate the energy away, and the surface temperature has dropped, the next contact of a gas molecule with the surface will exchange energy in the other direction. Initially, the temperature will drop, but in the steady state, the temperature of the surface will remain at the non-atmospheric level. The mass at the steady state temperature has increased, but the temperature has not. The “missing radiation” from the surface was used to increase the gas temperature to that previously enjoyed by the surface. As soon as that has be accomplished, the system is once again in equilibrium.

January 14, 2012 8:10 am

Michael Reed,
I believe you have made a profound point. Engineers such as myself struggle for years with expert help to understand these devilishly complex technical questions/issues. Without having gone through that, you must rely on someone who has. Find one you trust and ask him or her.
Failing that, I will answer. No, we are not. The earth has been far warmer and far cooler than now, with far higher, but not much lower, concentrations of CO2. There is no correlation between the two according to data from many sources. The “average temperature of the earth” is nearly impossible to ascertain, and we have only been recording temperature data in more than just a few places since 1851, and not accurately enough to determine whether changes mean anything. For example, climate scientists debate changes on the order of 0.01 degrees C, using data from weather thermometers accurate to +-1 degree C, obviously ridiculous. Despite this, most of them agree that the average temperature of the Earth probably has risen 0.7 degrees C since 1851! Does that seem like a lot? Not to me. Has it changed more than that and faster than that many times? Yes it has.
There, glad I could clear that up for you!

Tim Folkerts
January 14, 2012 8:17 am

I haven’t had time to carefully read all the replies, but several comments and conclusions in the posts seem to forget one important fact.
Air cools as it rises.
Rising air is gaining gravitational potential energy and must be losing other energy. It does this by expanding and cooling. This is the physical reason behind the lapse rate. So even if we had a planet with a uniform temperature of 255 K at the surface, only the bottom layers of the atmosphere would warm to this temperature. Temperature in the the upper layers would drop off at approximately the lapse, so the “average temperature” of a transparent atmosphere will always be less than the surface temperature.
COROLLARY. The idea that “falling air will get heated by gravity as it falls” ignores the fact that gravity had equally cooled the air as it rose. If you added NEW air at the top (perhaps from a small comet hitting the earth), then this new air falling would have warming effect (at least temporarily).
**********************************
A second very minor point is that the energy balance must also include geothermal energy. On earth this is a minor effect (on the order of 1 W/m^2), so the energy emitted will be slightly larger than the energy absorbed. On the moon this would be an even smaller effect. On Jupiter, however, this is a significant effect, warming the “surface” well above the SB temperature. (http://en.wikipedia.org/wiki/Internal_heating#Gas_giants)
********************************
Finally, I want to reiterate that I agree with Willis and his overall conclusions.

A physicist
January 14, 2012 8:21 am

Alexander Feht says: Boltzmann killed himself, because the “consensus” among most of the respected, published and peer-reviewed “scientists” of his time was that Boltzmann was nuts.

I have to say, it’s pretty darn thrilling to see WUWT folks taking such an interest in thermodynamics and transport theory! There is perhaps no better subject than transport theory to unite mathematics, physics, engineering, biology, ecology, and economics. And conversely, an solid grasp of transport theory makes it far easier to appreciate these topics as aspects of a unified whole … which is perhaps why three of the greatest scientists of the 20th century were attracted to it:
(1) Albert Einstein for Brownian motion and black-body radiation,
(2) Paul Dirac for a (still largely classified) theory of U235 isotope transport in centrifuges, and
(3) John von Neumann for the computational theory of detonation waves and
     boundary layer transport (H-bombs, aircraft, and missiles)
Regarding Alexander Feht’s dubious assertions, it is true that Boltzmann killed himself, and it is true too that Boltzmann faced opposition to his ideas. But opposition is very commonly encountered in science, and perhaps a more likely explanation is Boltzmann’s lifelong history of depression, worsened by grief over the unexpected death of his eldest son (from appendicitis).
On the other hand, perhaps it is well to reflect upon the celebrated opening lines of David Goodstein’s physics textbook States of Matter:

“Ludwig Boltzmann, who spent much of his life studying statistical mechanics, died in 1906, by his own hand. Paul Ehrenfest, carrying on the work, died similarly in 1933. Now it is our turn to study statistical mechanics. Perhaps it will be wise to approach the subject cautiously.”

So if you should find yourself starting to feel a bit “down” reading this particular WUWT topic … then please take a break!   🙂

bacullen
January 14, 2012 8:33 am

Only got partway thru the replies so if this has been said before ……”never mind!”.
W’s argument only applies if the absorptivity and emissivity (albedo) of the planet’s surface are identical. He is correct if his ideal atmosphere, where convection, conduction, adiabatic lapse rate (thus gravity) play no part, has zero absorptivity and emissivity. Also not stated and assumed (I assume) is that any H2O is inert, like the atmosphere, and must have zero heat of vaporization/crystallization, and absorptivity = emissivity.
If these apply W’s simplistic argument is correct and gravity cannot play a part but in the real world Willis has previously shown here that excess heat is transferred to the upper atmosphere by condensing water vapor in thunderheads and radiated into space to keep the Earth’s temp withing a narrow range. Brilliant Willis!!!!!!
Now looking at N&Z’s and HJ’s argument that gravity (adiabatic lapse rate) plays a part; the elevator explanation is: Assume that the air is dry. All other properties are the same. It warms from contact with the warmer surface. A warm bubble breaks away and rises. As it rises it cools thru expansion, radiating some of that heat energy, as long wave IR, into space or warming some of trace GHGs. (These then radiate some of that miniscule energy, as LWIR, into space as they cool.) As the cold air bubble sinks back to the ground it cools the surrounding warmer air and surface absorbing the remaining energy NOT radiated into space. So N&Z and HJ are correct; gravity plays a part in transmission of some energy into space.
Now add water vapor. As Willis explained previously, the amount of energy transferred to space thru LWIR is dramatically increased because of the large amount of energy released when the water vapor condenses into clouds, driving the clouds to ever higher altitudes and ever more efficient transmission of the released energy into space. N&Z and HJ are still correct but the overall contribution by gravity now becomes a far smaller proportion of the total heat removed, possibly insignificantly small.
BC

Kelvin Vaughan
January 14, 2012 8:40 am

My roof stored negative energy last night. It was covered in frost. When the sun hit it in the morning, energy that would otherwise have heated the tiles was used up in melting the ice and evaporating the resulting wet surface.

Vince Causey
January 14, 2012 8:40 am

simpleseekeraftertruth says:
January 14, 2012 at 6:10 am ,
Your description was based on the idea of an IR photon from the sun being absorbed on the way in, and showing that 50% will be reflected back into space. The idea is that, in agregate, the co2 will have a cooling effect.
However, you have ignored the fact that most of the energy reaching the Earth from the sun is not in the IR, but in the visible spectrum. This bulk of energy goes straight through the CO2 and reaches the surface. It is then absorbed by the ground and the oceans and is emitted as IR. So, although the CO2 has intercepted the small amount of incoming IR and sent it back into space, there is a much larger flow of outgoing IR on which to act. This means the cooling effect is much less than the warming effect.

Kevin Kilty
January 14, 2012 8:41 am

michael hammer says:
January 13, 2012 at 9:45 pm

Not only integrating sphere, Michael, but also thermal IR sensors and cameras. One has to know the true emissivity in order to sense temperature accurately.

Nick Stokes says:
January 13, 2012 at 9:57 pm
“Now, I’m happy for folks to comment on this proof”
Seems to me to be sound.
The notion of emissivity for S-B as usually expressed is a surface emissivity. For a partly transparent medium like a gas, it’s a bit more complicated.

It is quite a lot more complicated, which is why in engineering we do two things to simplify. To avoid having to integrate emitted power throughout the entire volume of gas onto the point of interest, we resort to a “effect length” of path through the gas that depends on the geometry of the enclosure, which makes the problem quasi-one dimensional. For example, the effective length of path to one surface in an enclosure between infinite planes is 1.8 times the distance between the planes.
Second, we find the “emissivity” of the gas as a function of partial pressure of H2O and CO2 (the two common gases involved in IR) and temperature from a chart, or from a program that represents the chart. Problems in which there is a temperature gradient in the gas are more complex yet. Active gas in an enclosure is not a simple problem.
I agree quite generally with Willis on this post. One general comment I would make is about the common use of the term “radiation” as in radiation=sigma*T^4. What we actually calculate is emitted power; i.e. power (watts) emitted per unit area of surface into a hemisphere above that surface. Not keeping in mind the units of things leads to a lot of confusion.

January 14, 2012 8:44 am

Willis says he disagrees with this notion:
“somehow a combination of gravity and a transparent, GHG-free atmosphere can conspire to push the temperature of a planet well above the theoretical S-B temperature, to a condition similar to that of the Earth.”
Wayne says: “No Bill, if the air was ever warmer than the surface the air would transfer by conduction that energy from the air TO the surface, always warmer to cooler. It would not just accumulate. Eventually an equilibrium would be established.”
Peter Sorenson says: “Willis has forgotten about the convective heat transfer. and the mass transfer modes of moving heat.”
There is no planet that derives heat from a multitude of surrounding suns. [snip: My friend, it’s called a “thought experiment”. -w]

ferd berple
January 14, 2012 8:56 am

The argument that theory trumps observation is what has led so many astray. Here is a practical demonstration of a device that is theoretically impossible.

Kevin Kilty
January 14, 2012 8:56 am

David says:
January 14, 2012 at 7:46 am

am not certain Willis’s response makes sense to me. The lapse rate may be constant, but it is a constant VARIATION,which appears to be predicated on g and Cp. (IE, the greater the gravity, the higher the specific heat of the atmosphere) What does “specific heat emanate from? If specific heat, which is the heat capacity per unit mass of a material, then the more materials there are per volume, then the greater heat per volume. Therefore an atmosphere of more (denser) material, will have a higher specific heat content then a thinner atmosphere. The lapse rate will be the same in both atmosphers, just the starting point or temperature will be different.
What am I missing here?

First, let’s clear up two separate uses of the term “lapse rate”. Willis refers to the dry adiabatic lapse rate. This is a decrease in temperature per height gain in a rising air parcel. It is a process — adiabatic decompression.
Second, the term “lapse rate” in the atmosphere is the general decline in temperature of the troposphere with altitude. It is the end result of a whole series of processes that contribute to a final temperature distribution — one of these processes is adiabatic sinking or lifting of air, so is radiant emission.
When we use the term specific heat, we are not speaking of heat, but rather how much heat is needed to raise the temperature of one kilogram of material one degree celsius.
Cp is a measure based one-kilogram mass of material. If you increase density by jamming more kilograms into a volume, the specific heat remains the same because it is per unit mass. The heat capacity per unit volume increases, but not the specific heat. So thinner air has less heat capacity, but not less specific heat.

January 14, 2012 8:58 am

Oh, and let’s not forget the ocean.

Phil's Dad
January 14, 2012 9:01 am

I’m not sure about lapse rate warming being the answer as lapse rate cooling in the convection cycle would tend to cancel it out however… does gravity (which can do work) add energy to the equation such that radiation in + work done by gravity = radiation out? (Where the heck does gravity come from anyway?)

Jim D
January 14, 2012 9:06 am

Willis is right, but they might quibble (though they didn’t think to make this suggestion) that clouds radiate at colder temperatures, so this effect goes in the right direction to balance the energy. However, the cloud-top distribution that is observed does not help to close the energy balance.

Vince Causey
January 14, 2012 9:07 am

Conservation of energy dictates that the amount of energy radiated from the Earth’s system must be equal to the energy it receives. Under the GHG hypothesis, it is argued that this balance occurs at a place called “top of atmosphere”, while any level below this is allowed to be at a higher temperature.
The gravity hypothesis must similarly involve a TOA radiation that is equal to the incoming radiation, and would allow that the temperature at the Earth’s surface be higher than this. Willis premise is that there cannot be a TOA radiating less that the surface, because to do so, the atmosphere would need to absorb outgoing IR, and without GHG’s it cannot do so.
But, if the lower atmosphere is warmed by the gravity effect, there would indeed by a temperature gradient, with temperature declining as you go higher. At this point, it could be imagined that there is a TOA higher up radiating the same amount of energy that is received from the Sun.
But. . .the crucial point is, these molecules in the atmosphere cannot radiate. To do so would require molecules of N2 and O2 to emit photons. As far as I am aware, they do not. The only way energy can leave the Earth is by radiation, and only the surface can radiate. This does lead to the violation that Willis states – the ground is supposedly warmed by a gravity compressed atmosphere, yet the energy can only leave the planet by radiating from the surface, which implies more energy radiating than being received.
However. . . What if we imagine some GHG’s are added to the atmosphere (just like our good ol’ Earth, actually). What happens if we suppose the gravity hypothesis to be correct? The ground is warmed by compression, and this extra outgoing radiation is absorbed by the GHG’s in the atmosphere. There can now be a TOA effect, radiating exactly the same amount of energy into space as is received.
Does this sound as if GHG’s and gravity both play a role in regulating the Earth’s temperature? Could be. The moral is – never thow out the baby with the bath water, Willis.

evilincandescentbulb
January 14, 2012 9:08 am

The Love of AGW Theory “elevator speech”
Free enterprise capitalism and America for that matter are an evil that must be crushed through govcernment regulations on all factors of production and taxing productivity. Fear of AGW is what informs us because the ends justify the means.

lateposter
January 14, 2012 9:13 am

If I understand Jelbring, the elevator speech version of his paper is “the temperature at the surface has to be higher than at altitude due to the adiabatic lapse rate, even without greenhouse gasses”. He then (it seems to me) confuses that with the greenhouse effect.
Willis’ proof appears to me to be unassailable, but his statement that “they say that the combination of gravity plus an atmosphere without greenhouse gases (GHGs) is capable of doing what the greenhouse effect does” is only true for Jelbring because Jelbring re-defines the greenhouse effect as the adiabatic lapse rate.
Leonard Weinstein’s post above has the best explanation of the difference.

ferd berple
January 14, 2012 9:20 am

Let me see if I have this correct. An N2 O2 atmosphere without GHG does not cool by radiation.
Thus, without GHG, the earth’s surface would need to be warmer than it is now, to radiate the equivalent amount of energy to space as is currently radiated by the surface and atmosphere together.
Thus, the only effect that GHG can have on the surface is to cool the surface.
(atmosphere + surface) ^4 is greater than (surface) ^ 4
(atmosphere + surface) ^4 is equal to (hotter surface) ^ 4
Since we know that radiation in must equal radiation out, if N2 and O2 do not radiate, then the only conclusion that is possible is that GHG cools the surface.

Genghis
January 14, 2012 9:21 am

I think some seem to be missing the obvious part here. If you take a volume of gas at a given temperature and compress it, the same gas in a smaller volume now has a higher temperature, due to the compression and resulting density.
Let’s take a long thin tube of air in thermal equilibrium (100km long lets say), and extend if from the surface of the earth outward.
Gravity is going to compress the entire column of air down to about 17km. That compression heats the air and establishes the lapse rate, greatest density at the bottom and less density at the top. The part of the column of air that is most compressed (heated) is closest to the earth with the part at the top of the column not compressed much at all. The air will be denser and warmer closest to the surface (higher compression) gradually getting less dense and cooler as the altitude (less compression) increases. “Even though there is a non-uniform temperature distribution, the column of gas will still be in thermal equilibrium because the non-uniformity simply arises due to the decreasing density of the atmosphere with altitude.”
Now for the fun part. If this column of air is in a perfect insulator, it will be completely stable, warmer at the bottom transitioning to colder air at the top.
Under Willis’s steady state though, he is essentially placing a heating/cooling element at the bottom of the column of gas. This will act much like a pot of boiling water with convection cells rising and falling in response to the comparative temperature differences at the surface boundary layer. While the equilibrium temperature of the atmosphere won’t change, the atmosphere itself will be in constant dynamic change striving to reach equilibrium.
Willis’s heating/cooling surface will remain in equilibrium, emitting exactly as much radiation as it receives.
This by the way is a much better description of how our atmosphere actually works than the greenhouse gas theory.

Kevin Kilty
January 14, 2012 9:24 am

ferd berple says:
January 14, 2012 at 8:56 am
The argument that theory trumps observation is what has led so many astray. Here is a practical demonstration of a device that is theoretically impossible.

Badly done experiments can prove almost anything–thankfully here one of the participants at least tries to explain why the experiment does not debunk Newton. This thing works for the same reason that thrust deflectors on a jet engine slow the jet airplane. Newton rules!

Bill Hunter
January 14, 2012 9:25 am

“You are doing nothing but guessing and speculating, often in impossible directions. Gravity can’t run an engine of any kind. If it could, you’d have perpetual motion.”
Willis, you need to give what I wrote more thought than that! Does convection work without gravity? No! Gravity is a condition necessary, not a condition sufficient.
I am not a scientist but have experience in practical passive solar design. One can maintain water in an insulated tank at a warmer than average temperature using passive solar principles and convection by placing the collectors below the storage.
The poor emissivity of the bulk of gases in the atmosphere acts as the insulation.
The irrationality I posited was a semantic irrationality. Claiming an average temperature at the collector (surface) when what is being measured is the water in the insulated tank (air 6ft above the surface). Thus the average temperature of the surface needs to be lower than claimed because they are instead measuring the stored temperature.
This is actually pretty simple when you recognize accepted rules of radiation that something is as good of a radiator as it is as an absorber. In building insulation one does this by placing shiny non-radiative surfaces in the insulation space. The foil is hotter, approaching the insulated space temperature, and radiates less.
Assuming the average temperature of the surface is 288K by measuring air 6 feet above a convecting surface provides a one-way path for heat into the atmosphere that becomes trapped there. There is no convection process to deliver it back to the surface as in the solar collector system placed below the storage.
I think the only way you could disprove this is by actually measuring the surface, the real surface, and still come up with 288K. As a check using the NASA budget of 30% of solar energy transferred via convection one can calculate that as 84% of the greenhouse effect. Throw in a factor for the earth’s surface not being 1.0 but maybe more like .9, a dash of UHI, a pinch of cloud IR reflectivity, and maybe a spoonful of error resulting from weather stations favoring low altitude locations and you may have the recipe.
Keep in mind that its the diurnal cycle and perhaps the curavture of the earth that makes this happen. If the globe received it average radiation uniformly the passive solar water system would not work either.

ferd berple
January 14, 2012 9:37 am

Mathematical proof that GHG cools the surface of planet earth
In an atmosphere with GHG
total energy incoming from sun = net energy emitted to space by GHG atmosphere + net energy emitted to space by surface(1)
In an atmosphere without GHG (non radiating),
total energy incoming from sun = net energy emitted to space by surface(2)
Therefore this can be rewritten as:
net energy emitted to space by atmosphere + net energy emitted to space by surface(1) = net energy emitted to space by surface(2)
Which then becomes:
net energy emitted to space by GHG atmosphere = net energy emitted to space by surface(2) – net energy emitted to space by surface(1)
Since we know that “net energy emitted to space by GHG atmosphere” > 0
We can rewrite this as
net energy emitted to space by surface(2) – net energy emitted to space by surface(1) > 0
Thus
net energy emitted to space by surface(2) > net energy emitted to space by surface(1)
Since we know that net energy radiated to space is a function of temperature, we can then say:
Temperature surface(2) > Temperature surface(1)
QED

Alan D McIntire
January 14, 2012 9:44 am

There’s a cart/horse problem not mentioned previously. The constitution of an atmosphere is affected by chemistry- With a warming of the planet, much of the carbon in Venus’s rocks went into the atmosphere- cool the planet by other means, and much of Venus’s CO2 would wind up back in the soil. In other words, one could argue that Venus has a dense atmosphere because it’s hot, rather than argue that it’s hot because it has a dense atmosphere.

Coldish
January 14, 2012 9:45 am

Thanks, Willis, for another interesting post.
Edim says:
January 14, 2012 at 1:01 am
“How would Earth’s surface temperature change if atmospheric pressure were doubled, that is, increased to 2 atm by adding more N2 (and nothing else)?”
Leonard Weinstein’s comment (above) seems to be relevant to Edim’s question. I don’t claim expertise in this field, I’m still learning, and would welcome correction, but as I currently understand it this is what might happen.
1. The mass of the atmosphere would be aproximately doubled.
2. The newly added N2 would be quickly mixed with the existing atmospheric components, included the radiative gases.
3. The mean height of emission of out-going long-wave radiation (OLR) would be increased. By how much I don’t know, but I don’t think it would be doubled.
4. If the lapse rate remained the same the average surface temperature would rise. By how much would depend on the magnitude and sign of the feedback factor.

Bill Hunter
January 14, 2012 9:49 am

An addendum to the above. Most people apply an electric pump to their passive solar designs (making it a little less than passive) because they want the collectors on the roof and the insulated storage in the basement. So then convection works against you so you have to provide a pump for the transport of warm water to the basement during the day and a valve to limit convection of the warm water in the basement to the roof at night. But the lack of emissivity of the storage still makes it work.

Gina Becker
January 14, 2012 9:57 am

As a chemical engineer, I support Willis’ conclusions completely. It’s nonsense that a GHG free atmosphere will affect the temperature. The field of climate science is full of such nonsense, and it doesn’t help the skeptics cause to unskeptically perpetuate their own.

PeterGeorge
January 14, 2012 9:59 am

One can’t have an atmosphere at 0K – absolute zero – because it would be a solid, not a gaseous atmosphere.
At any temperature above 0K the atmosphere must be radiating, and the energy it radiates must get replaced somehow, or it would cool and liquify, then solidfy.
In your model, where is the atmosphere getting its energy? What is its temperature? How much is it radiating?
Does this atmosphere have a lapse rate? Is it, or is it not warmer at the bottom of the pile than at the top? Why?
If we add more gas to this atmosphere will the difference in temp between the top and bottom of the atmospheric column increase, decrease, or remain unchanged?
In my limited experience with solving physics problems, one needs a complete, coherent model. Otherwise one gets silly, meaningless answers. I’ve never seen such a model in any discussion of the “greenhouse effect.” Why is that?

JimOfCP
January 14, 2012 10:03 am

So, Willis. Did you choose the 9-12 micron range because it hits the water and CO2 main bands? Some other reason?

ferd berple
January 14, 2012 10:06 am

Guest Post by Willis Eschenbach
TO CONCLUDE: I’m interested in people who can either show that my proof is wrong
There is no free lunch. If the GHG atmosphere is radiating, then there must be a reduction in radiation somewhere else.

Hans Jelbring
January 14, 2012 10:08 am

lateposter says: January 14, 2012 at 9:13 am
No you don´t understand the content of my E&E, 2003 paper correctly. Willis has been putting words in my mouth that I have never said and he refuses to adress what is written in my peer reviwed paper, since 8 years. There certainly are no “Elevator version” of my paper which is based on first principal physics. It means that what I have written is either true or false. There is nothing inbetween. After 8 years nobody has falsified it and Willis attempt here is subclass for a number of reasons.
See the reference given by Tallbloke here 3:35 PM “The Greenhouse Effect as a function of atmospheric mass” and get firsthand information. I certainly appreciate a number of comments in this thread where the authors can discriminate between an approximate model and to what degree its properties can be applied to any real planetary atmosphere. There is little or any use for Willis model in this respect but there is plenty of use with the model I have describe. The former cannot be verified in any planetary atmospheres. The one I describe can be verified by observational evidence in Venus, Titan, Jupiter, Saturn, Uranus, Neptun. Mars fails for a simple reason. Its atmosphere is not massiv enough. The earth´s atmosphere is in the middle between Mars and Venus and is especially tricky to handle. There are so many physical processes that are involved changing the static average tmperature rate from -9.8 K/km to about 6.5 K/km. Some poeple in this thread has notice the importance of atmospheric surface mass per unit area. This is the first and last comment I will do on this thread.
Thanks
Hans Jelbring

Joel Shore
January 14, 2012 10:08 am

Thanks, Willis, for the nice post.
Bill Illis says:

What is the physics explanation for why temperatures of a gas/star/planet increase as it is gravitationally compressed?

If an object is truly undergoing gravitational collapse then you can have some gravitational potential energy being converted to other forms of energy (in particular, thermal energy). Hence, there truly is another source of energy besides the energy being received from the sun.

Why does matter do this?
I think the answer to this question will point to the answer about an atmosphere in gravitational equilibrium.

Yes…Comparison to this case shows why for gravitational collapse, you can truly have more energy coming out than you are receiving from the sun. However, for gravitational equilibrium, all that you can have is energy being moved around and, at the end of the day, you have to have only as much energy being emitted back out into space as is being absorbed from the sun.
So, the Earth is not undergoing gravitational collapse and hence it must be emitting ~240 W/m^2 back into space. And, indeed, measurements from satellites confirm this is what it is doing. The fact that the surface is emitting 390 W/m^2 has nothing to do with an internal energy source but is simply because some of the emissions from the surface are absorbed by the atmosphere, i.e., there is a radiative greenhouse effect.
erl happ says:

But the cooler latitudes in fact emit more energy than is acquired in solar radiation. The warmer latitudes emit less. The denser atmosphere increases the residence time and the amount of energy stored in the system, hence the temperature.
Last sentence is the elevator statement

And, one easily shown to be nonsense.
First of all, what we are interested in is the global energy balance. Yes, radiative energy does not balance locally because there is considerable movement of energy around on the Earth, as well as storage and release of energy. That is a good reason why we look at global energy balance and not local balance.
Second of all, your notion that some sort of energy storage can be invoked is not reasonable. What you would have to argue is that there is such a huge store of energy that the Earth can continuously emit 150 W/m^2 more than it absorbs without cooling down. Sorry, but this ain’t going to happen. To a good approximation, any energy storage that is occurring (e.g., due to plants converting sunlight into chemical energy) is balanced by energy release that is occuring (e.g., due to plants decaying). Also, the amounts involved are pretty small. For example, despite the fact that we are going through fossil fuel reserves much faster than they were created, we are only releasing about 0.02 W/m^2 of thermal energy by burning them, which is 4 orders of magnitude less than the 150 W/m^2 deficit.

Alan D McIntire
January 14, 2012 10:09 am

Willis Eschenbach and others have assumed that without water vapor, the lapse rate for a planet would be constant,
It wouldn’t. Lapse rate also depends on temperature. As the temperature cools with height, the
lapse rate drops
let atmospheric density = r., pressure= p, temperature = T, gas constant for dry air = R,
lapse rate = L.
T0 = ground temperature, p0= ground pressure, height = H
Then
L = (gr/RT)(dT/dp)
With constant density,, at current surface pressure, the lapse rate would b
:L = g/R = 3.42 C per kilometer. Such an atmosphere would have a specific finite height.
With constant temperature, the pressure would be proportional to the log of the pressure
height 2 – height 1 would be 29.3* ground temperature *( lnp1/lnp2)
With constant lapse rate
p = p0 ( 1 – (LH/T0))^(g/RL)
Needless to say, in the real world atmospheric density is not constant, nor does it drop off
exponentially with height- which would imply a constant temperature atmosphere, nor
is the lapse rate constant, It’s a messy average of those 3 ideal cases.

adolfogiurfa
January 14, 2012 10:09 am

…because the atmosphere is transparent, this means that the planet is radiating to space more energy than it receives. This is an obvious violation of conservation of energy, so any theories proposing such a warming must be incorrect.
OMG!..The Earth has no lid!…We urgently need to release the millions of degrees under the ground down there, according to Al Gore!

Kevin Kilty
January 14, 2012 10:09 am

Phil’s Dad says:
January 14, 2012 at 9:01 am
I’m not sure about lapse rate warming being the answer as lapse rate cooling in the convection cycle would tend to cancel it out however… does gravity (which can do work) add energy to the equation such that radiation in + work done by gravity = radiation out? (Where the heck does gravity come from anyway?)

Let’s put this gravitational work in its proper perspective…
[SNIP: Let’s not. Lets stick to the topic of the thread. -w.]

Joel Shore
January 14, 2012 10:13 am

ferd berple says:

Mathematical proof that GHG cools the surface of planet earth

You are just embarrassing yourself. An atmosphere with GHGs doesn’t just emit radiation, it also absorbs radiation from the surface. And, in fact, it absorbs more from the surface than it emits back out into space. (It has to emit as much in total as it absorbs overall, but some of what it emits goes back to the surface, not out into space.)

There is no free lunch. If the GHG atmosphere is radiating, then there must be a reduction in radiation somewhere else.

And, what law of physics is this…and how does it pertain to anything being talked about?

ferd berple
January 14, 2012 10:18 am

Gina Becker says:
January 14, 2012 at 9:57 am
It’s nonsense that a GHG free atmosphere will affect the temperature.
Stand at sea level in an atmosphere free of GHG. Now stand at the top of a mountain in an atmosphere free of GHG. There will be an affect on temperature.
The problem comes in assuming that GHG warms the surface. In fact the opposite is true. GHG cools the surface, in the same way it cools that atmosphere, by radiating LWR to space that would otherwise remain trapped in a non-radiating atmosphere.
This effect is most noticeable in the upper atmosphere, where temperatures can reach 1500C in the absence of GHG cooling.

January 14, 2012 10:19 am

The contributor known as wayne previously summarised my position on this issue which has been in my mind for several years as a very important aspect of the climate debate.
Willis did not address wayne’s version so here it is in my words:
[SNIP: vague, wandering, unscientific, and off topic. -w.]

January 14, 2012 10:23 am

OK, I made a fool of myself with “Stefan-Boltzmann”.
My mistake.

Andy West
January 14, 2012 10:24 am

hmmm… my elevator attempt above no doubt falls apart if convection stops, as it eventually will if no photons at all can ever be emitted, as indeed roy spencer points out. Isothermic. But that calls into question the usefulness of the model, as some have noted above. If the original papers had used this model, then fair game, but Tallbloke seems to suggest this was not the case. maybe a more interesting model would be to use just nitrogen and oxygen at earth ratio and pressure. Awful lot of it, so still significant emission despite low emmissitivity. Just a thought…

Capo
January 14, 2012 10:39 am

@ Willis
Your proof is of course correct. Did you realize why Jelbring and N&Z-“theories” give quite nice values? The deeper reason is the lapse rate, the dry lapse rate given by g/c_p. There it is, the effect of gravity. The “theories” are equivalent to lapse rate calculation, but they miss the central point: Where to start with, what is the right height h?
The answer ist given by ghg-theory: it’s the height, where the ghg-concentration is optical thin enough, that IR-radiation is able to escape to space.
(Therefore I like the following explanation of GHG-theory more: Adding more GHGs will elevate the level, where radiation can escape to space. It’s colder there, so earth emits less energy than it receives. The atmosphere will warm, until it radiates in the elevated level the same energy as before.)
Another indirect proof:
Imagine a planet moving around a black hole or wandering through empty space without a star like the sun. According to Jelbring or N&Z this planet should have the same temperatur as the planet moving around the sun. Nonsense.

TimC
January 14, 2012 10:46 am

Willis said (to Crosspatch): “I will repeat it again. If there are no GHGs in the atmosphere, the atmosphere will not and cannot radiate energy. That’s the whole point. The only thing that can radiate is the surface. You keep claiming the atmosphere will radiate. It will not.”
I don’t profess to any real knowledge of thermodynamics but isn’t this then postulating (a) an atmosphere with mass, (b) capable of convection and conduction, and (c) with its own localised temperatures and gradients (separate from the surface temperature, although no doubt coupled to it, with lags).
Why can’t such an atmosphere radiate? Suppose, instead of surrounding a planet, it was just a cloud of the (hypothetical) transparent gas, in space. Would there be no way of physically detecting it’s presence, outside the gas envelope itself?
I’m smelling an impossible premise here, somewhere.

Kevin Kilty
January 14, 2012 10:48 am

ferd berple says:
January 14, 2012 at 10:06 am
Guest Post by Willis Eschenbach
TO CONCLUDE: I’m interested in people who can either show that my proof is wrong
There is no free lunch. If the GHG atmosphere is radiating, then there must be a reduction in radiation somewhere else.

There is no principle called “conservation of radiation”. There is conservation of energy and radiant energy is but a part contributor to that principle.

Jim D
January 14, 2012 10:48 am

The problem with ferd berple’s argument is that just because GHGs reduce the amount of surface radiation escaping to space, you can’t conclude that there is less surface radiation entering the atmosphere, and actually it turns out it has to be more because a GHG atmosphere has to be cooler than the surface due to the lapse rate (or gravity if you like).

Eric Barnes
January 14, 2012 10:53 am

Willis Eschenbach says:
January 14, 2012 at 12:32 am
So how do you not understand Huffman’s explanation?
Throw out unrealistic assumptions about blackbodies, greybodies, albedos, etc. and the complications surrounding them.
It’s simple recognition that the temperature profile of earth and venus are very close given similar pressures and considering their relative distances from the sun in spite of massive differences in concentrations of CO2. It’s natures results and natures experiment.
Can you explain why the temperature/pressure profiles are so close? Here’s the link.
http://theendofthemystery.blogspot.com/2010/11/venus-no-greenhouse-effect.html
Surely it’s a rather amazing coincidence if there is a greenhouse effect.

Stephen Greener
January 14, 2012 10:54 am

Convince me that the molten core of the earth has no effect on the temperature of the planet surface with or without an atmosphere or a solar source. [SNIP: Not on this thread, I won’t. -w]

Eric Barnes
January 14, 2012 10:59 am

NoIdea says:
January 14, 2012 at 6:14 am
Excellent explanation NoIdea. 🙂

Gary Pearse
January 14, 2012 11:00 am

– Willis, – is your GHG-less atmos transparent? or just approximately so for gubmint work.
– There is mass in this atmosphere in the form of gas atoms that photons do collide with (and are absorbed).
– Beers Law T= I/I sub o (Transmissivity = incident photons/transmitted photons through a slab of air) which equals e^(-sigma l N); sigma is absortion cross-section of gas atoms/molecules; l is thickness of air and N = number of atoms/molecules in the path.
-N increases as the air pressure increases – greater excitation in the lower atmos
-Energy absorbed (and eventually re-emitted) E =Planck cons*c (speed of light)/lambda wavelenght of light (not just IR).
-Imagine an atmos pressure at the bottom that is near liquifaction to exaggerate the picture.

ferd berple
January 14, 2012 11:01 am

Stephen Wilde says:
January 14, 2012 at 10:19 am
Thus Oxygen and Nitrogen are involved despite being relatively non radiative.
Oxygen and Nitrogen participate fully in adding to the process of conduction
in its competition with radiation.
Exactly. In the absence of GHG, the atmosphere would not be able to lose energy to space. Any energy lost to the atmosphere from the surface would eventually be returned to the surface.
In contrast, with a GHG atmosphere, energy lost from the surface to the atmosphere can then be radiated to space by the atmosphere and need not be returned to the surface.
Thus, without a GHG atmosphere, S-B tells us that the surface of the planet must warm to radiate this excess energy, that is no longer being radiated away by the atmosphere.

Tim Folkerts
January 14, 2012 11:11 am

ferd berple says:
>The argument that theory trumps observation is what has led so many
>astray. Here is a practical demonstration of a device that is theoretically impossible.
and
>Mathematical proof that GHG cools the surface of planet earth
Unfortunately, Fred, you don’t quite understand the theory involved in either case. The blow-your-own-sail is counter-intuitive to many people, but fully within the rules of newtonian mechanics. The greenhouse is also and counter-intuitive to many people, but it also follows the rules of physics.

Richard M
January 14, 2012 11:11 am

Eric Barnes says:
January 14, 2012 at 10:53 am
Willis Eschenbach says:
January 14, 2012 at 12:32 am
So how do you not understand Huffman’s explanation?
Throw out unrealistic assumptions about blackbodies, greybodies, albedos, etc. and the complications surrounding them.
It’s simple recognition that the temperature profile of earth and venus are very close given similar pressures and considering their relative distances from the sun in spite of massive differences in concentrations of CO2. It’s natures results and natures experiment.
Can you explain why the temperature/pressure profiles are so close? Here’s the link.
http://theendofthemystery.blogspot.com/2010/11/venus-no-greenhouse-effect.html
Surely it’s a rather amazing coincidence if there is a greenhouse effect.

Not a coincidence at all if there is a maximum greenhouse effect. All the planets would warm up until they reached that maximum. If that maximum just happened to be determined by the mass and the gravitation of those planets then we would see exactly what Huffman sees. However, for the surface to warm above the SB calculations there must be an effective radiation altitude above that surface.
We don’t need to discard the GHE. It’s real, it just has limits that haven’t been discovered because of simplifications made to climate models.

ferd berple
January 14, 2012 11:13 am

Willis Eschenbach says:
January 14, 2012 at 10:50 am
So your argument is that on a planet with a transparent GHG-free atmosphere, the surface is continually losing energy by conduction … riiiight, that’s the ticket
I think you have this backwards. In a non GHG (non radiating) atmosphere, any heat lost by the surface to conduction must be returned to the surface – for example at the poles. In contrast, in a GHG atmosphere, heat lost by the surface to conduction can then be radiated to space by the GHG atmosphere.
Thus, the GHG atmosphere MUST result in a lower average surface temperature as compared to a non GHG atmosphere, due to the energy lost via atmospheric GHG radiation.

PeterGeorge
January 14, 2012 11:37 am

I believe Willis’ proof is not correct.
In a real world with a no-GHG atmosphere, the atmosphere still absorbs heat from the surface through conduction and convection. And the near-surface temperature of the atmosphere is not the same as the “skin” temperature of the solid or liquid surface.
If one adds more atmosphere, there will be more conduction/convection. The “skin” temperature will decrease and the skin will radiate less. The near surface gas temperature, however, will rise and the radiation from the atmosphere as a whole will increase, maintaining a total balance.
In simple terms, the sand on the beach will not burn your feet as much, but the air you are immersed in will be warmer and will be radiating more.
When we talk about the “surface” temperature, what are we talking about, the “skin” or the air? We should at least be clear about that, because the “skin” can get very, very hot. With radiation increasing as T^4, hot beach sand radiates a hell of a lot more than the same beach sand, irradiated in the same way by the same sun, but cooled by a brisk wind.
In short, none of the models I’ve ever seen is either complete or coherent. Which explains the confusion.

GabrielHBay
January 14, 2012 11:38 am

Well, to stupid me it seems unbelievable how many here seem not to understand the difference between energy and temperature. Thanks to the few voices in the wilderness who clearly do understand. (One thing I do seem to remember from my university days too many decades ago) When I got to Willis’ talk about temperature issues somehow conflicting with conservation of energy (or something to that effect) I just skipped to the comments.. And why, pray tell, should it be possible to definitively explain the physics of it all in an ‘elevator speech’ of a few lines? Who was it who said that simplicity can also be the essence of untruth?
Gabriel van den Bergh

PhilC
January 14, 2012 11:39 am

berple GHGs can only radiate into space long wave / infra red photons intercepted from the surface and stored in internal energy states. They cannot emit additional photons that they haven’t captured emerging initially from the surface of the planet.
AIUI The effect of GHGs is that they STORE energy in the vibrational energy states corresponding to the infra red radiation energies. This energy may be later released as heat. The rate of spontaneous emission has a reverse relationship with frequency, so the energy is retained for a relatively long time (minutes, hours).

January 14, 2012 11:40 am


What? Everything with a temperature radiates. I don’t care WHAT a substance is, it will cool. It will cool until it reaches equilibrium with its background.
I don’t know why, but some folks seem to think that all things radiate at all temperatures. They don’t.

No one said it radiates at all temperatures. No one is even saying that all things radiate IR. What they”re saying is that everything radiates at SOME frequency. At least that’s my understanding… correct me if I’m wrong. U of Virginia seems to agree.
http://galileo.phys.virginia.edu/classes/252/black_body_radiation.html
Any body at any temperature above absolute zero will radiate to some extent, the intensity and frequency distribution of the radiation depending on the detailed structure of the body.
So simplifying the earth to only consider IR is a gross oversimplification. For example, the earth was also hit by UV rays. Some of the material in the earth will absorb those UV rays and increase the energy of the earth. Conduction within the earth will transfer some of that energy to other materials, which may have a different emissivity… say it emits in IR. So in that instance we have IR being generated from the surface that did not arrive as IR!
So, an atmosphere that aquires heat content from the surface of the earth (and it MUST, right? your atmosphere can’t be at absolute 0 or it’d be a solid) has the possibility of radiating that energy as microwaves, etc, etc. Reducing the heat content of the atmosphere and the surface.
To totally ignore all emissivity except IR seems a gross oversimplification.

ferd berple
January 14, 2012 11:41 am

Tim Folkerts says:
January 14, 2012 at 11:11 am
The greenhouse is also and counter-intuitive to many people, but it also follows the rules of physics.
No, you have it backwards.
Nature does not follow the rules of physics. Physics describes the actions of Nature, and the description is imperfect, due to the existence of the “unknown” which for all practical purposes is infinite in size.
I have shown a very simple mathematical demonstration of why the GHG theory does not describe nature. If you wish to refute this, show where the mathematics in in error. You cannot do this via rhetoric, you must do so via mathematics.
The gravitation theory of surface temperatures has demonstrated a predictive power across multiple examples. Something the radiative theory is yet to demonstrate.

January 14, 2012 11:43 am

“No one said it radiates at all temperatures. ”
Actually, all temperatures but absolute 0. Duh. 🙂

lateposter
January 14, 2012 11:47 am

I thank Dr. Jelbring for his response, and I apologize if I misrepresented his paper. But my comments were based on reading it, not second-hand from Willis. I thought it was easy to understand, and I’m not claiming to have found any statement in it that wasn’t true, but the definition of greenhouse effect implied by the paper seems to be different from what other people mean by it.
I understand the greenhouse effect to be something that raises the effective radiation height, which together with a relatively constant lapse rate, results in a higher surface temperature, as explained by Leonard Weinstein.
Dr. Jelbring defines the greenhouse effect quite clearly as “The average global surface temperature minus the average infrared black body radiation temperature, as observed from space”. But this definition cannot be applied to his model, because it “neither receives solar radiation nor emits infrared radiation into space” and the GE “is independent of the absolute average temperature of the model atmosphere”. In it’s place, he identifies the GE with the lapse-rate-induced temperature difference (“the temperature difference (GE) between the surfaces”), dropping the all-important black body radiation temperature reference point.

Eric Barnes
January 14, 2012 11:48 am

Richard M says:
January 14, 2012 at 11:11 am
Not a coincidence at all if there is a maximum greenhouse effect.
Convenient explanation.
So we’ve maxed out at 390ppm CO2 and our current level of water vapor/methane?
Watch how far you back up. The cliff of never ending rationalization is near.

January 14, 2012 11:58 am

It just occurred to me, when Hansen said..
http://wattsupwiththat.com/2011/12/20/hansens-arrested-development/#more-53430
The precision achieved by the most advanced generation of radiation budget satellites is indicated by the planetary energy imbalance measured by the ongoing CERES (Clouds and the Earth’s Radiant Energy System) instrument (Loeb et al., 2009),, which finds a measured 5-year-mean imbalance of 6.5 W/m2 (Loeb et al., 2009). Because this result is implausible, instrumentation calibration factors were introduced to reduce the imbalance to the imbalance suggested by climate models, 0.85 W/m2 (Loeb et al., 2009).

Was he talking only about the IR portion of the spectrum?
http://en.wikipedia.org/wiki/Clouds_and_the_Earth's_Radiant_Energy_System

Each CERES instrument is a radiometer which has three channels – a shortwave channel to measure reflected sunlight in 0.3 – 5 µm region, a channel to measure Earth-emitted thermal radiation in the 8-12 µm “window” region, and a total channel to measure entire spectrum of outgoing Earth’s radiation.

If so, conversion of other wavelengths into emitted IR might account for some of that.

Edim
January 14, 2012 11:58 am

“The problem comes in assuming that GHG warms the surface. In fact the opposite is true. GHG cools the surface, in the same way it cools that atmosphere, by radiating LWR to space that would otherwise remain trapped in a non-radiating atmosphere.”
The oposite is often true in life. But I digress, I agree with this. The question is, how strong is the effect. Probably not very strong.

GabrielHBay
January 14, 2012 12:00 pm

Willis:
Crosspatch and others keep claiming that oxygen and nitrogen are greenhouse gases, for example saying:
crosspatch says:
January 14, 2012 at 12:39 am
“1. The non greenhouse gas atmosphere is a perfect conduction insulator to space, it can’t radiate its heat out.”
What? Everything with a temperature radiates. I don’t care WHAT a substance is, it will cool. It will cool until it reaches equilibrium with its background.
I don’t know why, but some folks seem to think that all things radiate at all temperatures. They don’t.
——————————————————————————–
So is only IR relevant? Are we saying that a substance transparent to IR but having heated up through energy gain by conduction or by compression cannot cool down by any mechanism other than (reverse) conduction or expansion? No radiation? So it will retain its heat indefinitely if none of these options apply? Or are we saying it cannot gain energy via conduction? Which is it, or am I missing something? Crosspatch makes more sense to me… sorry.
Gabriel van den Bergh

Phil.
January 14, 2012 12:00 pm

kcrucible says:
January 14, 2012 at 5:27 am
[NOT SO. Repeat after me, “GHG-free atmosphere”. The surface is the only thing that can radiate. w.]
You keep saying that, but it’s BS. Being “transparent” to IR just means that it can’t absorb energy via RADIATION. That does not mean that it can’t absorb energy by CONDUCTION. And ANY mass that has a higher energy than the stable state will then emit energy.

No it won’t because being transparent means that it has an absorptivity of 0 and by Kirchoff’s Law an emissivity of 0. Fundamental misunderstanding of the physics of gases.

Tim Folkerts
January 14, 2012 12:01 pm

Fred says
>The argument that theory trumps observation is what has led
>so many astray. Here is a practical demonstration of a device
>that is theoretically impossible.
And
>Thus, the GHG atmosphere MUST result in a lower average surface
>temperature as compared to a non GHG atmosphere, due to the
>energy lost via atmospheric GHG radiation.
Unfortunately, Fred, you are wrong on both counts. The blow-your-own-sail is perfectly within the laws of newtonian mechanics. The greenhouse-effect-warms-the-earth is perfectly within the laws of thermodynamics and radiation.

Frumious Bandersnatch
January 14, 2012 12:02 pm

Interesting analysis Willis, but I admit a problem I have with it is that if a GHG atmosphere cannot radiate heat, than how can the surface of the blackbody do so, since it is also non GHG? Also, are conduction and convection limited to an atmosphere or do they also play a role in the surface (and below) of the planetoid? It seems to me these are salient points not addressed by your post.

G. Karst
January 14, 2012 12:02 pm

There are mind-sets here which are almost impossible to overcome. I have limited my own mindset to the following:
Gravity compression of atmos is continuous process, not a one time batch compression. Conduction then convection ensure this continuity.
All gases when energized by conduction, above ambient, will radiate energy. Convection moves all heated gases, to locales, where it is above ambient. GHG or non-GHG matters not a twit. As long as it is matter… it matters. Poor radiators require higher delta T to transmit the same E as efficient radiators. This higher delta T is found at altitude.
Thermal energy conducted from ground (surface) to atmos is intercepted future radiant heat. Conservation of energy is maintained, as surface IR rate decreases due to conducted/convected heat loss.
Pressure induced lapse rate does seem to set the limits or mixture of radiative/conduction/convection rates… according to my present mind-set. That is all…as you were. GK

Admin
January 14, 2012 12:09 pm

I’ve closed the other two threads on this subject since they were getting a bit ragged and Shore-worn, and directed everyone here to this thread.

GabrielHBay
January 14, 2012 12:14 pm

On the wisdom or otherwise of an “elevator speech”:
Ok, I refreshed my memory. From the foreword of Aldous Huxley’s Brave new world: This is what was lurking in my mind:
“The soul of wit may become the very body of untruth. However elegant and memorable, brevity can never, in the nature of things, do justice to all the facts of a complex situation. On such a theme one can be brief only by omission and simplification. Omission and sim­plification help us to understand — but help us, in many cases, to understand the wrong thing; for our compre­hension may be only of the abbreviator’s neatly formu­lated notions, not of the vast, ramifying reality from which these notions have been so arbitrarily abstracted.”
Gabriel van den Bergh

January 14, 2012 12:19 pm

Roy Spencer says: January 14, 2012 at 4:09 am
As I have discussed on my blog, the observed temperature lapse rate of the atmosphere only describes how temperature CHANGES with height *IF* the atmosphere is convecting. It says nothing about what the temperature will be, in an absolute sense, which is an energy budget issue. If the atmosphere cannot absorb/emit IR, it would become isothermal, and all convection would cease.
I hate to argue against authority, but an isothermal atmosphere is unstable and will not remain that way. It will convect until it reaches the adiabatic profile, only then will convection cease.
N&z predicts only how a temperature will vary from the starting grey-body temperature, not the initial temperature itself.

Bart
January 14, 2012 12:19 pm

FTA: “The Stefan-Boltzmann equation specifies how much radiation is emitted at a given temperature.”
Only for a blackbody in equilibrium. The corrected statement is

The Stefan-Boltzmann equation specifies the maximum amount of radiation that is emitted at a given temperature for a body in thermodynamic equilibrium.

There are many loopholes in that description which might allow non-GHG heating to be physically realizable. I discuss some of these here. Your “QED” is not, in fact, demonstrative.

Leonard Weinstein
January 14, 2012 12:22 pm

Willis,
I have stated the cause of the greenhouse effect in basic terms in a previous reply. The issue I want to discuss is your claim that if the amount of greenhouse gases was constant, but non greenhouse gases (O2 and N2) increased, there would be no change in ground level temperature. That is incorrect. Increasing the mass of the atmosphere not only increases surface pressure (linearly by the increase in mass), but also results in greater thickness (not a linear process). The mechanism for increase in greenhouse heating is simply that the gases would be mixed at all altitudes, so the average location of outgoing radiation would be increased some due to the greater thickness of the more massive atmosphere. The increase would be a nonlinear effect, and doubling mass of non-greenhouse gases would only increase temperature slightly for Earth, but it would definitely increase. It takes both greenhouse gases and the lapse effect to heat above a non-greenhouse level, but it is the effective average altitude that locks the temperature on the lapse rate curve. The effective lapse rate is not density dependent in the Troposphere until near the Tropopause, and for Earth, not temperature dependent.

Bill Hunter
January 14, 2012 12:23 pm

Willis: “So your argument is that on a planet with a transparent GHG-free atmosphere, the surface is continually losing energy by conduction … riiiight, that’s the ticket”
According to NASA, Trenberth, and every global energy budget I have seen thats the case. Its somewhere in the range of a 100 watts global 24/7 average.
[SNIP: I’ll say it real slow. Traaaanspaareeent GHG-freee aaaatmooospheeereeee. NASA, Trenberth, and the energy budgets are about non-transparent atmospheres containing GHGs. The rest of your post is about that, and hence way off topic. -w.]

GabrielHBay
January 14, 2012 12:30 pm

@ C Karst:
Love your mind-set…
Gabriel van den Bergh

George E. Smith;
January 14, 2012 12:31 pm

“”””” Alexander Feht says:
January 14, 2012 at 12:55 am
Why everybody here imitates Mr. Eschenbach, using hyphen in “Stefan-Boltzmann temperature”, “Stefan-Boltzmann equation”, and “Stefan-Boltzmann constant” expressions, as if “Stefan” and “Boltzmann” were two people who developed this formula?
Stefan Boltzmann was one man, his last name was Boltzmann, his first name was Stefan, and the correct way to use his name is “Boltzmann’s equation” or “Stefan Boltzmann’s equation” (if you insist for some reason on repeating his first name all the time) but not “Stefan-Boltzmann equation.” The “S-B” abbreviation, in this context, is incorrect. “””””
Balderdash; and that’s the polite comment.
Boltzmann’s name was Ludwig.
Stefan was a totally different person.
All that either of them did, in this instance, was to successfully integrate from zero to infinity wavelength range, the Planck Radiation formula for a completely fictitious, non-existent ideal absorber called a Black Body, whose only required property is to totally absorb any and all electro-magnetic radiation that falls on it in any direction, and originating from ANY source or multiplicity of sources, anywhere, having ANY Temperature or Temperatures, without restriction.
This is not to denigrate ANY other scientific contributions either of them made; and Boltzmann’s were significant.

Gary Palmgren
January 14, 2012 12:31 pm

There are two many comments to study so I’ll just put in the elevator speech.
[SNIP: I specifically asked for elevator speeches about two theories, Jelbring’s and N&Z’s. I also asked you to leave your own pet theory out of the mix. In response, you give us the elevator speech on your pet theory. Fail. -w.]

kiwistonewall
January 14, 2012 12:32 pm

Wills is correct in that the Earth mainly cools by emission from its surface. Gravity cannot heat the atmosphere, just organize the molecules into a heat gradient.
Wills is wrong, in that all gases will absorb & emit at all frequencies. However, greenhouse gases absorb & emit at specific frequencies, and this absorption is many orders of magnitude greater than “black-body” emission from non greenhouse gases. Gases without dipoles (Oxygen, Nitrogen, Argon) can ONLY absorb & emit Earth frequency thermal radiation during collisions. This effect is vanishingly small at low temperature and pressure,
Our atmosphere is a grey body – in that there is a small amount of black-body type absorption and emission. This is from dust, aerosols, water droplets and (very minor) gas molecules in collision. This part of atmospheric radiation obeys the T^4 rule. This varies greatly due to cloud cover,
But Greenhouse gas absorption is at specific frequencies and depends on the proportion of molecules over a specific energy level. It does not obey the T^4 law. Depending on the thermal signature coming from the Earth (day/night) emission/radiation may increase or decrease.
It is too simplistic to say that if the atmosphere warms then more GHG radiation will be returned to the Earth’s surface. Firstly, thermodynamics will require the warm air to cool by expansion, Warming of the atmosphere by capture of outward thermal radiation is dwarfed by convection and latent heat effects, Secondly, since for all absorption and emission there is full equality (a good absorber is an exactly good emitter) then there is always more GHG emission heading to space.
Thus the contradiction I noted in an earlier post: If GH gases send MORE radiation back to the Earth’s surface, then they send even more radiation outward,
I can illustrate this in a simple energy balance equation:
S – Energy in from the Sun net of albedo and outer atmosphere reflection)
E – Earth Surface radiation without the GH Effect. (So S = E)
GB – Greenhouse gas ‘Back; radiation. Since the Earth is a Sphere, simple geometry requires slightly more GH gas emission to leave to space than returns to Earth. I use 2% extra as an approximation in this illustration.
In the Green house effect we have at the Earth’s Surface:
Energy emitted = E + 0.98 * GB (The Earth needs to emit the extra energy in non GH absorbing bandwidths)
In the atmosphere:
Outward: 1.02 * GB
Downward: 0.98 * GB
So We have S in and E + 0.98GB + 1.02GB outward,
OR: The Earth cools by twice any heating effect of greenhouse gases.??????
Surely there is a problem with my logic, but I can’t see it.

wayne
January 14, 2012 12:33 pm

Think I’ll just let the readers here pass judgment. It started with a comment from Michael.
[SNIP: No, I think I’ll just let the readers focus on the science and not worry about your complaints. -w.]

Gina Becker
January 14, 2012 12:35 pm

The point is that a greenhouse-gas-free planet’s SURFACE temperature is set in stone by its emmisivity, once it reaches equilibrium with the sun’s energy. The atmosphere also reaches an equilibrium. It’s surface temperature is independent of any qualities of the atmosphere that are not radiation absorbing. Adding more oxygen or nitrogen, making it thicker or thinner or non-existent, doesn’t affect the surface temperature. The surface temperature of the earth remains the same, soley based on the emissivity–unless radiation absorbing gases are added. If greenhouse gases are added, then the surface temperature is changed.
Again, as skeptics, we should not fight the nonsense of the alarmists with our unskeptical nonsense.

PaulR
January 14, 2012 12:37 pm

Willis Eschenbach wins the thread by default. No one has met the challenge.
“A simplified model of Earth will be considered. The model planet does not rotate. It
neither receives solar radiation nor emits infrared radiation into space.”
If that model assumes the mechanical work of introducing an atmosphere where there was none, and then does not permit the energy of that work (compression as described by Gas Laws) to radiate into space then of course the temperature will rise but it is a circular argument. As W. Eshenbach has also explained it promotes transient effects to equilibrium states by making radiation to restore equilibrium impossible by stipulation.

gnomish
January 14, 2012 12:38 pm

i suppose you can statistically raise the average temperature of an atmosphere-free planet by coating it with anything that redistributes heat better than conduction of the solid material because it carries energy to places it would never get due to not receiving any radiation.
but it can’t raise the temperature of the heat source; it can only cool that.

Willy
January 14, 2012 12:39 pm

Willis: Your proof is pretty tight but not yet perfectly so. What follows is a theoretical counter argument with some specific points that could be tested.
1. Your proof applies to the radiating surface of the planet. Conservation of energy limits that surface temperature to at or below the theoretical S-B limit . On earth, 70% of the time, we are discussing the first few microns of ocean (IR penetration limit).
2. The practical near surface temperature, outside that few micron range, can exceed that limit if a mechanism exists for maintaining a sharp positive heat gradient relative to the actual radiating surface.
3. Short wave solar radiation penetrates the radiating surface delivering energy to the liquid below. This can maintain a positive gradient relative to the radiating surface and hence the average temperature of the liquid below can exceed that of the radiating surface.
4. Evaporative heat transport, since it occurs within the radiating surface, can maintain a sharp temperature gradient between the top few microns of liquid and both the liquid below and atmosphere above.
5. The effectiveness of evaporative heat transport is proportional to the probability an evaporated molecule will be convected away from the surface which in turn is proportional to atmospheric density.
6. Due to the heat gradient maintained by evaporative heat transport, the practical near surface temperature (outside of those few microns) can exceed the theoretical S-B limit by an amount regulated by atmospheric pressure without regard to the composition of the atmosphere.
Concluding: Your proof is valid within a few micron range. But it fails, for wet windy planets, if the practical near-surface temperature is substituted for the actual radiating surface. This is because evaporative transport can suppress the temperature of the actual radiating surface relative to the surrounding matter.

Leonard Weinstein
January 14, 2012 12:39 pm

G. Karst,
Your statement “All gases when energized by conduction, above ambient, will radiate energy.” is wrong. All gases that are diatomic or have more than 2 atoms per molecule will absorb and radiate some range of radiation. In general, the diatomic gases only absorb and radiate at short wavelengths (typically in the UV). Water vapor, CO2, Methane, etc, absorb and radiate at wavelengths typically found at near ambient temperatures on Earth. These wavelengths are related to excited vibration energy states of the molecules, and occur only at specific ranges of frequencies. Gases do not act like solid and liquids, as black or gray bodies. The specific interaction frequencies are the cause of the atmospheric greenhouse effect (along with aerosols, which do act like black bodies, but are being neglected in this discussion). If an atmosphere had only O2 and N2, it would only heat by conduction and convection with the solar heated ground (except for a very small UV absorption, and this could not be radiated back due to the air temperature). Convective mixing would cause a lapse rate to form, but no radiation would cool the top of the atmosphere, so once it warmed initially, the radiation balance would only be due to the absorbed solar energy, and ground to space radiated energy.

January 14, 2012 12:41 pm

Willis Eschenbach says:
January 14, 2012 at 1:36 am
Go away, sir, your impudence knows no bounds. You haven’t a tenth of the knowledge you claim.
I made a mistake, and admitted it. My childhood memories let me down. I was so sure I remembered it correctly that I didn’t check before posting (something I usually do). It will serve me right to be more prepared next time.
Which doesn’t give you any ground to dismiss out of hand anything else I said or may say in the future, or to make sweeping and insulting conclusions about the volume of my knowledge.
By commanding everybody who disagrees with you to “go away,” you are making yourself more and more ridiculous every time. Not to mention that your insulting language, which you alone are allowed to use with impunity on this forum, makes me even more determined to keep you in check.
Before your arrival on the scene, Mr. Eschenbach, this site was almost 100% civil. Now, with your non-moderated outbursts, whatever are the merits of some of your articles, it stinks of favoritism.

gnomish
January 14, 2012 12:42 pm

practical illustration:
heating a piece of metal with a torch.
if you want the whole thing hot, it works best to blow the hot gases back and forth over the entire thing rather than just one spot.

BarryW
January 14, 2012 12:47 pm

I’m still thinking about this but one thing you said in a reply to a comment is wrong. Your GHG atmosphere does radiate. It receives energy from the surface by conduction. To have a temperature it radiates even if it can’t absorb IR.

George E. Smith;
January 14, 2012 12:54 pm

“”””” Genghis says:
January 14, 2012 at 9:21 am
I think some seem to be missing the obvious part here. If you take a volume of gas at a given temperature and compress it, the same gas in a smaller volume now has a higher temperature, due to the compression and resulting density “””””
Not true !
The same gas in the smaller volume, now has a higher Temperature, DUE TO THE WORK DONE ON IT TO COMPRESS IT.
The volume (V) of gas has some surface area(A) depending on the geometry of the container, and neglecting the weight of the gas itself, that surface experiences some pressure P everywhere on the surface.
So the total force pressing on the container, and vice versa is given by F = P.A
So now if you move every pointon that container surface inward by an infinitessimal distance ds, the total wark done is force times distance W = F.ds = P.A.ds = P.dV, where A.ds = dV the change in Volume.
It has nothing whatsoever to do with the density of the gas, and furthermore, the heating that results because all of that work done ends up as waste heat, is a purely transient event, so it is NOT a new stable state of the gas; which will eventually return to the ambient Temperature with a reduced volume, and an increased density.

Richard M
January 14, 2012 12:55 pm

Eric Barnes says:
January 14, 2012 at 11:48 am
[Richard M says:
January 14, 2012 at 11:11 am
Not a coincidence at all if there is a maximum greenhouse effect. ]
Convenient explanation.
So we’ve maxed out at 390ppm CO2 and our current level of water vapor/methane?
Watch how far you back up. The cliff of never ending rationalization is near.

I was simply pointing out there does exist another simple explanation. Is it true, well one shouldn’t ignore all possibilities dealing with physics. You can heat water forever and it won’t get much warmer than 100C. Why not 1,000,000C? The answer we all know has to do with the physics of water.
In fact, the maximum could occur at much lower concentrations of CO2 simply because CO2 is not the only greenhouse gas. Remember, I said a maximum GHE, not necessarily a maximum if CO2 was the only GHG. Also, it could be not an actual maximum, just a threshold where increases take so much additional GHGs that the result is effectively a maximum.
All it really takes is a catalyst that enhances the GHE at low concentrations so it maxes out before one would normally expect. In a gravitation field with constant downward acceleration we might not have a linear response.

January 14, 2012 12:56 pm

Stephen Wilde says:
January 14, 2012 at 10:19 am
Willis, my post, as above, awaits your attention 🙂
I think it is as good an elevator speech as you could reasonably expect.

January 14, 2012 12:58 pm

I feel sorry for Mother Nature–she so desperately wants to randomize the universe that she’ll even apply forth-power gain to motivate radiation in the cause of entropy. But then she comes up against gases like Nitrogen, Oxygen and Argon and regardless of her futile frustration, there is nothing she can do as these stubborn gases hold their acquired energy forever. Couple thermal energy into them and there it stays for all eternity.Then CO2 comes into play and increases the temperature of these gases even more. Oh, how our Mother is vexed and powerless. I wish I could send her a bouquet of flowers and a bottle of wine.

tallbloke
January 14, 2012 12:59 pm

Willis says
“TO CONCLUDE: I’m interested in people who can either show that my proof is wrong”

I have shown Willis that his proof is wrong here:
http://wattsupwiththat.com/2012/01/13/a-matter-of-some-gravity/#comment-863688
[Tallbloke, I snipped that because it had no scientific content. Post your elevator speech or give it up. -w.]
It is an easy to understand demonstration that his assertion that Hans Jelbring’s 2003 E&E paper violates the laws of thermodynamics because of considerations regarding radiation to space is incorrect. I provided him the same demonstration in email yesterday. The demonstration consists of two lines from the very paper Willis attacks. They are:
“A simplified model of Earth will be considered. The model planet does not rotate. It
neither receives solar radiation nor emits infrared radiation into space.

Willis has been unresponsive regarding this demonstration both in email yesterday and on this thread he subsequently posted here at WUWT.
Joel Shore made similar accusations, also unsupported by fact, against Nikolov and Zeller. I have offered him a guest post in which to lay out his objections formally. This offer was declined.
It is now a matter of public record that Willis has been formally alerted to the failure of his ‘proof’. I will record this on my website in a new post. Since Willis hasn’t responded to my demonstration of the failure of his ‘proof’ here, any whining about his inability to respond on my website due to his self banning will be met with the derision and ridicule it deserves in a place his snipping scissors cannot reach.
I will not have people who ignore correct formal scientific points directly offered to them multiple times and who then continue to cast demonstrably false slurs against reputable scientists posting comments on my site. They can cry “censorship” all they like, but it doesn’t cut it with me, especially when they have censored parts of adverse comments here on this thread, and deleted others completely.
[Tallbloke, first give us your elevator speech about Jelbring’s hypothesis. Until then, I will not believe you understand it well enough to “prove” anything. You have not shown that my proof has “failed” as you claim, record it on your website or not.
Finally, what “demonstrably false slurs” by Joel are you talking about? You keep making accusations without a shred of proof or even a hint of evidence, which I suppose shouldn’t surprise me by now, but still does. I snipped the last bunch, but I left these in so people can see your willingness to make unsupported accusations.
-w.]

ferd berple
January 14, 2012 1:05 pm

Consider a simple analogy. [SNIP: Consider instead giving and elevator speech or disprove my proof, but no vague handwaving. -w.]

Anton Eagle
January 14, 2012 1:06 pm

Good article Willis.
I think I am being persuaded by your argument. But, I have one lingering question.
Although I fully understand that N2 and 02 are not IR absorbers (different bond energy), my question is the following. Lets say I manage to heat up some non-GH gas through some other means other than a radiative process… and then isolate it so that conduction and convection are no longer possible… will the non-GH gas not radiate? Will the temperature just hold constant forever?
That seems unlikely, but I admit I am unsure. Even if they do not radiate in the IR spectrum, it still seems like the non-GH gas would still eventually radiate all its energy away… perhaps at some other frequency?
So, even if a non-GH gas atmosphere were transparent to IR, it still seems like it could participate in radiative processes once heated. If a planet heats its atmosphere through conduction and convection, it still seems like the atmosphere could then radiate some of that energy away to space, even if its not at IR frequencies.
-Anton

Archonix
January 14, 2012 1:07 pm

[SNIP: If you wish to accuse me of being disingenuous, quote my words or go home. I won’t stand for this kind of vague nasty accusation, that’s a slimy tactic. And if you believe a man like Tallbloke, who censors scientific opinion that he disagrees with, you are an idiot. -w.]
Well, Willis, since you took out the substantive part of my post and then called me an idiot I really have nothing more to say to you on the subject except, as you have opened that door: you are an arrogant, insufferable ideologue who refuses to countenance the possibility that other ideas may be better than your own. You have constantly belittled and attacked anyone who disagrees with you, characterise your opponents as ignorant fools, accuse them of the very tactics you use against anyone whoholds a differing opinion to your own and act as if you’re God’s own gift to the internet. In short, sir, your condescension ill behoves a scientist as you claim to be.
Consider this a formal protest at your behaviour, your arguments and your continued presence on this site.
Graham Dawson
[Your protest is noted. I also note that you have said absolutely nothing scientific in your post at all, you just whined about what a terrible awful person I am.
I called you an idiot because you obviously believed the bull that Tallbloke was spouting. You didn’t ask me if Tallbloke was right, you didn’t check your sources, you just started out your post by saying I was being disingenuous but didn’t say where.
Now, you have come back and once again, just like in your first post, you start out with the accusations. Now the problem is that I’m an arrogant insufferable ideologue … well, that’s all good to know, and I’m sure it does one important thing—it might, just might, make people forget that there isn’t a scrap of science in this post.
w.]

NoIdea
January 14, 2012 1:09 pm

Thanks Willis but I’ll pass. I don’t think there is any more ‘science’ in Huffman’s claims than that. That is perhaps his weakest point – he does not bother to debate the detail. The Venus stuff is essential for the simple reason that he claims it is the comparison between the two that is the critical issue. There is no ‘runaway’ GE on Venus – the temperature at various pressure levels conforms to the profile of the ‘standard atmosphere’. Were the earth’s atmosphere substantially deeper, then the temperature near the surface would be higher. I suspect that means that the temperature in say a deep hole (mine) on the earth would be higher (absent any geothermal effects that is).
I thought the point of the elevator speech was to describe in concise terms what is happening. And I think it’s pretty clear what he claims. How deep into science you’d need to go to prove/disprove his analysis I have no idea.
My question was merely to what extent does his claim differ from N&Z, as to me they appear largely similar. I’m in no position to judge however.

January 14, 2012 1:09 pm

Well said, TB.
My post appears on your blog too but I refined it slightly, above.
I want to hear Wilis’s reply and if he can show that my formulation is incorrect then c’est la vie 🙂
Mind you, I do entirely agree with Willis’s fine ‘Thermostat’ idea but still think he should have extended it globally as I have done.

P Wilson
January 14, 2012 1:12 pm

If the proof is in contradiction: The basal metabolic rate of a human is 58wm2. A human is on average 2m2 thus the average human emits around 100m2 of radiation. YET: Infrared devices record humans as a glowing patch, yet the ambient background is dark. Thus at night the earth emits FAR LESS than 100wm2.
The S-B equation is a gross oversimplification. It is a theoretical tool that expresses the relation between temperature and radiation emitted from a surface. As it is a single equation, it is applied to all surfaces, gas or liquid or solid without any regard whatsoever for their properties. Air for example has little very low heat capacity, and is not even a surface, but a 3 dimensional gas which emits virtually nothing compared to a solid. Solids themselves give off very little radiation at standard/average temperatures.
the absurdity of the results from this equation is not science.
In fact, very little radiation is given off by surfaces at normal temperatures. In terms of the earth, it is probably about a tenth (1/10) of the radiation given off by a human.
The SB equation for example gives water at freezing (0c) as 315Wm2, if 5.67051 x 10-8 x K4.
That is the equivalent heat emission of three combined people at body temperature on an average day.

Robert Austin
January 14, 2012 1:13 pm

Wow! Too many posts to read in detail so I skimmed and hit what appeared to be the best ones.
My conclusion:
I would second Willis’s elevator speech and fully agree with his thought experiment as posed.
Some of the fallacies (in my opinion) demonstrated by posters to my mind are:
-gravitational compression elevating surface / atmospheric temperature.
-atmospheric convection having a role in this thought experiment.
-non greenhouse gases emit significant black body long wave radiation.
This kind of esoteric topic seems to generate a great deal of heat, so to speak. I hope you and tallbloke can kiss and make up. The same for tallbloke and Joel Shore.

Jim D
January 14, 2012 1:21 pm

Part of the confusion is that Jelbring defines the GHG effect differently from just about everyone else, as mentioned above by lateposter, and as you can see from the paper. If you eliminate solar and IR radiation, you are also eliminating the greenhouse effect and all you have left to explain everything is gravity, so what is being explained is not the greenhouse effect, but just the classical lapse rate effect, which is also explained in the first chapter of any atmospheric textbook.

Bill H
January 14, 2012 1:24 pm

ferd berple says:
January 14, 2012 at 11:01 am
Exactly. In the absence of GHG, the atmosphere would not be able to lose energy to space. Any energy lost to the atmosphere from the surface would eventually be returned to the surface.
In contrast, with a GHG atmosphere, energy lost from the surface to the atmosphere can then be radiated to space by the atmosphere and need not be returned to the surface.
Thus, without a GHG atmosphere, S-B tells us that the surface of the planet must warm to radiate this excess energy, that is no longer being radiated away by the atmosphere.
———————————————————————————-
with the absence of a pathway for IR to “radiate” the black body will warm to the point of equilibrium that CONVECTION will control the balancing of incoming and out going energy.
in the absence of convection (even in a non GHG atmosphere) the black body will warm until it begins burning. without one or the other to remove heat the body will get very hot.. a non conductive atmosphere will stop energy loss at night
Convection passes heat from molecule to molecule and with each transference, a loss of energy to space. Gravity compresses the molecules in close proximity and as it diminishes allows further space between them.. Thus gravity controls the rate of convection.
as a layman its a rather simple concept.

January 14, 2012 1:26 pm

Now suppose we add an atmosphere to the planet, a transparent GHG-free atmosphere. If the theories of N&K and Jelbring are correct, the temperature of the planet will rise.
But when the temperature of a perfect blackbody planet rises … the surface radiation of that planet must rise as well.
And because the atmosphere is transparent, this means that the planet is radiating to space more energy than it receives. This is an obvious violation of conservation of energy, so any theories proposing such a warming must be incorrect.

This is not correct. Add an atmosphere to a rotating planet with an inhomogeneous temperature distribution, hottest at the equator, coldest at the poles. If you use this as your baseline temperature instead of an ideal blackbody, adding an atmosphere enables lateral heat transport (whether or not the atmosphere radiates, surely it convects and conducts). This move heat from the hot equator towards the colder poles, which produces net heating as in an increase in mean surface temperature for the same insolation.
The reason your last statement isn’t correct is that things aren’t linear. T^4 is not linear. ^4 \ne . This is what davidmhoffer has been pointing out in the other thread. As long as one can transport heat in air or water, one can achieve “heating” of the average temperature while maintaining balance.
However, this doesn’t effect your general argument. I have no idea what N&Z could possibly be saying either. Some of what I read sounded like complete nonsense — connecting PV = NkT to the effect, for example, and making noises about gravity being a “source” for additional heating or the like.
The only thing that I can imagine that makes sense is this. If the atmosphere of a planet is thick enough that you cannot see the ground, and is layered so that relatively transparent gases live outside relatively opaque gases, you end up achieving energy balance at some atmospheric height that leads to some equilibrium temperature there (recall you have BCs of “low pressure, cold” somewhere above the surface at the top of the transparent zone). You then have an e.g. adiabatic lapse rate all the way to the ground. So the ground is much warmer. That suffices to describe Venus — somewhere high above the surface, there is a maximum height from which energy directly radiates, and from that height down the atmosphere gets hotter because of the increase in pressure and equilibrium.
The Earth is completely different, though — the surface radiates directly and there is a lapse rate with an atmosphere in between that radiates energy from many heights and temperatures (but only in certain bands). The lapse rate is due to gravity and thermodynamics, sure, but that’s not news, nor is it the “cause” of the warming. As you say, in a perfectly transparent atmosphere with no lateral heat transport there would be no surface warming caused by the atmosphere, because the atmosphere would be in quasi-static thermal equilibrium with the surface and would not lose energy at all on the outside (air-vacuum) interface. The only way I can think of that it would
“warm” is by buffering the incoming heat so that it produced a smaller warming in the first place, then released it to slow the cooling as well, again by moving the temperature everywhere closer to uniform.
rgb

Alan Wilkinson
January 14, 2012 1:27 pm

Roy Spencer’s comment is incontestable. If the atmosphere is transparent to radiation then there is no convection, it must become isothermal via conduction and cannot alter the radiating surface temperature at equilibrium.
Dr Jelbring seems to contest his model is based on a transparent atmosphere reference but leaves me no wiser as to what if anything it proves.

January 14, 2012 1:28 pm

The previous reply once again ate my < type brackets, sorry, that were supposed to denote averages. It said "T to the fourth average is not equal to T average to the fourth". A very common problem in the discussion is conflating the two as if they are somehow equal.
rgb

George E. Smith;
January 14, 2012 1:31 pm

[SNIP: stick to elevator speeches or disproving my proof. Vague meanderings need not apply, nor long expositions on basic theory. Sorry, w.]

ferd berple
January 14, 2012 1:33 pm

Jim D says:
January 14, 2012 at 10:48 am
The problem with ferd berple’s argument is that just because GHGs reduce the amount of surface radiation escaping to space, you can’t conclude that there is less surface radiation entering the atmosphere
At no time did I discuss surface radiation entering the atmosphere. One the contrary my proof relies upon “net energy escaping to space”.
A large part of the confusion in climate science is a result of analysis of GROSS energy, which leads to the problems of calculating energy transfer between the surface and atmosphere, convection, back radiation, etc. etc. My proof purposely avoids this confusion.
As soon as you consider only the net transfer in energy from the surface to space and from the atmosphere to space, the problem is crystal clear. There is no need to calculate the relative transfer between the surface and atmosphere, because in the end it is the radiation to space that must balance. Radiation in = radiation out.
A second part of my proof was to compare two planets, one with a radiating atmosphere and one without. Rather than avoid the confusion of trying to calculate temperature, I used the simplifying device of an inequality. My point wasn’t to show how much effect GHG has, only the sign of the effect.
My proof shows that the standard model of GHG theory has the sign reversed. In which case this lends weight to the gravity model of regulating planetary surface temperature, and assigns GHG a secondary role in redistributing energy between the surface and atmosphere.

tallbloke
January 14, 2012 1:35 pm

Wow, Willis’ censor’s scissors have been busy here in the last few minutes. Good job I screenshotted relevant posts.

January 14, 2012 1:36 pm

George E Smith said:
“furthermore, the heating that results because all of that work done ends up as waste heat, is a purely transient event, ”
Hi George,
I’m a fan of your work but you’ve missed something there.
It isn’t called a gravitational CONSTANT without good reason. Joel Shore and others make the same mistake.
Gravity is a continuous renewing process which replenishes itself over time. No one knows why or how, not even Einstein, but we must live with it.
So it is NOT a transient effect and the result is PERMANENT all other things being equal.
Without that effect the Ideal Gas Law would not be a Law.

Phil.
January 14, 2012 1:37 pm

TimC says:
January 14, 2012 at 10:46 am
Willis said (to Crosspatch): “I will repeat it again. If there are no GHGs in the atmosphere, the atmosphere will not and cannot radiate energy. That’s the whole point. The only thing that can radiate is the surface. You keep claiming the atmosphere will radiate. It will not.”
I don’t profess to any real knowledge of thermodynamics but isn’t this then postulating (a) an atmosphere with mass, (b) capable of convection and conduction, and (c) with its own localised temperatures and gradients (separate from the surface temperature, although no doubt coupled to it, with lags).
Why can’t such an atmosphere radiate? Suppose, instead of surrounding a planet, it was just a cloud of the (hypothetical) transparent gas, in space. Would there be no way of physically detecting it’s presence, outside the gas envelope itself?
I’m smelling an impossible premise here, somewhere.

An atmosphere of Argon fits the bill, M=40, can conduct and convect, will have a lapse rate, this atmosphere will not radiate. No impossible premise.

Craig Moore
January 14, 2012 1:39 pm

I humbly ask that everyone stop the derision, ridicule, slurs and snide remarks. I have been a fan of both Tallbloke and Willis. Either take it to the alley or transcend the silliness.
Here’s my simple understanding. Take a water bottle and fill with boiling water or with equal temperature gases. Don’t care. Hold up your hand an inch away. Does the heat radiate to warm your hand? Isn’t the atmosphere under pressure from gravity also the the gas in the water bottle?
By the way can’t both positions be true that there is a GH effect and gravity/atmosphere pressure working together?

kwik
January 14, 2012 1:39 pm

[SNIP: stick to elevator speeches or disproving my proof. Your snark is not appreciated. Sorry, w.]

Tester
January 14, 2012 1:47 pm

It is very easy to test backradiation theory. Put a mirror towrads ground approx 2 meters high on a sunny day so that it reflects all IR from the ground to sunny place on a ground. Now measure temperatures from this place where mirror reflects all IR and surroundings. Result from that test is there is no difference in temperature on ground in any spot. Why so, mirror is obviously cooler than the ground and radiation from cooler object can’t warm warmer objects. If you somehow can warm the mirror over the temperature of the ground you might get some little difference depending of the temperature difference. So backradiation warming is pure bulls**t from cooler atmosphere to warmer ground.

D. Patterson
January 14, 2012 1:47 pm

Gentlemen, experiments at forty paces.

tallbloke
January 14, 2012 1:48 pm

Willis.
You have been wholesale deleting peoples responses to this thread to leave it looking like most people agree with you.
[Bull. I have been deleting just what I said I would delete, and I should have deleted more. Your pretended surprise that I would actually do what I said I would do doesn’t become you.]
At least I was clear direct and upfront about who was not being allowed to post at the talkshop and the reasons why. You have been removing comments altogether because they disagree with you. It’s all being recorded at the talkshop in real time. Any justification you thought you had for complaining about Joel being barred from posting is out of the window.
[Oh, I see, my subsequent actions after the fact mean that your banning Joel was correct and proper … ]
There was scientific content in my first reply on this thread, and it also contained a reply about the situation with Joel. Your removal of it shows who the real censor is. You are doing yourself terrible damage here. Stop now.
[Tallbloke, every post can be claimed to have scientific content, including yours. I said that I would snip things that were off-topic, which your post most assuredly was. If you (or anyone else) think your post contained actual science that has been deleted incorrectly, then post the scientific part again and we can discuss it. -w.]

ferd berple
January 14, 2012 1:53 pm

Ken Coffman says:
January 14, 2012 at 12:58 pm
Then CO2 comes into play and increases the temperature of these gases even more.
CO2 reduces the temperature of the atmosphere, according to WP which is my bible:
Within the mesosphere, temperature decreases with increasing altitude. This is due to decreasing solar heating and increasing cooling by CO2 radiative emission. The top of the mesosphere, called the mesopause, is the coldest part of Earth’s atmosphere.[1] Temperatures in the upper mesosphere fall as low as −100 °C (173 K; −148 °F),[2] varying according to latitude and season.
http://en.wikipedia.org/wiki/Mesosphere

January 14, 2012 1:54 pm

Gravity does effect temperature, i.e. without gravity there can be no atmosphere and therefore the atmosphere following the basic gas laws at each altitude level DELAYS or slows the rate of radiation back to space via it’s specific heat. Venus is an example of this, given the 90 bar pressure at the surface we expect to see high temperatures maintained even at night time due to the pressure-temperature relationship. Due to the thickness of the atmosphere even the night time side of Venus is quite hot unlike airless Mercury or the Moon on the dark side that approach but do not reach absolute zero.
Moreover it’s not just the atmosphere that slows the rate of IR back to space via it’s specific heat property, it’s also the type of regolith that absorbs energy and then releases that energy according to its own physical properties via density and conductivity, specific heat. E.g. a stone heated in the sun absorbs that heat all the way to the core if the heat is applied to it long enough depending upon it’s material composition, size and shape; then once the sun drops (planet rotates) the stone being slower to release it’s heat than air radiates it’s energy in the IR band back to space for hours until it reaches equilibrium temperature with the surrounding air.
Your argument fails the test of reality because you make the false assumption of time scale and rate of heat conductivity. The fact that the Moon’s surface does not achieve absolute zero during the two weeks of darkness as it revolves around the earth demonstrates this concept of time delayed IR radiation. Basic thermodynamics tells us that the rate of heat transfer (even via radiation) decreases as the temperature differential decreases. Call it a heat storage effect if you will.

January 14, 2012 1:54 pm

Published elevator speech: Ned Nikolov, Ph.D. & Karl Zeller, Ph.D.
Based upon the analysis of data for eight planetary bodies [Mercury, Venus, Earth, Moon, Mars, Europa, Titan and Tritonus] they derived a simple yet robust formula for predicting a planet’s mean surface temperature as a function of only two variables: 1) TOA solar irradiance and 2) mean atmospheric surface pressure.
Plain speaking elevator speech: Harry Dale Huffman
All the supposedly learned theorizing by one and all is precisely worthless, because everyone uses it to ignore the simple, definitive fact that disproves the tyrannously-promulgated carbon dioxide greenhouse effect, and reveals the radiative transfer theory as unconnected from the real thermodynamics of the atmosphere.

Bomber_the_Cat
January 14, 2012 1:58 pm

kcrucible, January 14, 2012 at 11:40 am
OK, I think by now most people should have got the message that the major constituents of the atmosphere, nitrogen and oxygen. do not absorb or emit long wave infrared radiation. Consequently, they play no part in the greenhouse effect and they cannot cool the atmosphere by radiating energy to space. It is not a difficult concept, so those who haven’t got that yet are not going to get it.
“Simplifying the earth to only consider IR is a gross oversimplification”. No it isn’t. This is what the earth radiates from its surface.
“We have IR being generated from the surface that did not arrive as IR!”. Yes, that is quite possible, in fact it is what happens. So, you got something right.
“So, an atmosphere that acquires heat content from the surface of the earth ….. has the possibility of radiating that energy as microwaves, etc, etc.”.
The wavelengths that a body is able to radiate at are determined by its temperature. The spectrum of wavelength distribution is governed by Planck’s Law. This specifies how much power can be emitted at each wavelength and this is what determines whether it emits UV, infrared, visible, radio waves or microwaves etc. and in what proportion. The temperature of the Earth’s surface dictates that the earth will emit in the infrared. The temperature of the atmosphere dictates that it will also emit in the infrared, if it is able to emit at all. If a body cannot emit within the range of Planck prescribed wavelengths for its temperature, then it will not emit anything. It cannot just decide to emit in some other part of the electromagnetic spectrum instead. Therefore, only the infrared region is pertinent to the greenhouse effect. It is not a gross oversimplification – it is called physics.

Fred Souder
January 14, 2012 1:58 pm

Willis,
I am not sure if anyone has mentioned this yet, so please forgive me if it has been mentioned. I am in a hurry to paint the family room before my wife gets home so I haven’t thoroughly checked the thread.
You said in a comment earlier that the potential energy of the molecules in the upper atmosphere is greater than at the surface, thus, we have a lapse rate (or something to that effect). If we double the amount of atmosphere, then we would increase the potential energy of the molecules at the top of the atmosphere. Maybe about 1.2 times for the first doubling in my hasty estimation (gravitational potential doesn’t double, and gas is compressible, so something less than the sqrt of 2).
If we increase the potential E of the top molecules of gas, then we increase the KE of the lower levels and we have temperature rise. I don’t see how it matters whether this is a greenhouse gas or not. Maybe I am not understanding where this fits into your argument when you brought it up in the thread.

January 14, 2012 2:00 pm

Willis said:
“Since there is gravity, the atmophere will have a “dry adiabatic lapse rate”, which means that the temperature must drop with altitude. The atmosphere will warm until the bottom layer of the atmosphere has the same temperature as the surface, and has the dry adiabatic temperature profile above the surface. It will neither gain nor lose energy after that, and will be stable with no bulk motion.”
Ok, you’ve accepted the gravity induced dry adiabatic lapse rate.
And you seem to accept that the warming is from the solar irradiated surface and that the lapse rate is supported by conduction from the surface.
That is then the baseline gravity induced GHE as per N & Z and the Ideal Gas Laws. Nice and stable and set by gravity and atmospheric mass alone.
Then one introduces GHGs which have two effects.
They absorb more energy due to their radiative characteristics.
They then radiate 50% up and out of the system and 50% back down to the surface.
The 50% sent upward reduces total system energy content because it is lost to space. That is a cooling process.
The 50 % sent downward destabilises the gravity induced GHE but in turn provokes more convection and on a water planet energises the water cycle too.
Now, convection and the water cycle are cooling mechanisms (evaporation has a huge net cooling effect of 5 to 1 – see latent heat of vapourisation) so that 50% sent downward must be all or mostly negated unless you can show otherwise and the N & Z data seems to show that the negation is pretty much complete.
Which leaves the (admitted) gravitationally induced GHE firmly in control does it not ?
Checkmate ?

Archonix
January 14, 2012 2:00 pm

The reason, I feel, that people are not paying attention to the model is that it removes the one element that’s necessary to understand earth’s atmosphere. It radiates. Your model is an exercise in pointlessness – it doesn’t model that one tiny little detail that makes earth’s atmosphere tick; it’s full of gases that radiate all over the place and because of those gases radiating all over the place – but specifically radiating outwards at the top – the atmosphere can’t reach an equilibrium and stop circulating. That outward radiation introduces an imbalance that in turn provides a mechanism to create work, by cooling gases much further than lowering their pressure would do in the non-radiating atmosphere your model describes.
That’s why I said you were being disingenuous. Your model doesn’t model anything approaching reality, which is also why I described it as a strawman: it’s a parody of the argument you’re trying to refute with key elements removed.
Finally, since I’m sure my other comment won’t get posted I’m going to rephrase one part of it: you have accused me of being an idiot for supporting the opinion of someone who is, as far as I can tell, making an honest and effective rebuttal of your model. I never attacked you personally, though I did question your motivations. I would be pleased if you never posted on this site again and you can consider this my formal protest against your continued presence.

January 14, 2012 2:05 pm

Willis said:
“If there are no GHGs, the surface must radiate (to space, since there are no GHGs) the amount of energy it absorbs. Its radiation is fixed and unchangeable”
You forgot something.
The Ideal Gas Law means that the warmest molecules of air are at the surface.
Those molecules are at a higher temperature than the average for the atmosphere.
Thus they will inhibit upward energy transfer more than would be the case if the atmosphere were at a cooler average temperature throughout.
That will give a higher surface temperature than predicted by the S-B equation.

Phil.
January 14, 2012 2:06 pm

crosspatch says:
January 14, 2012 at 12:39 am
“1. The non greenhouse gas atmosphere is a perfect conduction insulator to space, it can’t radiate its heat out.”
What? Everything with a temperature radiates. I don’t care WHAT a substance is, it will cool. It will cool until it reaches equilibrium with its background. Greenhouse has absolutely nothing to do with radiation, it has to do with absorption. Something can be completely transparent to LWIR and still cool down by radiation. A greenhouse gas is something that absorbs LWIR. It has nothing to do with what it radiates. So I can have a non greenhouse gas that heats via direct contact … conduction. It will STILL radiate that heat away. It just didn’t heat by absorbing LWIR.

No it won’t, you need to learn about the physics of gases.
Willis’s theoretical planet, uniformly illuminated from all directions, in the absence of an atmosphere will be at a uniform temperature of Tsb. Add a transparent atmosphere and initially the gases next to the surface would heat up to close to the surface temperature, briefly cooling the surface, as gases conduct poorly the temperature will fall off rapidly with height. This would be unstable so convection cells would form and the temperature profile will approach the adiabatic lapse rate . As the same radiation is incoming and less is leaving because the surface is cooler and the atmosphere can’t radiate the surface will warm up to the original Tsb with a temperature profile at the adiabatic lapse rate.
Double the mass of the atmosphere and the pressure everywhere will double, the surface will achieve the same temperature, Tsb, and the atmosphere will have the same profile, -g/Cp. The only difference would be the time to achieve equilibrium.
Increase the gravity of the planet and the surface pressure will increase and the lapse rate will change but the equilibrium temperature will be the same Tsb, again the only difference would be the time to achieve equilibrium.
Therefore in the absence of GHGs there is no ‘gravitational enhancement’.

markus
January 14, 2012 2:06 pm

Willis, You have your ladder upside down.
LWR reaches and excites the atmosphere to a equal state of LWR emission relative to the atmospheric pressured state of of it’s molecules. The atmosphere reaches a maximum saturation of LWR before any surplus LWR reaches the Earths surface and excites it to it’s maximum saturation of LWR.
In turn, some of that absorbed energy is radiated by the surface back to the atmosphere.
None of reflected energy from the Earths surface is absorbed by greenhouse gases (GHGs) in the atmosphere, as they are already excited to a state of maximum LWR.
INMO, LWR steps down to the surface but uses a elevator to get back up. Due to the different atmospheres of Earth, strats & tops, It’s like, two steps down, one step up.
How does LWR from our Sun pass through GHG’s with no effect but the same GHG’s are not effected by the Earths surface emissions? Your steps mean a gas sphere held by gravity, would have the same T as deep space.

ferd berple
January 14, 2012 2:06 pm

Willis Eschenbach says:
January 14, 2012 at 1:41 pm
Tallbloke, I said I would snip. I’m snipping. Don’t pretend to be surprised, it makes you look meretricious. Screenshot all you wish, I have nothing to hide.
Willis, censorship has no place in the discussion. What sets WUWT apart from RC is the ability to express ideas right or wrong. Lose that right, and tyranny will follow.
“I disapprove of what you say, but I will defend to the death your right to say it”

January 14, 2012 2:07 pm

“That’s why the surface can get hot, because some of the energy radiated by the surface is absorbed by the atmosphere so we’re not emitting to space more that we’re absorbing.”
Which is why the equilibrium temperature rises intead ?

Jim D
January 14, 2012 2:08 pm

ferd berple, yes, you didn’t discuss surface radiation entering the atmosphere, but that is part of the surface radiation, and if you are equating radiation to temperature, you have to consider it too because temperature affects both parts.

markus
January 14, 2012 2:09 pm

JEEZ. I Meant;
How does LWR from our Sun pass through GHG’s with no effect but the same GHG’s are effected by the Earths surface emissions?

Mario
January 14, 2012 2:16 pm

Willis
As usual a clear and simple post.
Your reductio ad absurdum as presented with note 1 is all you need to convince me that there is a problem with N&Z.
Does such a planet exist, probably not.
Is the Earth more complex, yes. But it does not take anything away from your argument.
Dr Brown.
In the planet that Willis describes it is surrounded by mini suns. Incoming energy is evenly distributed over the entire surface, pole to equator. As presented I do not see any lateral heat transport, rotating or not.
Is my understanding correct?

Alan Wilkinson
January 14, 2012 2:23 pm

Anton Eagle says:January 14, 2012 at 1:06 pm
.”.. will the non-GH gas not radiate? Will the temperature just hold constant forever?”
Anton, non-GH gases like O2 and N2 don’t radiate IR (vibrational energy transitions) because they have no dipole moment – except when they are momentarily and rarely produced by collisions. So they will slowly leak energy. However that process is very slow relative to others.

Bill H
January 14, 2012 2:27 pm

Willis,
I think I understand the point you are making.
A body absent the ability to either radiate or convect energy, it will collect energy until it gets to a fission state. thus a star which then radiates in all bands..
There in lies the problem with N&K. in their attempt to show black body balance they have nulled their hypothesis. And possibly yours also…. nothing in the universe is static, including gravity. it is constantly changing..
By creating a planet with an atmosphere of any substance which is clear, convection becomes the controlling “balancing” act. Now gravity plays a roll as does the type of gas and the size of its molecules. thus the rate of convection. The lapse rate is what will dictate the temp at which it balances (equilibrium). Now mix gases and it really becomes fun..
when one looks at atmosphere challenged planets their own night time radiation out ways the daytime collection usually by a factor of 3/2. the reverse is true during day time hours where thy reach 500K while at night they can drop to below 100k. by adding a clear atmosphere you attempt to create a thermal blanket to “balance” your outcome.. however, there is always loss with any atmosphere.
I’m not sure what it is they are trying to say other than confuse the subject with their paper. That being said, certain gases will allow warming with their compaction near the surface with convection due to their molecular weight and gravity holding them there..
a warm surface will warm the immediate area above it and depending on wind speed and its make up it will hold that thermal layer close. this will a allow warming.even absent an IR reflecting gas.
Bill

January 14, 2012 2:29 pm

Wow I come back from work and find this thread has boomed.
Firstly, people please stop it with the “all gasses radiate” meme.
It all depends on the vibrational mode.
Co2 has 4 fundemental vibrational modes. 2 of them are called scissoring, one is called asymmetrical stretching. These 3 modes ARE infrared active.
But even CO2 has a mode that is not responsive to infrared.The 4th mode, the symmetrical stretching, is not INFRARED active because the vibration does not cause a change in the dipole moment of the molecule.
One further point. Just because a molecule is symmetrical, does NOT mean it is invisible to IR.
So long as the vibration can change the dipole moment, it is active in IR. That is why, symmetrical molecules such as Carbon Monoxide and Iodine Chloride ARE active in IR.
But does any of this mean Nitrogen and oxygen can’t absorb and emit radiation? No, of course not, BUT IN THE INFRARED REGION…..THEY CANNOT AND DO NOT.
http://orgchem.colorado.edu/hndbksupport/irtutor/IRtheory.pdf
http://www.ems.psu.edu/~bannon/moledyn.html

ferd berple
January 14, 2012 2:31 pm

Willis Eschenbach says:
January 14, 2012 at 2:03 pm
Took me a minute to figure out what is wrong with this. It is in the assumption that “net energy radiated to space is a function of temperature”. It is, but for the surface it is also a function of the amount of energy absorbed by the atmosphere. That’s why the surface can get hot, because some of the energy radiated by the surface is absorbed by the atmosphere so we’re not emitting to space more that we’re absorbing. As a result, the last step of your proof is incorrect.
I’ve already accounted for that energy, by using “net” energy. The problem with your rebuttal is that you are double counting, confusing net with gross.
To explain, here is the problem in diagram form: “===>” denotes energy flow.
space <==A== surface ghg ==C==> space
In my calculation, I’m only considering energy flows A and C. You are adding in flow B. However, flow B is meaningless and only serves to confuse, because it does not take part in the flow to space. It is only the flow to space that is subject to the equivalence, radiation in = radiation out.
Therefore from above:
A + C = solar energy in = radiation out to space
However, on an earth with no GHG, atmospheric radiation to space = 0,
space <==D== surface no ghg ==F==> zero radiation to space
D + F = solar energy in = radiation out to space
However, since F = 0, this becomes
D + 0 = solar energy in = radiation out to space
Therefore, since solar energy remains the same
A + C = D (because F = 0)
Since C > 0, for GHG atmosphere
D > A
Since D and A vary as 4th power of Temp
Temp(D) > Temp(A)
Therefore the surface will be hotter on a planet with a non radiant (non GHG ) atmosphere.
QED

ferd berple
January 14, 2012 2:34 pm

arghh html spoils my posting above. The use of gt and lt characters goofed things up. I will repost.

January 14, 2012 2:38 pm

Willis
I’d be grateful if you would take the time to respond to my post at
http://wattsupwiththat.com/2012/01/13/a-matter-of-some-gravity/#comment-863674
thank you

Jim D
January 14, 2012 2:43 pm

Willis, you are probably not understanding Jelbring’s paper because you are trying to read too much into it. All he says is that two layers S and A in the atmosphere, separated by a distance D, under the adiabatic assumption will have a temperature difference give by gD/cp where g is for gravity (9.81 m/s/s), and cp is the atmospheric heat capacity (1000 J/kg/K). This temperature difference he defines as his GE (greenhouse effect). It is apples when the real GE is oranges.

ferd berple
January 14, 2012 2:43 pm

Willis Eschenbach says:
January 14, 2012 at 2:03 pm
Took me a minute to figure out what is wrong with this. It is in the assumption that “net energy radiated to space is a function of temperature”. It is, but for the surface it is also a function of the amount of energy absorbed by the atmosphere. That’s why the surface can get hot, because some of the energy radiated by the surface is absorbed by the atmosphere so we’re not emitting to space more that we’re absorbing. As a result, the last step of your proof is incorrect.
I’ve already accounted for that energy, by using “net” energy. The problem with your rebuttal is that you are double counting, confusing net with gross.
To explain, here is the problem in diagram form:
first some terminology:
“==X==)” denotes energy flow from left to right, called X.
“(==Y==)” denotes two way energy flow between right and left, called Y.
“(==Z==” denotes energy flow from right to left, called Z.
Here is the model I described,
space (==A== surface (==B==) ghg ==C==) space
In my calculation, I’m only considering energy flows A and C. You are adding in flow B. However, flow B is meaningless and only serves to confuse, because it does not take part in the flow to space. It is only the flow to space that is subject to the equivalence, radiation in = radiation out.
Therefore from above:
A + C = solar energy in = radiation out to space
However, on an earth with no GHG, atmospheric radiation to space = 0,
space (==D== surface (==E==) no ghg ==F==) zero radiation to space
D + F = solar energy in = radiation out to space
However, since F = 0, this becomes
D + 0 = solar energy in = radiation out to space
Therefore, since solar energy remains the same
A + C = D (because F = 0)
Since C > 0, for GHG atmosphere
D > A
Since D and A vary as 4th power of Temp
Temp(D) > Temp(A)
Therefore the surface will be hotter on a planet with a non radiant (non GHG ) atmosphere.
QED

pochas
January 14, 2012 2:44 pm

I feel compelled to add to the confusion.
It doesn’t matter whether the atmosphere is transparent or opaque. If the atmosphere is transparent the surface will be at the radiating temperature and the atmosphere will get cooler as you ascend. If the atmosphere is opaque the top of the atmosphere will be at the radiating temperature and it will get warmer as you descend.
How does the transparent atmosphere develop the adiabatic lapse rate? In Willis’s scenario it doesn’t. However on a real planet the polar surface is cooler than the equatorial surface. Because a gas is transparent does not mean it cannot conduct heat, so when the bottom layer is heated from the surface it expands just like the opaque gas, it rises and cools just like the opaque gas until the adiabatic lapse rate is recovered. The cold polar surface cools the gas by conduction so that the gas densifies and creates the necessary driving force for poleward convection which results in development of the lapse rate.
How does the opaque gas develop the adiabatic lapse rate? In Willis’ scenario it doesn’t. However on a real planet the equator is a net receiver of energy and the poles are a net radiator of energy (cosine effect) this will cause density differences in the upper atmosphere that cause poleward motion of the top of the atmosphere where it cools and descends just as it does on earth. Convection develops when there is a temperature difference, whether it is at the surface (transparent atmosphere) or at to top of the atmosphere (opaque atmosphere like Venus).
Willis places the earth inside a Dyson Sphere where everything is in a universal thermal equilibrium, temperature is the same everywhere, and there is no driving force for convection, no change in the system entropy and no creatures running around happily increasing entropy.
The real world is in a far-from-equilibrium situation which is completely different. We have a high temperature compact source of radiation in the sun and a low temperature sink in the 3 K temperature of the background radiation of space. The energy flows that take place to maximize the entropy of the universe produce the temperature differences that are the driving force for convection and for developing the adiabatic lapse rate common to all planetary atmospheres.
How about that, Willis?

Bebben
January 14, 2012 2:46 pm

Me, I’m with tallbloke.
Science is not settled in elevators.

markus
January 14, 2012 2:48 pm

You have been told Hans : Prove it or p**s off.
Kinda like what the general public would like to say to all the theoretical climate scientists out there.

Spiny Norman
January 14, 2012 2:50 pm

I’m not a scientist, so I feel like I am sticking my head into the lion’s mouth … BUT … in willis’ experiment, isn’t the only way he could have a completely (100%) transparent atmosphere would be if that atmosphere had no mass? Others have pointed out that any gas, no matter what it is comprised of, has at least SOME ability to store and radiate? If that’s true, then willis’ thought experiment gas must have some unusual property … if that unusual property is that it has no mass, then doesn’t that make it irrelevant as a disproof of the claim that the action of gravity (which requires mass) can heat the surface? I guess my question is this:
Willis, in your thought experiment … what are the properties of this proposed atmosphere that would enable it to be completely transparent?

carlbrannen
January 14, 2012 2:53 pm

Willis is correct.
If you assume that the only thing that radiates heat is the earth’s surface, then an atmosphere can’t heat the surface up.
I think that where most of the arguers against are wrong is in failing to consider the situation in equilibrium. Yeah gasses can get warm due to changes in pressure, etc., but that doesn’t matter. In equilibrium, all we need to consider is net heat flow. Since the gases only have thermal contact with the surface, the equilibrium result is that there is no net heat flow between the atmosphere and the surface. On the other hand, the heat flow into the surface is from the “sun”, and the heat flow away from the surface through radiation have to match.
Temporarily, heat could also flow into the planet’s interior, but this would be trapped in the interior, and heat flow out of the interior would cool down the interior. These are not things that go on in equilibrium so there is no net heat flow. Note that the temperature at the core will be (very very slightly) different from the surface temperature but that no heat will flow. The temperature difference is due to the small difference in potential energy. If the reader has trouble believing this, then see Tolman(1930) which is discussed and generalized in Phys. Rev. A 30, 1461–1464 (1984), “Spherically symmetric heat conduction in general relativity”, U. F. Wodarzik, http://pra.aps.org/abstract/PRA/v30/i3/p1461_1
This situation is analogous to the situation with the atmosphere (again for this GHG free atmosphere). The atmosphere will be slightly warmer at the lowest elevation, but only very very slightly. There will be no circulation. This is the equilibrium condition and it’s not the most realistic model of a planet but this whole argument went off the deep end about three decades ago.

steve fitzpatrick
January 14, 2012 2:56 pm

Tallbloke,
Please don’t behave like an idiot. You are completely wrong on the technical issues, and worse, wrong to ban someone from your blog who says that utter nonsense is in fact utter nonsense. Get a grip man, and stop embarrassing yourself.

David Coe
January 14, 2012 3:01 pm

Elevator explanation
A gas which absorbs no radiation also emits no radiation. It has an emissivity of zero
If such a gas were to envelope a planet it would absorb energy from the planet surface resulting in an increase in gas temperature (the kinetic energy of its molecules would increase).
Eventually the gas would reach an equilibrium condition whereby it’s temperature would equal that of the planet surface. Since the gas cannot radiate energy (zero emissivity) there would be no heat loss from the gas into space, no heat transfer through the gas and therefore no temperature gradient. The whole gas would be at the same temperature as the planet surface.
The radiative balance of the planet would be undisturbed by the presence (or absence) of the gas at equilibrium.
The temperature of the planet would not be affected by the gas.
This is simply a question of thermodynamics. Gravity has no part to play and does not influence the planet temperature.
You, sir, are thus vindicated.
David Coe

tallbloke
January 14, 2012 3:03 pm

Willis says:
[Tallbloke, first give us your elevator speech about Jelbring’s hypothesis. Until then, I will not believe you understand it well enough to “prove” anything. You have not shown that my proof has “failed” as you claim, record it on your website or not.

No Willis, before we can move on to a discussion of the science, you need to acknowledge that Hans Jelbring defined his model planet as one which does not radiate to space, and therefore your allegation that his model breaks the laws of thermodynamics because of radiation to space considerations fails. No-one needs to understand the rest of the paper to see this simple point of logic. So acknowledge it, apologise to Hans for blackguarding him for the last eight years, and then we can move on.

January 14, 2012 3:08 pm

Willis is considering a highly idealised case, but for that highly idealised case (perfectly IR transparent, and therefore non-radiating, atmosphere and uniform constant surface illumination, thermal equilbrium established) his argument is obviously correct.

Billy Ruff'n
January 14, 2012 3:15 pm

With all the snipping going on this layman is hesitant to step into the elevator, but….
A while back NoIdea said: “Willis is correct…if the atmosphere cannot absorb or emit IR, then the surface must emit thermal radiation at the same average rate as it absorbs solar radiation…”
Before that Dr. Spencer said pretty much the same thing.
But solar radiation is not the only source of heat on the planet — I can think of two others that have nothing to do with the sun: one is independent of the composition of the atmosphere (or even the existence of an atmosphere — the molten core. The other heat generated by the friction created between the atmosphere and the surface where the force necessary to generate the friction is rotational force, not movement of air masses.
I’ll retract my head from the debate and go watch Tebow and Brady duel with a pigskin.

G. Karst
January 14, 2012 3:17 pm

Leonard Weinstein says:
January 14, 2012 at 12:39 pm
G. Karst,
Your statement “All gases when energized by conduction, above ambient, will radiate energy.” is wrong.

Everything you have stated is correct (thank goodness), however it is NOT an absolute, across all conditions.
If we consider a gas that has reached the edge of space where rarefied to the density where conduction is unlikely, but continue to input energy, the gas will begin to radiate energy at the same rate as the input. Irregardless of the gas. It’s apparent temperature will increase until it does. Knowing what gas it is only tells us what part of the spectrum will dominate emissions. Otherwise non GHGs would be our ticket to infinite temperatures. Given a continuous energy input with no conduction output and you will soon have a gas with luminous characteristic qualities (plasma if necessary). Down here on the surface conduction, then convection, prevents this from happening. The lapse rate dictates it so… I think.
This is all off the top of my head. Not sure how satisfying it will be. Your comments are always high quality, and I don’t want to muddy them. GK

January 14, 2012 3:21 pm

Is this good enough to be an “elevator speech” I’ve tried to break it down but the “atmosphere effect” is a bit more complex than the “poorly-named” and laughable “greenhouse” theories.
The adequately-named “atmosphere effect” works as follows:
• Thermal conductive bodies on the earth emit energy in the form of longwave radiation.
• The mass of atmospheric gases have a constant thermal potential of absorbed longwave radiation.
• The sum of longwave radiation is absorbed until the thermal conductive potential of the atmosphere is reached.
•Once the sum of the thermal conductive potential of the atmosphere is reached longwave radiation is no longer absorbed, nor is it trapped by a potential thermal constant of an atmospheric gas.
• As a result, the potential of thermal Transient conduction above the earths surface is greater than it would be without an atmosphere.

Related references and wikipedia Links:
Wien’s displacement law: http://en.wikipedia.org/wiki/Wien%27s_displacement_law
Planck constant: http://en.wikipedia.org/wiki/Planck_constant
Fourier’s law http://en.wikipedia.org/wiki/Heat_conduction#Fourier.27s_law
Atmosphere definition: http://dictionary.reference.com/browse/atmosphere
Effect definition http://dictionary.reference.com/browse/effect

January 14, 2012 3:22 pm

Wow I have read all the comments in this post and now I am far more confused than when I started.For the next thought experiment can you use a planet with beer for an ocean and can it have small tropical islands and lots of hula dancers? It may not help with the science but it will keep my head from hurting,, well till I take a dip in the beer ocean.

gbaikie
January 14, 2012 3:24 pm

Richard M says:
January 14, 2012 at 11:11 am
“…. All the planets would warm up until they reached that maximum. If that maximum just happened to be determined by the mass and the gravitation of those planets then we would see exactly what Huffman sees. However, for the surface to warm above the SB calculations there must be an effective radiation altitude above that surface.
We don’t need to discard the GHE. It’s real, it just has limits that haven’t been discovered because of simplifications made to climate models.”
On Earth there is 390 parts per million of CO2 or .04% or 4/10,000th. And Venus has 92 of earth atmospheres, or 92 times 2500 more CO2 than earth. If CO2 added .01 watts per square meter
for earth, Venus could be 230,000 times .01 watts per square meter or 2300 watts per square meter. And having more atmosphere could mean watts keep in the atmosphere rather than make it to space. It could be that CO2 in the amount earth radiates .02 watts, and around half leave, whereas on Venus only say 1/10th of energy leaves.
It seems CO2 can add some heat if exposed to energy of certain frequencies of electromagnetic wavelengths.
Not that know anything about it, but maybe CO2 can absorb longwave IR and via collisions or other absorption of wavelength, emit the energy at a higher wavelength- thereby actual warm a surface which is warmer than the CO2 gas. Or the temperature CO2 gas has nothing to do with temperature it emits. The temperature of any gas is the velocity of the molecules- which has to do with a gas emissivity. There is connection though, because CO2 is suppose to absorbing a “low temperature” wavelength and of course it can emit the same wavelength- and if it doing that it can’t heat up something of same temperature. Nor can block longwave IR by any means other absorbing it.
Anyways it does seem to matter in regards to CO2 and heating earth- even if it heated by 10 watts per square meter, that is insignificant as in loss in the noise,

January 14, 2012 3:25 pm

Man, I am almost getting tired of coming to this web page any more. It seems the intelligence level of any person allowed to post is being quickly dropped to the point of mentally disabled.
The no GHG theory works like this:
• Energy from the sun heats the surface of the Earth.
• The surface of the earth emits energy in the form of thermal long-wave radiation and transfers heat to the air through conduction.
• The heated air transports the energy upwards into the atmosphere through convection. As the heated air rises it releases some of the energy to space through emission of long-wave radiation. (The idea that any gas is non infrared absorbing and emitting is a myth and nothing more, Both Oxygen2 and Nitrogen2 absorb and emit infrared radiation, we will ignore this for the moment.)
• This raises the point where the 1:1 ratio of incoming and outgoing radiation exists off the physical rocky surface of the Earth and with our current atmosphere that point is about 5km off the ground.
• As a result of the atmosphere moving the point of radiation balance, the lapse rate comes into effect and causes the added warmth at ground level. (The lapse rate it determined by the mass of the atmosphere.)

Editor
January 14, 2012 3:25 pm

I’m late to this party, and maybe this has already been said (285 comments, whew), but so long as we are only considering radiative energy transfer, nothing in Willis’ argument says that the ATMOSPHERE can’t be heated by gravity. As long as that heat doesn’t radiate into space, or irradiate the planet surface (Willis’ “transparent, GHG-free atmosphere”), it doesn’t affect the SB equilibrium temperature.
So at least that narrows where to look for how to resolve the seeming conflict between ideal gas law SB equilibrium: the conflict only arises when we start taking kinetic energy transfer into account. Does that help?

January 14, 2012 3:27 pm

I crossposted my comment to Tallbloke’s Talkshop. So far it is still awaiting moderation. Two later comments have made it through.

darkobutina
January 14, 2012 3:30 pm

Willis,
The way you argue your case, or rather lack of it, you would fit really well with the RealClimate. Why dont you introduce new NULL hypothesis: “I am always right and whoever doesn’t like it they can go elsewhere”.
db-uk

January 14, 2012 3:32 pm

My comment is now posted. Thanks Roger.

Bebben
January 14, 2012 3:32 pm

“Because either you can tell us, simply and clearly, how your “Jelbring effect” is supposed to work, or you are worse than useless and should just audit the discussion because you have nothing to add.”
This is a sad state of affairs. I hate to say this, but there’s a need of a moderator to moderate the moderator.
This debate seems to be over.
Time-out.

Dan in Nevada
January 14, 2012 3:36 pm

Willis,
Your whole argument is pointless. Nikolov & Zeller argued two points:
1) the currently accepted understanding of how to apply physics to a planetary body’s surface temperature is wrong. (i.e. the laws of thermodynamics, Stefan-Boltzman, etc. are correct, they are just being misapplied)
2) based on the (in their view) correct application of physics, they claim to be able to explain what governs a planetary body’s surface temperature.
To clarify, you have to accept (1) in order to understand (2), if they are correct.
What you, Joel Shore, and others keep saying is “based on the currently accepted understanding of how to apply physics to a planetary body’s surface temperature, N&Z’s (2) is clearly wrong” (paraphrasing). Well, yes, I think even they would agree to that. So what?
The only important argument (so far) is whether “the currently accepted understanding of how to apply physics to a planetary body’s surface temperature” is correct or not. So far, all they have come up with is an unsubstantiated and unsupported assertion that it is not. Unless and until they can make their case, the status quo “wins” by default. You don’t have to do anything at all. But it’s clearly a waste of time to argue something that even they would agree with.

gbaikie
January 14, 2012 3:38 pm

“The temperature of the planet would not be affected by the gas.
This is simply a question of thermodynamics. Gravity has no part to play and does not influence the planet temperature.
You, sir, are thus vindicated.”
The problem with that is you assume that earth in sunlight is absorbing the same amount energy regardless of the earth’s surface temperature.
It doesn’t.
Cold pavement absorbs more energy than hot pavement.
The Non GHE gas is like a battery. It doesn’t add energy but it stores it.
On top surfaces the gas takes the energy away- it recharges. If gas did not do this, the surface would warm more and radiate more energy.
The gas is taking energy which would have otherwise be radiated into space.

Steve from Rockwood
January 14, 2012 3:41 pm

If gravity is so important then why is the density of the atmosphere not proportional to the mass of the planet?

George E. Smith;
January 14, 2012 3:41 pm

“”””” Stephen Wilde says:
January 14, 2012 at 1:36 pm
George E Smith said:
“furthermore, the heating that results because all of that work done ends up as waste heat, is a purely transient event, ”
Hi George,
I’m a fan of your work but you’ve missed something there.
It isn’t called a gravitational CONSTANT without good reason. Joel Shore and others make the same mistake.
Gravity is a continuous renewing process which replenishes itself over time. No one knows why or how, not even Einstein, but we must live with it.
So it is NOT a transient effect and the result is PERMANENT all other things being equal.
Without that effect the Ideal Gas Law would not be a Law. “””””
Well Stephen,
I’m not really looking for fans; just the truth and reality.
So I’ll just take my apparatus up to the space station and do the experiment up there in ZERO GRAVITY.
So my gas will still warm up when I compress it, and this time I don’t even need my legal disclaimer about neglecting the weight of the gas, since it is essentially zero; well it is at least orders of magnitude less than the weight was on earth.
So the gas heats by the amount of work I do on it
Just think of a part of a Carnot cycle experiment. But I stop everything with the volume reduced, the quantity of gas unchanged, the pressure and density higher, and the Temperature now slightly elevated above the ambient. The excess “heat” will now conduct through the walls of my apparatus to the surroundings, and the Temperature will come back down to the ambient equilibrium I had before I changed the volume. The amount of gas will still be the same, the volume will still be the same (I made the container out of Invar), so the density will still be the same; but the pressure will settleat a slightly lower pressure in accordance with the gas law. The final Temperature after about five times the thermal time constant will be within 1% of the original Temperature; as I said a transient event..
And the graviational constant (of Einstein ?) is irrelevent to the problem.
And Joel does make some misteaks; but I seriously doubt that he makes that one; he is quite careful.

tallbloke
January 14, 2012 3:42 pm

Unfortunately the link where people can actually see what Hans Jelbring said in his 2003 paper was lost in the carnage of Willis’ censorship spree. Here it is again:
[SNIP: No, you cannot use my thread to drive traffic to your site. Nice try though. -.w]

January 14, 2012 3:43 pm

THANKS WIllis, for setting a great example for how scientifically-oriented Global Warming skeptics should react to a new theory. The N&Z theory reaches conclusions that are favorable to our negative view of the official climate Team hysteria. However, N&Z appear to violate the bedrock science principle of conservation of energy.
Perhaps Ned Nikolov, when he posts the N&Z “reply paper” here at WUWT, will make it clear just what their theory encompasses, and how it avoids violating conservation of energy.
In a comment on my “Unified Climate Theory May Confuse Cause and Effect” thread, I suggest some scientifically sound ways to calculate a warming effect that exceeds the 33K of the conventional accounting.
Until I was forced to think about it by the N&Z theory threads, I did not fully realize the implications of the fact that the 33K conventional number for the Atmospheric “greenhouse” effect (GHE) compares our actual Earth with a very strange imaginary Earth. It must not only have a GHG-free atmosphere of the same mass, but that atmosphere must lack not only CO2 but must also be free of water vapor. So, that imagined Earth would have to be ocean-free as well. Lacking clouds and ice, it would also have to be painted to achieve an albedo of 0.3, equal to that of the actual Earth. Quite an artificial construct! But, it does get us to a mean surface temperature of (no more than the S-B limit) 255K which is 33K less than the measured mean Surface temperature of about 288K.
I was also inclined to accept that the mean temperature of our Moon is around 250K, as published in some mainline sources. That now seems to be on the high side, but I do not think the Moon is as cold as the 155K apparently required by the N&Z theory.
So, even if the N&Z theory turns out to be misguided, it has raised my awareness and understanding, at least a bit.
Let us consider how could we get a number greater than the conventional 33K warming effect of a GHG-Atmosphere? Well, here are two possibilities:
(1) As you state in your initial posting, any atmosphere, even without GHGs, distributes surface heat energy more evenly. The denser the atmosphere the greater the uniformity. That will increase the mean temperature of a planet above the mean of a similar planet that has no atmosphere. This effect, as you correctly state, is good only up to the S-B temperature associated with the mean incoming radiation from the sun.
(2) A planet that already has an atmosphere with GHGs could warm further if we dump a big load of non-GHGs into that atmosphere. Even after equilibriation, the additional pressure/density may result in warming due to the pressure broadening of CO2 and other GHG spectral lines, but again only up to the S-B limit.
So, if one starts with a barren Earth, with no water and no atmosphere, and adds an Atmosphere with both non-GHG gases (e.g., N2) and GHG gases (e.g., CO2), plus oceans full of water to produce the most effective GHG (water vapor), the mean Surface temperatures will be increased by more than the 33K of the conventional accounting.
However, despite the efforts of those who commented on the previous two N&Z theory threads here at WUWT, I cannot see how that additional warming could possibly lead to 133K.
– Ira
PS: As Pops says (January 13, 2012 at 11:17 pm} “One tiny nit, Willis”. Pops is correct that you described night vision goggles (NVG) incorrectly when you wrote:

… A curious fact is that almost everything around us is continually radiating energy in the infrared frequencies. You, me, the trees, the ocean, clouds, ice, all the common stuff gives off infrared radiation. That’s how night-vision goggles work, they let you see in the infrared. …

Yes, NVGs do “let you see in the infrared” but it is in the NEAR infrared, just a bit past the visual range (about 0.7 to 0.9 microns). NVGs do not rely on the radiation given off by everything around us. They are merely image-intensifiers that multiply the reflected moonlight or starlight by tens of thousands of times. NVGs do not work when there is no light, such as with an overcast sky at some distance from populated areas. (While working on avionics systems for Special Forces helicopters, I was invited along on a couple of night-time, low-altitude training missions and had the opportunity to use NVGs.)
As Pops also noted, there are sensor devices called Forward Looking Infrared (FLIR) that work in the FAR infrared (centered around the 10 micron peak blackbody emission of the Earth, where there happens to be a good atmospheric window). FLIRs do depend upon everything continually radiating energy in the infrared. (A highlight of FLIR imagery is seeing cows glow in the dark, because they emit more far infrared than the grass below them. Cow flops glow as well.)

ferd berple
January 14, 2012 3:45 pm

Willis Eschenbach says:
January 14, 2012 at 3:18 pm
Ferd, you claimed that the atmosphere in my example in the head post would be warmer than the surface.
What I said was “for example at the poles”.

gbaikie
January 14, 2012 3:52 pm

“BUT … in willis’ experiment, isn’t the only way he could have a completely (100%) transparent atmosphere would be if that atmosphere had no mass? Others have pointed out that any gas, no matter what it is comprised of, has at least SOME ability to store and radiate? ”
It’s not radiating- it’s convection. Energy is transferred by conduction, convection and radiation- that’s all folks. At room temperature or earth’s temperature heat is best tranfered via conduction. Next, generally is convection, and lastly radiation.
Or there is nothing stopping anyone from transporting energy via light- or micowave, but wires generally easier- small wires can conduct a lot of energy. Of course wires can annoying, and one can transfer energy in different ways.
So the atmosphere is 5.1 x 10^18 kg and the gas molecules are traveling at around 1000 mph.
The energy is 5.1 x 10^18 kg of mass traveling at 1000 mph.

Fred Souder
January 14, 2012 3:54 pm

People keep saying that if you double the amount of gas in the atmosphere, you will keep the same temperature profile because the lapse rate is -g/Cp. These same people keep saying that the temperature of the surface will still be the same after the doubling of mass.
I keep checking this, and maybe my math is wrong, but after a few doublings you will get a temperature below absolute zero for the top of the atmosphere. Seems unlikely. Either the -g/Cp does not apply (unlikely), my math is bad (pretty easy exercise, but somewhat likely), or the assumption that the temperature at the surface will stay the same is invalid.

gbaikie
January 14, 2012 4:01 pm

pochas says:
January 14, 2012 at 2:44 pm
“I feel compelled to add to the confusion.
It doesn’t matter whether the atmosphere is transparent or opaque. If the atmosphere is transparent the surface will be at the radiating temperature and the atmosphere will get cooler as you ascend. If the atmosphere is opaque the top of the atmosphere will be at the radiating temperature and it will get warmer as you descend.
How does the transparent atmosphere develop the adiabatic lapse rate? ”
The gas doesn’t cool. The atmosphere is colder. But the velocity of the gas remains roughly the same. The velocity of gas molecules is the gas temperature. Stop gas velocity, get absolute zero, speed up gas to some fraction of lightspeed and if they gas are interacting with thing, the gas is hot. On earth average speed of gas molecule is about 500 m/s.
What changes as you go to higher elevation is the gas become less dense. Gas molecules traveling same speed but lower density have less ability to warm things like thermometers.

LazyTeenager
January 14, 2012 4:11 pm

And because the atmosphere is transparent, this means that the planet is radiating to space more energy than it receives. This is an obvious violation of conservation of energy, so any theories proposing such a warming must be incorrect.
——————-
Thanks Willis, this works for me.

tallbloke
January 14, 2012 4:13 pm

QUOTE MY WORDS. I have no recollection of saying anything of the sort, I don’t know the context, I don’t know if you understood me, I haven’t a clue what you are referring to. STOP THIS VAGUE ACCUSATORY BULLSHIT AND QUOTE MY WORDS.
I can imagine Hans Jelbring is probably thinking similar thoughts so if you don’t want to have the link to his paper I provided on your thread because you think I’m placing it to drive traffic to my site then copy his paper and upload it here and link it in the headline post so people can judge what he is saying for themselves instead of having to trust your VAGUE ACCUSATORY BULLSHIT
Thank you.
TB.

willb
January 14, 2012 4:17 pm

@Willis Eschenbach
One thing puzzles me with regard to your elevator speech about the greenhouse effect: you don’t include or even acknowledge the role of the lapse rate. My understanding is that it is fundamental to the greenhouse effect and just as important to the GHE as the presence of greenhouse gases. Say for example that the lapse rate is zero. Doesn’t this imply there will be no greenhouse effect regardless of the presence of vast quantities of greenhouse gases in the atmosphere? If the lapse rate is zero, I believe the first three bullets of your elevator speech will still be valid, but your fourth bullet, the conclusion, would be incorrect.
(I have seen arguments that a lapse rate of zero is possible, but I personally am not persuaded.)

Bill Hunter
January 14, 2012 4:19 pm

David Coe says:
January 14, 2012 at 3:01 pm
“Elevator explanation”
If it were true that a GHG-free atmosphere would find an equilibrium at the average temperature of the surface, why can I tomorrow go out with a few hundred dollars of materials build a solar water heating system that the water in it is higher 24/7 than the average surface temperature of the location I build it at?
This is without any power running strictly on solar energy and convection. Its somewhat a cinch to achieve 10degC even with a storage vessel that is only modestly insulated. With the only means of cooling the upper atmosphere being the wind and conduction with the surface its not going to cool the upper atmosphere at all and equilibrium is really somewhere between the maximum temperature of the surface and the average temperature. I don’t think we can explain all the greenhouse effect with that, but where I have a problem is in the perceived need to have one mechanism explain everything.

LazyTeenager
January 14, 2012 4:20 pm

Michael Hammer says
Willis, I agree with amonst everything you said with one exception. I question the emissivity of fresh snow. Yes cetainly water and ice have IR emissivities very close to 1 but fresh snow is a mixture of ice and air with each “layer” or crystal a few to a few 10′s of microns thick.
———–
Yes Michael, but I think you need to consider the situation at visible and ir wavelengths is different. At visible wavelengths the Water in snow is transparent. At ir wavelengths it is essentially “black”.
As for igloos a comparison needs to be made between the insulatng effect due to low thermal conductivity of snow and the ability of snow to absorb IR. I am guessing the first predominates.

PaulR
January 14, 2012 4:20 pm

This thread should be closed. Willis Eschenbach is driven to exasperation now, writing “God damn it” and “Is stupidity contagious?” Willis is right in his approach in the original post but if he has reached the limit of his patient and polite endurance then it is time to close up shop for the day. Some people just can’t be reasoned with, but being unpleasant at them won’t work either.

ferd berple
January 14, 2012 4:25 pm

ferd berple says:
January 14, 2012 at 2:43 pm
Willis Eschenbach says:
January 14, 2012 at 2:03 pm
That’s why the surface can get hot, because some of the energy radiated by the surface is absorbed by the atmosphere so we’re not emitting to space more that we’re absorbing. As a result, the last step of your proof is incorrect.
I disagree with Willis. I believe he is double counting. To answer Willis more completely, here is the GHG model I described,
space (==A== surface (==B==) ghg ==C==) space
I believe Willis is saying that because the surface radiates both through A and B, so lets replace the net flow B with H as follows:
space (==A== surface ==H==) ghg ==C==) space
Flow C takes part of its energy from the surface to ghg net of flow B, plus any energy absorbed directly from the sun by the GHG. Thus we can say that:
H + solar absorbed by GHG = C
Since solar absorbed by GHG > 0, then we can say than in all cases with a GHG atmosphere, that:
(result a) H A + H
D > A + H
Since D and (A+H) vary as 4th power of Temp
Temp(D) > Temp(A+H)
Therefore the surface will be hotter on a planet with a non radiant (non GHG ) atmosphere.
QED

PaulR
January 14, 2012 4:26 pm

Paul, I don’t follow this. Why are you stipulating a simplified model of earth that doesn’t receive solar radiation or emit IR? How could it not emit IR? What does this have to do with my statement? Color me confused.
Hans Jellbring stipulates that in his gravity-greenhouse model, not you. The distinction you made between transient versus equilibrium temperatures (with a graphic) was in a different thread.

tallbloke
January 14, 2012 4:26 pm

Willis opines:
Tallbloke goes on and on about the N&Z and the Jelbring theories, but he can’t summarize either one … I find that significant.

More innuendo. As you know full well from the email I sent you yesterday, I am awaiting the response to comments paper from N&Z before trying to summarize their work. Why are you misleading people like this? It reflects very poorly on you.
By the way, when will you be snipping Roy Spencer’s reply for not being an elevator speech?

ferd berple
January 14, 2012 4:29 pm

aargh – my post again got trashed by html mistaking LT for the start of an html block

gbaikie
January 14, 2012 4:29 pm

“The most obvious way of transporting “heat” is to simply transport the physical mass of particles that comprise the material under discussion. That process is called CONVECTION and is the main process by which “heat” is transported from a car engine to the radiator, by pumping hot water from the heat source (engine) to the heat sink (radiator).
The other process for transporting heat from one body of material to another, is to place them into contact, so that the particles of one body can collide with and exchange energy with the particles of the other body,without the particles themselves being largely moved from their positions.
That process is called CONDUCTION, and is the mechanism by which the “heat” contained in the “hot” engine block is conveyed to the cooling water, that will then carry it to the radiator by convection.”
Ok. good description
“There is NO way to transport “heat” in the total absence of a physical material that has a rest mass. Furthermore the concept of Temperature has NO meaning, in the absence of materials with mass (real matter).
In particular Electromagnetic Radiation is NOT “heat” nor is it a means of transporting “heat”.
It IS a mechanism for transferring ENERGY; just not HEAT ENERGY.
Electromagnetic Radiation ENERGY can be converted to “HEAT” energy, when it is absorbed in SOME REAL PHYSICAL MATERIAL HAVING MASS, that is unable to convert that energy to ANY OTHER form of energy, such as ELECTRICITY, for example, or SOUND (ACCOUSTIC) energy.
For example, so far as we know, ordinary amounts of electromagnetic radiation energy having ANY frequency or wavelength if absorbed by water, do not result in either electricity, or sound forms of energy; essentially THE MAJORITY of the EM radiation energy absorbed by water is converted to waste “heat”. The only significant exception is when certain RESTRICTED frequencies or wavelengthsof EM radiation energy are re-emitted from the water, as molecular resonance radiation spectral lines or bands.
We get oodles of electromagnetic radiation energy from the sun; we get essentially ZERO “HEAT” from the sun, since there is no real physical material medium to transport any heat from sun to earth.”
Hmm. Energy is transferred, by conduction, convection, and radiation.
You are saying heat is transferred by conduction, convection but not radiation.
Now I would use an analogy that energy of the sun is sort of like room temperature.
In that the distance from the sun is the sun heat. The sun 150 million kilometers closer is 5000 something K, at earth distance it is determined by S-B.
At earth distance the sun is about 120 C [390 K]. As in 390 *390 *390 *390 times .0000000567
being 1311 watts per square meter of solar flux.
What is wrong with thinking of the Sun at earth distance is about 390 K?
And follows thermodynamic laws. The sun can’t warm above 390 K?
I realize it’s not exact. The sun can make plant grow and one have forest fire at 2000 K.
But wrong with using this as approximation- generally the Sun is 390 K at earth distance?

Vergent
January 14, 2012 4:30 pm

“The Stefan-Boltzmann equation specifies how much radiation is emitted at a given temperature. It states that the radiation increases much faster than the temperature. It turns out that radiation is proportional to absolute temperature to the fourth power.”
If we are talking about a rotating planet like the moon. It has a hot side and a cold side. At noon it would be cooler say 10C because of the larger thermal mass, and therefore radiating less. At midnight it would have >>10C warmer to radiate the same energy. The average temperature would have to rise to balance the radiation.
QED

ferd berple
January 14, 2012 4:32 pm

Here is another try at posting html
ferd berple says:
January 14, 2012 at 2:43 pm
Willis Eschenbach says:
January 14, 2012 at 2:03 pm
That’s why the surface can get hot, because some of the energy radiated by the surface is absorbed by the atmosphere so
we’re not emitting to space more that we’re absorbing. As a result, the last step of your proof is incorrect.
I disagree with Willis. I believe he is double counting. To answer Willis more completely, here is the GHG model I
described,
space (==A== surface (==B==) ghg ==C==) space
I believe Willis is saying that because the surface radiates both through A and B, so lets replace the net flow B with H as
follows:
space (==A== surface ==H==) ghg ==C==) space
Flow C takes part of its energy from the surface to ghg net of flow B, plus any energy absorbed directly from the sun by the
GHG. Thus we can say that:
H + solar absorbed by GHG = C
Since solar absorbed by GHG &gt 0, then we can say than in all cases with a GHG atmosphere, that:
(result a) H &lt C
Here is our other model, the non GHG atmosphere, that does not radiate:
space (==D== surface (==E==) no ghg ==F==) zero radiation to space
D + F = solar energy in = radiation out to space
However, since F = 0, this becomes
D + 0 = solar energy in = radiation out to space
Therefore from above:
A + C = solar energy in = radiation out to space
and
D + 0 = solar energy in = radiation out to space
Therefore we can say
A + C = D
and from (result a) above
D = A + C &gt A + H
D &gt A + H
Since D and (A+H) vary as 4th power of Temp
Temp(D) &gt Temp(A+H)
Therefore the surface will be hotter on a planet with a non radiant (non GHG ) atmosphere.
QED

ferd berple
January 14, 2012 4:33 pm

does anyone have a solution so that html doesn’t see the LT and GT characters as html commands?
[Try: & l t ; and: & g t ; (eliminate the spaces between the ampersand, the letters, and the semicolon. Test it on the Test page. ~dbs, mod.]

tallbloke
January 14, 2012 4:33 pm

Willis sez:
I have no responsibility and no desire to spread Jelbrings ascientific BS around

Your discourse is all circumference and no centre.

Ed Fix
January 14, 2012 4:37 pm

Willis, I believe your proof contains a couple of fundamental errors.
I haven’t read the Nikolov and Zeller or Jelbring papers in any great detail, and I haven’t read the comments here, so I don’t know what others have already posted. However, I do understand a bit about S-B radiation, as well as gravitational heating, starting from first principles. So here’s my first draft of an “elevator speech” about gravitational heating and S-B radiation of an atmosphere. I haven’t spent a lot of time refining it, so at this point, the elevator would have to be in a tall building.
I’ll start by stating a couple of assumptions. First, by non-GHG atmosphere, I mean an atmosphere that is perfectly transparent to electromagnetic radiation at all wavelengths. Second, we need to decouple heat and temperature. One way people tend to confuse themselves is by thinking of heat and temperature as the same thing. They’re not. Heat is the kinetic energy of the molecules within a body–either their average velocity in a fluid, or vibrational amplitude in a solid. Temperature is not a measure of energy by itself, just as voltage alone is not a measure of electrical power. It’s a measure of the intensity or concentration of the heat energy within the body. A large body containing a given amount of heat energy has a lower temperature than a smaller body containing the same amount of heat energy.
Now to your errors:
1. “But when the temperature of a perfect blackbody planet rises … the surface radiation of that planet must rise as well.”
No, that’s incorrect. As the temperature of a S-B radiating body rises, the peak of its radiation spectrum shifts to shorter wavelengths. This says nothing about the total energy radiated. You are conflating heat with temperature. The energy emitted by the filament of a 25 watt incandescent bulb at 6000K (if you could build such a thing) would be MUCH less than that emitted by the sun, but have the same spectrum. And if you could build a 25 watt bulb with an 8000K filament (a materials problem, not a theoretical one) it would have a higher temperature S-B spectrum, but no increase in total energy emission. So, yes. It is possible to emit lower energy at the same temperature, or the same energy at higher temperature.
2. You have ignored the fact that even a non-GHG atmosphere is itself a S-B radiating body. Even absent the property of intercepting outgoing longwave radiation, it gains heat energy by conduction, through contact with the planet’s surface. The atmosphere then re-radiates that energy through S-B radiation, but some of it goes back to the surface to be re-absorbed, converted back into heat until it’s transferred to an impinging air molecule again (speeding it up a bit), or radiated as S-B radiation. The overall effect is to slow the radiation of the earth’s heat into space (even without the greenhouse effect), and that necessarily raises the temperature of the atmosphere and the surface. It doesn’t, however, increase the amount of energy radiated–just changes the temperature and spectrum. The S-B radiation of an amorphous body of gas is how we can measure the temperature of gas clouds in interstellar space.
As for gravitational heating, here’s a thought experiment. Imagine a transparent tube in interstellar space (no gravity), 200 km long, one square meter cross sectional area. Fill this tube with some amount of an ideal gas at 0K. That is, each molecule of gas in the tube is motionless. Without gravity, everything remains motionless WRT the tube.
Now instantaneously put an earth-size planet in contact with one end of the tube. All the gas begins to accelerate toward the planet (now the bottom) end of the tube. There will be collisions between gas molecules. That means pressure must increase at the bottom as it decreases at the top. The ideal gas law PV=nRT tells us that with increasing pressure the temperature of the gas will rise. This increased temperature causes the gas to begin emitting S-B radiation.
If no extra energy is imparted to the gas from the surface of the planet, the gas will eventually all settle to the bottom end of the tube, back at 0K. However, the gas’ temperature did rise, and EM energy was emitted. The planet doesn’t have any less gravity than it did before, so where did the energy come from? I don’t know. If you want to say that violates conservation of energy, I can’t dispute it. Maybe if we really knew what gravity is, we could answer the question.
However, if we continuously add 240 W at the bottom of the tube, the gas will reach some equilibrium. When a falling air molecule collides with the bottom, and gets a little extra kick as it rebounds up (conduction heating) gravity still continuously accelerates it back down, therefore continuously keeps the temperature elevated. The fact that energy radiated must equal insolation doesn’t mean the temperature near the ground can’t be above the S-B temperature.
Gravitational heating is real. Does it violate conservation of energy? I dunno.

LazyTeenager
January 14, 2012 4:38 pm

Anythingisposible says
Higher surface pressures elevate surface temperatures by suppressing convection because the greater the weight of the overlying atmosphere, the more energy (heat) is required to enable the process to get under way.
—————-
Nup. There is no indication that higher pressures suppress convection. Convection is driven by differences in buoacy due to temperature differences. That’s all.
The counter examples are easy. Water when heated convects even though it has a much higher density that air. Water convects even at extremely high pressures; witness the behavior of water near deep sea volcanic vents. The atmosphere of Venus convects and thereby establishes a temperature profile defined by the adiabatic lapse rate. The atmosphere of jupiter convects. The horizontal version of this can be seen with a telescope.
The vertical temperature profile of the earths atmosphere is determined by convection. Therefore convection does happen.

tallbloke
January 14, 2012 4:40 pm

Hans Jelbring’s peer reviewed 2003 E&E paper:
The “Greenhouse Effect” as a Function of Atmospheric Mass
http://ruby.fgcu.edu/courses/twimberley/EnviroPhilo/FunctionOfMass.pdf

LazyTeenager
January 14, 2012 4:42 pm

beaker says:
January 13, 2012 at 10:42 pm
Just a small correction. Night vision uses a silicon detector and amplifies the available ambient light. Silicon is not responsive in the MW or LWIR, 3-5um and 8-12um, the naturally occuring atmospheric IR windows where us radiometric guys make our measurments. A FLIR or Forward Looking Infrared “see’s” thermal differentials without visible light. I believe you are referring to a FLIR, not night vision.
————
Almost.
There are 2 kinds of night vision. There is a kind which depends on amplifying existing night light levels and another kind which depends on converting themal IR into visible light.

gbaikie
January 14, 2012 4:45 pm

“I realize it’s not exact. The sun can make plant grow and one have forest fire at 2000 K.
But wrong with using this as approximation- generally the Sun is 390 K at earth distance?”
Or re-phrase this. The Sun has Planck curve [or it roughly follows Planck curve]. When objects at distance from the sun are radiating a similarly close approximation of a Planck curve. Is this not heat being radiated from one object to another?

Editor
January 14, 2012 4:51 pm

Willis wrote:[SNIP: No, you cannot use my thread to drive traffic to your site. Nice try though. -.w]If this were Willis’ site, I would have no problem with this response to Tallbloke’s censorship of his views at Tallbloke’s Talkship Turnabout is fair play, as the saying goes. But this is NOT Willis’ personal blog and tit for tat is not a principled response to censorship. Two wrongs do not make a right, and if Willis is going to moderate comments on his own post, I think he has an obligation to be especially non-censorious.
Anthony’s usual policy is non-censorship of any on-topic civil (or moderately uncivil) discussion. There is nothing the least bit uncivil in Tallbloke posting a link to Hans Jelbring’s paper, and the fact that it is hosted at Talbloke’s website certainly does not make it uncivil.
I, for one, would like to have a link to Hans Jelbring’s paper available. For all who agree, Tallbloke’s posting is here:
[SNIP]
Will Willis censor me too? Nothing against Willis. Great post, by the way. But the censorship standards of Anthony’s blog should not be arbitrarily altered.

Nullius in Verba
January 14, 2012 4:58 pm

Entertaining thread, and interesting challenge.
OK, this first elevator speech may seem off topic at first, but it is relevant.
1. Shallow pool of water, transparent to visible, absorbs IR in 20 microns.
2. Heat comes in, hits bottom, radiated upwards.
3. Water absorbs and re-radiates up and down.
4. Energy radiated downwards absorbed by bottom. Warmer than it would be without water.
5. Every 20 micron thick layer does the same. Pond explodes.
There are other ways of doing this, I just happen to like this one.
[SNIP: I’m glad you like it, but since it has nothing to do with the subject of the thread, I’ve snipped it. w.]

LazyTeenager
January 14, 2012 5:00 pm

Berényi Péter says:
January 13, 2012 at 10:52 pm
How would Earth’s surface temperature change if atmospheric pressure were doubled, that is, increased to 2 atm by adding more N₂ (and nothing else)?
————-
There would be no change. If there was no GHG at all.
The only route for energy to escape the earth with no GHG present is by surface thermal IR emission.
The energy budget must balance, so solar energy in must equal thermal energy out.
The rate of surface thermal emission is determined by just one thing: it’s temperature.
So adding more non-IR gas makes no difference.
————–
If you did have some GHG to start off with it is not clear to me how things would turn out. The vertical extent of the atmosphere would be higher, the GHG would also extend to a higher altitude but have a lower partial pressure at those altitudes and so on. In other words there are a bunch of countervailing factors that would need to be considered. There might even be no change.

January 14, 2012 5:06 pm

If as you claim the atmosphere is above the S-B temperature and the surface is still at S-B temperature,
Nope. The surface normal to the sun is at a higher temperature than the atmosphere and thus through entirely mechanical means the atmosphere is heated.
At night the temperature of the surface is lower than the atmosphere through radiation from the surface. So now the surface is at a lower temperature than the atmosphere. The atmosphere is convecting at the same time but lets keep it simple. The atmosphere now is mechanically transferring energy to the surface, raising its temperature and thus the surface radiates the heat.
Heat only flows from warmer to cooler surfaces and this is nothing more than conductive, mechanical heat flow between dissimilar materials, 100% within the laws of thermodynamics.

tallbloke
January 14, 2012 5:07 pm

Willis sez:
That’s it? You’ve run out of real points

I didn’t run out of them so much as have them deleted by your censor scissors.
[BULL: I quoted your whole entire post, that was all of it, uncensored. w.]

steve fitzpatrick
January 14, 2012 5:09 pm

Willis,
Some people simply can’t understand the concept. That is why I long ago gave up trying to explain.
Tallbloke,
Please get a grip man, you are embarrassing yourself. You are completely mistaken, and only digging an ever deeper hole. Take a deep breath and really think about Willis’ thought experiment; you owe it to yourself to figure this one out.

Tim Folkerts
January 14, 2012 5:13 pm

I skipped a few of the posts toward the end, but I figured out the “elevator speech” for the Jelbring Effect.
STEP 1: Instead of a greenhouse gas like CO2 at the top of the atmosphere (TOA) that radiates at some IR wavelengths and is transparent at other IR wavelengths, use a greenhouse gas at the TOA that radiates at ALL IR wavelengths (ie is “black” to IR = emissivty = 1).
Or in his words “[The model planet globe] (G) and the atmosphere (AT) are surrounded by a concentric, tight, black spherical shell. “
STEP 2: Apply all the standard physics to the rest of the analysis.

A quick read-through makes me think that his discussion of STEP 2 is right (or close to right). The lapse rate will cause the temperatures to increase below the TOA due to gravity, independent (to a considerable extent) of greenhouse gases. His “shell” at the TOA is taking the place of the GHGs at the TOA in terms of radiative effects.
The “Jelbring effect” is simply postulating something besides CO2 to radiate from the TOA. The shell wouldn’t have to be “black” (emissivity = 1), but the emissivity would have to be definitely greater than 0. Without the shell, the temperature enhancement would disappear (ie an atmosphere without a “lid” and without GHGs, which puts the TOA at ground level).
The “Jelbring effect” should actually work pretty well for Venus, where a permanent cloud layer acts as the “black shell”, since the cloud should be pretty close to “black” for IR light. The fact that the clouds are far from black for visible light would reduce the “Jelbring effect” but not eliminate it
————————————————————————
PS I have concluded that the ability to allow visible light thru is only a part of the “greenhouse effect”. Focusing too much on this aspect can obscure other important aspects, like lapse rate and IR within the atmosphere — but that would take an entire post to explain the interconnections (and some time beforehand to figure it out and figure out how to explain it). So his postulate that the surface is “black” to visible light and not just to IR light would make some difference, but would not prevent the surface from being warmer than the shell at the TOA.

January 14, 2012 5:13 pm

heat will be constantly flowing 24/7 from the warmer atmosphere to the cooler surface … and that violates conservation of energy. -w.
Our assumption is that the atmosphere has zero radiative absorption. Solar radiation at 10,000 kelvin is striking the surface of the Earth at an energy of 1366 w/m2 heating that surface. That surface is now warmer than the atmosphere and energy flows through conduction to the atmosphere through mechanical means. In conduction heat ALWAYS flows from the warmer system to the cooler system.
At night the warmer atmosphere transfers energy through conduction to the ground where it is then radiated to space.
You are getting too hung up on radiative transfer when it is not the only means of exchanging energy.

P Wilson
January 14, 2012 5:14 pm

try a cup of tea at 80C instead. Leave it 1 hour it cools to 15C.
This infers the body of liquid cools because it is emitting radiation
This is the point of contention:
The radiation from the body of liquid doesn’t simply go into the atmosphere and increase its temperature. It thermalises with its surrounding atmosphere (15C). The prior radiation is no longer a form of heat. Neither does that radiation go into the floor boards through gravity. It disappears as radiative energy.
There is a tendency to regard radiation/heat as a permanent factor, probably due to this erroneous S-Bolzmann equation, and its assumptions, which is a prize piece of thought experiment absurdity, yet stubbornly accepted as a gospel truth thrown from heaven.

Dan in Nevada
January 14, 2012 5:15 pm

Ed Fix says:
January 14, 2012 at 4:37 pm
THANK YOU, Ed. On this topic (celestial body temperature), the insistence here on an equivalence between temperature and power has really had me bugged. I’ve begun wondering why thermometers don’t come marked in units of watts/mtr^2. My intuition and the dim memories of my thermodynamics courses 35 years ago tell me it’s much more indirect than that. Thanks for a lucid explanation.
Dan

ferd berple
January 14, 2012 5:17 pm

ferd berple says:
January 14, 2012 at 2:43 pm
Willis Eschenbach says:
January 14, 2012 at 2:03 pm
That’s why the surface can get hot, because some of the energy radiated by the surface is absorbed by the atmosphere so we’re not emitting to space more that we’re absorbing. As a result, the last step of your proof is incorrect.
To answer Willis more completely, here is the GHG model I described,
space <==A== surface <==B==> ghg ==C==> space
I believe Willis is saying that because the surface radiates both through A and B, this allows the surface temp in the GHG model to increase.
I agree with Willis, so lets replace the net flow B with H as follows to see what happens:
space <==A== surface ==H==> ghg ==C==> space
Flow C takes part of its energy from the flow from the surface to ghg (flow H), plus the net energy absorbed directly from the sun by the GHG that re-radiates as C. Thus we can say that:
H + net solar absorbed by GHG reradiated as C = C
Since net solar absorbed by GHG > 0, then we can say than in all cases with a GHG atmosphere, that:
(result a) H < C
Here is our other model, the non GHG atmosphere, that does not radiate:
space <==D== surface <==E==> no ghg ==F==> zero radiation to space
D + F = solar energy in = radiation out to space
However, since F = 0, this becomes
D + 0 = solar energy in = radiation out to space
and from above:
A + C = solar energy in = radiation out to space
Therefore we can say
A + C = D
and from (result a) above
D = A + C > A + H
therefore
D > A + H
Since D and (A+H) vary as 4th power of Temp
Temp(D) > Temp(A+H)
since
Temp(D) = surface temp of planet with non GHG atmosphere
Temp(A+H) = surface temp with GHG atmosphere
Therefore the surface will be hotter on a planet with a non radiant (non GHG) atmosphere.
QED
While I agree with Willis that the inclusion of H has allowed the surface temperature of the GHG surface to come closer to the surface temperature of the non GHG planet, the GHG planet still has a lower surface temperature.

PaulR
January 14, 2012 5:17 pm

Ed Fix: If no extra energy is imparted to the gas from the surface of the planet, the gas will eventually all settle to the bottom end of the tube, back at 0K. However, the gas’ temperature did rise, and EM energy was emitted. The planet doesn’t have any less gravity than it did before, so where did the energy come from? I don’t know. If you want to say that violates conservation of energy, I can’t dispute it. Maybe if we really knew what gravity is, we could answer the question.
The extra energy came from falling down the gravity well and stopping/colliding at the end of the fall. It is a transformation of potential energy in the form of the “altitude” of the gas particles into the kinetic energy of linear motion, then vibrational modes for IR. The gas will never make it to absolute zero even given infinite time because of starlight falling on the planet and gas.

P Wilson
January 14, 2012 5:19 pm

You could use a fulcrum and lever, from which you infer the metaphysics of force and resistance.
You cannot argue that once you exert force, then let go, that the energy you used to pull the lever remains permanently locked into the apparatus. It goes back to a state of potential/inertia and force/resistance are no longer operating.
Much the same with radiation. Heating a stone doesn’t lead to transfers of heat from the stone to heat up the air around it which then travels off somewhere else. It disippates

Vergent
January 14, 2012 5:21 pm

On a rotating planet without an atmosphere, there is a big temperature difference between night and day. Add an atmosphere, and the daytime high temperatures are lower because of conduction and convection. Therefore the daytime radiation is reduced. At equilibrium, the nighttime increase in radiation must equal the day reduction.
“The Stefan-Boltzmann equation specifies how much radiation is emitted at a given temperature. It states that the radiation increases much faster than the temperature. It turns out that radiation is proportional to absolute temperature to the fourth power.”
Because of the S-B equation, the night time temperatures must rise more than the daytime temperatures were reduced.
Therefore, the average temperature must rise.

tallbloke
January 14, 2012 5:21 pm

Alec Rawls says:
January 14, 2012 at 4:51 pm
Willis wrote:[SNIP: No, you cannot use my thread to drive traffic to your site. Nice try though. -.w]If this were Willis’ site, I would have no problem with this response to Tallbloke’s censorship of his views at Tallbloke’s Talkship Turnabout is fair play, as the saying goes.

I haven’t censored anything said by Willis or Joel on my site. It’s all there. I won’t provide links since Willis thinks I’m trying to boost my hit count by doing so, but check the end of the suggestions page and the post about Joel Shore and scientific discourse, it’s all there. Every word said by both of them.
I have told Joel he’s not going tto get the opportunity to disrupt the various relevant gravity threads, but offered him a guest post in compensation for this. Willis then banned himself in protest.
By the way, I like the idea of a talkship. When Mickey Mann makes me rich I’ll consider this further. 😉

Phil.
January 14, 2012 5:31 pm

Willis I read Jelbring’s paper out of interest and frankly it’s nonsense. Basically he shows that an atmosphere has an adiabatic lapse rate which depends on g, calls that the greenhouse effect and declares victory. It’s idiotic! I read it twice because I couldn’t believe anyone could be that stupid. I like your model planet in this thread because as I showed above it’s possible to show no graviational effect, if it doesn’t work for this it won’t work in a more complex system.

hmccard
January 14, 2012 5:36 pm

Anthony Watts says:
January 14, 2012 at 12:09 pm
I’ve closed the other two threads on this subject since they were getting a bit ragged and Shore-worn, and directed everyone here to this thread
—-
… and leave the moderating to Willis, eh? (Or is this a new moderating policy?) Although I have been a lurker here at WUWT for several years, I don’t know what the average number of snips per thread is for your blog. However, IMO, the number of snips by Willis in this thread must be at least a 6-sigma outlier. FWIW, the number of self-snips on this thread suggests to me that others share my disappointment.
Willis, I prefer to use the stairs …

tallbloke
January 14, 2012 5:41 pm

PaulR says:
January 14, 2012 at 5:17 pm
Ed Fix: If no extra energy is imparted to the gas from the surface of the planet, the gas will eventually all settle to the bottom end of the tube, back at 0K. However, the gas’ temperature did rise, and EM energy was emitted. The planet doesn’t have any less gravity than it did before, so where did the energy come from? I don’t know. If you want to say that violates conservation of energy, I can’t dispute it. Maybe if we really knew what gravity is, we could answer the question.
The extra energy came from falling down the gravity well and stopping/colliding at the end of the fall. It is a transformation of potential energy in the form of the “altitude” of the gas particles into the kinetic energy of linear motion, then vibrational modes for IR.

Hi Paul,
Aren’t you missing Ed’s point though? He’s asking why it is that the gas gained that energy, when it hasn’t been subtracted from the gravity of the planet. Maxwell and Faraday burned the late night candles working on that problem too. Maxwell gave up on it in the end, basically saying gravity didn’t fit with the laws of mechanics too well. The best he could come up with was to say the ‘lines of force’ were like a web pervading space continually.

PaulR
Reply to  tallbloke
January 14, 2012 6:21 pm

Given the existence of a force field such as gravity, that automatically implies energy and differences of energy due to position in space. We don’t need to know why gravity exists to understand the physics of motion, temperature, radiation and planets with atmospheres.

January 14, 2012 5:46 pm

Ed Fix says:
January 14, 2012 at 4:37 pm
As for gravitational heating, here’s a thought experiment. Imagine a transparent tube in interstellar space (no gravity), 200 km long, one square meter cross sectional area. Fill this tube with some amount of an ideal gas at 0K. That is, each molecule of gas in the tube is motionless. Without gravity, everything remains motionless WRT the tube.
Now instantaneously put an earth-size planet in contact with one end of the tube. All the gas begins to accelerate toward the planet (now the bottom) end of the tube. There will be collisions between gas molecules. That means pressure must increase at the bottom as it decreases at the top. The ideal gas law PV=nRT tells us that with increasing pressure the temperature of the gas will rise. This increased temperature causes the gas to begin emitting S-B radiation.
If no extra energy is imparted to the gas from the surface of the planet, the gas will eventually all settle to the bottom end of the tube, back at 0K. However, the gas’ temperature did rise, and EM energy was emitted. The planet doesn’t have any less gravity than it did before, so where did the energy come from? I don’t know. If you want to say that violates conservation of energy, I can’t dispute it. Maybe if we really knew what gravity is, we could answer the question.

Uh, how do we account for all of the energy expended putting that earth-sized planet in contact with one end of the tube instantaneously?
If we could turn gravity on and off in such a manner, I can see all sorts of energy that could be created as long as it also didn’t take even greater amount of energy to turn that gravity on and off.
Where gravity isn’t a constant, maybe such things are possible?

kbray in california
January 14, 2012 5:47 pm

“…But I digress, my point is that passions are running high on this topic, so let’s see if we can keep the discussion at least relatively chill …
TO CONCLUDE: I’m interested in people who can either show that my proof is wrong, or who will give us your elevator speech about the science underlying either N&K or Jelbring’s theory. …”
Willis wrote this post and put parameters on the type of comment he was looking for.
Although the parameters are unusual, I think it’s ok to: “Snip it if it don’t fit it.”
The choices are:
prove it wrong,
or write an elevator speech.
I don’t like to see the contention between thinking minds, but this exchange seems to have a history to it and that’s a separate issue…. so… “keep the discussion at least relatively chill …”
kbray.

Vergent
January 14, 2012 5:49 pm

On a rotating planet without an atmosphere, there is a big temperature difference between night and day. Add an atmosphere, and the daytime high temperatures are lower because of conduction and convection, and a larger thermal mass to apply the energy to. Therefore the daytime radiation is reduced. At equilibrium, the nighttime increase in radiation must equal the day reduction.
“The Stefan-Boltzmann equation specifies how much radiation is emitted at a given temperature. It states that the radiation increases much faster than the temperature. It turns out that radiation is proportional to absolute temperature to the fourth power.”
Because of the S-B equation, the night time temperatures must rise more than the daytime temperatures were reduced.
Therefore, the average temperature must rise.

January 14, 2012 5:53 pm

Dear Willis,
I have no dog in this fight, I’m just a bystander. But you obviously do, and it seems so in a bad way. I don’t know or care who bit you on the ass, but by the timbre of your responses they struck a nerve. Some of your comments are excerpted below, all quoted as you prefer. I always enjoyed your writings and mostly agreed with the content, but you’ve come off the rails. Sorry that you’ve lost it emotionally…. Your responses belittle you and that is the sad part, and you can never tale any of it back, it’s out there forever.
Please clip this response as this is a personal message. What’s important is that you re-read these quotes below, maybe after a couple of beers. This is all you in the mirror, and it’s not pretty. Feel free to contact me directly if you feel like lambasting me as well. My name directs to my website and my email.
Best,
J.
“That’s it? That’s my crime? You are busting me because I called ten to the minus eighth tiny? You are a waste of bandwidth, sir. I specifically requested that people not bother me with this kind of petty nit-picking, and yet here you are.
Come back when you have something of substance to say, and my advice for your optics would be to leave off the “OMG”, it makes you sound like a Valley Girl.”
“Alexander, the amazing thing about you is that your bull is so convincing I always have to shake my head. Here’s what Wikipedia says about what they call the “Stefan-Boltzmann law”, complete with hyphen:
The law was deduced by Jožef Stefan (1835–1893) in 1879 on the basis of experimental measurements made by John Tyndall and was derived from theoretical considerations, using thermodynamics, by Ludwig Boltzmann (1844–1906) in 1884.
Go away, sir, your impudence knows no bounds. You haven’t a tenth of the knowledge you claim.”
“Why is it that when a charming fellow like you make some idiotic statement that clearly violates conservation of energy, they feel they have to finish it off by insulting my understanding? Medica, cura te ipsum …”
“PS—I note that you have not given us the elevator speech for your “theory of gravitational enhancement”. Until you rectify that egregious omission, why are you opening your mouth about your theory? I do love that your theory claims that you can’t have a GHG-free atmosphere, that’s a new one for me.”
“Tallbloke, I said I would snip. I’m snipping. Don’t pretend to be surprised, it makes you look meretricious. Screenshot all you wish, I have nothing to hide.”
“Because either you can tell us, simply and clearly, how your “Jelbring effect” is supposed to work, or you are worse than useless and should just audit the discussion because you have nothing to add.
This is crunch time, Hans, this is where the rubber hits the road. There’s enough heat already, give us some light. Give us your elevator speech about your grand theory that will revolutionize science. Explain your stupendous ideas clearly in a few well-thought-out sentences. You’ll never have a better or larger audience for your words.
You and I have spent hours on this topic. I still don’t understand how the “Jelbring Effect” works. So fight my ignorance, let me know clearly and cleanly just what happens first, and what happens next, see my example of an “elevator speech” in the head post.
Or walk away, I truly don’t care, Hans. This kind of stuff hurts my head, which I should have examined for posting this thread …”
“The problem was not your error. We all make those.
It was that you were acting like a jerkwagon about it, insulting people for their stupidity when you were the one being stupid.
And the problem is not that you made a fool of yourself, it’s not all about you, really. The problem is that you insulted everyone, and you still haven’t apologized for that, only for your “it’s all about me” mistake …”
“I do love people who start out by saying they don’t profess to any real knowledge, and end up by telling me that what I’ve said is impossible …
TimC, you are right. You don’t even have enough knowledge to ask intelligent questions. I don’t wish to be cruel, but in such a situation, just listen and learn, OK?”
“Unfortunately the link where people can actually see what Hans Jelbring said in his 2003 paper was lost in the carnage of Willis’ censorship spree. Here it is again:
[SNIP: No, you cannot use my thread to drive traffic to your site. Nice try though. -.w]”
“God damn it, Leonard, QUOTE MY WORDS. I have no recollection of saying anything of the sort, I don’t know the context, I don’t know if you understood me, I haven’t a clue what you are referring to. STOP THIS VAGUE ACCUSATORY BULLSHIT AND QUOTE MY WORDS.
Is that so hard to understand?”
“Is stupidity catching? I provided chapter and verse from the IPCC showing that all gases will NOT absorb and emit at all frequencies, and you come back without a cite at all and tell us in essence ‘They do too radiate’!
No, kiwi, they don’t. Hie thee to a textbook, your slip is showing.”
“The point is that you were a dickhead about it, insulting me, and insulting the other posters for “following my lead”. Nor did you apologize for acting like a prick, you only say “sorry I made a mistake”. That’s why I said “go away”. You insult people, then you pretend there was no insult.”
“Well gosh, Alexander, if you don’t like the site then … I hate to say it, but in that case here’s a novel idea—go away. I will not blow in your ear and pat your tummy and pretend to be your friend when you come in here to tell us all what absolute jerks we are and you lambaste people for following my lead. You want someone to blow in your ear, you’ll have to apply to another thread. Here, on the other hand, you might actually learn something. Up to you, I’m not banning you.”
“Dissatisfied with being able to ban people and post whatever you want on your own website, you now want to tell me what we should post on this website? … TB, you’re losing the plot. I have no responsibility and no desire to spread Jelbrings ascientific BS around, that’s up to you and him. If folks here want to find it I have faith that they can, their google-fu is strong …
I notice you still haven’t provided an elevator speech explaining H&N’s theory, which certainly fits with my belief that you don’t understand it any better than I do … yet despite not understanding it, you ban people for saying it violates conservation of energy.
How does that work again?”
“I swear, some people just don’t know when to quit. BZZZZZT! Sorry, wrong. Next contestant, please.”
“That’s it? You’ve run out of real points and are reduced to simple insults? Disappointing, but OK, I’ll play if that’s what you want, although I’d prefer to discuss the science.
Here’s one from my childhood, an insult for a cowboy:
You’re all hat and no cattle!
Gosh, isn’t this fun? Your turn …”

tallbloke
January 14, 2012 5:58 pm

Willis sez:
NOTE 1: Here’s the thing about a planet with a transparent atmosphere. There is only one object that can radiate to space, the surface.

But earlier in the post Willis Sez:
A curious fact is that almost everything around us is continually radiating energy in the infrared frequencies. You, me, the trees, the ocean, clouds, ice, all the common stuff gives off infrared radiation.
Can I add oxygen and Nitrogen (AKA nearly all the air) to this list Willis? If so, what prevents the heat conducted and convected from the surface into the air being radiated to space by the GHG free model atmosphere as well as the surface?

Alan Wilkinson
January 14, 2012 5:58 pm

Ed Fix: “The planet doesn’t have any less gravity than it did before, so where did the energy come from?”
Gravity is a (classical physics) force, not an energy. Forces are unchanged by work done by or against them. The energy came from the original work done in separating the planet and the gases and is now being returned when they are brought back together.

beaker
January 14, 2012 6:00 pm

Willis said:
Thanks, beaker, but that’s not true. There are two kinds of what are usually called night vision devices, image enhancement and thermal imaging. I’m speaking of the second of these. See here for details.
Those of us in the industry always chuckle at what you can find on the web. Thermal imaging is not night vision. Night vision is not thermal imaging. Each has it’s own specific use. Thermal imaging is useless if there are no radiating bodies in the FOV. Night vision is useless if there is no ambient light, even starlight. Each uses a completely different technology. I have been working 20 years with each.
Willis, you have your specialty. I have mine.

Alan Wilkinson
January 14, 2012 6:03 pm

tallbloke: “Can I add oxygen and Nitrogen (AKA nearly all the air) to this list Willis? If so, what prevents the heat conducted and convected from the surface into the air being radiated to space by the GHG free model atmosphere as well as the surface?”
No. Because anything that radiates in the infra red spectra is a GHG model by definition of the GHG process.

Dr. Dave
January 14, 2012 6:09 pm

Willis,
I may be risking a dreaded [snip] here, but I’ll take my chances. I read your piece and I agree with you. It’s not entirely different from what Dr. Roy Spencer wrote on his site a few days ago. I have plodded through something like 400+ comments. They have been mostly entertaining, some informative, some that produced laughter and some that induced cringing. But now I am worrying that your blood alcohol content is dangerously low. Rather than engaging the gormless cavilers or pursuing a slap fight with Tallbloke, why not step away from the keyboard and have a couple of beers?
This stuff is interesting, but it’s not all that damn important. You’re just not your usual erudite, polite and cheerful self. I recommend a moderate dose of alcohol.

tallbloke
January 14, 2012 6:11 pm

Anthony Watts says:
January 14, 2012 at 12:09 pm
I’ve closed the other two threads on this subject since they were getting a bit ragged and Shore-worn, and directed everyone here to this thread.

Lol, nice one Anthony. Sorry to hear you had to close two threads though on such an important topic though. The Shore effect in action. It Shore isn’t going to happen on my site.

Vergent
January 14, 2012 6:23 pm

Willis,
Your error is this. You said:
“But when the temperature of a perfect blackbody planet rises … the surface radiation of that planet must rise as well.”
This is true for a body of uniform temperature.
What was needed for your “proof”:
“But when the average temperature of a perfect blackbody planet rises…the surface radiation must rise.”
This is not true. consider the moon; cool the hot side, and then warm the cold side until the total radiation is equal. You will have warmed the cold side more because of the S-B equation. The average temperature will go up.
Vergent

tallbloke
January 14, 2012 6:24 pm

Alan Wilkinson says:
January 14, 2012 at 6:03 pm (Edit)
tallbloke: “Can I add oxygen and Nitrogen (AKA nearly all the air) to this list Willis? If so, what prevents the heat conducted and convected from the surface into the air being radiated to space by the GHG free model atmosphere as well as the surface?”
No. Because anything that radiates in the infra red spectra is a GHG model by definition of the GHG process.

I thought GHG’s were the gases that absorb IR as well as emit it. Can we have the definition of GHG agreed please Willis.
Thanks
TB.

tallbloke
January 14, 2012 6:29 pm

Willis, how come everyone else is expected to stick to elevator speeches but you get to publish long self pitying diatribe? Now that Anthony has redirected the refugees from the “Shore worn” N&Z threads here, you need to relax, and relax the rules.

ferd berple
January 14, 2012 6:30 pm

Mathematical proof that GHG cools the surface of planet earth
terminology:
==X==> denotes energy flow from left to right, called X.
<==Y==> denotes two way energy flow between right and left, called Y.
<==Z== denotes energy flow from right to left, called Z.
In an atmosphere with GHG
(1) space <==A== surface(r) <==B==> ghg ==C==> space
total energy incoming from sun = net energy emitted to space by GHG atmosphere + net energy emitted to space by surface(r)
(2) A + C = solar energy in = radiation out to space
In an atmosphere without GHG (non radiating),
(3) space <==D== surface(n) <==E==> no ghg ==F==> zero radiation to space
total energy incoming from sun = net energy emitted to space by surface(n)
(4) D + F = solar energy in = radiation out to space
However, since F = 0, this becomes
(5) D = solar energy in = radiation out to space
Assume that in (1) above the surface is warmer than the atmosphere, and the net energy flow is positive from surface to GHG atmosphere. (1) can then be rewritten as:
(6) space <==A== surface ==H==> ghg ==C==> space
Flow C takes part of its energy from the flow from the surface to ghg (flow H), plus the net energy absorbed directly from the sun by the GHG that re-radiates as C. Thus we can say that:
(7) H + net solar absorbed by GHG reradiated as C = C
Since net solar absorbed by GHG > 0, then we can say than in all cases with a GHG atmosphere, that:
(8) H < C
from (2) we have:
A + C = solar energy in = radiation out to space
And from (5) we have
D = solar energy in = radiation out to space
Therefore we can say
(9) A + C = D
and from (8) we have
H < C
Therefore
(10) D = A + C > A + H
Therefore
(11) D > A + H
Since D and (A+H) vary as 4th power of Temp by S-B
(12) Temp(D) > Temp(A+H)
Since
Temp(A+H) = surface(r) temp with GHG atmosphere
Temp(D) = surface(n) temp of planet with non GHG atmosphere
Therefore, because of (12) the surface will be hotter on a planet with a non radiant (non GHG) atmosphere.
QED

January 14, 2012 6:33 pm

The if we separated the atmosphere including the oceans from the earth it would be a small ball of ice floating in space just like a large comet, wouldn’t the temperature of the earths surface be greater as a black body under a source of energy without an atmosphere?.

eyesonu
January 14, 2012 6:36 pm

Very interesting thread.
There is certainly no Pal Review here.
This is science review. What went wrong over the past 20 years? I hope this is the new normal which would be the old normal prior to Pal Review.

January 14, 2012 6:36 pm

*Sorry, Then, if…

Ed Fix
January 14, 2012 6:36 pm

PaulR says:
January 14, 2012 at 5:17 pm
The extra energy came from … a transformation of potential energy … into the kinetic energy of linear motion…

Paul, you must have had the same physics text I did. Was it “Halliday and Resnick”, like I used, or “Halliday, Resnick and Walker” like my son used?
Basically, I can’t get past the fact that if an object is released in a gravitational field, it accelerates without gravity being depleted in any way. [SNIP: please, no philosophical speculations on gravity. It’s a field. To move against it takes energy. To move with it gives you energy. No one knows why. w.]

LazyTeenager
January 14, 2012 6:39 pm

Genghis says
Gravity is going to compress the entire column of air down to about 17km. That compression heats the air and establishes the lapse rate, greatest density at the bottom and less density at the top.
———–
So calculate the temperature developed under this scheme.
I think you will be surprised that the temperature you get bears no relation to the actual lapse rare either measured or calculated from the known theory of lapse rate..

Alan Wilkinson
January 14, 2012 6:44 pm

tallbloke: “I thought GHG’s were the gases that absorb IR as well as emit it”.
Absorption and emission are symmetric processes. If it emits then it absorbs in the same manner afaik.

LazyTeenager
January 14, 2012 6:45 pm

crosspatch says:
January 13, 2012 at 11:24 pm
I have used light amplification night vision devices that also had IR capability along with an IR search lamp built into the device for use in completely dark conditions (inside of something where there is no ambient light to amplify).
————
These are near infrared devices which are different again from thermal Infrared imaging devices.

tallbloke
January 14, 2012 6:50 pm

Willis: a little bit of reasonableness goes a long way with me, so thanks for that. It’s way past bedtime here so I’ll sleep on it and hope that the N&Z response to comments update is in my inbox when I awake. After I’ve digested that, and kicked it around for a while with my friends, I’ll be closer to formulating a brief summary.
You shouldn’t try to hurry good science. Savour it and mull it around before describing it.
Cheers
TB.

u.k.(us)
January 14, 2012 6:56 pm

Willis Eschenbach says:
January 14, 2012 at 5:41 pm
….”I defy you to find anything in that statement about him disrupting your poor little thread.”
=========
Poor little thread ?,
Rein in that horse, Willis.
We are trying to spread information, or have I been reading the wrong blog for the last 3 years ?
Petty arguments, achieve nothing.

eyesonu
January 14, 2012 7:00 pm

Willis, I believe I understand what you tried to do here. That was to try to factor out one of sooo many variables regarding the chaotic nature of the atmosphere. The results of your noble attempt has become chaotic.
It seems that nothing is settled or that a consensus has been reached.
Thank you for generating such a discussion.

Willy
January 14, 2012 7:08 pm

Willis says: “We’re not discussing the earth, we’re discussing a planet with no water (“GHG-free atmosphere”). So your points have no meaning in this discussion.”
Thanks for taking the time to read my remarks. I should have been more clear on the point that I was not using the GHG properties of water vapor. I was using its evaporative cooling effect. So lets specifically ignore radiative properties and only consider the effect of evaporative cooling.
[No, lets not. No water. No evaporation, thanks. Read the head post again. Follow the instructions. w.]

LazyTeenager
January 14, 2012 7:14 pm

Wayne says
Also, if there is any absorption of radiation by the air, then more radiation would be absorbed lower due to the increased density and that can be from both solar radiation and surface infrared radiation. That too causes more warming lower.
—————
Depends. This would only be true if the green house density changed. Since the original question was about changing N2 only, you need to justify why the green house gases have changed. If CO2 is the only GHG to be considered then the claim that more radiation would be absorbed is likely wrong.

Editor
January 14, 2012 7:18 pm

Willis wrote:

While this is not my site, it is my thread, and I said I would snip anything off-topic. Now I do that and you want to complain.

A link to the paper that is the subject of the post is off topic? Personally, I can’t imagine writing a post without linking to the subject.
[SNIP: I pointed out that Tallbloke above has linked to the paper. The rest of your post is merely complaining. Where is your elevator speech in defense of the theory? . w.]

LazyTeenager
January 14, 2012 7:28 pm

Hoser quotes Willis and says
A curious fact is that almost everything around us is continually radiating energy in the infrared frequencies. You, me, the trees, the ocean, clouds, ice, all the common stuff gives off infrared radiation.
>>>>>>>>And the air.
No. The air does not radiate as a black body.
The air radiates according to the ability of its component molecules to radiate. Molecules radiate electromagnetic radiation according to the same rules that apply to a radio aerial. In other words to produce IR emissions from a molecule the bond vibrations need to produce an oscillating electric field.
Vibrations of a homonuclear diatomic like N2 or O2 cannot do this at all. No way, no how.
Asymmetric molecules like CO can emit.
Symmetric molecules like CO2 can emit via bending modes that are asymmetrical and cannot via bending modes that are symmetrical.
So Willis is correct and Hoser is wrong.

LazyTeenager
January 14, 2012 7:32 pm

Higley7 says
If I remember my light physics correctly, light energy is lost by destructive interference laterally during transmission through the atmosphere. I do not know what proportion is lost but the sky is blue because of the lateral dispersion.
———–
Sorry no.
Nothing to do with
1. Lateral dispersion
2. Interference
Look up the definitions of these terms and while you are at it look up Rayleigh scattering.

Dr Burns
January 14, 2012 7:35 pm

I think that Willis’ elevator model is flawed. A more realistic model is …
[SNIP: Put your alternative theories and models on your own thread, please. We’re discussing a particular model here, not your model. Thanks, w.]

evilincandescentbulb
January 14, 2012 7:36 pm

How hard can it be?
Elevator speech: Venus …
[SNIP: It is harder than you think, because I asked for an elevator speech about the N&Z or Jelbring theories, not about Venus. That’s not only off-topic, it’s off-planet. w.]

LazyTeenager
January 14, 2012 7:36 pm

Crosshatch says
Model the atmosphere as a bunch of concentric surfaces that are transparent to IR but still radiate
————-
That would be a violation of a physical law.
If the atmosphere is transparent to IR it is impossible for it to radiate IR.

Leonard Weinstein
January 14, 2012 7:38 pm

Willis,
In your response to me at 3:52 pm, you responded to the following:
Leonard Weinstein says:January 14, 2012 at 12:22 pm
Willis,
I have stated the cause of the greenhouse effect in basic terms in a previous reply. The issue I want to discuss is your claim that if the amount of greenhouse gases was constant, but non greenhouse gases (O2 and N2) increased, there would be no change in ground level temperature.

God damn it, Leonard, QUOTE MY WORDS. I have no recollection of saying anything of the sort, I don’t know the context, I don’t know if you understood me, I haven’t a clue what you are referring to.

In fact I was referring to your response to Genghis at 12:19 am, where you responded to the following:

4. The thicker and denser the atmosphere, the higher the near surface atmospheric temperature will be.

I don’t think so. The dry adiabatic lapse rate is g / Cp, where g is gravity and Cp is the specific heat of the atmosphere. The lapse rate does not vary with elevation, which means that Cp doesn’t vary with density, so I don’t see how a denser atmosphere would perforce be warmer.

I used the example of increasing density without increasing greenhouse gas content to show how that statement is incorrect. It is true the lapse rate does not change, but the altitude of outgoing radiation does, and that does change the near surface temperature some. I know you know that but the statement as made was wrong.

1DandyTroll
January 14, 2012 7:42 pm

[SNIP: bad humor w.]

Leonard Weinstein
January 14, 2012 7:42 pm

Willis,
The reply I just made above is not an exact quote of your words, so I understand the confusion. It is, however, equivalent in meaning.

Tim Folkerts
January 14, 2012 7:43 pm

Willis says January 14, 2012 at 7:22 pm: “so far, no one has shown that they understand either one. Including you and Jelbring and Nikolov, none of you has been willing to give a simple, clear explanation of the claimed theory.”
At the risk of being redundant, I did give an explanation of the “Jelbring effect” as a variation of the standard greenhouse effect, but it seems to have been lost in all the smoke. So, to quote myself from earlier:
———————————————–
STEP 1: Instead of a greenhouse gas like CO2 at the top of the atmosphere (TOA) that radiates at some IR wavelengths and is transparent at other IR wavelengths, use a greenhouse gas at the TOA that radiates at ALL IR wavelengths (ie is “black” to IR; ie emissivty = 1).
Or in his words “[The model planet globe] (G) and the atmosphere (AT) are surrounded by a concentric, tight, black spherical shell. “
STEP 2: Apply all the standard physics to the rest of the analysis.

evilincandescentbulb
January 14, 2012 7:47 pm

Elevator speech: You’re pretty quick on the snip-button … [SNIP: Yes, I am. I asked for elevator speeches about the theory of either N&Z or Jelbring. Yours is neither, so I snipped it. w.]

Ed Fix
January 14, 2012 7:54 pm

Willis Eschenbach says:
January 14, 2012 at 5:48 pm
…you claim that non-GHG gases radiate in the IR, so you’ll have to rebuild the parts of your theory that depend on that incorrect claim.

Willis, you’re conflating absorption with emission. Any body, solid or fluid, emits electromagnetic radiation with a spectrum determined by the S-B relationship. A body’s transparency spectrum is independent of its emission spectrum. They’re separate processes.
A transparent body of gas, however, is different from a solid body in that we can see EM radiation emitted throughout the volume, not just at the surface. Specifically, at those wavelengths where the gasseous body is transparent, we see the emissions from the entire depth at full intensity. However, at those wavelengths the intervening gas can absorb, we see a notch in the intensity spectrum. Carbon dioxide happens to absorb a wavelength of EM near the peak of the earth’s overall S-B radiation spectrum (about 15 micrometers), but it re-emits S-B radiation in a smooth spectrum depending on its temperature only. Just like any other body of matter.
So I reiterate, an atmosphere containing a greenhouse gas gains heat both by conduction through contact with the surface, and by absorbing the earth’s (and the surrounding atmosphere’s) S-B radiation at some wavelength, while a non-GHG atmosphere is transparent to all EM wavelengths, and gains heat energy only by conduction. Both, however, emit their own S-B radiation in a spectrum determined only by their temperature.

Come back when you have spent the time refining it

I may do that at some point, but I’ve spent far too much time on this today already.

evilincandescentbulb
January 14, 2012 8:00 pm

Elevator speech: N&Z do not believe you can draw conclusions about the effects fo GHGs on the climate without understanding the effects of any atmospheric gas on climate–irrespective of the composition of the gas–when atmosheric pressure is added as a variable.

Tim Folkerts
January 14, 2012 8:05 pm

Ed Fix mistakenly thinks: “Both, however, emit their own S-B radiation in a spectrum determined only by their temperature.”
Sorry Ed, but the ability to emit IR is the same as the ability to absorb IR. If this were not true, it would be easy to violate the laws of thermodynamics. Google “Kirchhoff’s law of thermal radiation” (for example http://en.wikipedia.org/wiki/Kirchhoff%27s_law_of_thermal_radiation) for more info.

Jim D
January 14, 2012 8:11 pm

Elevator speech on Jelbring:
The atmosphere is warmer as you go down in it because of the adiabatic lapse rate (g/cp) therefore greenhouse gases have nothing to do with the adiabatic lapse rate (true). The adiabatic lapse rate causes the greenhouse effect (false).
Elevator speech on Nikolov and Zeller:
You can fit a four-parameter curve to surface temperature over blackbody temperature ratio of seven solar system atmospheres just as a function of mass (not albedo or composition or clouds!). Therefore these other things don’t matter. It doesn’t matter that you can’t derive this curve except by a mathematical fit to the data. Who needs to explain why it fits?

Derek Miller
January 14, 2012 8:17 pm

Willis Eschenbach says:
January 14, 2012 at 4:51 pm
Sure. If there is no way for a gas to radiate energy, and it is isolated, it will stay at the same temperature indefinitely. I think N2 absorbs/emits very, very, very weakly in the IR, so it might cool over thousands of years, but basically indefinitely for practical purposes.
__
Uh, no. “All matter with a temperature greater than absolute zero emits thermal radiation”! The only difference between “greenhouse” gases and the others is that they absorb IR LOTS better than the others–they don’t emit IR any differently. Think about it like this, heat Xg samples of CO2 and Argon to the same temperature and put into separate Dewars (or do it at home in a thermos!). Measure the temperature after 1 hour. I can guarantee that they would be at the same temperature to within the error of the experiment.

Jim D
January 14, 2012 8:19 pm

By the way, N&Z believe the blackbody temperature of the earth to be 154.3 K in disagreement with just about everyone else including Jelbring who believe it should be 255 K, so their mathematical fit may be flawed at this basic level.

Dr Burns
January 14, 2012 8:20 pm

Willis, I feel you are getting a little emotional. There is no justification in your SNIP.
The flaws in your model include ignoring the earth’s 70% cloud cover and assuming that the earth is primarily a solid rather than a liquid surface. These flaws allow only radiation as the heat transport mechanism. Radiation is insignificant between the earth’s surface and clouds, where the major heat transport mechanisms are convection and evaporation.
No wonder you find GHG’s having a major impact when you only consider radiation. You should be considering radiation from cold clouds to space rather than radiation from a warm earth surface.

willb
January 14, 2012 8:24 pm

@Willis Eschenbach
Here is my best shot at an elevator speech explaining the N&Z “greenhouse effect”. The N&Z effect works as follows:
• The sun transfers energy via radiation to the earth, warming the earth.
• The surface of the earth transfers energy into the (non-GHG) atmosphere through conduction, heating the atmosphere.
• Earth’s gravity causes an altitude-dependent temperature and pressure lapse rate to form in the atmosphere. The atmospheric temperature and pressure drop as altitude increases with the temperature ultimately dropping to that of deep space (~3K).
• Atmospheric gases heated conductively at the earth’s surface then convectively rise through the pressure lapse rate.
• As the temperature of the atmosphere drops towards that of deep space, the rising gases transition through a number of phase changes (e.g. gas => liquid => solid-a => solid-b). At the occurrence of each phase change the atmosphere releases latent heat in the form of radiation.
• Some of the radiating energy from the released latent heat is directed downward through the gaseous, IR-transparent atmosphere and is absorbed by the earth.
• As a result of absorbing that latent heat energy from the atmosphere, the surface is warmer than it would be in the absence of the (non-GHG) atmosphere.

Alan Wilkinson
January 14, 2012 8:33 pm

JIm D, great elevator attempt. However you may be over-generous in awarding one true point to Jelbring. I doubt the (observable) adiabatic lapse rate is independent of GH gasses since without radiation there is no convection and therefore as Roy Spencer points out the atmosphere becomes isothermal by conduction (violating the adiabatic premise)?

Jim D
January 14, 2012 8:42 pm

Alan Wilkinson, yes there is a debate on the point of what the laps rate would be with no GHGs. Some believe isothermal, some adiabatic (well mixed), some in between (slightly stable).

eyesonu
January 14, 2012 8:44 pm

As I said previously, the science is not settled. There is a lot to be digested here.
Damn if I know the answer! There is certainly no consensus.
I have a view point, but uncertain enough to not express it. When in doubt, keep my mouth shut but my eyes wide open. I’ll keep my eyes wide open.
Willis tried to break it down to components, but it seems much more involved that a simple solution as expressed by many. Chaotic? Idono. Maybe the whole shebang is more than can be modeled if we can’t get one single factor agreed upon. There are many.

Tim Folkerts
January 14, 2012 8:47 pm

Roy Spencer is wrong.
I didn’t think I would say that about a basic physics idea, but he is incorrect when stating “If the atmosphere cannot absorb/emit IR, it would become isothermal, and all convection would cease. ”
It would become isentropic, not isothermal. At equilibrium, both the temperature and the density will decrease with altitude. This is the required condition for the stopping of convection, not being isothermal. This is directly related to the idea of “adiabatic lapse rate”.
See http://mysite.du.edu/~etuttle/weather/atphys.htm
or
http://maths.ucd.ie/met/msc/PhysMet/PhysMetLectNotes.pdf

Phil's Dad
January 14, 2012 8:55 pm

The temperature of the radiating surface would be unaffected by pressure changes in a transparent atmosphere. If that is all the post is saying I agree. I am less sure that N&Z are dealing with transparent atmospheres in equilibrium when they hypothesise that the green house effect is negligible overall.
Cue lift music
* A “greenhouse”-gas-free atmosphere (by definition one which can not be involved in long wave radiation) will still pick up heat from the surface by conduction and mix it throughout the atmosphere by convection cells until it reaches stability (something like the surface heat at the warmest point / the equator normally, but your sphere of suns means all points will be equal so I would accept a lower adiabatic profile in this case).
* Until this balance is reached overall radiation-out will be lower than radiation-in as some of the energy at the surface is conducted away from the radiating surface and stored by the atmosphere.
* It can thus act as a reservoir for heat, resulting in a temperature at any given height above the ground surface depending on the density (heat is stored by the molecules. Less molecules per cubic foot = less heat per cubic foot = lower measured temperature) but changes to the atmospheric heat content would be very slow as exchange can only happen at the point of contact between surface and atmosphere.
* If you add more of this “transparent” atmosphere [which is, I think, the mechanism N&Z are talking about] it will be able to hold more heat overall but it must still get this extra heat from conduction at the surface unless it is already warmed elsewhere (e.g. volcanic [again as N&Z suggest if I recall correctly]).
* Because gravity is present, the density of that increased atmosphere near the surface will increase (I.e. there will be now be more of the atmosphere present near the surface than before) and it follows that there will be more heat present (per cubic foot as it were) leading to higher near surface temperatures.
And for the lift journey back down.
* It is tempting to think that the heat in the conductive bottom layer of the atmosphere can not exceed that of the surface from which it first gets that heat energy.
* The atmosphere has however built up a store of heat as described above in the total volume of atmosphere.
* If that is compressed as described because volume has been added to it (note I am ignoring for now the act of compression – assuming measurement after the compression is complete) the amount of heat can (temporarily) exceed that at the surface.
* By definition it can not radiate this heat away. It will however pass some of this heat back to the ground to restore equilibrium at the conductive layer. (Much more slowly than conductive warming as convection constantly interferes.)
* That will temporarily (but for a very long time) result in more radiation out than in, without breaking any laws.
In the very fullness of time a transparent atmosphere would always return to match the radiating surface (at its long term warmest point) but this would be a very (very) slow process (cooling more slowly than it warms) and in shorter time scales temperature would be pressure dependant as N&Z suggest.
So Wills and N&Z can both be right but not on the the same timescale.
Does this apply to the Earth? Obviously not entirely as we do not have a transparent atmosphere, we have other stuff like night and day and a single light source, spin, wobble…
Does that mean N&Z idea makes no contribution at all?

David
January 14, 2012 9:01 pm

Willis Eschenbach says
January 14, 2012 at 8:05 pm
Willis, are you saying that if I am suspended in space, and a stream of 8,000 degrees N2 jets by me at say 10′ away, (nothing but the vacume of space between me and the gas) I will feel nothing and my thermometer will register nothing, but if that stream of gas, again with 10′ of the vacume of space separating me, streams past, I will feel emense heat from the radiating CO2, but none from the non radiating N2?

Allen63
January 14, 2012 9:03 pm

Well, I didn’t read 400 comments. So, maybe someone already suggested this.
The “Planet” is not uniformly heated by a thousand suns — there is just one. [SNIP: No, it’s evenly heated. If you want to discuss your situation, write your own post. w.]

January 14, 2012 9:04 pm

Willis says,
WE’RE NOT TALKING ABOUT THE EARTH! REPEAT AFTER ME! WILLIS IS DISCUSSING A HYPOTHETICAL PLANET, NOT THE EARTH!
OK, lets imagine the atmosphere on this hypothetical planet (that is evenly irradiated) has five times the mass of the solid portion. And, within the bounds of the hypothetical example imagine a second planet where the atmosphere has just 1% of the mass of the solid portion, the diameter of the sold portions in each case being similar. So, they both gather the same amount of radiation.
Then let us imagine that the only means of heat transfer to space is via radiation from the solid portion. But the atmosphere’s in each case are heated by conduction, i.e. collision of molecules.
The atmospheres will not be equal in temperature from top to bottom. Temperature will relate to density.
Will the equilibrium temperature of the atmosphere at two meters above the surface be greater in the planet with the denser atmosphere?
I say yes, the planet with the denser atmosphere will impart more energy to a thermometer.
Add a greenhouse gas to the denser atmosphere. It will become cooler and so will the surface of the planet because the surface that is emitting will be enlarged and there is still the same amount of energy to go round.

January 14, 2012 9:05 pm

Alan Wilkinson says:
January 14, 2012 at 8:33 pm

as Roy Spencer points out the atmosphere becomes isothermal by conduction

Precisely. And on a spherical body like a planet, the atmosphere can only become isothermal when it reaches the temperature of the warmest part of the surface, i.e. the equator.
This means the temperature of the ATMOSPHERE WILL BE HIGHER than the average temperature of the SURFACE as defined by the SB equation.
This has profound implications for the greenhouse hypotheses.

David
January 14, 2012 9:08 pm

Open question; Surface heat conducts to the non GHG atmosphere, Can non GHG molecues conduct heat to the surface?

David
January 14, 2012 9:13 pm

David says:
Your comment is awaiting moderation.
January 14, 2012 at 9:01 pm
Willis Eschenbach says
January 14, 2012 at 8:05 pm
Willis, are you saying that if I am suspended in space, and a stream of 8,000 degrees N2 jets by me at say 10′ away, (nothing but the vacume of space between me and the gas) I will feel nothing and my thermometer will register nothing, but if that stream of gas IS COMPOSED OF CO2 and jets past, again with 10′ of the vacume of space separating me from the gas, I will feel emense heat from the radiating CO2, but none from the non radiating N2?
This clarifies my poorly worded question at 8:05.

January 14, 2012 9:14 pm

NoIdea says:
January 14, 2012 at 1:09 pm

Were the earth’s atmosphere substantially deeper, then the temperature near the surface would be higher. I suspect that means that the temperature in say a deep hole (mine) on the earth would be higher (absent any geothermal effects that is).

Dunno if you will get to see this in such a long thread, but you are correct. The Mediterranean has been waterless (or almost so) several times in Earth history. At the bottom, the estimated atmospheric temperature would have reached ~80C. Some of the minerals formed there by evaporation only form at high temperatures. Or so I was taught…

January 14, 2012 9:19 pm

Baa Humbug said @ January 14, 2012 at 9:05 pm

Precisely. And on a spherical body like a planet, the atmosphere can only become isothermal when it reaches the temperature of the warmest part of the surface, i.e. the equator.
This means the temperature of the ATMOSPHERE WILL BE HIGHER than the average temperature of the SURFACE as defined by the SB equation.
This has profound implications for the greenhouse hypotheses.

In Willis’s pedagogical example, the toy planet is lit uniformly by many suns. Every part of its surface is “at the equator”.

jorgekafkazar
January 14, 2012 9:31 pm

Thus spake Willis: “…Oxygen may well emit visible if it gets as warm as the sun, I don’t know….”
I do know what oxygen does when it gets “warm” enough. It’s called aurora borealis in this hemisphere. At 900°C +/- 300°. Fairly cool, relatively speaking.
Like it or not, the elevator speech concept is valid. I think Nikolov & Zeller and Jelbring have overlooked some critical point necessary to make their case. Either that, or they’ve failed to explain some key concept or assumption. Maybe next month they’ll connect the dots.

January 14, 2012 9:33 pm

Dr Burns said @ January 14, 2012 at 8:20 pm

The flaws in your model include ignoring the earth’s 70% cloud cover and assuming that the earth is primarily a solid rather than a liquid surface. These flaws allow only radiation as the heat transport mechanism. Radiation is insignificant between the earth’s surface and clouds, where the major heat transport mechanisms are convection and evaporation.
No wonder you find GHG’s having a major impact when you only consider radiation. You should be considering radiation from cold clouds to space rather than radiation from a warm earth surface.

There are no clouds in Willis’s pedagogical example. That’s the whole point of the example: to simplify in order to teach a very basic lesson that few seem willing to learn. It’s as if everyone wants to become Beethoven without learning the rudiments of musical composition.

Vergent
January 14, 2012 9:40 pm

Willis,
Your error is this. You said:
“But when the temperature of a perfect blackbody planet rises … the surface radiation of that planet must rise as well.”
This is true for a body of uniform temperature.
What was needed for your “proof”:
“But when the average temperature of a perfect blackbody planet rises…the surface radiation must rise.”
This is not true. consider the moon; cool the hot side, and then warm the cold side until the total radiation is equal. You will have warmed the cold side more because of the S-B equation. The average temperature will go up.
Vergent

Vergent
January 14, 2012 9:44 pm

last tag.

Bob Fernley-Jones
January 14, 2012 9:44 pm

Willis Eschenbach @ January 14, 7:02 pm
Quick test of the debate mechanisms here;
Willis, you wrote in part to Vergent:

“… all I’m talking about at the moment is the situation spelled out in the head post. This is a body of uniform temperature. As a result, what other bodies in other situations do is not relevant…”

So what the…. Is this sphere rotating at infinite rate whilst retaining an atmosphere, and how is perfect distribution of energy from equator to poles obtained? (have you cancelled the cosine law?)

Vergent
January 14, 2012 9:50 pm

sorry about interjecting facts into a political forum.

Phil's Dad
January 14, 2012 9:56 pm

I modify my previous to say that the friction free atmospheric temperature can only exceed the average surface temperature if that surface is unevenly warmed which in the given model would not happen. In that model near surface atmospheric temperature would be increased with increased pressure (for a long time perhaps) time but would never exceed that of the surface (a perfectly dense atmosphere would approach the surface temperature)

January 14, 2012 9:58 pm

erl happ :January 14, 2012 at 9:04 pm said:
Hypothetical planet (that is evenly irradiated) within the bounds of the hypothetical example
1. has five times the mass of the solid portion.
2. a second planet where the atmosphere has just 1% of the mass of the solid portion, the diameter of the sold portions in each case being similar.
So, both gather the same amount of radiation… Heat transfer to space is via radiation from the solid portion….the atmosphere’s in each case are heated by conduction, i.e. Molecular collision.
The atmospheres will not be equal in temperature from top to bottom. Temperature will relate to density.
…….the planet with the denser atmosphere will impart more energy to a thermometer (at 2 mtr above surface).
Add a greenhouse gas to the denser atmosphere.
It will become cooler and so will the surface of the planet because the surface that is emitting will be enlarged and there is still the same amount of energy to go round.

This from Earl Hap looks like an excellent model, and has an interesting result! –
I can’t fault his thinking – I look forward to more discussion on this.

PaulR
January 14, 2012 9:59 pm

Bob Fernley-Jones says:
January 14, 2012 at 9:44 pm
So what the…. Is this sphere rotating at infinite rate whilst retaining an atmosphere, and how is perfect distribution of energy from equator to poles obtained? (have you cancelled the cosine law?)
Yes he did cancel the cosine law, he did it by assuming a planet equally lit by thousands of small suns. Read the OP? Twice?

January 14, 2012 10:16 pm

jorgekafkazar said @ January 14, 2012 at 9:31 pm

Like it or not, the elevator speech concept is valid.

I usually find that if I can’t distil my learning into elevator speech, it’s because I really don’t understand what I thought I learnt.

Bill Hunter
January 14, 2012 10:19 pm

Willis,
My reading of Jelbring is that the surface is going to be warmer than absolute zero even if there is no solar input because the adiabatic lapse rate reigns supreme.
And since there is no such thing known as minus K an absolute zero surface cannot exist on a planet surface with more than a trifling atmosphere.
If indeed your model of radiative equilibrium at the surface mandates an absolute zero surface then the upper atmosphere of this planet would have to be measured in minus kelvin something that does not exist. So the concept of a radiative equilibrium in dark space is in conflict with other physical laws like the ideal gas law.
My apologies to Jelbring if I got that wrong.
And whats wrong with the radiative model?
I like the passive solar water heater system with collectors below the storage analogy.
The water warms in the collector coils and convects up the pipes into the storage tank and out of the bottom of the storage tank cooler water drops down into the coils to be warmed in a convection driven cycle. The collector coils are cooled by this process so they remain below radiative equilibrium until the entire system has warmed to that level.
And I am not talking about radiative equilibrium at the average blackbody temperature of a rotating planet but I would say that would be radiative equilibrium with the current radiation rate say around noon time if you managed to crank your efficiency up to 100% (which you can’t but you can get close).
You can only get close with very good insulation on the elevated water storage tank. But what is better insulated than a non-radiative atmosphere? The answer is nothing.
It can’t lose heat by conduction or radiation out the sides or the top. It can only lose heat out the bottom by conduction and common air is one of the best insulators around. You can’t build stuff this good at least you can’t economically at building scales. Maybe a polished vacuum bottle will rival it with the bottom closed.
At the end of the 24 hour cycle you will find the system to average considerably warmer than the average radiation would dictate because it is far less efficient at cooling than it is at warming.
The surface remains relatively cool even when the air is warm. Its hard to detect or measure this though as this coolness is confined to an infintessimally thin surface layer with strong temperature gradients immediately above and below that radiating surface.
So the sand feels hot on your foot because your foot is more conductive than the air and its drawing subsurface heat into your foot.
Then of course you have to enter latent heat of evaporation into this that is even more effective at transferring heat to the atmosphere in a one way route than conduction.
As a kid I lived in the desert. We used to carry those old canvas water bags and hang them off the radiator cap of our Model A. The evaporation kept the water relatively cool despite outdoor atmospheric (Stevenson screen height) temps in excess of a hundred degrees (F).
But even at that temp the atmosphere was not as hot as it would have been in the absence of convection.
With 1200 watts burning down through a cloudless desert sky the temperature was not the 225degF such a radiation level mandates at equilibrium, nor was the ground though we often joked about frying eggs on the pavement nobody could.
So what you have is a lot of daytime damping going on via conduction amplified by convection. At nighttime the damping of the cooling is far less.
Add greenhouse gases to the atmosphere and you probably change nothing as the additional heat it intercepts is offset by the heat it radiates. You no doubt will dispute that but I have to wonder why a ground molecule at 288K can radiate all its heat up and gas molecule cannot. I see this area as the borderline of AGW mumbo jumbo.
I think the completely passive solar water heating system that one can build for a few hundred bucks proves that a simple radiative equilibrium model in a GHG free atmosphere is pure BS.

gbaikie
January 14, 2012 10:22 pm

[SNIP: gbaikie, either give us your elevator speech or falsify my model, your ramblings go nowhere. w.]

January 14, 2012 10:25 pm

Except I’m far too lazy to do it, it would be interesting to count the number of posts that show evidence of the commenter having actually read the headpost, never mind understood it. It must be alarmingly small!
I occasionally drink with one of those lucky guys who gets to program a Cray with numerical simulations; of weather, rather than climate. When the talk gets to CAGW, there has on occasion been considerable bitterness between us, including said programmer being very dismissive of the level of scientific understanding of WUWT readers. In the past I have defended WUWT. Today I think it’s time to eat some crow. Mind you, he seems to have remarkably little grasp of basic Newtonian mechanics even though he has a Doctorate in astrophysics.
We all have our blind spots.

anna v
January 14, 2012 10:31 pm

Dear Willis, I have not read the numerous comments, so somebody may have said the following already:
In your gedanken experiment:
Now suppose we add an atmosphere to the planet, a transparent GHG-free atmosphere. If the theories of N&K and Jelbring are correct, the temperature of the planet will rise.
A transparent to radiation atmosphere is a contradiction. For an atmosphere not to have a heat capacity it means that no matter ( atoms, molecules) is there . If there is matter then there exists a heat capacity. This matter will be radiating at some part of the electromagnetic spectrum with a modified SB law ( I think it is T^6 ), and the surface temperature will be ameliorated accordingly because of that. I am saying that the experiment is unphysical and cannot be used as proof.
I do not know the claims of the people you are arguing about.
( I will not digress into the oxymoron of talking of surface temperatures when all the fuss of AGW is made with temperatures measured at 2 meter height.)

Vergent
January 14, 2012 10:40 pm

Willis,
Your error is this. You said:
“But when the temperature of a perfect blackbody planet rises … the surface radiation of that planet must rise as well.”
This is true for a body of uniform temperatureAND PRESSURE.
What was needed for your “proof”:
“But when the average temperature of a perfect blackbody planet rises…the surface radiation must rise.”
This is not true. consider the moon; cool the hot side, and then warm the cold side until the total radiation is equal. You will have warmed the cold side more because of the S-B equation. The average temperature will go up.
Vergent

Vergent
January 14, 2012 10:43 pm

Whatt is amusing here is the time it took to achieve a logical conclusion.

Graham of Sydney
January 14, 2012 10:47 pm

“…while the temperature can be lower than the theoretical S-B temperature, as is the case with the moon”
Actually it’s 20° cooler by day and 60° warmer by night, the net result being a surface that is 40° warmer than the S-B prediction. S-B applies to a surface and takes no account of heat absorbed and re-radiated by the subsurface, an observation made by other readers here.
http://www.ilovemycarbondioxide.com/pdf/Greenhouse_Effect_on_the_Moon.pdf
NASA have long known about this contradiction to the greenhouse theory, but presumably sat on it for sinister reasons that are all too familiar now.
http://climaterealists.com/?id=5783

gbaikie
January 14, 2012 10:49 pm

“Willis, are you saying that if I am suspended in space, and a stream of 8,000 degrees N2 jets by me at say 10′ away, (nothing but the vacuum of space between me and the gas) I will feel nothing and my thermometer will register nothing, but if that stream of gas IS COMPOSED OF CO2 and jets past, again with 10′ of the vacuum of space separating me from the gas, I will feel immense heat from the radiating CO2, but none from the non radiating N2?”
A stream indicates they all going in one direction- they aren’t hot unless they hit something.
But even if they are hitting each other and not you, there is no heat to you.

Vergent
January 14, 2012 10:56 pm

Nice try, 20 down bye day and 60 plus by night, is a global temperature rise of 20C. This negates the premise of the argument.

January 14, 2012 11:10 pm

[SNIP: please, no philosophical speculations on gravity. It’s a field. To move against it takes energy. To move with it gives you energy. No one knows why. w.]
Since Q= U + W (simply stated)
W= FD (simply stated)
W is a path function which means W1-2 can be more or less than W2-1 then I think you have stated the N&Z idea. Gravity causes work(compression) that adds energy to the atmosphere. Convection work reduces energy in the atmosphere and the two do not have to be equal.

Bob Fernley-Jones
January 14, 2012 11:11 pm

PaulR @ January 14, 9:59 pm

Yes he [Willis] did cancel the cosine law, he did it by assuming a planet equally lit by thousands of small suns. Read the OP? Twice?

Sorry Paul, in a hurry, I overlooked that minor point of model definition. Me go and smack my wrist!

Vergent
January 14, 2012 11:13 pm

They are probably going to erase it but from 20% into this thread until 80% The authors of this thread were bitchen an moaning and smitten at each other.

Vergent
January 14, 2012 11:14 pm

U know what I mean.

gbaikie
January 14, 2012 11:16 pm

Open question; Surface heat conducts to the non GHG atmosphere, Can non GHG molecues conduct heat to the surface?.
Yes. Any gas does this.
It doesn’t mean it will occur quickly.
Put anything cold into room at room temperature. The main way it could lose heat is thru conduction, reduce the losses in conduction and main way it warms is due to air molecules- convection.
Have cold jacket, hang from chair it should mostly warm from convection. If put near fire in fireplace, it would be largely from radiation. Radiation works good, if it’s 2000 F

ferd berple
January 14, 2012 11:19 pm

I’d appreciate hearing back from anyone willing to wade through my post of
ferd berple says:
January 14, 2012 at 6:30 pm
I cleared up the points Willis and a few others had and the result still comes out the same. The crux of my approach was to not try and calculate the specific amounts of the various energy fluxes. Rather to simply deal with them as inequalities, as it is much easier to say if one flux is larger than another, even if you can’t say by how much. This allows one to deal with the uncertainties and arrive at an answer that would otherwise be impossible to resolve.

January 14, 2012 11:25 pm

Phil’s Dad says:
January 14, 2012 at 8:55 pm
“* By definition it can not radiate this heat away. It will however pass some of this heat back to the ground to restore equilibrium at the conductive layer. (Much more slowly than conductive warming as convection constantly interferes.)”
That part doesn’t sound right, heat is thermal energy in transit and energy will travel through the path of least resistance to restore equilibrium, less resistance equals shorter distance and shorter distance equals a more efficient transit of energy.
Principle of least action: http://en.wikipedia.org/wiki/Principle_of_least_action

Paul Dennis
January 14, 2012 11:27 pm

Willis,
494 responses and counting…wow!
Your redcutio ad absurbum argument is elegant and shows conclusively that in there can be no gravitational effect. End of as far as I’m concerned.
What is an interesting physics question is in your model what will the temperature distribution in the non-GHG atmosphere? I think it might be constant throughout with no convection and therefore no adiabatic lapse rate. The atmosphere in this model is effectively a gas column in an adiabatically bound state, heat conduction throughout the column will lead to an equilibrium of constant temperature distribution. I think Roy Spencer may have indicated this to be the case as well in his earlier comment.
Thank you for an elegant disproof of gravitational theorems.

ferd berple
January 14, 2012 11:46 pm

ps: the correction I added was the energy flux H, which is the net flow of energy from the surface to the ghg atmosphere, which several readers mentioned as missing from the proof.
The solution relies on the inequality H < C. Specifically that
net energy flux from surface to ghg < net energy flux from ghg atmosphere to space.
Keeping mind that we are talking net energy flux, this assumption seems supportable. The ghg atmosphere must itself absorb energy from the sun, which it then radiates to space. Thus in equilibrium the net transfer from the atmosphere to space must be greater than the net transfer from the surface to the atmosphere.
If this inequality does not hold, then that would be an obvious starting point to attack the proof.

Alan Wilkinson
January 15, 2012 12:01 am

Tim Folkerts, I don’t think you are correct for the conditions Willis specified (accepting Anna V’s point that they are unphysical at the limit). The adiabatic conditions are not satisfied at equilibrium so an isothermal condition is created and is neutral towards convection. With the atmosphere at the same temperature as the surface there are no sources of hot or cold spots to disturb it.

dr.bill
January 15, 2012 12:05 am

When I opened this thread there were already about 500 comments, so I haven’t read the bulk of them, and if someone else has already said what I’m about to say, I offer them my apologies.
Willis: I’m completely ignoring your “elevator speech” request, so feel free to just ignore this comment. For the rest of you, I will create a contradiction of the type that Willis appears to appreciate. Here’s what I have to say:
(1) I’ll assume that the “suddenly added atmosphere” is at a lower initial temperature than the surface of the planet. I will use the term “surface” to refer to the hard physical surface of the planet, and the term “air” to refer to the molecules of the atmosphere, regardless of what species these molecules might be. I will also ignore the possibility of radiation produced by scattering of any kind, and from the natural radiation of the atmosphere in the far IR region, even though both of these will necessarily occur.
(2) What happens just after the air is added is that the lowest thin layer will absorb energy from the surface by conduction, thus slightly reducing the temperature of the surface. That warmed thin layer of air will then conduct to the cooler air above it, will thus cool itself, and then be ready to accept re-heating by conduction from the surface. Under this logic, the process should then continue, propagating upward until the entire atmosphere has reached a single uniform temperature, namely the one possessed by the surface before any of this started.
(3) So we would seem to have an isothermal atmosphere, and things are back to the way they were at the start, except for a brief interlude during which the surface cooled a bit while heating the air, and then came back up to its original temperature. But there’s a problem with this scenario, namely that you can’t have an isothermal atmosphere in a gravitational field if it isn’t being heated (given energy) from the top.
Digression: We do have (approximately) such an isothermal situation in our real atmosphere, i.e. in the stratosphere, which has a fairly uniform temperature from the tropopause right up to the stratopause. Why does that occur? It’s because the top of the stratosphere is where the incoming UV starts to split up air molecules (notably diatomic oxygen, thus producing ozone). This, among other things, converts UV energy into thermal energy for whatever is up there. The effect gets reduced as you progress downward, so there is less heating in the mid-stratosphere than in the upper part, and by the time you get down to the tropopause the effect has run its course. The overall effect is that the decrease in temperature that “should” have continued upward from the tropopause has been negated by the absorption of incoming UV energy higher up, and the whole region is at a fairly uniform temperature (as long as the UV keeps coming).
(4) Back to the steady-state and supposedly isothermal atmosphere attained without the injection of extra energy at the top, the problem is that all of those molecules, which are supposed to have the same Kinetic Energy (same temperature), also now have gravitational Potential Energy. The system, however, is supposed to have reached a stable state where nothing is changing, so at each level the total energy of any thin layer should remain fixed. Another way to say this is that dU = 0.
(5) From basic thermodynamics, we have the relationship that dU = CpdT + gdh, so if dU is zero, then dT/dh = -g/Cp, which means that there is a lapse rate of the usual kind, and things get warmer as you go down. This implies that the new temperature of the air at the surface must be higher than the surface was at the beginning. I have no clue what the surface itself is doing, or what it’s temperature might be, but the AIR at the surface is warmer.
And that’s my take on the matter…
/dr.bill

TimC
January 15, 2012 12:24 am

Willis said “TimC, you are right. You don’t even have enough knowledge to ask intelligent questions. I don’t wish to be cruel, but in such a situation, just listen and learn, OK?”
A bit snippy, surely. I only have a science masters from long ago (my career was law), but I am still trying to learn – I thought that’s what this site was about …
From Wikipedia (I know, I know): “Thermal radiation is electromagnetic radiation generated by the thermal motion of charged particles in matter. All matter with a temperature greater than absolute zero emits thermal radiation.”
And: “A greenhouse gas (sometimes abbreviated GHG) is a gas in an atmosphere that absorbs and emits radiation within the thermal infrared range.”
Am I right (to ask a question directly this time, rather than offer a comment) that SB is concerned with total thermal (ie electromagnetic) radiation – whether it falls within the IR, visible or UV parts of the spectrum?

Alan Wilkinson
January 15, 2012 12:25 am

“The atmospheric temperature and pressure drop as altitude increases with the temperature ultimately dropping to that of deep space (~3K).”
Objects in space have a temperature, space itself does not. 3K is the background microwave radiation of space. A non-radiating object (gas) would maintain its temperature.

Paul Dennis
January 15, 2012 12:30 am

Willis,
I don’t think I agree with you here and am with Roy Spencer on this point. I think the the lapse rate does require convection. In an adiabatically bound column of gas in a gravitational field, which I think your model is, the temperature is constant throughout the column. The tradeoff between kinetic and potential energy has been discussed as the ‘Loschmidt effect’ and treated by the following authors:
Coombes, Ch. A. and Laue, H., 1985, Am. J. Phys, v53, 272-273
Velasco, S., Roman, F.L. and White, J.A., 1995, Eur. J. Phys. v17, 43-44
Qualitatively, since both the kinetic energy of the molecules and the number density of the molecules decreases with height, the average molecular kinetic energy does not necessarily decrease with height. The average molecular kinetic energy is the summation over all values of molecular energies divided by the number of molecules in any specific volume element. This average kinetic energy is constant throughout a column of gas in a gravitational field.
Interestingly there is an interesting discussion of this over at Tallblokes. I’m with Roy Spencer on this one. However, I’m also going to go back and give some more consideration to the lapse rate to see if I’ve missed something. Keep up the good work.

Tenuc
January 15, 2012 12:49 am

Vince Causey says:
January 14, 2012 at 9:07 am
“…But, if the lower atmosphere is warmed by the gravity effect, there would indeed by a temperature gradient, with temperature declining as you go higher. At this point, it could be imagined that there is a TOA higher up radiating the same amount of energy that is received from the Sun.
But. . .the crucial point is, these molecules in the atmosphere cannot radiate. To do so would require molecules of N2 and O2 to emit photons. As far as I am aware, they do not. The only way energy can leave the Earth is by radiation, and only the surface can radiate…”

Some very good points made in your post, Vince, except all matter does in fact radiate…
“Radiation:
Thermal energy emitted by matter as a result of vibrational and rotational movements of molecules, atoms and electrons. The energy is transported by electromagnetic waves (or photons). Radiation requires no medium for its propagation, therefore, can take place also in vacuum. All matters emit radiation as long as they have a finite (greater than absolute zero) temperature.”
http://www.eng.fsu.edu/~shih/eml3015/lecture%20notes/radiation.htm
At higher temperatures matter emits more photons. Also the number of photons being emitted depends on the density of the matter. The temperature of the atmosphere will be the same as the surface of the planet at the boundary between the solid and gas. It is also obvious that the surface of the planet contributes significantly more photons as it is significantly denser than air (~3000 kg/m3 vv ~1.225 kg/m for air at sea level and at 15°C).

RobB
January 15, 2012 12:50 am

I know this isn’t the model Willis used but what is the outcome if the IR transparent atmosphere can radiate energy?
I’m interested in the limiting cases of trace GHG gas in the atmosphere, what happens in a IR opaques atmosphere and also a GHG free atmosphere that includes the water cycle but without the IR properties of the water vapour.

RobB
January 15, 2012 12:57 am

“I know this isn’t the model Willis used but what is the outcome if the IR transparent atmosphere can radiate energy?”
Just to add, I think this is the most interesting model in that it appears to act something like the earth’s atmosphere but can obviously be seen to let surface radiation directly into space, so doesn’t have any direct GHG absorption properties

Konrad
January 15, 2012 12:59 am

Willis,
Do I have a simple “elevator explanation of the N&z hypothesis”? Yes. Are you getting it? No.
Not until you apologise to Roger and many others.
[SNIP: Well, I guess we’ll never hear your brilliant elevator speech if that’s the case. I gave my apology to roger above, that’s done, not sure why you bring it up again but that’s it. w.]

Dr Burns
January 15, 2012 1:09 am

Willis said
>>I don’t care if I ignored cloud cover, I’m talking about a THOUGHT EXPERIMENT THAT HAS NO CLOUDS.
Your model is quite pointless if you’re not trying to understand what goes on in the real world. A black solid sphere with a transparent atmosphere that has no convective circulation, no lapse rate, no clouds, no evaporation, no water and upon which only radiation acts, what do expect ?

dr.bill
January 15, 2012 1:41 am

re Willis Eschenbach, January 15, 2012 at 12:52 am :

However, statement (2) is where the train of thought gets derailed. The adiabatic lapse rate doesn’t magically disappear. It is still g / Cp, just like before. The molecules at the top of the atmosphere don’t magically convert their potential energy into kinetic energy. The atmosphere will not become isothermal. It will become isentropic.

I see your point, Willis. The logic I used in coming to my conclusion involved nothing but quasi-static reversible adiabatic processes, which by definition are also isentropic, but I was ignoring my own point (5) while thinking through point (2).
As far as Potential Energy is concerned, all of the molecules had some of that to start with, but now they’ve had extra energy added from below. I was assigning all of that to temperature increases in the molecules above, but forgetting that some of it will go into pushing those molecules to a higher altitude than they were at the start, so the atmosphere will expand until a balance is reached.
Back to the drawing board… ☺
/dr.bill

Bryan
January 15, 2012 1:50 am

Paul Dennis says:
“Willis, I don’t think I agree with you here and am with Roy Spencer on this point. I think the the lapse rate does require convection.”
It does not require convection in our adiabatic atmosphere

Bart
January 15, 2012 1:53 am

Willis Eschenbach says:
January 14, 2012 at 3:49 pm
“Since my description is of a system at equilibrium, I fail to see how any of that applies, so my QED is not disturbed by your claims about systems not at equilibrium”
Wrong, Sir. Wrong!

In thermodynamics, a thermodynamic system is said to be in thermodynamic equilibrium when it is in thermal equilibrium, mechanical equilibrium, radiative equilibrium, and chemical equilibrium. The word equilibrium means a state of balance. In a state of thermodynamic equilibrium, there are no net flows of matter or of energy, no phase changes, and no unbalanced potentials (or driving forces), within the system. A system that is in thermodynamic equilibrium experiences no changes when it is isolated from its surroundings.

You have a conductive and convective medium in direct contact with your surface with which you are exchanging heat continuously. The entire surface-atmosphere system can be said to be in equilibrium, but you cannot break up that system and claim the individual pieces are in equilibrium. That would be the equivalent of drawing a free body diagram in mechanics and claiming that there are no contact forces, so the separated bodies move only according to the external forces applied directly to them.
What you are continually failing to understand is that SB only works for a body in thermodynamic equilibrium. When you are not in thermodynamic equilibrium, then the energy distribution is not, in general, going to be Planckian. And, SB does not, in general, apply.

kwik
January 15, 2012 1:55 am

Willis, I think you have stopped using only the word “radiate”, when you mean “radiate in the IR spectrum”. Later in the thread you use “radiate in the IR spectrum”. Good.
This is what created some confusion early in this thread.
Okay;
You have agreed that non-GHG gases conduct energy.
Elevator speech for N&Z;
The non-GHG-gas can increase its energy via conduction at the surface.
This means the gas gets a higher temperature. Via conduction.
Via “bouancy” it will rise.(Figure of speech; Its density lowers, so it rise)
When ascending it looses its energy (via conduction with other gas molecules) until “bouyance” equals gravity. Then it stops.
The air-parcels close to ground that ascended is replaced by others, creating a circulation.
Conduction increase with increased density.
Density increase when the athmosphere has more mass.
This is not a perpeteum mobile; The energy comes from the sun.
It does not violate any laws. It radiate finally to space at TOA, instead of at surface.
So looking from space, you see the same energy radiated..
Not sure I am correct but I have decoded N&Z to this.
Call me stupid and ask me to go away if you like. I dont care.

Steven Hoffer
January 15, 2012 1:56 am

by the time i got to to this thread there was far too many posts to read them all. I’ll stick with an “elevator” explanation as i see it.
If a sphere in space has black body characteristics,
If a clear atmosphere has no interaction with incoming radiation,
If the incoming radiation Is equal on all sides
If there is no motion of the sphere to stir up the atmosphere with a coriolis effect.
1. The black body sphere with no atmosphere will radiate exactly as much energy out as it recieves, and have a temperature relating to its physical properties.
2.the addition of an atmosphere initially causes an energy imbalance, since the atmosphere will recieve energy from the black body surface through conduction. convection occurs.
3. eventually every molecule in the atmosphere contains the same amount of energy
4. when every molecule has the same amount of energy, convection stops, as does conduction. the system becomes stable and stagnant. no energy moves in or out of the atmosphere.
5. The black body sphere again radiates exactly as much energy as it recieves.
6. one square foot of the atmosphere at high elevation contains less energy than one square foot of atmosphere at low elevation strictly due to atmospheric density. the “Temperature” is supposedly the same at the top and bottom.
7. the temperature of the black body must be the same as the temperature of the atmosphere, whether or not a black body molecule contains as much energy as an atmosphere molecule, they have the same temperature.
8. The temperature of both the black body and the atmosphere is determined by the whichever material of the two requires less energy to increase temperature 1C.
9. thus when the atmosphere requires less energy to increase its temperature, it will determine the temperature of the black body. if the black body needed less energy to increase temperature, it would determine the temperature of the atmosphere.
Possibly no one thinks about the system like this but there is a disparity between the Heat Capacity of atmospheric gases and solids like rock, and no one has ever considered that the black body could just as easily affect the temperature of the gas as the other way around.
without doing an ounce of math I would contend that the end temperature of the system would HAVE to depend exclusively on the material requiring LESS energy per degree of temperature increase.
I have followed the warming debate for several years now, but do not have any physics or chemistry education beyond a poorly remembered highschool education followed by a couple of mispent years taking introductory courses at university. I’m NO EXPERT, and the things i DO remember are admittedly a little bit foggy. I don’t believe i’ve violated the law of conservation of energy here, nor do i think this thought experiment follows through to the real world. the real world has more variables. What I do think I’ve done is show how a black body could change temperature due to the presence of a non radiating atmosphere.

kwik
January 15, 2012 1:57 am

Correction ; When I say “It radiate finally to space at TOA, instead of at surface.” I meant the energy that was removed from the surface via conduction.

John Marshall
January 15, 2012 2:15 am

I am disappointed that Willis cannot be bothered to read all I blogged. I ask a question and he ignores it and SNIPs it.
Willis claimed that it is impossible for a planet to radiate more heat than it receives from its sun. NOT TRUE! Jupiter ans Saturn both do this, ask any astronomer.
If he can’t be bothered to answer this relevant question then my estimation of him as a scientist and person have gone down the pan.

Paul-in-UK
January 15, 2012 2:27 am

I’m just learning and trying to make sense of both sides of the argument. My interpretation is this:
The Earths’ surface … [SNIP: I’m talking about the situation in the blackbody planet described in the head post. Please restrict your comments to that, and leave the Earth for another thread. Thanks. w.]

Geoff Sherrington
January 15, 2012 2:35 am

LazyTeenager says:
January 14, 2012 at 7:28 pm
Hoser quotes Willis and says
A curious fact is that almost everything around us is continually radiating energy in the infrared frequencies. You, me, the trees, the ocean, clouds, ice, all the common stuff gives off infrared radiation.
>>>>>>>>And the air.
No. The air does not radiate as a black body.
The air radiates according to the ability of its component molecules to radiate……..
Why not pop back and read my sole contibution way at the beginning. January 14, 2012 at 2:45 am
If an atmosphere of mainly nitrogen and oxygen DID give off copious amounts of IR, the FLIR camera would not see distant objects because the atmosphere would obliterate their presence. It DOES see distant objects at wavelengths far away from the response of the human eye, like 20 microns or so. As a former spectroscopist, I’m prepared to conceded – without going back to the texts – that a small amount of energy at some frequencies that do not trouble a FLIR device might indeed exist, but on a scale so small as not to upset this hypothetical.
Now, having ploughed through all of the above, we have to agree that Willis was correct, with the exception of tiny effects that do not detract from his thought experiment. And yes, there was one correction, that a change in temperature, via Wein’s displacement law, changes the peak wavelength of an emitting body, but is silent about the total energy under the curve.
This used to be first year textbook stuff. Stefan-Boltzmann, Wein, Laws of Thermodynamics, Kirchoff. Are they no longer on the required reading list?

anna v
January 15, 2012 2:47 am

Dear Willis
Transparency is in the eye of the beholder. For transparency to work in your argument the atmosphere should be transparent to all electromagnetic frequencies, which is not possible for matter as we know it. To have heat capacity, the molecules must have kinetic energy and interact by scattering, the scattering happens because of exchange of photons, photons are electromagnetic radiation at some frequency, even if very low..

tallbloke
January 15, 2012 2:54 am

Willis sez:
a “gedanken” experiment in German, they were a great favorite of Albert Einstein for the same reason that I use them—to simplify complex questions so that they can be understood.

There is utility in gedanken experiments, and also danger. Things can be simplified to the point of meaninglessness wrt the real world you hope to gain insights into via the thought experiment. Another danger is that having appeared to prove something via such an experiment, insights of real value in longer than elevator speech length tracts may mistakenly get rejected a priori and without proper consideration.
As Hans Jelbring pointed out to Willis above, there is no elevator version of his peer reviewed 2003 paper. It isn’t overly long, but clearly longer than Willis is prepared to countenance. If Willis had read it carefully in 2003, he wouldn’t have made the error of thinking that his own gedanken experiment falsifies it.
Willis said to me above:
[Tallbloke, every post can be claimed to have scientific content, including yours. I said that I would snip things that were off-topic, which your post most assuredly was. If you (or anyone else) think your post contained actual science that has been deleted incorrectly, then post the scientific part again and we can discuss it. -w.]
Thank you. It’s a disproof of your assertion that Hans Jelbring’s paper is falsified by your argument that:
“The proof is by contradiction. This is a proof where you assume that the theorem is right, and then show that if it is right it leads to an impossible situation, so it cannot possibly be right.
So let us assume that we have the airless perfectly evenly heated blackbody planet that I spoke of above, evenly surrounded by a sphere of mini-suns. The temperature of this theoretical planet is, of course, the theoretical S-B temperature.
Now suppose we add an atmosphere to the planet, a transparent GHG-free atmosphere. If the theories of N&K and Jelbring are correct, the temperature of the planet will rise.
But when the temperature of a perfect blackbody planet rises … the surface radiation of that planet must rise as well.
And because the atmosphere is transparent, this means that the planet is radiating to space more energy than it receives. This is an obvious violation of conservation of energy, so any theories proposing such a warming must be incorrect.
Q.E.D.”

But in Hans Jelbring’s paper, the model planet by definition does not radiate to space. At the beginning of section 2.1 in his paper he states:
“A simplified model of Earth will be considered. The model planet does not rotate. It
neither receives solar radiation nor emits infrared radiation into space.”

Your ‘disproof’ of Hans Jelbring’s 2003 paper therefore fails.
QED
Now, Willis, somewhere in the foregoing mess of comments and deletions, you asserted that my argument is incorrect, but you did not in any way shape or form say anything to back up that assertion. So let’s have the discussion you offer above, and you can start by enlarging on your rejection of my argument.
Cheers
TB.

Kev-in-Uk
January 15, 2012 2:56 am

I have a relatively simple question. Are we considering that gravity does ‘work’ in the context of an atmosphere of some ‘mass’ being attracted downwards towards the centre of a body? If gravity is doing ‘work’ by attraction, then it must logically produce some energy somewhere, even in simple kinetic terms, if I ‘pull’ something towards me, I am exerting a force and doing ‘work’, the rubbing of the object (e.g. a molecule) against other molecules and producing tiny amounts of frictional heat as a by-product (if you like) of my ‘work’. So, in terms of Willis’ transparent non radiative atmosphere, it makes no difference if the atmosphere has some ‘mass”. Gravitational induced kinetic energy (i.e. movement or work) might be small, but it is clearly present and increases as an acceleration due to constant gravitational force? By conservation of energy, the gravitational ‘pull’ must conserve to some other form of energy once the object being ‘pulled’ comes to rest or hits another object – what’s wrong with it being converted to heat?

Bob Fernley-Jones
January 15, 2012 2:58 am

Willis Eschenbach @ January 15, 12:31 am
Willis, you triumphantly proclaimed to me:

Read the head post, Bob. And if you can’t figure out the answer to your question, don’t come back.

Willis, you clarioned don’t come back, according to YOUR beliefs?
No Willis; you are very, very naughty! You must explain how a planetary body could possibly ever have a uniform body temperature, and even if it is contained in your fertile imagination, what has that got to do with the price of cheese anyway?
Put another way, what has your very silly model got to do with the price of cheese, or Cornish Pasties or anything?

Paul Dennis
January 15, 2012 2:59 am

Bryan says:
“It does not require convection in our adiabatic atmosphere”
As I said before I think convection is needed. The definition of the dry adiabtaic lapse rate is:
“The dry adiabatic lapse rate (DALR) is the rate of temperature decrease with height for a parcel of dry or unsaturated air rising under adiabatic conditions.”
http://en.wikipedia.org/wiki/Lapse_rate
A rising air mass is a convecting air mass. The parcel of air does work during expansion and therefore cools. If there is no convection, which I think is the outcome of Willis’ thought experiment then the temperature of the atmospheric column will become that of the surface of his black body. This is covered in the following paper:
Coombes, Ch. A. and Laue, H., 1985, A paradox concerning the temperature distribution of a gas in a gravitational field. Am. J. Phys, v53, 272-273

Kev-in-Uk
January 15, 2012 3:12 am

I should have added that if we have a moving atmosphere, e.g. via convection due to incoming radiation warming a surface, which then warms the atmosphere and sets up convection currents, then we have a situation where gravity constantly acts to ‘repull’ molecules back down after they have risen via convection. Hence, it is not beyond the realms of possibility to see that the atmosphere is ‘warmer’ than the sum of the incoming radiation as a result of the ‘work’ of gravity?

January 15, 2012 3:18 am

Stephen Wilde says:
January 14, 2012 at 10:19 am
The contributor known as wayne previously summarised my position on this issue which has been in my mind for several years as a very important aspect of the climate debate.
Willis did not address wayne’s version so here it is in my words:
[SNIP: vague, wandering, unscientific, and off topic. -w.]
Incredible.
It was precise, on topic and a clear rebuttal to Willis’s contentions.
Anthony needs to realise that tjhis might as well be a Realclimate thread.

January 15, 2012 3:20 am

A guest author gets to moderate and snip comments on his own thread…..this is not good?

January 15, 2012 3:36 am

Here it is again (approximately, since I didn’t save the slightly refined version).
“The heat within the flames of a campfire would get hotter with more molecules (increased air
density) around the flames because the heat energy is moved away more slowly
due to the higher number of molecular collisions.. So the density of the air
around the campfire increases the temperature gradient from fire to
observer. In effect the denser atmosphere obstructs radiation leaving
conduction relatively more important. Conduction is a slower process than
radiation so the temperature within the flames rises
In exactly the same way the denser the Earth’s atmosphere the hotter the
surface becomes and the steeper the temperature gradient upward because
space remains at the same temperature but the surface gets hotter. Just as
with the campfire the solar energy hitting Earth’s the surface is moved away
more slowly because the role of slow conduction is enhanced relative to that
of fast radiation as a result of the greater atmospheric density.
Density of the air at the surface is a result of the strength of the
gravitational pull of the entire planetary mass and the mass (not
composition) of the atmosphere.
So, indirectly, through pressure and then density, gravity does determine
the lapse rate and it is mass dependent and not composition dependent so
Oxygen and Nitrogen are involved despite being relatively non radiative.
Oxygen and Nitrogen participate fully in adding to the process of conduction
in its competition with radiation.
The ability of increased conduction to slow down radiative energy loss is
what relegates radiative processes to a secondary role and explains why it
is gravity rather than radiation that sets the lapse rate.
Gravity and density alter the balance between fast radiation and slow
conduction. If one reduces radiation and increases conduction the heat
content and temperature will rise given the same energy input.”
I think it sidesteps a lot of the problems if one regards density as
shifting the balance from fast radiation to slower conduction giving a rise
in equilibrium temperature as a consequence.
After all, no atmosphere means an immediate turnaround of energy i.e.
radiation straight in and straight out pretty much instantly. As soon as one
adds an atmosphere capable of CONDUCTION which includes non GHGs then the
conduction takes away from the efficiency of the radiation process by
slowing energy dissipation down which is what then leads to the higher
equilibrium temperature. The denser the atmosphere the more conduction takes
place before the radiative energy can be released to space and the higher
the equilibrium temperature rises.
So, radiative processes are not in control because they are subject to
interference from density and the consequent increase in conduction.
Convection and the water cycle then act to try to reduce the slowing effect
on energy dissipation of more conduction but can never get back to the
efficiency of raw, in/out, radiation.
Neat isn’t it ?”
If it is wrong then a simple explanation as to why would be preferable to deletion.

January 15, 2012 3:43 am

Stephen Wilde says:
January 15, 2012 at 3:18 am
Anthony needs to realise that tjhis might as well be a Realclimate thread.
I am with you Stephen…so many guest experts pushing AGW themes, the PDO and solar influence on climate do not exist, as well as any new science on gravity heating.
WUWT?

January 15, 2012 3:47 am

I am also still awaiting responses from Willis on the following points:
1) Willis said:
“Since there is gravity, the atmophere will have a “dry adiabatic lapse rate”, which means that the temperature must drop with altitude. The atmosphere will warm until the bottom layer of the atmosphere has the same temperature as the surface, and has the dry adiabatic temperature profile above the surface. It will neither gain nor lose energy after that, and will be stable with no bulk motion.”
Ok, you’ve accepted the gravity induced dry adiabatic lapse rate.
And you seem to accept that the warming is from the solar irradiated surface and that the lapse rate is supported by conduction from the surface.
That is then the baseline gravity induced GHE as per N & Z and the Ideal Gas Laws. Nice and stable and set by gravity and atmospheric mass alone.
Then one introduces GHGs which have two effects.
They absorb more energy due to their radiative characteristics.
They then radiate 50% up and out of the system and 50% back down to the surface.
The 50% sent upward reduces total system energy content because it is lost to space. That is a cooling process.
The 50 % sent downward destabilises the gravity induced GHE but in turn provokes more convection and on a water planet energises the water cycle too.
Now, convection and the water cycle are cooling mechanisms (evaporation has a huge net cooling effect of 5 to 1 – see latent heat of vapourisation) so that 50% sent downward must be all or mostly negated unless you can show otherwise and the N & Z data seems to show that the negation is pretty much complete.
Which leaves the (admitted) gravitationally induced GHE firmly in control does it not ?
Checkmate ?
2)
Willis said:
“If there are no GHGs, the surface must radiate (to space, since there are no GHGs) the amount of energy it absorbs. Its radiation is fixed and unchangeable”
You forgot something.
The Ideal Gas Law means that the warmest molecules of air are at the surface.
Those molecules are at a higher temperature than the average for the atmosphere.
Thus they will inhibit upward energy transfer more than would be the case if the atmosphere were at a cooler average temperature throughout.
That will give a higher surface temperature than predicted by the S-B equation.
3)
Willis said:
“That’s why the surface can get hot, because some of the energy radiated by the surface is absorbed by the atmosphere so we’re not emitting to space more that we’re absorbing.”
Which is why the equilibrium temperature rises intead ?
In each case I have quoted Willis’s specific words as he has requested of others several times in this thread.

Bomber_the_Cat
January 15, 2012 3:51 am

Ed Fix says:
January 14, 2012 at 7:54 pm
“Carbon dioxide happens to absorb a wavelength of EM near the peak of the earth’s overall S-B radiation spectrum (about 15 micrometers), but it re-emits S-B radiation in a smooth spectrum depending on its temperature only. Just like any other body of matter.”
No it doesn’t Ed. It can only emit at the wavelengths it can absorb, so CO2 emits at 15 microns (it also has some other distinct absorption lines around 2.7 and 4.2 microns, from memory). But it does not emit as a blackbody. Thin gases do not behave as blackbodies (see my earlier posts).
Tenuc says:
January 15, 2012 at 12:49 am
“All matters emit radiation as long as they have a finite (greater than absolute zero) temperature.””
Tenuc, we can and do measure the absorption and emission properties of atmospheric gases. You can find such rsults anywhere on the internet. Atmospheric gases do not behave like blackbodies and only emit at specific, well defined wavelengths. Oxygen and Nitrogen, for example, do not emit in the long wave infrared part of the spectrum. If you want to reconcile this with your citation, just consider that these gases have an emissivity of zero in this spectral region. Thin gases do not behave as blackbodies (see my earlier posts).

bananabender
January 15, 2012 3:55 am

The heating of the atmosphere is a purely physical process according to the Ideal Gas Law.
“The Elevator speech”:
Gravity acts as piston compressing the atmosphere.
The atmosphere is then heated by this compression (think of a bicycle pump getting hotter as you pump).
As you get closer to the Earths surface it becomes hotter because the atmospheric pressure is greater (or cooler with increasing altitude and lower pressure).
Convection, evaporation and condensation transfers heat from the surface to upper atmosphere.
Heat radiates then from the top of the atmosphere to space (an infinitely large heat sink).

johndo9
January 15, 2012 4:19 am

I have just been re-reading Hans Jelbring. He does indeed start his model with- Quote “The model planet does not rotate. It neither receives solar radiation nor emits infrared radiation into space.”
With no energy passing through there is no trapping of energy and no Greenhouse Effect.
As such its pretty much irrelevent to what Willis has defined here.
His condition that the atmosphere cannot absorb or emit any radiation requires all radiation to be to or from the surface.
As others have shown the “average surface temperature” is below what Willis called the “theoretical Stefan-Boltzmann (S-B) temperature.”
He is right, all the atmosphere can do is raise the “average surface temperature” up towards the “theoretical Stefan-Boltzmann (S-B) temperature.”
So to Nikolov and Zeller and the elevator speech. But first its important to understand they use a GHG atmosphere,
Quote
“An increase in atmospheric emissivity does indeed cause a warming at the surface as stated by the current theory. However, Eq. (3) is physically incomplete, because it does not account for convection, which occurs simultaneously with radiative transfer. Adding a convective term to (3) (i.e. Eq. 4) dramatically alters the solution by collapsing the difference between Ts, Ta and Te and virtually erasing the GHE”
I thought the elevator speech from
astonerii says:
January 14, 2012 at 3:25 pm
was doing fairly well except maybe for the lapse rate bit, but no comment from Willis.
Perhaps I can give it a try (apologies to Nikolov and Zeller if I get too far off).
The N & Z process works as follows (note it does include GHGs):
The surface loses energy by conduction to the atmosphere.
The majority of this energy rapidly convects to the troposphere.
The surface of the earth emits energy in the form of thermal longwave radiation.
Some of that energy is absorbed by greenhouse gases (GHGs) in the atmosphere.
In turn, some of that absorbed and convected energy is radiated by the atmosphere back to the surface.
As a result of absorbing that energy from the atmosphere, the surface is warmer than it would be in the absence of the warm (GHGs) atmosphere.
Thats the speech.
The overall temperature seen from space is still the “theoretical Stefan-Boltzmann (S-B) temperature”, unfortunately their section on pressure and energy is contradictory.
They say “Pressure by itself is not a source of energy!” and in the same section ” the atmosphere ……….is in and of itself a source of extra energy through pressure.)
They are confusing correlation with causation, but there are many more questions about how all this works.
However I better stop there as this thread has had lots of snip and delete (it must be rivalling “Realclimate”).
So I must not incur the wrath of Willis for being off subject.

LazyTeenager
January 15, 2012 4:20 am

Hail Core says:
January 14, 2012 at 1:40 am
I have talked with one engineer who is working infrared warmer manufacturer company. They have tried to warm air with infrared warmer and they have never managed to do so. They have even set tens of radiators to work at same time (hundreds kilowatts), no measurable results in air temperature. So the claims that infrared radiation warms atmosphere can’t be correct
————
The observation about IR radiant heaters sounds correct but your interpretation is wrong.
1. Over the short distances within a room you are not going to see much absorption and therefore only a small temperature rise.
2. The absorption band for IR radiation matches room temperature at around 25C. It does not match the temperature of a radiant heater. Typically these operate around 800C. Again this means only a small fraction of the heater output is absorbed.

anna v
January 15, 2012 4:21 am

anna v says:
January 15, 2012 at 2:47 am
Dear Willis
Transparency is in the eye of the beholder. For transparency to work in your argument the atmosphere should be transparent to all electromagnetic frequencies, which is not possible for matter as we know it. To have heat capacity, the molecules must have kinetic energy and interact by scattering, the scattering happens because of exchange of photons, photons are electromagnetic radiation at some frequency, even if very low..

I want to elaborate this.
What is black body radiation? why do bodies at a given temperature radiate even if not a perfect black body?
In a solid the atoms and molecules are on a three dimensional grid, vibrating about their position there with three degrees of freedom. As they vibrate they move in the left over electric fields of each other (Wan der Waals forces are related), motion in electric fields generates radiation and this radiation gets out of the solid at the frequencies where the solid is transparent and thus the solid cools because energy is lost.
In gases there is the continuous scattering of molecules in the fields of each other, and again radiation is induced which leaves and cools the gas.
Quantum mechanics modifies this picture by having photons mediate the interactions, sometimes off shell, some times on and leaving taking some of the energy of the interaction away.
Thus one cannot have matter as we know it of a given temperature not radiating in some frequencies.

Roger Clague
January 15, 2012 4:43 am

‘Energy is radiated by the atmosphere back to the surface’.
This is not consistent with the 2nd law of Thermodynamics. Heat flows from hot to cold.
Willis accepts that the Ideal Gas Law ( IGL) calculates the observed lapse rate. The IGL assumes all the small molecules in air behave the same. There is no GHG/non-GHG split. Why is the GHG radiation theory needed?
It is needed so that CO2 can be said to cause warming
The comment by Geoff Sharpe, which I agree with, is off topic and be snipped.
I dislike the snide patronising snips by the author. Please get a proper moderator

Louise
January 15, 2012 4:46 am

Wow – the word ‘Snip’ appears over 100 times on this thread – free and open debate or censorship?

Louise
January 15, 2012 4:48 am

“And if you believe a man like Tallbloke, who censors scientific opinion that he disagrees with, you are an idiot. -w.”
Have we not witnessed Willis doing exactly that over and over on this thread?
How many comments like mine didn’t even make it past moderation?

Alexander Harvey
January 15, 2012 5:04 am

Hi Willis,
“So let us assume that we have the airless perfectly evenly heated blackbody planet that I spoke of above, evenly surrounded by a sphere of mini-suns. The temperature of this theoretical planet is, of course, the theoretical S-B temperature.
Now suppose we add an atmosphere to the planet, a transparent GHG-free atmosphere. If the theories of N&K and Jelbring are correct, the temperature of the planet will rise.
But when the temperature of a perfect blackbody planet rises … the surface radiation of that planet must rise as well.
And because the atmosphere is transparent, this means that the planet is radiating to space more energy than it receives. This is an obvious violation of conservation of energy, so any theories proposing such a warming must be incorrect.”
Sparse, essential, logical, what is there not to like. Congrats!
Alex

Peter Czerna
January 15, 2012 5:14 am

@bananabender et al.
I have already commented on the fallacy of the adiabatic compression of an atmosphere.
The nonsense is still surfacing in this thread, so I have to repeat myself (before Willis wakes up and starts shouting at us all again for not doing what he wants us to do).
Adiabatic compression cannot warm an atmosphere in any lasting way.
– When a gas is compressed adiabatically its temperature is raised as long as no heat escapes from the gas.
– In the real world, near-adiabatic compression usually only happens when a gas is compressed very quickly, as, for example, in a bike pump.
– If you hold the gas compressed, heat will gradually escape by conduction/convection/radiation and the temperature will fall until it reaches ambient temperature again. A compressed atmosphere will just leak heat to its surroundings and return to ambient temperature.
A gas is not hotter just because it is denser. If it were, liquified gases would not be possible.
How can gravity apply continuous work compressing the atmosphere?
When does it decompress? (If a gas decompresses it loses heat (because of the work it is doing in expanding) and its temperature falls).
In other words a one-off compression will not permanently raise the temperature of an atmosphere.
Furthermore, unlike the piston of a bike pump, gravity is not doing work on the atmosphere, in the same way that a ball rolling down an inclined plane or just falling to earth is not having work done on it by gravity.
There is therefore no adiabatic heating of an atmosphere as a result of gravity.
NB: The air of an adiabatic (‘katabatic’) wind, which is forced downwards by circulation and topography and so warms and dries out (the infamous ‘Föhn’ in the Alps)
has work done on it as a result of its motion.

Peter Czerna
January 15, 2012 5:16 am

@anna v
Exactly so…

wayne
January 15, 2012 5:25 am

Anna, may I intercept?
That is a very, very needed clarification, believe me, and well stated!
Would you also generally agree with a general view that a gas’s gray body radiation is weaker as compared to solid and liquids at the ratio of the distance between the molecules, therefore weaker electric field? Without explicit knowledge on that aspect that seems the most logical view I have gathered. That would put air’s gray body radiation at about two+ magnitudes less than something like water. I am also assuming that also a gas can absorb gray body radiation in about the same ratio (Kirchhoff’s).
And please, one more, could you clarify what is meant by “on shell / off shell”? (electron shells of course)

wayne
January 15, 2012 5:46 am

Stephen Wilde:
January 15, 2012 at 3:18 am
Stephen, thanks, but let’s just drop it. I think Willis got my point. (but lets say an apology later would not be rejected☺)

tallbloke
January 15, 2012 5:58 am

Willis Eschenbach says:
January 15, 2012 at 3:17 am
Now, my understanding may be wrong, and my proof may not apply to Jelbring. But since not one person has stepped forwards to say that they understand and can explain Jelbrings hypothesis, we don’t know, do we?

Willis, thank you for your more fully fleshed out thoughts on my contention, and your recognition that you could be wrong. I will email your full comment to Hans Jelbring and together we will offer a considered reply in due course.

bananabender
January 15, 2012 5:59 am

LazyTeenager says:
January 15, 2012 at 4:20 am
Incorrect.
Almost 100% of IR radiation will be absorbed by the CO2 and water vapour in only a few metres of air.
The wavelengths of IR radiation at both 25C and 800C are similar and both will be absorbed.
The reason why there is no heating is because the absorbed radiation is converted to kinetic energy which results in convection. Gases can only be heated in confined spaces.

Ian W
January 15, 2012 6:09 am

Well Willis you asked – and my elevator speech – I will stick to simple concepts and laws and not use any maths…
Errors
• Assumption that non-GHG gases (transparent to radiation) are somehow incapable of being at a temperature above absolute zero. You seem to think that only the GHG molecules have any heat content (kinetic energy)? Obviously FALSE
• Ignoring conduction – you happily accept that GHG can warm other gases in the atmosphere by ‘collision’ with non GHG but for some reason do not accept that the surface molecules warm the atmospheric molecules by collision. Obviously FALSE
• Confusion of ‘atmospheric heat content’ with ‘atmospheric temperature’ . Obviously FALSE
Elevator speech – without mentioning radiaiton
1. Atmospheric temperature is a measure of the sum of the kinetic energy of all the gas molecules in the volume being measured.
2. If the number of molecules in the volume with the same kinetic energy increases the sum of the kinetic energy of all the gas molecules in the volume is higher – therefore as in (1) the temperature is higher.
3. Pressure is a measure of the number of gas molecules in a volume. Pressure increases toward the bottom of the atmosphere due to the weight of molecules above. Therefore, there are more molecules in a given volume at the bottom of the atmosphere. If all molecules in the atmosphere have the same kinetic energy then as in (1) and (2) the temperature is greater at the bottom where there are more of those molecules in a volume than at the top. Pressure is also proportional by temperature as the gas molecules collide and ‘jostle for space’. (This is Charles’ Law – one of the gas laws)
4. The molecules at the bottom of the atmosphere are given kinetic energy by collision with the high energy vibrating molecules of the surface then in turn they collide with other gas molecules transferring the kinetic energy upward from the surface
5. As the molecules at the bottom of the atmosphere increase in kinetic energy raising local pressure and temperature that volume of atmosphere expands (Charles law) becomes lighter and moves higher (Archimedes principle)
6. As the gas rises higher and expands the pressure caused by the atmosphere above it reduces the number of molecules in that volume reduces and therefore the number of molecules with a particular kinetic energy in the volume reduces so the temperature reduces. (see 1 and 2)
7. As the process from 4 -> 6 operates cooler air is drawn in to replace the rising hotter air and a convection current starts. Indeed, convection is a simple term to describe the process 4 -> 6.
8. Convection is a warm air current that takes sensible heat from the hot surface upward in the atmosphere until the expansion cools the volume to the level of the ambient atmosphere. As the planet is rotating Coriolis forces affect the currents of both rising warm air and inflowing cooler air moving the heat energy from the equator to the poles.
I believe that the above describes how the atmosphere will warm from the surface to the top with a lapse rate that is directly due to the gas laws. Indeed convection is the major heat transport in the atmosphere to the tropopause (which literally means the place at which atmospheric convection currents stop ).
Now it is postulated (firmly) by Willis that the gases Nitrogen and Oxygen ‘cannot radiate heat’ therefore the atmosphere will continue to warm by conduction of sensible heat and convection. But according to the Willis postulate never cool as it can only heat up. However, if we add some gas molecules to the mix that can radiate heat energy – say CO2 molecules, all of a sudden we have gas molecules that will radiate the kinetic energy that they have obtained by collision with Nitrogen and Oxygen. So CO2 instead of warming the atmosphere actually is one of the gases that is essential to cool the atmosphere.
I could go on with the effect of water vapor from the surface but I believe we have reached your basement floor. 🙂

Vince Causey
January 15, 2012 6:14 am

I’ve been wondering about the assertion that N2 and O2, being non GHG’s, can neither absorb nor radiate IR radiation. I considered a scenario of a cloud of N2 in space, of sufficient mass to be gravity bound into a ball. Let’s say this ball of gas becomes heated, either by gravitational compression or by the application of energy from some temporary external source.
The question I would like to pose to anyone interested, is if this gas ball cannot radiate IR, then in what way can it loose heat into space? If there is no way for the gas to loose energy, then the only conclusion is that it must remain at that temperature for ever. This is of course, an absurd conclusion to reach.
By the same logic as this thread has used, if it is impossible by the laws of physics for the gas to remain at the same temperature for ever, then there are only two conclusions: either the premise that the gas cannot radiate energy into space is false, or there is some way other than by radiation of loosing energy, which I have overlooked.

TimC
January 15, 2012 6:22 am

Willis: thank you. May I then come back with another question, for I’m still having trouble with this.
Your hypothetical atmosphere (by definition not absorbing or emitting radiation within the thermal infrared range) nevertheless has its own mass so must have its own gravitational field (additional to that of the planet itself without an atmosphere).
This, to a second body at a distance, will presumably cause work (again, in addition to that of the planet without an atmosphere), therefore additional heat. This is not caused by conduction or convection – what can be the cause, other than radiation? Where in the spectrum does this radiation (if it be such) lie?

steveta_uk
January 15, 2012 6:27 am

From Stephen Wildes elevator speech (must be a rather slow elevator, or a tall building): January 15, 2012 at 3:36 am

After all, no atmosphere means an immediate turnaround of energy i.e.
radiation straight in and straight out pretty much instantly. As soon as one
adds an atmosphere capable of CONDUCTION which includes non GHGs then the
conduction takes away from the efficiency of the radiation process by
slowing energy dissipation down which is what then leads to the higher
equilibrium temperature. The denser the atmosphere the more conduction takes
place before the radiative energy can be released to space and the higher
the equilibrium temperature rises.

Takes away from the efficiency? How? I can see that this transiently reduces the heat available for radiation, so should transiently reduce temperature, but you seem to argue that the same amount of heat has to be releases, so the temperature must RISE to compensate.
But this simply isn’t so. The temperature would transiently fall, as the atmosphere takes away heat, until is reaches an equilibirum state with the surface, at which point, since there is no possible source of outgoing radiation OTHER THAN THE SURFACE, then Willis’s balance is restored.

David
January 15, 2012 6:30 am

I haven’t read all the comments, so excuse me if this point has already been covered, but I am puzzled by some claims implying that a non-GHG, such as nitrogen, cannot radiate energy. Have I understood this correctly? I thought the definition of a non-GHG was that it does not absorb radiant energy passing through it, in other words it is transparent to radiation at all wavelengths. But it does not follow that it cannot radiate energy acquired in some other way. Consider this thought-experiment. A planet-sized ball of nitrogen is heated by a nuclear-powered source at its center, which is ‘switched on’ at T1. Heat is transferred by conduction to the nitrogen in contact with the heat source, and by conduction and convection to the rest of the nitrogen. The temperature of the nitrogen increases, and its volume expands, doing some work against gravity. By doing work, which substitutes potential energy for kinetic energy (heat), the temperature of the nitrogen is lowered by expansion, but it will not, I think, cool down to its original temperature by this process alone. At T2 the internal heat source is switched off and removed. At this stage the ball of nitrogen has a higher temperature and a larger volume than at T1. Unless it can reduce its temperature, its volume is in equilibrium, with gravity balanced by the higher pressure exerted by the gas molecules. Since the ball of nitrogen is in the vacuum of space, and not in contact with any possible recipient of heat, it cannot lose heat, and reduce its temperature, except by radiation. If it cannot radiate, it will remain indefinitely at a higher temperature than the surrounding space (and possibly other distant bodies). But this would violate the Second Law of Thermodynamics. Therefore it must radiate.
What have I misunderstood?

bananabender
January 15, 2012 6:39 am

“But what such an atmosphere cannot do is raise the temperature beyond the theoretical maximum average temperature for that given level of incoming radiation. That’s against the law … of conservation of energy.”
Completely and utterly wrong.
The temperature of the atmosphere can be raised (almost infinitely) according to the Ideal Gas Law pV=nRT without any need for (the totally imaginary) Greenhouse Effect. In theory the surface of a planet could even become hotter than the Sun if it had a sufficient atmosphere pressure.
The primary role of the Sun is to provide enough heat energy to keep the atmosphere in a gaseous state and maintain the vapour pressure. The pressure then heats the planet. The surface temperature of every planetary body in the Solar System is a function of the atmospheric pressure and gravity. Mercury and Mars are relatively cool because they have virtually no atmospheric pressure. Jupiter, Neptune, Uranus and Titan are warmer because they have very high surface pressures.
A blackbody is a purely theoretical concept. It is defined as a cavity radiator of zero thickness. The last time I checked the Earth is a 6×10^24kg solid sphere.

David
January 15, 2012 6:48 am

Willis Eschenbach says:
January 15, 2012 at 12:28 am
David says:
January 14, 2012 at 9:08 pm
Open question; Surface heat conducts to the non GHG atmosphere, Can non GHG molecues conduct heat to the surface?
Sure.
w
—————–
Thank you Willis, Now I have a folllow up. The second law of thermodynamics, as applied to radiating heat, allows energy to flow both ways, just the net flow of energy is from warmer to cooler, until both systems equalise, the tendency towards entropy is not contradicted. Is this true of conducted heat also?
Now another question Sir. Your statements concerning non GHG I have read. Answering this question will help clairify my understanding. Are you saying that if I am suspended in the vacume of space, and a stream of superheated 8,000 degrees N2 jets by me for 24 hours, at say 10′ away, (nothing but the vacume of space between me and the gas) I will feel nothing and my thermometer will register nothing, but if that stream of gas IS COMPOSED OF CO2 and jets past, again with 10′ of the vacume of space separating me from the gas, I will feel emense heat from the radiating CO2, but none from the non radiating N2?

wayne
January 15, 2012 7:10 am

Willis, back to the science.
It seems so many here were under their breath condemning you for taking a classic consensus climate science and IPCC view of the world in your example. A perfectly radiating planet, in essence massless, with equal light at every point and an atmosphere that can never shed even a watt of energy, that seemed to peeved most. We have heard such a story for years and it is totally meaningless.
But if you insist, you are right best I can tell. The best that planet could do is by conduction heat the entire atmosphere to the same temperature as the surface from bottom to top when an equilibrium was established, even if a gravity caused temperature gradient also was present. By a gravity caused temperature gradient I mean the dry adiabatic lapse rate (DALR=g/Cp) that is not merely a descriptive ratio but has, by some unknown physical process, a tendency for temperatures to sort by themselves, albeit very slowly, warmer deeper in the gravity well matching that ratio.
So, you win, but I don’t see that proving anything. If that enabled natural lapse rate did occur all you would have to do is add the ability for that atmosphere to shed energy. As anna v was saying, just let the N2 be real N2, it does radiate though weakly. In that case you have given the atmosphere the ability to shed that excess energy seen at the top and that gradient would slowly emerge.
But also in that case the N2 can by Kirchhoff’s law also absorb though equally weakly radiation and since there is more density low, more energy would be absorbed near the surface than in the thin gas at top. That one factor would pull the surface above your theoretical S-B temperature though not up to +14°C. Add water at the surface and it seems it would. And I’m not sure that ‘real’ DALR is even necessary, it seems to be strictly from the density, not pressure.
If you see flaws in that let me know.

Richard M
January 15, 2012 7:22 am

johndo9 says:
January 15, 2012 at 4:19 am
I have just been re-reading Hans Jelbring. He does indeed start his model with- Quote “The model planet does not rotate. It neither receives solar radiation nor emits infrared radiation into space.”
With no energy passing through there is no trapping of energy and no Greenhouse Effect.
As such its pretty much irrelevent to what Willis has defined here.
His condition that the atmosphere cannot absorb or emit any radiation requires all radiation to be to or from the surface.
As others have shown the “average surface temperature” is below what Willis called the “theoretical Stefan-Boltzmann (S-B) temperature.”
He is right, all the atmosphere can do is raise the “average surface temperature” up towards the “theoretical Stefan-Boltzmann (S-B) temperature.”
So to Nikolov and Zeller and the elevator speech. But first its important to understand they use a GHG atmosphere,

No they don’t. From their abstract:
“the so-called Greenhouse Effect is in fact a Pressure-induced Thermal Enhancement (PTE), which is independent of the atmospheric chemical composition.”
If they did use a GHG atmosphere I think they would be on the right track. The profile of an atmosphere is by and large independent of the chemical composition. Therefore, it is possible to determine the GHE from looking at that profile (which is what they did for other planets). However, you must have some radiating gases that are carried by thermal energy to higher altitudes otherwise you end up with the situation Willis is discussing. If K&Z had specified GHGs were required then I think this entire discussion would be on the right track. The GHE is defined by the pressure distribution as they stated. But, it is if and only if you have sufficient GHGs present.
Maybe a chemist can help with an example. Think of a solution that is stable until you add a small amount of a catalyst. All of a sudden a big change occurs. Only a certain amount of the catalyst is required and, after that certain point, more of the catalyst does nothing. That is what happens with the GHE. The GHGs are the catalyst.

January 15, 2012 7:26 am

It isn’t called a gravitational CONSTANT without good reason. Joel Shore and others make the same mistake.
Gravity is a continuous renewing process which replenishes itself over time. No one knows why or how, not even Einstein, but we must live with it.
So it is NOT a transient effect and the result is PERMANENT all other things being equal.
Without that effect the Ideal Gas Law would not be a Law.

Only rarely does something leave me speechless. What does this mean? I really, truly have to say “nothing”, but I’m willing to have it explained to me.
In the meantime, permit me to explain it to you as I understand it, which is really pretty well.
Gravity isn’t a “continuously renewing process”. Gravity is an interaction between two massive objects that results in a force of attraction between them. Within irrelevant corrections associated with relativity or things like “dark” matter or energy, the form for gravitation is:
F_{12} = – G M_1 M_2 / r_{12}^2
It is always attractive and acts along the “right line” connecting two point-like masses. Spherically symmetric masses behave like (produce the same gravitational field as on the outside of) a point mass at the origin with the same total mass. Hence this field describes the force of gravity near the surface of the Earth.
Near the surface of the Earth (inside the atmosphere) variation with “height” varies the radius r_{12} by a trivial fraction — the Earth is 6400 km in radius, and most of the atmosphere is contained within 32 km of the surface, a variation of around 0.5%. For that reason, one makes little error by assuming that “Near Earth” gravity is a constant field, producing a force of the form:
F = mg
where g = G M_earth/R_earth^2
Gravity has (and pay careful attention here) nothing whatsoever to do with the Ideal Gas Law. I really mean that. Nothing whatsoever. Gravity is not mentioned anywhere in the derivation of the IGL. It plays no role. It is irrelevant. The gravitational force between gas molecules is so tiny as to be completely ignorable, and besides, it doesn’t have the right form to lead to an IGL if that were the only source of interaction. The IGL is derived using a hard sphere interaction or no interaction but a presumption of equilibrium and hence equipartition.
Gravity is constantly being proposed as a source of heat in discussions on this site, usually (AFAICT) by people who are really pretty clueless about what gravity is, what it does, what energy is, where it comes from (in the context of gravity) and so on. I will provide you with a single example of how to use gravity “correctly” as a source of energy delivered to the Earth from outside of the Earth, and one that shows you how people use it incorrectly.
Correct usage is templated by the killer asteroid. It begins far away, where its (negative) potential energy is very small. It falls toward the Earth, being pulled downward by the force of gravity. This force speeds it up. Speeding it up increases its kinetic energy. From the point of view of energy conservation, as its kinetic energy increases, its potential energy decreases, which is in fact the case, becoming more negative (negative and larger in magnitude) is such a way that the total mechanical energy (sum of kinetic and potential) of the asteroid remains constant.
For those unfamiliar with “kinetic” and “potential” energy, think of this as your checking and savings account. Kinetic energy is energy of motion, like your checking account contains money that can easily be mobilized. Potential energy is energy stored in a field interaction that can be freely transformed into kinetic energy, much as your savings account can receive money from checking or vice versa. Moving money back and forth doesn’t affect the amount of money you have, but it does change money in one form/venue to money in the other. The only thing the metaphor doesn’t help you understand is that nature exerts force in the direction that favors the transfer of money from savings to checking, forces push in the direction that decrease potential energy.
So, the asteroid falls to Earth, speeding up all the way, hits the Earth and transforms basically all of its kinetic energy to heat. A lot of heat in the case of an asteroid. Less heat in the case of e.g. small meteors. This heat represents an actual gain in the Earth’s energy budget, because it came from outside of the Earth and ended up inside the Earth (including the atmosphere). It remains there, warming things, until radiation (the only way the Earth can lose energy) rebalances the Earth towards dynamical equilibrium with Mr. Sun.
Incorrect usage is, well, everything else. The problem is that the total gravitational potential energy of the atmosphere is very, very nearly a constant no matter how it moves around, up or down in convection, side to side as wind. Gravity produces a force that sometimes opposes motion, sometimes supports it. No net work is done by gravity as gravity acts on the air, and gravity itself is not the source of the free energy that provides the actual pushes that move air around.
Almost all — and by “almost” I mean 0.9999… with many nines of it — of the energy that actually moves air around, moves it up and down and sideways — comes from one place and one place only. Mr. Sun. The sun warms the surface, the surface warms the air, the air expands (reducing density), and experiences a buoyant force as colder denser air seeks to displace it. It rises (increasing gravitational potential energy) as colder air falls in behind it (decreasing gravitational potential energy) in a near-zero-sum game whose source of energy was the heat that warmed the air packet and altered its density, not gravity. If there were no external heating, no inhomogeneity in density, there would be no net buoyant force and the air would stratify, unmoving, at which point gravity would do no work whatsoever, not even balanced work on chunks of air.
In order for gravity to release energy that appears as actual heat, in other words, some mass has to go downhill (because just rolling a ball downhill is also an example of the good paradigm). Sure, rolling something down hill gives you energy, but you can’t roll the same ball downhill twice without first pushing it back up to the top, and to do that requires a source of energy that is not gravity. Gravity can store energy (energy from somewhere else) and release it later, but it cannot act as a source of energy unless all of the mass ends up at the bottom of the hill when you are done, and once it gets there nothing happens until something else pushes it up once again.
Can gravity ever act as an energy source that releases heat in a gas? Sure. It is the primary source of heat in stars as they form, right up to where they “ignite” — fusion kicks in — as a source of non-gravitational free energy. Brown dwarfs and black dwarf stars are stars whose primary source of heat is a slow gravitational collapse that is literally “squeezing” gravitational energy out in the form of blackbody radiation, a process that can take billions of years. Jupiter’s surplus heat may come from a similar process, because Jupiter is not that far away from the mass range that would have caused it to be a brown dwarf instead of a gas giant. But the Earth? It is tiny. Its atmosphere long, long ago equilibrated so that solar heating is the only meaningful source of energy, with tiny corrections from the internal heat of the Earth bleeding out through the surface, from the tides, from radioactivity, even from magnetic induction (it is a conducting ball spinning in the Sun’s magnetic field, which means that it is very weakly being magnetically “braked”, a process that releases an impressive number of Joules in a year but that is utterly irrelevant as far as being a thermally significant source of heat).
So please — can we stop speaking of gravity as a source of heat? Seriously, and permanently? Gravity is not a source of heat in the Earth’s atmosphere. Not ever. No, no, no. Gravity is not irrelevant — it is the reason that there is a lapse rate in the thermal gradient of the atmosphere — but it provides no energy to the processes that maintain that gradient. That energy has to come from somewhere else, and there is only one somewhere else and it is a million miles across, 93 million miles away, and converting over 4 million metric tons of mass to energy every second some tiny fraction of which makes its way, after meandering around inside it for 100,000 years, to the Earth.
I still do not understand the N&Z paper any more than Willis does — it seems to imply that gravity is providing heat and that PV = NkT is somehow relevant to the process, but the former, as I make clear above, is nonsense and the latter is equally irrelevant — the lapse rate may have something to do with the structure of dynamic equilibrium in an atmosphere, but it isn’t providing heat or altering the way heat is absorbed or emitted per se. It only becomes important along with differential outgoing radiation, radiation with different temperatures in different bands, which is the “atmosphere warming effect”, a.k.a. “the greenhouse effect”, no matter how you dress it up and no matter that it has nothing to do with the way greenhouses actually function.
I’m still working on plausible non-greenhouse sources of warming, or ways that the “simple” greenhouse effect is not so simple — differential warming or cooling due to differential lateral transfer of heat, differential warming or cooling due to modulation (or internal feedback of) albedo. Until I understand what N&Z are trying to assert as a hypothesis I will reserve judgement, but so far I don’t understand what they are trying to assert.
rgb
[COMMENT: Robert, many thanks for this post. For those unfamiliar with him, Robert is a physicist who teaches this stuff for a living. I would strongly recommend that anyone who believes that gravity can do ongoing work or be the source of ongoing heat read this post two or three times. -w.]

Reply to  Robert Brown
January 15, 2012 7:44 am

Brown
What’s missing from the whole scenario is “work”.
This Wikipedia entry from thermodynamics sums it well:

In thermodynamics, work performed by a system is the energy transferred to another system that is measured by the external generalized mechanical constraints on the system. As such, thermodynamic work is a generalization of the concept of mechanical work in mechanics. Thermodynamic work encompasses mechanical work plus many other types of work, such as electrical or chemical. It does not include energy transferred between systems by heat, as heat is modeled distinctly in thermodynamics. Therefore, all energy changes in a system not a result of heat transfer into or out of the system are thermodynamic work.

In the case of our model planet, the atmosphere gravitationally attracted to the spherical mass would be a static system unless perturbed by an energy input of some kind. The atmosphere would essentially be like a layer of paint on the sphere, static and unmoving.
Add external energy (our solar light source for example), and the surface warms, and the atmosphere roils, parcels of atmosphere are lifted and returned to the surface as they cool.
Think lava lamp.
Work is being done in the lifting, and in the lifting the adiabatic lapse rate applies. Rising (expanding) air cools, sinking (compressing) air warms. That’s a product of work due to the external energy input.
Shut off the energy, and the model planet goes back to steady state equilibrium with the atmosphere clinging like paint again.
Like you, I can’t see any way “that gravity is providing heat and that PV = NkT is somehow relevant to the process”.

Eric Barnes
January 15, 2012 7:27 am

Willis Eschenbach says:
January 14, 2012 at 3:14 pm
As I said to someone else who wanted to abuse me because I don’t understand Huffman, if you’re such a dang expert, where’s your elevator speech on the Huffman Effect?

No abuse intended Willis. I don’t see how someone can read Huffman and not understand. It’s about as simple as it gets. This is the only post you’ve made that’s left me scratching my head. I’ve enjoyed all the others.
. I’m happy to disagree and hope you are as well 🙂
Take Care.
Eric

Richard M
January 15, 2012 7:32 am

To avoid confusion, I’m NOT stating that the GHE is a chemical reaction, that is simply an analogy. What happens is the GHGs become well mixed in the atmospheric profile due to pressure and heat. Hence, the GH effective radiating altitude gets set very high even with low concentrations. Because the atmospheric profile is changed very little by adding additional GHGs, that altitude does not increase if things like additional CO2 are added. It is that height that determines the overall GHE.

Tim Folkerts
January 15, 2012 7:37 am

Willis Eschenbach says:
January 15, 2012 at 2:50 am

kwik says:
January 15, 2012 at 1:55 am
Elevator speech for N&Z;
The non-GHG-gas can increase its energy via conduction at the surface.
This means the gas gets a higher temperature. Via conduction.
Via “bouancy” it will rise.(Figure of speech; Its density lowers, so it rise)
When ascending it looses its energy (via conduction with other gas molecules) until “bouyance” equals gravity. Then it stops.
The air-parcels close to ground that ascended is replaced by others, creating a circulation.

True so far, but the circulation cannot continue forever.

(I Hope I got that formatting right and people know who said what)
Actually, Kwik made a mistake even within these lines when he said: ” it looses its energy (via conduction)…”
The rising air doesn’t loses much energy from conduction because air is such a poor conductor. Mostly the air rises “adiabatically”, neither gaining nor losing energy. The air GAINS gravitational energy and LOSES internal energy (ie it cools off).

jocky scot
January 15, 2012 7:38 am

What’s the temperature of space ?

geoff
January 15, 2012 7:47 am

Given the model world Willis postulates, it seems evident he is correct. Since the gas molecules cannot absorb or emit radiation at these frequencies, they can only heat or cool by conduction or expansion. Initially you get convection, but eventually the air near the surface is at surface temp and the temp of the air smoothly decreases with altitude. All the molecules at any given altitude all have the same temp (or very, very nearly so) so conduction ceases. Convection ceases. The atmosphere no longer absorbs any energy from the surface, so it is now irrelevant. Willis wins. Since Jelbring seems to postulate the same simplistic world it would seem he loses.
Move one step closer to reality with a rotating world illuminated from one direction and you no longer have an equilibrium case. Conduction and convection will not cease, and the atmosphere can indirectly radiate by warming the surface on the dark side by conduction. The air thus being cooled can moderate the peaks of temp (again by conduction) when that side is sunlit again. Since radiation is related to the fourth power of temp, clipping the peaks reduces a disproportionate amount of radiant energy. This reduces the total radiation of the surface, thus raising its average temp from what it would be without a transparent atmosphere.
So Willis is right for Willis’ world. All models are simplifications, but I think he went one simplification too far when trying to show GHG are required to raise the surface temp.

f---you
January 15, 2012 7:47 am

[snip – site policy violation, fake name edited also – Anthony]

coturnix
January 15, 2012 7:49 am

[snip – in your previous comment (under a different handle) you used the F-word, and it was caught by our spam filter. If you have something to say, put your name on it, otherwise don’t comment again. I have no tolerance for people who abuse site policy from the comfort of anonymity – Anthony]

January 15, 2012 7:54 am

motion in electric fields generates radiation and this radiation gets out of the solid at the frequencies where the solid is transparent and thus the solid cools because energy is lost.
In a solid the radiation gets out at the surface, and comes out at the frequencies where the solid is opaque — you have it exactly backwards. Atoms or molecules absorb or emit symmetrically, or nearly so (the laser is an example of an exception, but requires special preparation and states that line up “just right” with an external driver). The whole point of blackbody radiation is that if a solid is a “perfect absorber” — a black body — it is also a “perfect emitter” in all frequencies and hence generates a BB spectrum if it has a given temperature as derived by Planck, subject to his quantization hypothesis.
Real matter is generally not a perfect absorber, and pure atomic or molecular matter — single species — tend to have specific quantum structure such that they absorb and emit primarily in transitions between specific discrete energy levels. Things are also different when one considers transparency and penetration of real matter, especially for “transparent” fluids. In that case radiation doesn’t come out of “the surface” — it comes out of a (usually exponentially attenuated) depth from the “surface” (if there is one — in the case of the atmosphere the notion of a “surface” is largely artificial with the possible exception of structural interfaces such as the tropopause (which still have some depth, but it is small compared to the overall range of variation). This is quite relevant for BB radiation — the outgoing IR isn’t released from a “surface”, but rather from a rather complicated volume of atmosphere. Remember that no matter how large the CO_2 cross section in the atmosphere, CO_2 is only 0.03% of the atmosphere (and the atmosphere itself is exponentially attenuating) so the “emitting surface” is kilometers if not tens of kilometers in depth. Water is even more complicated, as it forms clouds.
There is a lot I’m still working on understanding in all of this, don’t get me wrong. It’s just that your assertion above is exactly, diametrically wrong regarding the transparency, it is the opposite of the case.
rgb

DavidB
January 15, 2012 7:59 am

Just to avoid confusion, I previously posted a comment as’David’, giving a thought-experiment about a ball of nitrogen in space. I have noticed that at least one other person has commented under the name ‘David’. They are not me. I have only posted one comment (so far).

January 15, 2012 8:03 am

Think lava lamp.
Well said, Anthony, and an excellent example. Or, as I suggested on another thread, consider the rolling turnover a boiling wort when making beer — heat input at the bottom from the flame, heat output at the top from evaporation and convection, heat transport from bottom to top due to differential density and buoyancy such that the temperature of the beer remains in a steady state in between.
In neither case does gravity do net work as the wax blobs rise and fall or as the wort turns over. It isn’t irrelevant. Imagine heating the beer from the top. Hmm, no turnover, no rolling convection, no real boiling — the wort heats and evaporates at the same spot, and conduction would take forever to heat the beer. Try turning a lava lamp upside down and turn it on (heat at the top!) Actually don’t, because the bulb might overheat and start a fire because cooling of the bulb through the liquid would almost completely disappear and the lava lamp wouldn’t “lava” — the wax would melt once, fall, and sit at the cold bottom forever.
rgb

January 15, 2012 8:04 am

Anna V,
You say “Thus one cannot have matter as we know it of a given temperature not radiating in some frequencies.” Consider a sample of helium gas at standard temperature and pressure (roughly 270K, 1atm). At what frequencies do you think it will radiate? What transitions will cause this? And what are the relevant transition dipole moments?

Phil.
January 15, 2012 8:05 am

Willis Eschenbach says:
January 15, 2012 at 12:13 am
willb says:
January 14, 2012 at 8:24 pm
@Willis Eschenbach
Here is my best shot at an elevator speech explaining the N&Z “greenhouse effect”. The N&Z effect works as follows:
• The sun transfers energy via radiation to the earth, warming the earth.
willb, thanks for the elevator speech. First step looks good.
• The surface of the earth transfers energy into the (non-GHG) atmosphere through conduction, heating the atmosphere.
OK
• Earth’s gravity causes an altitude-dependent temperature and pressure lapse rate to form in the atmosphere. The atmospheric temperature and pressure drop as altitude increases with the temperature ultimately dropping to that of deep space (~3K).
Kinda, although it ignores the thermosphere that may not be significant.

Indeed but it’s easier to ignore that. 🙂
If it’s N2 atmosphere it’s not clear why the temperature of deep space is relevant that would require N2 to have a far microwave absorption which I don’t think it does otherwise Penzias and Wilson at Bell Labs wouldn’t have been able to measure the cosmic background?
• Atmospheric gases heated conductively at the earth’s surface then convectively rise through the pressure lapse rate.
Not sure what “through the pressure lapse rate” means, but OK.
• As the temperature of the atmosphere drops towards that of deep space, the rising gases transition through a number of phase changes (e.g. gas => liquid => solid-a => solid-b). At the occurrence of each phase change the atmosphere releases latent heat in the form of radiation.
OK

Not if we consider a N2 atmosphere, if we have a 20ºC surface at 100kPa we’ll hit -60ºC (~210K) at 33kPa based on the dry adiabat, below the triple point so the liquid phase is out of play. To get to the sublimation curve you’d need to get to ~60K which probably wouldn’t happen below 150km, so no N2 snow!
• Some of the radiating energy from the released latent heat is directed downward through the gaseous, IR-transparent atmosphere and is absorbed by the earth.
Whoa, whoa, whoa. The atmosphere is GHG-free. How does the released energy radiate anywhere, when the atmosphere has no GHGs, and as a result can’t radiate in the IR? I fear your explanation dies there …

No latent heat either.

Steve from Rockwood
January 15, 2012 8:06 am

The outer planets are gaseous but have high internal core temperatures caused by gravitational compression. On Earth we should expect our atmosphere to be heated constantly merely by the force of gravity. This heating is likely explained using the ideal gas law PV = NkT. We should expect both a temperature and pressure gradient that both increase toward the surface of the earth. The greater the density of the atmosphere the greater the temperature gradient.
Gravity is a force that heats objects by compression without any help from the sun. Otherwise the core temperature of Neptune and Pluto would be closer to absolute zero.

January 15, 2012 8:07 am

What’s the temperature of space ?
Your choice of “it has no temperature” (a vacuum has no temperature), or 3 K (the vacuum isn’t empty, it contains electromagnetic energy in the form of blackbody radiation leftover from the big bang).
The latter is probably the right temperature to use as far as cooling the Earth via radiation is concerned. Turn off the sun, wait “a while”, and the Earth would cool to 3 K in equilibrium with “space”.
But 3K is close enough to “0 K” that you wouldn’t go far wrong using it instead as a simpler idealization, at least until the Earth cooled to temperatures that were approaching 3 K.
rgb

coturnix
January 15, 2012 8:08 am

Do your worst, antiscientific censorship freak.

beng
January 15, 2012 8:09 am

For those with the question of what a non-GHG atmosphere would be like, I don’t think it’s too hard to envision. The surface will have temp profile similar to the moon — very cold at night & very hot in day. The air just above the surface will follow the surface temp (by conduction), and eventually the entire atmosphere will end up as nearly isothermal (same temp bottom to top) and close to the average of the surface temp. If you take our current solar input, such a planet would have surface temp’s like the moon, and an isothermal atmosphere (only a tiny lapse rate) of approx the moon’s avg temp.
Of course, day/night cycles will cause temperature cycles in the surface/very near air temp profiles, but the GHG-free atmosphere can NEVER IN AVG be warmer than the avg surface temp — that’s the only means of it getting heated (by conduction). And no, not by “compression” – compression is work, which happened only when the planet first formed. A stable, static pressure, no matter how high, is NOT compression. Yes, there are local updrafts/downdrafts, but the net compression is zero for a non-accreting atmosphere.

Paul Dennis
January 15, 2012 8:09 am

Robert Brown and Antony,
you are correct in this description of convection and work. However, in Willis’ model, with even heating of the planetary surface there will be no temperature differences and the atmosphere will not convect. Under such conditions I think a uniform temperature distribution will arise, and not a thermal gradient resulting from the lapse rate.

Reply to  Paul Dennis
January 15, 2012 8:15 am

@Paul Dennis – yes, that’s what I was alluding to with the paint layer in equilibrium analogy. We are in agreement

coturnix
January 15, 2012 8:10 am

I used to think wuwt was a legible place, instead it turned into comment-erasing freakodrom. WUWT is dead to me as well as many other people i am sure. uoyckuf
REPLY:Oh reverse f-word (misspelled), clever. Comments that don’t meet policy, like fake names and f-words? THAT’s what you are defending? Don’t let the door hit your butt on the way out – Anthony

January 15, 2012 8:12 am

You say “Thus one cannot have matter as we know it of a given temperature not radiating in some frequencies.” Consider a sample of helium gas at standard temperature and pressure (roughly 270K, 1atm). At what frequencies do you think it will radiate? What transitions will cause this? And what are the relevant transition dipole moments?
So you’re asserting that a container of helium gas, confined inside a perfectly transparent container in intergalactic space, would still be at 270 K after a million years? Or a million seconds?
Yes, I understand your argument, but the helium atoms are not non-interacting and they are made of matter. When they collide (as they frequently will) the charge they are made up of will oscillate and they will still cool. Slowly, perhaps, but you have second order effects to consider. When they collide, the non-interacting energy levels all shift and second order (two-photon) transitions are enabled, are they not?
rgb

Martin Mason
January 15, 2012 8:14 am

With no atmosphere the earth will absorb radiation from the sun and will emit it 100% as LW radiation to space. The earth acts as a black body whose temperature will be as calculated from SB. The near surface temp is zero.
With an atmosphere, the earth’s surface absobs radiation from the sun, heats up and emits some energy as radiation, some via conduction and convection and the latter is dominant. the earth’s surface is cooler than the BB temperature as it is transfering heat to the atmosphere. The near surface air is far hotter than the zero atmosphere case simply because it has heat capacity and can receive heat via conduction from the surface
Heat from the surface is carried upwards by convection and radiates out to space. Radiation out balances radiation in and energy is conserved.
Near surface air already has internal energy because of its pressure and its equilibrium temp is a function of it’s own PV and the heat transferred to it from the earth’s surface.
If the pressure of the near surface air is higher then it will have a higher equilibrium temp than the lower pressure air as it has higher internal energy to start with Energy is conserved.
All valuecan be quantified and no radiation effect is needed.
I’m no expert but where I believe Willis is wrong is that he is confusing surface temp and near surface air temp. He is also wrong about an atmosphere without GHGs not being able to radiate to space.

coturnix
January 15, 2012 8:15 am

Good riddens. My hit won’t make much difference to you rating, but thats the best I can do. Bye, big oily slave.
REPLY: LOL! Such maturity, and then there’s the big oil barb, heh. OK, we’ve had our fun, you’ve earned permanent troll bin status, all subsequent comments go to the bit bucket. – Anthony

January 15, 2012 8:17 am

“So it is NOT a transient effect and the result is PERMANENT all other things being equal.
Without that effect the Ideal Gas Law would not be a Law.”
“Only rarely does something leave me speechless. What does this mean? I really, truly have to say “nothing”, but I’m willing to have it explained to me.”
Then I’ll try to put it better.
Within a gravitational field the pull of gravity is a constant force.It is not akin to a single one off pressurisation, it is akin to continuous pressurising. Therefore there is work being done continuously but only as long as there are energetic molecules for the gravitational field to work on.
That is not to deny that all the energy in the molecules comes initially from the sun. Gravity simply governs the distribution of energy within the atmosphere in accordance with the Ideal Gas Law.
With a gaseous atmosphere the Ideal Gas Law applies. A temperature gradient is set up from surface to space with the warmest molecules at the surface.
It is the action of gravity on the molecules of the atmosphere that creates that outcome.
No solar input, no energy to be distributed and no lapse rate.I don’t think that anyone says that the gravity itself is the source of the energy.

January 15, 2012 8:20 am

The rising air doesn’t loses much energy from conduction because air is such a poor conductor. Mostly the air rises “adiabatically”, neither gaining nor losing energy. The air GAINS gravitational energy and LOSES internal energy (ie it cools off).
Agreed, Tim. Also even if it did lose energy to “something”, what could it be? There are only two choices — other air (mixing and thermalizing via convection and diffusion) and “everything else” via radiation. The former doesn’t help — yes, you are moving heat uphill, but there’s only a certain distance “uphill” you can move it as heat in matter. Sooner or later, you have to remove the heat thus transported, and only radiation removes heat from the Earth altogether. Otherwise you simply heat the air overhead until warmed air at the bottom doesn’t rise any more and you once again have a static lapse rate. That’s the way actual greenhouses work — they trap warmed air so it cannot convect heat away. Sort of.
rgb

January 15, 2012 8:26 am

To avoid confusion, I’m NOT stating that the GHE is a chemical reaction, that is simply an analogy. What happens is the GHGs become well mixed in the atmospheric profile due to pressure and heat. Hence, the GH effective radiating altitude gets set very high even with low concentrations. Because the atmospheric profile is changed very little by adding additional GHGs, that altitude does not increase if things like additional CO2 are added. It is that height that determines the overall GHE.
This seems correct to me, although somebody (Tim F.?) asserted otherwise. Indeed, I suspect that the Earth is largely insensitive to changes in GHG concentrations, and might even operate the opposite way than expected in some cases. Once you are “opaque” you are opaque, and making it twice as opaque doesn’t really happen. I am actually curious as to whether anybody knows anything concrete about this, as it has bothered me for some time. I’d expect alterations in the outgoing radiation profile due to doubling CO_2 concentration to be, well, almost impossible to detect, as radiation of IR from the troposphere is going to still be radiation of IR from the troposphere. Is the troposphere going to move? Will the adiabatic lapse rate change? Why, exactly, is radiative balance going to change?
rgb

January 15, 2012 8:31 am

“The temperature would transiently fall, as the atmosphere takes away heat, until is reaches an equilibirum state with the surface, at which point, since there is no possible source of outgoing radiation OTHER THAN THE SURFACE, then Willis’s balance is restored.”
Willis’s balance will be restored but at a higher equilibrium temperature because the lapse rate resulting from the Ideal Gas Law places the warmest molecules of the atmosphere right at the surface.
Therefore the surface becomes warmer than the average temperature of the atmosphere. The more mass in the atmosphere the greater the warming effect at the surface as compared to what the S – B equation would predict.

January 15, 2012 8:34 am

Within a gravitational field the pull of gravity is a constant force.It is not akin to a single one off pressurisation, it is akin to continuous pressurising. Therefore there is work being done continuously but only as long as there are energetic molecules for the gravitational field to work on.
But (as I explained) this turns out not to be the case. No work is being done by gravity.
You should, perhaps, take a look at the first law of thermodynamics. It states basically that the heat added to or taken out of a system, plus the external work done on a system, has to equal the change in the internal energy of a system. It is the law of conservation of energy in (a fairly thin and transparent) disguise. Now imagine an adiabatic fluid. That is, one where no heat is being added or taken away. If gravity did work on it, the temperature of the fluid would increase without bound. Gravity is constantly adding work, right? No energy is leaving the box (it’s adiabatic). So the energy content of the box has to constantly increase.
Hence, this statement violates the first law, and is simply not correct.
rgb

January 15, 2012 8:36 am

Anthony Watts says:
January 15, 2012 at 7:44 am
Think lava lamp.

Hey, I’ve got one of those.
If you leave it on long enough, doesn’t the mixture stop moving with the two primary substances now reversed (stuff that was on the bottom is now on the top)?
Equilibrium perhaps?

Reply to  JohnWho
January 15, 2012 8:40 am

@JohnWho
True, but if we wanted the lava lamp to model Earth’s atmosphere, we’d have to turn the lamp off/on in 50% duty cycle to simulate rotation, in which case equilibrium would not be a permanent state – Anthony

January 15, 2012 8:42 am

Robert Brown 8:12, I’m not asserting anything in that post; I’m just asking Anna V to back up her confident expresions of opinion with some elementary calculations.
I am quite familar with two photon processes, the presence of small dipole moments in isotopically mixed diatomics (such as 14N-15N), and the potential effect of transitory complex formation (such as Ar/N2). I asked about He because these effects will be particularly small in that system.
A sample of He will eventually radiate its heat away, but the process will be incredibly slow. Certainly millions of seconds, quite possibly millions of years. Esesentially impossible to measure as it will be overwhelmed by radiation from the container in any real system.

January 15, 2012 8:43 am

“How can gravity apply continuous work compressing the atmosphere?”
By constantly restraining the propensity of molecules to fly off into space when they are energised by an external heat source such as the sun.
I am puzzled that so many far better qualified contributors than me seem to be unaware of that.
Gravity works ceaselessly as long as there are molecules with any energy at all attempting to escape its embrace.
That is why one can balance an object in a so called ‘permanent’ near Earth orbit so that it neither crashes to the surface nor flies off to space. It is a well established principle but not here it seems.

January 15, 2012 8:44 am

With a gaseous atmosphere the Ideal Gas Law applies. A temperature gradient is set up from surface to space with the warmest molecules at the surface.
It is the action of gravity on the molecules of the atmosphere that creates that outcome.
No solar input, no energy to be distributed and no lapse rate.I don’t think that anyone says that the gravity itself is the source of the energy.

This is sort of correct, but it isn’t “the ideal gas law” that applies or is relevant. I’m sorry, but it is a bit more complicated than that. Do you have the link to the Caballero PDF? It’s been posted now on a half dozen threads, and I’ve suggested to Anthony that he create a permanent link to it on the toplevel pages. If you want to see how the adiabatic lapse rate is established, it works through it. Truly, it involves things like the bulk compressibility of the gas, which establishes the structure of the pressure gradient in the first place, plus the way the bulk compressibility varies with temperature, plus the imposition of the adiabatic constraint to give you how the temperature and pressure and height all couple, and this is still in the simplest of circumstances.
The reason, by the way, that people are interpreting gravity “as the source of the energy” is that you keep saying things like WORK done by gravity, and as I just pointed out, that is incorrect. Work goes into the First Law, and if gravity did any net work at all there would be serious problems. So perhaps we could all agree if you just stop asserting that gravity is doing (relevant) work. It simply establishes a static equilibrium pressure gradient — nothing more. It is simple to derive the variation of pressure with height without mentioning temperature or work at all, assuming a constant bulk compressibility. It is a statics problem, static equilibrium. No work is done in static equilibrium, because nothing moves, and work is force through a distance.
rgb

January 15, 2012 8:51 am

By constantly restraining the propensity of molecules to fly off into space when they are energised by an external heat source such as the sun.
I am puzzled that so many far better qualified contributors than me seem to be unaware of that.
Gravity works ceaselessly as long as there are molecules with any energy at all attempting to escape its embrace.
That is why one can balance an object in a so called ‘permanent’ near Earth orbit so that it neither crashes to the surface nor flies off to space. It is a well established principle but not here it seems.

Seriously, you are starting to cause me actual physical pain. Gravity does no work on an object in a circular orbit, and this isn’t a useful description of how gravity binds molecules in a gas at an equilibrium temperature.
Stephen, I don’t want to discourage you — contributing is good and a good way to learn. But you need to either take a course in introductory physics or else study it on your own in some detail. You can visit my website and under the toplevel “Class” heading there are two semesters worth of intro physics textbook. You can learn how gravity and orbits work (among other things) there. I’ll try to put back the thermo chapters soon as well — I removed them for my classes this last year as we omitted thermo.
rgb

Roger Longstaff
January 15, 2012 8:55 am

Jonathon Jones,
Surely, the He container would lose energy to the 3K background of space, as long as it can be “heated” by the gas it contains (by molecular collisions)? The container will radiate at microwave fequencies.
Everything will suffer “heat death”, as the laws of thermodynamics dictate. Maximum entropy, minimum enthalpy. Or an I missing something?

January 15, 2012 8:56 am

A sample of He will eventually radiate its heat away, but the process will be incredibly slow. Certainly millions of seconds, quite possibly millions of years. Esesentially impossible to measure as it will be overwhelmed by radiation from the container in any real system.
Agreed, especially the latter as I have no good feel for the former. I was correcting Anna as well.
rgb

wayne
January 15, 2012 8:59 am

Stephen Wilde says:
January 15, 2012 at 8:43 am
“How can gravity apply continuous work compressing the atmosphere?”
By constantly restraining the propensity of molecules to fly off into space when they are energised by an external heat source such as the sun.
I am puzzled that so many far better qualified contributors than me seem to be unaware of that.
— — —
Stephen, you might have the right thought but are just be wording it incorrect, or just thinking of one side of it. Work is invariant of it’s path. So if a molecule starts to leave Earth and gravity pull it back to the same point no work has been performed, energy is identical. (and really what is happening is the molecule just followined it’s geodesic in the gravity curved spacetime, not ‘pulling’, though usually it is easier to see it that way)

Richard M
January 15, 2012 9:04 am

Robert Brown says:
January 15, 2012 at 8:26 am
Richard M: [To avoid confusion, I’m NOT stating that the GHE is a chemical reaction, that is simply an analogy. What happens is the GHGs become well mixed in the atmospheric profile due to pressure and heat. Hence, the GH effective radiating altitude gets set very high even with low concentrations. Because the atmospheric profile is changed very little by adding additional GHGs, that altitude does not increase if things like additional CO2 are added. It is that height that determines the overall GHE.]
This seems correct to me, although somebody (Tim F.?) asserted otherwise. Indeed, I suspect that the Earth is largely insensitive to changes in GHG concentrations, and might even operate the opposite way than expected in some cases. Once you are “opaque” you are opaque, and making it twice as opaque doesn’t really happen. I am actually curious as to whether anybody knows anything concrete about this, as it has bothered me for some time. I’d expect alterations in the outgoing radiation profile due to doubling CO_2 concentration to be, well, almost impossible to detect, as radiation of IR from the troposphere is going to still be radiation of IR from the troposphere. Is the troposphere going to move? Will the adiabatic lapse rate change? Why, exactly, is radiative balance going to change?

The change in atmospheric mass from burning fossil fuels converts O2 to CO2. This increases the mass slightly. So, 100 ppm increase of CO2 should increase the mass by .005%. This will change the profile height by even less then .005% due to gravitational compression. Pretty much undetectable.
As you stated the atmosphere is already absorbing all the radiation from the surface except for a couple of bands where increases will have little effect. The energy works it way up to higher altitudes where it is emitted to space. It gets emitted to space because the density of the GHGs drops dramatically once the gases are no longer well-mixed. It is the profile of the atmosphere that defines this point, not the concentration of any particular GHG. Having more GHGs might slow this process down a few micro-seconds but that is about it.
Of course, I could be wrong and I’m willing to listen to alternative descriptions.

Bryan
January 15, 2012 9:05 am

I said: “It does not require convection in our adiabatic atmosphere”
Paul Dennis says:
“As I said before I think convection is needed. The definition of the dry adiabatic lapse rate is:
“The dry adiabatic lapse rate (DALR) is the rate of temperature decrease with height for a parcel of dry or unsaturated air rising under adiabatic conditions.”
http://en.wikipedia.org/wiki/Lapse_rate
I used to think that still air was a very infrequent phenomena until I read the text below.
The dry adiabatic lapse rate is derived using the condition of hydroSTATIC equilibrium
for the still air (no convection) condition.
Convection itself is a closely related but not a necessary condition.
(See bottom of page 13)
In the absence of convection (still dry air) heated from the bottom and cooled at the top a temperature gradient would be set up and is in fact given by the DALR.
The dry lapse rate can be satisfied by diffusion(molecular conductive heat transfer)
This is what meteorologists call the neutral atmosphere.
The neutral atmosphere can be quite stable particularly at night.
See page 31 and the residual layer.
http://www-as.harvard.edu/education/brasseur_jacob/ch2_brasseurjacob_Jan11.pdf

January 15, 2012 9:06 am

“Work is invariant of it’s path. So if a molecule starts to leave Earth and gravity pull it back to the same point no work has been performed, energy is identical.”
Agreed in relation to a molecule that is following a geodesic path. But as regards most molecules work is being done constantly as it moves in and out of the geodesic with on average a net tendency for an energised (by the sun) atmosphere to try and escape to space.

January 15, 2012 9:11 am

“Seriously, you are starting to cause me actual physical pain. Gravity does no work on an object in a circular orbit, and this isn’t a useful description of how gravity binds molecules in a gas at an equilibrium temperature.”
I know that but most molecules are NOT in a circular orbit. I only gave that as an example of how there are always two forces working in opposition such that they can be finely balanced.
Thus molecules energised by the sun are constantly trying to depart and the gravitational field is constantly trying to restrain them. That is why one cannot equate the situation to a single one off pressurisation.
I can accept that my terms of expression may be flawed but the concepts are sound.

Paul Dennis
January 15, 2012 9:15 am

Bryan
thanks for that link. I’ll find time to read it but I don’t think it invalidates my point vis-a-vis Willis’ thought experiment. There is no cooling at the top of his atmosphere because it is a non-GHG atmosphere.
Now I’m off to read your link.

January 15, 2012 9:15 am

Of course, I could be wrong and I’m willing to listen to alternative descriptions.
That is then two of us, as this is exactly the way I was thinking. I’m trying to remember some of the mechanism that I’ve heard somewhere — that increasing concentration alters the line widths (pressure broadening) and hence changes the width of the hole? But this makes little sense — the pressure doesn’t really change, and it could change things in any direction.
Does anyone know the accepted explanation? I can’t believe that this is a great big “hole” in the reasoning, although with the crap about upwelling and downwelling anything is possible.
rgb

kwik
January 15, 2012 9:18 am

Okay, I have updated my N&Z elevator speech with Willis’ comments, and my answers.
Let us see how much of it that survive this time!
kwik says:
January 15, 2012 at 1:55 am
Elevator speech for N&Z;
-The non-GHG-gas can increase its energy via conduction at the surface.
-This means the gas gets a higher temperature. Via conduction.
-Via “bouancy” it will rise.(Figure of speech; Its density lowers, so it rise)
-When ascending it looses its energy (via conduction with other gas molecules)
until “bouyance” equals gravity. Then it stops.
-The air-parcels close to ground that ascended is replaced by others, creating a circulation.
Willis says:
True so far, but the circulation cannot continue forever.
kwik answers:
Remember, this is elevator speech for N&Z. Not your model.
I think they say the sun continously shines, and that drives the pumping.
I cannot find anything wrong with this.
kwik says:
Conduction increase with increased density.
Density increase when the athmosphere has more mass.
Willis says;
The mass of the atmosphere isn’t changing, and I don’t understand why
conduction would increase with density.
Kwik answers:
Remember, this is elevator speech for N&S. Not for your model.
(You asked for someone giving an elevator speech for N&Z)
I understood that this was their main argument; It is the different athmospheric mass on
the different planets, and gravity, that counts.
SB radiation in their paper is also there, but very small.
Conduction/convection is king. (For N&Z, me thinks )
When the total mass of the athmosphere increase, surely density increase
all the way down to the bottom.
When density increase, the molecules are more densily packet.
More densily packet, means more conduction. That sounds good to me. Experienced it
many times myself when cold at sea in Norway.
kwik says:
This is not a perpeteum mobile; The energy comes from the sun.
It does not violate any laws. It radiate finally to space at TOA, instead of at surface.
Willis says;
Sorry, but there’s no GHG’s in the atmosphere … so it cannot “radiate finally to space at TOA”.
Kwik answers:
When you say “cannot radiate”, I must assume you mean “cannot radiate IR”.
When I say “radiate”, I mean radiate on all frequencies.
Willis says:
All the best, thanks for giving it a try.
Kwik says:
All the best to you too, Willis.

January 15, 2012 9:22 am

“Now imagine an adiabatic fluid. That is, one where no heat is being added or taken away. If gravity did work on it, the temperature of the fluid would increase without bound.”
But we are adding heat from the sun and releasing it to space later on an altered timescale. I’m not suggesting that additional energy is coming from ‘gravity’.
Gravity has an effect on the molecules energised by the sun by incoming solar shortwave. It increases density the nearer the surface you get and that density increases the number of molecular collisions for a slowing down of the flow of energy through the system, an accumulation of energy within the system and a higher equilibrium temperature as a result.
The ‘work’ arises by virtue of the slowing down of the transmission of energy through the system.
I really don’t think my description is as flawed as you make out.

wayne
January 15, 2012 9:24 am

Stephen Wilde says:
January 15, 2012 at 9:11 am
I know that but most molecules are NOT in a circular orbit.
— — —
Oh Stephen… look up vis viva equation. It doesn’t have to be circular, ANY orbit will do.

Kev-in-UK
January 15, 2012 9:26 am

wayne says:
January 15, 2012 at 8:59 am
I don’t follow your reasoning – if a molecule is trying to escape gravity – it must have kinetic energy in the ‘escape’ direction – the gravitational force pulls it back, so must be countering that energy, ergo, applying force, and doing ‘work’? It doesn’t matter where the energy came from that the molecule got to have the kinetic escape energy, just that gravity counters that energy and brings it back, so it must be applying force and in moving the molecule back, must be doing work?
REPLY:Brownian motion. See the section on the Stokes equation (under the Einstien section) in this Wiki entry:
http://en.wikipedia.org/wiki/Brownian_motion
Note the diagram on the right, which looks just like our atmosphere vertical profile.
Anthony

Richard M
January 15, 2012 9:29 am

Robert Brown says:
January 15, 2012 at 9:15 am
[Of course, I could be wrong and I’m willing to listen to alternative descriptions.]
That is then two of us, as this is exactly the way I was thinking. I’m trying to remember some of the mechanism that I’ve heard somewhere — that increasing concentration alters the line widths (pressure broadening) and hence changes the width of the hole? But this makes little sense — the pressure doesn’t really change, and it could change things in any direction.
Does anyone know the accepted explanation? I can’t believe that this is a great big “hole” in the reasoning, although with the crap about upwelling and downwelling anything is possible.

I think the pressure broadening argument is in response to the claims of saturation. So, it is true that a little more radiation is absorbed. The claim is this leads to a log reduction and hence the doubling of concentration to get a linear temperature increase. However, I don’t think this addresses the problem of raising the effective radiation altitude at all. More radiation is captured but it still gets emitted at the effective radiation height. In fact, I think both of these work in parallel to cap the overall effect of adding additional GHGs.

January 15, 2012 9:31 am

I know that but most molecules are NOT in a circular orbit. I only gave that as an example of how there are always two forces working in opposition such that they can be finely balanced.
But there aren’t. What two forces are acting for particles in any orbit? I only count one — gravity! The problem is that your concepts are not sound. Most molecules aren’t in an orbit at all. The things that determine binding of gas molecules to a planet are:
a) Escape velocity (or escape energy). Molecules with a total mechanical energy greater than zero (on a scale where being at rest at infinity is zero) have escape energy. For a planet in crude terms, v_escape = \sqrt{2 G M/R} = \sqrt{2 g R} (the latter in the case of Earth, with g surface gravity and R the radius).
b) The Maxwell-Boltzmann distribution of velocities (or kinetic energies) of molecules in a gas at thermal equilibrium, per mass species. Smaller masses at a given temperature have more of a MB “tail” at any given temperature and are more likely to have escape speed. Consequently, light gases like hydrogen and helium tend to relatively quickly be lost (from Earth’s atmosphere) where O_2, N_2 and so on have much longer lifetimes. All of the components of the atmosphere are partially replaced as they are lost via e.g. outgassing from the crust — otherwise there would be no helium in the atmosphere at all.
Neither of these has much to do with PV = NkT, although one can probably connect the MB distribution to it somehow. Not any obvious way other than equipartition, but somehow.
rgb

January 15, 2012 9:36 am

What interests me about the gravitational/adiabatic hypotheses is the notion that the mass of the atmosphere could vary so much over time.
The warming if an IR transparent atmosphere by compression (Browning, Pauling, etc) can be completely independent of insolation and radiation.

January 15, 2012 9:38 am

wayne, most molecules aren’t in orbit at all.
They constantly vibrate with kinetic energy, fly about with convection everywhere and throughout all those movements of whatever type they are working against but within the constraints of gravity.
There are more molecules nearer the surface due to gravity, more collisions and conduction near the surface due to gravity, slower transfer of energy through the system near the surface due to the extra density caused by gravity.
If one slows down the rate of energy transfer through anything then the equilibrium temperature will rise and the mass of non GHGS will achieve just that They will do so in relation to density which is gravitationally induced.
I am astonished how far this thread has gone away from the basic realities.

Paul Dennis
January 15, 2012 9:43 am

Bryan,
I’ve had a quick look and see nothing that doesn’t agree with what I have said. Whilst the hydrostatic pressure distribution is used to calculate the dry adiabatic lapse rate, the key statement in the article is on the bottom of page 12:
“The temperature of an air parcel changes anytime its pressure changes, i.e., anytime there is movement in the atmosphere.”.
The key point is the air parcel has to experience a change in pressure by moving for it to cool. This is atmospheric convection.
Of course if there is a temperature gradient set up by warming at the base and cooling at the top then the heat transport can be by conduction. I don’t, however, see how heat conduction can lead to a temperature gradient defined by the DALR. Surely, the gradient will depend on the rate of warming at the base and cooling at the top. I couldn’t see anything on page 31 that referred to heat conduction.
Now if there is no heat loss by radiation at the top of the model atmosphere and the base of the atmosphere is at uniform temperature then convection won’t happen and conduction will ensure the atmosphere is at uniform temperature.
I agree with you this is not a useful model for Earth’s atmosphere in which variable heating at the base, convection, conduction and radiation are occurring.

January 15, 2012 9:45 am

“What two forces are acting for particles in any orbit? I only count one — gravity!”
Plus escape energy makes two.
In fact ANY energy in a molecule causes movement that counts as a second force interacting with gravity.
Are you getting lost in semantic detail and losing the bigger picture ?

Richard M
January 15, 2012 9:57 am

Dr. Brown,
Here’s a paper that discusses some of the issues.
http://climateclash.com/2011/06/21/does-the-tropopause-limit-carbon-dioxide-heat-trapping/
They get a reduced GHE to about .7C per doubling of CO2. I still think this is too high but the details are beyond my capabilities.

gbaikie
January 15, 2012 9:58 am

So, it seems to me the million worlds providing 240 watts, can not without some kind magic increase surface temperature above -18 C.
And therefore taking sun the energy energy and dividing by 4 is wrong.
The million suns are providing a million times more energy but do not increase temperature
above -18 C.
Whereas one sun providing 500 watts per meter could have temperatures above -18 C

wayne
January 15, 2012 10:02 am

Stephen Wilde says:
January 15, 2012 at 9:38 am
wayne, most molecules aren’t in orbit at all.
They constantly vibrate with kinetic energy, fly about with convection everywhere and throughout all those movements of whatever type they are working against but within the constraints of gravity.
There are more molecules nearer the surface due to gravity, more collisions and conduction near the surface due to gravity, slower transfer of energy through the system near the surface due to the extra density caused by gravity.
If one slows down the rate of energy transfer through anything then the equilibrium temperature will rise and the mass of non GHGS will achieve just that They will do so in relation to density which is gravitationally induced.
I am astonished how far this thread has gone away from the basic realities.
— — —
Just trying to help. You were speaking of a molecule leaving the atmosphere and being pulled back into the atmosphere by gravity and you were speaking of the work done. I see no work if the beginning and ending altitudes are identical and that molecule didn’t hit another molecule in the meantime. That molecule IS REALLY IN ORBIT (high eccentricy squashed orbit) during its short excusion so yes, the vis viva equation (conservation of energy) exactly appies here and yes, no work is performed. I’ll get into this matter of orbit mechanics later if you wish, maybe better later at talkshop.
Take Dr. Brown up on his offer of the books, I am. I always find new or forgotten things in each book even though I am pretty well versed already.

Bart
January 15, 2012 10:12 am

Willis Eschenbach says:
January 15, 2012 at 2:46 am
“I didn’t say the individual pieces are at equilibrium, Bart, I don’t even know what that would mean. I said, and you quoted, that it is the system that is at equilibrium.”
Aye yi yi. You are claiming SB consistent radiation from your surface. The surface is exchanging heat with a conductive and convective atmosphere. It is therefore NOT in thermodynamic equilibrium. Fail.
If you do not get it now, I suppose you will not get it. As my old EE prof used to fondly admonish the class, “if you do not understand (fill in the blank), you should drop the class.”

willb
January 15, 2012 10:17 am

Willis Eschenbach says:
January 15, 2012 at 12:13 am

Whoa, whoa, whoa. The atmosphere is GHG-free. How does the released energy radiate anywhere, when the atmosphere has no GHGs, and as a result can’t radiate in the IR? I fear your explanation dies there …

Willis, thanks for responding to my attempt at an elevator speech. Your comment above refers to the following statement I had made:
Some of the radiating energy from the released latent heat is directed downward through the gaseous, IR-transparent atmosphere and is absorbed by the earth.
Although the atmosphere has no GHGs, my argument for this statement is as follows:
• As the non-GHG gases rise, they will cool.
• When they cool sufficiently, they will go through a phase change.
• A phase change (such as going from gas to liquid) means the molecules have dropped down to a lower energy state.
• When this happens, photons are released. It is irrelevant whether the gas is GHG or non-GHG. The release of photons is required for energy balance to counteract the drop in molecular energy from the phase change.
• These released photons are the radiating latent heat energy in my statement above.

Bart
January 15, 2012 10:19 am

wayne says:
January 15, 2012 at 10:02 am
“That molecule IS REALLY IN ORBIT (high eccentricy squashed orbit) during its short excusion so yes, the vis viva equation (conservation of energy) exactly appies here and yes, no work is performed.”
You are 100% correct here. However, work is performed when that molecule is first energized (launched into orbit) and, upon collision with another particle, that work energy is exchanged. I’m not involved in your argument with Stephen – haven’t read enough of it and am wary of Stephen’s loose parlance – just trying to help facilitate your debate to focus on rigorous concepts.

Jeff Hagen
January 15, 2012 10:20 am

Mr. (Dr.?) Eschenbach,
With the stipulation that I am not familiar with the context of this discussion outside of your post nor how it relates to the debates regarding climate science, I have a potential thought about your question. Noted as well that I am not a scientist, but only an engineer. Apologies up front as well for not having read through the voluminous replies to your post before responding, so perhaps the following has already been addressed.
The core issue as I see it is that your hypothesis implicitly assumes that the only significant thermal coupling between the solid body and the gaseous atmosphere of the idealized planetary body in question is via radiation mechanisms. Admittedly without reference to any actual sources of data, it seems to me that we should intuitively assume that the conductive coupling between the surface of the solid body and the adjacent atmospheric gas will be a sufficient factor to noticeably affect the overall system balance. Introducing the presence of the gravity field to the question will cause a convective transfer mechanism within the gaseous atmosphere itself that should further enhance the effectiveness of the conductive coupling between solid and gas at the surface, thus further increasing the influence of the conductive component of coupling between the solid body and the atmosphere relative to the radiative coupling component.
In other words, the hot ground will warm adjacent cooler air, or hot air will warm adjacent cooler ground through direct contact at the molecular level and gravity will physically move hotter air away from the surface and cooler air to the surface so as to maintain a higher temperature differential and therefore more effective conductive coupling at the solid to gaseous interface at the surface and to distribute the energy more effectively throughout the atmospheric column that by internal conductive transfer within the atmospheric gas alone.
I believe that introducing the conductive and convective energy transfer mechanisms is within the bounds of your question as stated.
The conclusion is that even if we assume a perfectly transparent atmosphere with no radiative coupling between solid and gas and no direct solar heating of the gas, there is still a thermal coupling mechanism that introduces energy to the atmosphere. It is my assumption (without data) that this component will be substantial enough to have a noticeable impact on the balance of the idealized system in question.
I have not thought through clearly how this explanation explains the theories you cited as I have not read the full context of the broader question.
One thought to consider is that the solar energy absorbed into the atmosphere by the aforementioned conductive coupling will cause expansion of the atmospheric volume. As the atmosphere goes through day / night cycles, it will go through expansion and contraction cycles that could be considered as mechanical work against the gravity field that creates energy absorbed back into the solid body, thus raising it’s temperature above the minimum due to radiative solar heating alone. In other words, the gas warmed on the day side can not radiate back to space since it is perfectly transparent and thus can only cool by reheating the solid body, thus warming the night side. There will also be friction effects within the convective gas flow and within the warming and cooling solid body itself to consider. However, I am not sure if the day / night cycle is a permissible consideration within your stipulation of an evenly distributed, constant solar radiation.
Another thought to consider is that with the energy coupling to the atmosphere, the effective size of the absorptive body can be considered as larger than the solid body alone for purposes of the net energy balance. In other words, because some of the energy absorbed by the solid body is transferred into the transparent atmosphere, the solid body can absorb more radiative energy than it could were no atmosphere present. The net effect would be that the solid body is apparently absorbing an amount of radiation equivalent to a larger solid body, thus increasing its net energy balance above what it should be for perfect emissivity the actual surface area of the solid body and thereby causing its temperature to be elevated above the theoretical value of the solid body in a vacuum. Because the atmosphere is assumed to be perfectly transparent and thus unable to radiate, the presence of the atmosphere does not introduce any additional capacity for energy rejection. That is, within the ground rules of your question, introduction of the atmosphere permits a net planetary absorption of energy equivalent to a planet with a larger solid body, but maintains a net planetary emission equivalent to the actual size of the solid body, thereby skewing the net energy balance of the planet upward. The presence of the gravity field serves to more effectively distribute the energy throughout the atmosphere than gaseous conduction alone, thus further enhancing the affect of the atmosphere on the net energy balance. This explanation I believe does fit within the your stipulation of an evenly distributed, constant solar radiation. Again though, I have not looked at any data to see if it is a sufficient explanation for the values cited. Perhaps someone more educated than I on the relevant subjects can address whether or not this is a sufficient explanation.
Interesting question. Hopefully these thoughts are of some use to you even if not correct or not directly responsive to your question.

Jeff Hagen
Reply to  Jeff Hagen
January 15, 2012 10:31 am

Of course, I forgot to continue the thought that once equilibrium is reached, it doesn’t matter if the energy is distributed through the atmosphere by conduction or convection, the total will be the same. It is still necessary to introduce consideration of gravity though in order to keep the atmosphere from flying away from the planet. Without gravity, we would have to consider the energy conductively absorbed into the atmosphere as causing the atmosphere to depart the planet and thus not reaching equilibrium until the solid body was again in vacuum and thus violating the assumed conditions. Perhaps that is what was meant by taking gravity into consideration in the original theories?

Paul Dennis
January 15, 2012 10:23 am

Bart,
the surface in Willis’ thought experiment is exchanging heat with a conductive and ‘potentially’ convective medium, a thin, non-GHG atmosphere. However, the heat input in Willis’ model is uniform everywhere at the boundary between the surface and atmosphere. Therefore, there will be no convection. As the atmosphere is non-GHG it won’t lose energy by radiation loss at it’s upper surface (however the upper surface is defined). Thus it will heat up by conduction to the surface temperature. Equilibrium will be established when the atmosphere reaches the surface (Stefan-Boltzmann) and has a uniform temperature throughout. At least that’s my reading of Willis’ thought experiment.

Bart
January 15, 2012 10:25 am

The only effect of gravity in all of this is that it effectively forms the constrained space in which atmospheric molecules interact. It does no net work. This is very basic.

Jeff Hagen
Reply to  Bart
January 15, 2012 10:58 am

Bart says:
January 15, 2012 at 10:25 am
“The only effect of gravity in all of this is that it effectively forms the constrained space in which atmospheric molecules interact. It does no net work. This is very basic.”
Agreed. That’s essentially what I was trying to say in my reply to myself. (My apologies to all for not doing a better job of editing my original post before posting.)
If we assume as stipulated that the solar radiation is constant and uniform in time and space and we assume having reached a steady state, gravity is not adding any work. The catch is that if we don’t consider gravity then to reach a steady state we have to assume that the atmosphere has continued to absorb energy through convection until it has expanded to the point that it is effectively removed from the picture, thus departing from the core question. Without being familiar with the theories questions by the original post, I assume this is what they meant by taking gravity into consideration.
I still think that the core business here is that the question as stated can not be addressed purely on the basis of radiation, but must take into consideration conduction between solid and gas as well.

gbaikie
January 15, 2012 10:28 am

gbaikie says:
January 14, 2012 at 10:22 pm
“[SNIP: gbaikie, either give us your elevator speech or falsify my model, your ramblings go nowhere. w.]”
Well that post explained that earth with 10 times the gravity in a million sun world, would have
a much denser atmosphere and the air molecules would move slower than on earth.
And the temperature from the million sun which give 240 watts per square can not exceed -18 C.
So sun providing million times more energy than the sun, does not heat a planet as much as our sun does it earth distance- obviously our moon’s daytime temperature exceeds 120 C.
With million suns providing 240 watts per square meter the Moon would not get above -18 C.
Put a world 10 gees at earth distance from the sun with same amount atmosphere as earth with
no greenhouse gases and the average temperature would exceed earth’s average temperature.
And also Neptune doesn’t even receive 100 watts per square meter from the Sun, but exceeds Venus’s temperature- due to high gravity and large atmosphere.

January 15, 2012 10:30 am

Sorry to give the impression that there is any sort of ‘argument’ with wayne or Dr. Brown for that matter.
I am aware that my parlance is a bit loose for the experienced scientists here but I’m doing my best and picking up a lot along the way.
The only thing I would ask is that readers look past my parlance where it is weak and at the concept I am trying to express.
At some point the conclusions to all this are going to have to be readily comprehensible to the general public so a merging of mytype of style and the more rigorous statements is going to be necessary at some point.
Willis, I have reposted at:
Stephen Wilde says:
January 15, 2012 at 3:36 am
If it doesn’t make sense to you then there is little else I can add.

Bart
January 15, 2012 10:31 am

Paul Dennis says:
January 15, 2012 at 10:23 am
Bart,
Therefore, there will be no convection.
I know, but I left the term in there because conduction is not as powerful an effect, and the real world, to which we will return someday I suppose, has powerful convective action going on.
Equilibrium will be established when the atmosphere reaches the surface (Stefan-Boltzmann) and has a uniform temperature throughout.
There will always be a temperature gradient. There will always be a lapse rate. There is no equilibrium possible.

Paul Dennis
January 15, 2012 10:39 am

Willis,
you are absolutely right about thermal conductivity of gases. There is much confusion about thermal conductivity throughout this thread and I think much of it comes from peoples every day experience with vacuum flasks in which they understand that reducing the pressure in the cavity between the walls of a flask reduces the thermal conductivity. I fact the thermal conductivity does not reduce until the mean free path of molecules is greater than the distance between the inner and outer walls of the flask. A high vacuum is required for most flasks for this to be achieved. What does this mean for an atmosphere. It means that the thermal conductivity is constant irrespective of pressure/density as you have pointed out.

January 15, 2012 10:41 am

“It is a rambling inaccurate claim that the rate at which heat is conducted varies by the density of a gas. ”
Actually it isn’t.
It is a claim that if conduction increases relative to radiation then the rate of energy flow through the system declines.
Radiation leapfrogs across non GHGs at the speed of light but conduction from molecule to molecule of a non GHG is slower and the more dense the non GHGs the more radiation is replaced by conduction.
Can you rebut that for me ?

Bart
January 15, 2012 10:42 am

Jeff Hagen says:
January 15, 2012 at 10:20 am
“As the atmosphere goes through day / night cycles…”
Your post shows you are assembling all the pieces properly. However, I believe Willis has enforced uniform heating without day/night cycles in an attempt to form some basis from which to extrapolate additional behavior. Whether any such extrapolation is valid or useful is another argument to be waged.

Bart
January 15, 2012 10:45 am

Of course, the very notion of uniform spherical heating requires rather more than a few Suns. The atomic scientists, as I’m sure most here recall, had to construct a semi-uniform spherical shock wave to implode the plutonium in the Fat Man bomb. It’s not particularly easy to do, and requires a very artificial setup.

Bart
January 15, 2012 10:47 am

Willis Eschenbach says:
January 15, 2012 at 10:43 am
“The dry adiabatic lapse rate is a result of energy considerations. It does not depend in any way on conduction.”
No, but conduction depends on it.

steveta_uk
January 15, 2012 10:48 am

“slower transfer of energy through the system near the surface due to the extra density caused by gravity”
Willlis was very explicit in talking about IR from the surface leaving via a non_GHG, i.e. IR translarent atmosphere.
Exactly why do the IR photons “slow down” due to increased density of an IR-transparent gas?

Bart
January 15, 2012 10:50 am

Paul Dennis says:
January 15, 2012 at 10:39 am
“[In] fact the thermal conductivity does not reduce until the mean free path of molecules is greater than the distance between the inner and outer walls of the flask. It means that the thermal conductivity is constant irrespective of pressure/density as you have pointed out.”
Nearly constant. And, only at low altitude. And, the transition altitude will depend on lapse rate.

anna v
January 15, 2012 10:51 am

wayne says:
January 15, 2012 at 5:25 am
The Stefan–Boltzmann law when used for gases is multiplied by the emissivity of that gas, a measured value which has to be measured for all materials, since a true black body is rare.
I have once seen a derivation that gases follow a T^6 law instead of T^4, but cannot find it again :(.
And please, one more, could you clarify what is meant by “on shell / off shell”? (electron shells of course)
This terminology has to do with Feynman diagrams one would write to calculate the interactions quantum mechanically. Off ( mass)shell means the particle ( photon here) is virtual, between two vertices. On (mass) shell means that the photon leaves the molecule and becomes part of the “radiative pool” used to derive the black body radiation formula.

Bryan
January 15, 2012 10:54 am

Paul Dennis says: “Of course if there is a temperature gradient set up by warming at the base and cooling at the top then the heat transport can be by conduction. I don’t, however, see how heat conduction can lead to a temperature gradient defined by the DALR. Surely, the gradient will depend on the rate of warming at the base and cooling at the top. I couldn’t see anything on page 31 that referred to heat conduction.”
The neutral atmosphere is the atmospheric condition for;
1. ……still air
2….. or for air parcels moving at constant speed (no unbalanced force )…….Top of page 15.
The DALR gives the temperature rate for such atmospheric conditions.
These conditions are often met at night…page 31
“stable boundary layer capped by a NEAR NEUTRAL residual layer.”
If the air happens to be still then only two methods of heat transfer are left
1. diffusion or molecular collision (conduction process)
2. radiation
Yet there is no term for radiation in the gradient formula = – g/Cp
Solution
Both these methods must be involved and the radiative effects of CO2 are grossed up in the bulk thermodynamic quantity Cp.

Bart
January 15, 2012 10:55 am

steveta_uk says:
January 15, 2012 at 10:48 am
“Exactly why do the IR photons “slow down” due to increased density of an IR-transparent gas?”
Do you know how Cherenkov radiation is produced?

steveta_uk
January 15, 2012 11:01 am

Do you know how Cherenkov radiation is produced?
Yes – not like this 😉

January 15, 2012 11:01 am

“I was with you until the last point. The dry adiabatic lapse rate is a result of energy considerations. It does not depend in any way on conduction.”
Conduction involves energy considerations.
The dry adiabatic lapse rate results in a higher temperature at the surface. Since there are no GHGs the energy comes from sun to surface and is then supplied to the air by conduction. Conduction supplies the energy in the gas above the surface.
“No, that is the dry adiabatic lapse rate. It is not a “gravity induced greehouse effect”, there’s no GHG’s, remember?”
Any greenhouse effect there may be from GHGs is seperate from the gravitationally induced effect. Both may be validly termed greenhouse effects but from differing causes. I don’t think the issues can be resolved without recognising both.
Once you accepted the existence of the dry adiabatic lapse rate in a non GHG atmosphere you accepted the gravitationally induced component.
“First, there is no energy going either into or out of the atmosphere in my example in the head post”
That only applies AFTER equilibrium has been reached and I don’t disagree with you there. However any change in solar input or atmospheric mass of non GHGs will result in a change in equilibrium temperature.
“But if it does the surface is radiating to space more than it receives, which violates conservation of energy.”
The surface isn’t radiating to space more than it receives. The conductive energy acquired by the non GHG atmosphere is stored there and bounced back to and fro between surface and atmosphere by conduction.The lapse rate causes the warmest air molecules to be in contact with the surface and because those molecules are at a higher temperature than the average for the atmosphere they feed energy back to the surface more than if the atmosphere were at a uniform temperature throughout. That should cause a higher surface equilibrium temperature and the atmospheric storage reduces the outflow to space to match solar input at equilibrium.
“the fact that the atmosphere absorbs some of the surface radiation decouples the surface from space, and allows its temperature to rise without breaking any natural laws.”
Exactly, you got it. And all without GHGs.

January 15, 2012 11:04 am

“Exactly why do the IR photons “slow down” due to increased density of an IR-transparent gas?”
The IR photons don’t slow down.
Instead the incoming solar shortwave remains longer as kinetic energy passing between colliding molecules before it is released out again as IR.

Bart
January 15, 2012 11:09 am

steveta_uk says:
January 15, 2012 at 11:01 am
“Yes – not like this ;)”
My point was that the speed of light is dependent upon the material through which it flows, based on the material’s index of refraction.

gbaikie
January 15, 2012 11:11 am

A million suns which provides 240 per square meter will give world which has earth like atmosphere a temperature of 255 K [-18 C]
Earth at sun distance receives about 1300 watts.
A million worlds which provide about 1300 watts per square meter will give world which has earth like atmosphere the temperature will not not exceed 390 K [116 C].
The reason why earth isn’t 116 C is because it’s a sphere and rotates- it can radiate 4 times the amount of energy it receives.
So earth will warm up until it reach a temperature in which it radiates the same energy as it receives. The possible range is -18 C to around 110 C
Whereas with the million suns at 240 watts per square meter is in the range is not more than -18 C.
So the question isn’t want warms earth, the question is what cools earth. The atmosphere prevent 300 watt of the 1300 watts from reaching the surface- this knocks down highest temperature to below 370 K [90 C]. Next, convection lowers this highest temperature down by around 10 C.
And these temperatures are observable on earth.
Another factor is clouds. Another factor is the spherical shape- or hemisperical shape- at higher
latitudes the sun never provides 1000 watts per square meter.
End of elevator speech.

Richard M
January 15, 2012 11:14 am

Dr Brown,
If you’re still following the discussion there is another hypothesis that I saw at Tallbloke’s that also leads to reducing the impact of GHGs on temperature.
http://declineeffect.com/wp-content/uploads/2011/08/Pot-Lid-Sep-2011-v2.1.pdf

Juraj V.
January 15, 2012 11:15 am

Too much complicated, again. Surface is heated by a sunlight, and atmosphere (99% nitrogen and oxygen) is warmed from the surface by conduction and convection. Nitrogen and oxygen thus indirectly absorbs the energy of Sun. The higher density, the more molecules kicking the thermometer, which shows higher temperature. Very few molecules in the atmosphere (like Mars) – theoretical and blackbody temperature = 310K. Does not matter, that Mars thin atmosphere has 6,000 ppm of CO2.

January 15, 2012 11:16 am

Robert Brown said @ January 15, 2012 at 8:26 am

To avoid confusion, I’m NOT stating that the GHE is a chemical reaction, that is simply an analogy. What happens is the GHGs become well mixed in the atmospheric profile due to pressure and heat. Hence, the GH effective radiating altitude gets set very high even with low concentrations. Because the atmospheric profile is changed very little by adding additional GHGs, that altitude does not increase if things like additional CO2 are added. It is that height that determines the overall GHE.
This seems correct to me, although somebody (Tim F.?) asserted otherwise. Indeed, I suspect that the Earth is largely insensitive to changes in GHG concentrations, and might even operate the opposite way than expected in some cases. Once you are “opaque” you are opaque, and making it twice as opaque doesn’t really happen. I am actually curious as to whether anybody knows anything concrete about this, as it has bothered me for some time. I’d expect alterations in the outgoing radiation profile due to doubling CO_2 concentration to be, well, almost impossible to detect, as radiation of IR from the troposphere is going to still be radiation of IR from the troposphere. Is the troposphere going to move? Will the adiabatic lapse rate change? Why, exactly, is radiative balance going to change?
rgb

As I understand it, while CO2 is perfectly opaque over a range of frequencies, at the upper and lower bounds absorption/emission is less. Thus there is a small change in the atmosphere’s ability to absorb/emit as CO2 concentration rises above current levels. When I first became interested in such things, I too was initially puzzled as a back of envelope calc showed that only a 100 metre column of atmosphere was required for total opacity. Hans Erren who helped my understanding in this linked to a satellite photograph that showed Earth emitting some small amount of radiation at these boundaries.

jjthom
January 15, 2012 11:29 am

measurement of night time downward radiation
http://www.patarnott.com/atms749/pdf/LongWaveIrradianceMeas.pdf
basically (irradiance in watts/sqm
night1 275
day1 315
night2 290
day2 360
night3 300
day3 410
night4 340
day4 variable ~
night5 340
day5 390
night6 380
day6 375
night7 360
day7 360
night8 270
day8 300
So we know the solar output received in the dark is 0
We know that O2 and N2 have very very very little thermal radiation.
So where does all that downward radiation come from (at least 270W/sqm)
It can only be from GHGs.
During the day we receive about 380W/sqm
So the 270W/sqm is additional to the solar irradiance.
So the average night day temp difference is less than expected (no sideways conduction/wind required)

Bart
January 15, 2012 11:30 am

Some more grist for the mill. We all know that you do not have to dig very far, nor swim very low, before temperatures plummet. The Earth itself violates the SB conditions because it is not uniformly heated within, and is continually conducting/convecting heat down in daylight, and giving it up again at night.

Bart
January 15, 2012 11:35 am

All right, I’m going to do a Willis here, and declare victory. I am not proclaiming that the GHG hypothesis is wrong, or even mostly wrong. But, clearly the KO intended by using SB to “prove” that it must be the case is just a pat on the cheek.
A) SB holds only for an ideal radiating body at thermodynamic equilibrium.
B) The Earth’s surface is never at thermodynamic equilibrium.
Hence, the SB argument is riddled with holes. QED.

wsbriggs
January 15, 2012 11:40 am

Willis,
Your patience has been astounding! All the complaints about snipping, when you initally stated you would snip, and to what criteria you would snip. It was all up front in the original post.
The most disappointing thing about the whole thread is the number of posters, who, at least as I recall, have previously been able to respond coherently to a clearly posted challenge and who have subsequently in this post looked like goats in loco weed.
Students who responded in such a fashion to a hypothetical question in our theoretical physics oral examinations would have been retaking the entire examination in another six months – instant fail.
Perhaps thinking about layering the gas molecules a layer at a time around the sphere would help people understand that conduction is how the molecules gain energy from the sphere, and how, over time, the original steadystate energy balance is reasserted. Regardless of how many mono-atomic layers you add to the original layer, conduction will simply cause the gas molecules to reach the surface temperature of the sphere, and no higher.

Paul Dennis
January 15, 2012 11:54 am

Bryan,
The definition of a neutral atmosphere is one in which the local lapse rate for a particular parcel of air is the same as the lapse rate for the environment (the DALR). Therefore there are no residual forces on the parcel, no convection occurs and the atmosphere is stable. However, that doesn’t say how the temperature profile was intitially set up which is presumeably by convection of the atmosphere prior to the establishment of neutral conditions.
I’m going to spend some time studying this because I’m coming at this very much from a first principles approach rather than as someone who has studied these things and am very much aware I may have misunderstood some concepts. Thank you for pointing out this informative link and discussing with me.

willb
January 15, 2012 11:57 am

Phil. says:
January 15, 2012 at 8:05 am

Not if we consider a N2 atmosphere, if we have a 20ºC surface at 100kPa we’ll hit -60ºC (~210K) at 33kPa based on the dry adiabat, below the triple point so the liquid phase is out of play.

From Wikipedia:
Triple point of Nitrogen = 12.53 kPa
Triple point of Oxygen = 0.152 kPa

January 15, 2012 12:00 pm

“conduction will simply cause the gas molecules to reach the surface temperature of the sphere, and no higher.”
Increasing conduction relative to radiation ( which is what increased density at the surface achieves) will reduce the rate of energy flow through the system and result in a warming effect because total system energy content rises given a stable energy input.
Due to the dry adiabatic lapse rate the temperature is not uniform through the atmospheric column. The molecules at the surface are warmer than the average for the atmosphere.
The conductive interchange at the surface is skewed by the dry adiabatic lapse rate in favour of an increased conductive interchange at the surface for a raised surface equilibrium temperature..
In the case of the Earth some 240 Wm2 comes in. The surface reaches a temperature that would require 390Wm2 to go out to space but only 240Wm2 escapes at equilibrium.
The difference of 150Wm2 is attributable to the conductive and radiative interchange between surface and atmosphere. As Willis said, the atmosphere decouples the surface from space.
If there were no GHGs then the conductive interchange would be all we have and on the basis of the N & Z data the surface temperature can be calculated accurately without including a radiative component.
Thus on Earth the radiative component gets dealt with by non radiative energy transfer processes.
That means that for a planet without GHGs as per Willis’s proposed model the conductive surface/atmosphere interchange would be sufficient to keep the surface temperature much as it is now.
Does that qualify as an elevator speech ?

anna v
January 15, 2012 12:03 pm

Jonathan Jones says:
January 15, 2012 at 8:04 am

Anna V,
You say “Thus one cannot have matter as we know it of a given temperature not radiating in some frequencies.” Consider a sample of helium gas at standard temperature and pressure (roughly 270K, 1atm). At what frequencies do you think it will radiate? What transitions will cause this? And what are the relevant transition dipole moments?

Given the emissivity of helium it will have a gray body radiation spectrum appropriate to its temperature . Gray body is Stefan–Boltzmann law times emissivity. Small, I do not dispute it, but still there.
My point was that the thought experiment that Willis claims proves energy violation cannot be carried out with the physical matter we have. That’s all.

Kevin Kilty
January 15, 2012 12:07 pm

Kevin Kilty says:
January 14, 2012 at 10:09 am
Phil’s Dad says:
January 14, 2012 at 9:01 am
I’m not sure about lapse rate warming being the answer as lapse rate cooling in the convection cycle would tend to cancel it out however… does gravity (which can do work) add energy to the equation such that radiation in + work done by gravity = radiation out? (Where the heck does gravity come from anyway?)
Let’s put this gravitational work in its proper perspective…
[SNIP: Let’s not. Lets stick to the topic of the thread. -w.]

Well, it was on the topic of the thread which is gravity, work done by gravity, etc.. Haven’t you, who ever you are, gotten a little liberal with the scissors?

Alan Wilkinson
January 15, 2012 12:18 pm

Paul Dennis, I thought some more about the isothermal equilibrium postulate and concluded it is also unphysical since gravity would then be unable to prevent the escape of gas molecules at the top of atmosphere resulting in continuous loss of matter and energy. This is the difference between Willis’s model and a closed adiabatic column in gravity.
So I don’t think either the isothermal or isentropic models fit.

January 15, 2012 12:24 pm

I don’t see anything in either of those that says “all substances are required to emit thermal radiation until they are at absolute zero”, or even “all substances are required to emit thermal radiation at all temperatures”.

All objects are required to emit photons until they are at absolute zero.
Now you have a quote. Put it to use.

Paul Dennis
January 15, 2012 12:30 pm

Alan Wilkinson,
well of course there is a continuous loss of matter and energy at the top of the atmosphere because there will always be some molecules and atoms with the necessary escape velocity as a function of the Maxwell-Boltzmann distribution. Though only a very tiny fraction of molecules will have the necessary energy and the rate of loss will be small.
However, I do agree with you Alan that I haven’t given enough consideration to the boundary conditions at the top of the atmosphere and that the difference between Willis’ model and a ‘closed’ adiabatic column might well be significant. I’m finding the discussion very interesting and it’s posing some interesting problems that require some thought. Thanks for thinking about and responding to this.

January 15, 2012 12:37 pm

Willis said:
“But in my example, there is no compression going on. There is no variation in pressure at all. ”
and over at tallbloke’s
A Watts said:
“As far as I’m concerned, the idea that gravity provides energy input (beyond the one time initial compression during attraction and capture) is simply a misinterpretation.”
I’m amazed that so many modern scientists have that idea.
I learned decades ago that gravity is a constant force all pervasive and always acting.
The only reason we don’t all fall to the centre of the Earth is that the ground is denser than our bodies.
But for gravity every molecule of the planet right down to the core would float off into space.
At every moment our every movement and every movement of every atom molecule or elementary particle is being opposed by the constant gravitational field
It is not akin to a single pressurisation. It is akin to continuously renewed pressurisation and every time anything moves or is prevented from moving by a denser object work is being done and heat energy produced.
The Laws of Thermodynamics are preserved because the heat energy is produced from the change in momentum of molecules caused by movement within the gravitational field.
Heat isn’t produced by gravity itself but by the changes that the gravitational field imposes on matter moving through it.
A Watts said:
“Shut off the energy, and the model planet goes back to steady state equilibrium with the atmosphere clinging like paint again.”
Maybe so but a force is still being applied constantly drawing the ‘paint’ towards the centre of the Earth. That movement is blocked by the density of the ground beneath but the gravitational attraction is still present and to the extent that the ground restrains the paint from falling further an imperceptible amount of heat energy is being constantly produced.
Is that really a surprise to everyone but me ?

hmccard
January 15, 2012 12:38 pm

Willis Eschenbach says:
January 14, 2012 at 6:00 pm
hmccard says:
January 14, 2012 at 5:36 pm
… and leave the moderating to Willis, eh? (Or is this a new moderating policy?) Although I have been a lurker here at WUWT for several years, I don’t know what the average number of snips per thread is for your blog. However, IMO, the number of snips by Willis in this thread must be at least a 6-sigma outlier. FWIW, the number of self-snips on this thread suggests to me that others share my disappointment.
Willis, I prefer to use the stairs …
I don’t get this complaint. I said that, contrary to my usual practice, I was going to snip this thread to try to keep it on course, to prevent from wandering around the entire universe. I have done so, and now you want to complain that I’m snipping posts … what part of ‘I’m going to snip posts’ was unclear to you?
w
Willis, I was replying to Anthony’s comment. In my comment directed to you, I said that I prefer to use the stairs.

Paul Dennis
January 15, 2012 12:43 pm

Willis,
I’ve posed the question about dry adiabatic lapse rate. I think Roy Spencer has and Anthony too though neither explicitly. In your model the planet surface is uniformly heated and there is no loss of energy from the top of the atmosphere. Under such conditions convection is supressed and the dry adiabat temperature profile cannot be established. My understanding is you think that the lapse rate of temperature can be established without atmospheric movement. I’m not sure this is the case. The texts I’ve seen all suggest that convection is required to establish the dry adiabat lapse rate. e.g. www-as.harvard.edu/education/brasseur_jacob/ch2_brasseurjacob_Jan11.pdf page 13.
“An atmosphere left to evolve without exchanging energy with its surroundings will eventually achieve an adiabatic lapse rate due to the motion of air parcels up and down.”
In the absence of convection then I think the profile should evolve to one of a constant temperature. I’m prepared to be proven wrong on this but haven’t seen a convincing statement that the dry lapse rate can be established without atmospheric movement.

January 15, 2012 12:48 pm

Willis, you insulted me on the basis of a complete misreading of my post:
Stephen Wilde says:
January 15, 2012 at 10:41 am
“It is a rambling inaccurate claim that the rate at which heat is conducted varies by the density of a gas. ”
Actually it isn’t.
It is a claim that if conduction increases relative to radiation then the rate of energy flow through the system declines.
Radiation leapfrogs across non GHGs at the speed of light but conduction from molecule to molecule of a non GHG is slower and the more dense the non GHGs the more radiation is replaced by conduction.
Can you rebut that for me ?
Please do me the courtesy of a considered and polite response.

January 15, 2012 12:51 pm

Stark Dickflüssig said @ January 15, 2012 at 12:24 pm

All objects are required to emit photons until they are at absolute zero.

And where in Resnick & Halliday (or other physics text) will I find this quote, or its equivalent?

January 15, 2012 12:55 pm

As a very slight aside:
I’d like being able to use gravity in some manner that would provide for vitually unlimited energy.
Cavorite, anyone?

Paul Dennis
January 15, 2012 12:55 pm

Stephen,
I think Willis’ model is correct, bar one aspect which is that of the dry adiabatic lapse rate. I think that in the model the atmosphere will rise to a uniform temperature which is that of the planetary surface. This atmosphere is unable to radiate energy because it is composed exclusively of non-GHG molecules. Thus at equilibrium all the energy is radiated from the planetary surface. There is no conduction of heat through the atmosphere since the surface and atmospheric column are all at the same temperature.
I don’t know if this answers your questions but I’m satisfied that Willis’ reductio ad absurbum approach provides a proof that gravitational effects cannot raise the temperature of the planetary surface.

January 15, 2012 12:59 pm

OT, but someone above mentioned an isolated parcel of non-GHG remaining at a constant higher than 3K temperature since it can’t radiate photons. In the real Universe it will lose heat as particles of rock move through it by conduction to those particles. Heat death of the universe is slow, but inexorable.

anna v
January 15, 2012 1:00 pm

Robert Brown says:
January 15, 2012 at 7:54 am
Anna v :motion in electric fields generates radiation and this radiation gets out of the solid at the frequencies where the solid is transparent and thus the solid cools because energy is lost.

Robert Brown:
In a solid the radiation gets out at the surface, and comes out at the frequencies where the solid is opaque — you have it exactly backwards. Atoms or molecules absorb or emit symmetrically, or nearly so (the laser is an example of an exception, but requires special preparation and states that line up “just right” with an external driver). The whole point of blackbody radiation is that if a solid is a “perfect absorber” — a black body — it is also a “perfect emitter” in all frequencies and hence generates a BB spectrum if it has a given temperature as derived by Planck, subject to his quantization hypothesis.
But you say it yourself: “if a solid is a “perfect absorber” — a black body — it is also a “perfect emitter” .
I was thinking on the production of the photon side.

January 15, 2012 1:12 pm

Paul,
I think Willis is right about the dry adiabatic lapse rate and that you are right about movement being required to achieve it.
The important point is that it must form with or without GHGs. That is the nub of the issue.

January 15, 2012 1:13 pm

LO! thepompousgit maketh a great clamouring, as the tinkling of brass upon January 15, 2012 at 12:51 pm

Stark Dickflüssig said @ January 15, 2012 at 12:24 pm
All objects are required to emit photons until they are at absolute zero.
And where in Resnick & Halliday (or other physics text) will I find this quote, or its equivalent?

Are you saying it’s untrue, or are you just too lazy to do a text search on the single largest collection of human knowledge in all of history?
http://lmgtfy.com/?q=objects+emit+photons+absolute+zero

Bart
January 15, 2012 1:14 pm

I have been over this already upthread, but I want to be sure interested readers late to the game understand the principles that I am elucidating.
SB can fail to hold in at least two ways:
A) the energy distribution is skewed or non-uniformly distorted from the Planckian distribution by non-radiative, or at least non-uniform, energy dissipation mechanisms
B) the spatial distribution of radiation can be altered
On the first score, pro-GHG commenters have held that emissivities of common surface constituents have been analyzed, and bulk emissivities have been measured near unity. BUT, those measurements were carried out in particular environments which are non-representative, e.g., in the laboratory or in placid weather conditions in calm waters. Worldwide energy dissipation mechanisms and radiative conditions are hardly represented here.
Moreover, the measurements are typically not of the entire spectrum, but of selected bands, e.g., those performed by Niclòs et al – 2005, which looked at “four channels placed in the 8–14 μm region”. Overall emissivity is then extrapolated based an an assumed Planckian distribution. So, skewing or non-uniformity in the distribution would not be detected, and the claim of near unity emissivity becomes significantly a circular argument.
On the second, the spatial distribution can be altered particularly by absorption characteristics of neighboring masses. The angular dependence of emissions in the aforementioned reference is attributed to surface roughness of the waves. Imagine how it would vary with larger waves or choppy seas! Or, if measurements were done on land, with local flora and landmasses. The effective emissions must be scaled to what they would be for an equivalent perfectly smooth, flat plate to match SB conditions. There is no indication in this article that such scaling has been done, but the nosedive the curves take after 70 degrees suggests the integrated area loss could be substantial, and this is in calm conditions.
So, in sum, SB has not been demonstrated to be applicable, and it is a thin reed upon which to base the entire GHG argument.

January 15, 2012 1:19 pm

“It is not akin to a single pressurisation. It is akin to continuously renewed pressurisation and every time anything moves or is prevented from moving by a denser object work is being done and heat energy produced.”
For the avoidance of doubt I am NOT suggesting that it is the interaction of mass with gravity that causes the dry adiabatic lapse rate.I think that misinterpretation was placed on my words previously.
Instead it is the effect of gravity in creating greater density at the surface which then causes more collisional activity, more conversion of radiation to conduction and a slowing down of energy flow through the system.
The increased density at the surface amplifies the initial miniscule gravitational effect.

Myrrh
January 15, 2012 1:24 pm

Ed Fix says:
January 14, 2012 at 7:54 pm
Willis Eschenbach says:
January 14, 2012 at 5:48 pm
…you claim that non-GHG gases radiate in the IR, so you’ll have to rebuild the parts of your theory that depend on that incorrect claim.
Willis, you’re conflating absorption with emission. Any body, solid or fluid, emits electromagnetic radiation with a spectrum determined by the S-B relationship. A body’s transparency spectrum is independent of its emission spectrum. They’re separate processes.

http://wattsupwiththat.com/2012/01/13/a-matter-of-some-gravity/#comment-864675
========
This is the problem I’m having with trying to answer here, you can’t have it both ways, either everything above absolute zero emits ir or it doesn’t. If hot air doesn’t then it’s defying this world’s physics.
Hot air radiates out heat which is thermal infrared, which is thermal energy on the move by radiation. If it can’t get hot by absorbing thermal infrared (spectrum), then one of the other methods of getting hot must be in play; by transfer in conduction or convection, or by pressure or whatever.

Bart
January 15, 2012 1:25 pm

Paul Dennis says:
January 15, 2012 at 12:55 pm
“There is no conduction of heat through the atmosphere since the surface and atmospheric column are all at the same temperature.”
Incorrect. The column cannot be at the same temperature, because that would require progressively more energy stored in each progressively higher altitude spherical shell, which would be an unstable configuration.

Bart
January 15, 2012 1:28 pm

“In my thought experiment in the head post, at equilibrium there is no heat exchanged between the surface and the atmosphere.”
Then, you are thinking of something completely unphysical, and we are arguing how many fairies can dance on the head of a pin.

Paul Dennis
January 15, 2012 1:29 pm

Bart,
I don’t understand your comment. Can you elucidate please? Thanks.

Alan Wilkinson
January 15, 2012 1:36 pm

Bart: “The column cannot be at the same temperature, because that would require progressively more energy stored in each progressively higher altitude spherical shell, which would be an unstable configuration.”
Several problems with this claim. First, at higher altitudes the pressure is lower and there is less gas in each shell, therefore its total energy is diminished. Second, it is not unstable. An isothermal column is neutral to convection whereas a temperature gradient in either direction is not – being either stable or unstable according to direction.
However, as I noted above, the upper boundary of an isothermal atmospheric column is unphysical since it allows the gas to escape gravity.

Niels A Nielsen
January 15, 2012 1:48 pm

Excellent post, Willis E

Reed Coray
January 15, 2012 1:49 pm

Ferd Berple (14 Jan 9:37 am), I like your comment.
With some amplifications to your statement “Since we know that net energy [rate] radiated to space is a function of temperature, we can say that Temperature surface(2) > Temperature surface(1),” your logic seems to prove that the surface temperature of a sphere in the presence of a greenhouse-gas-free atmosphere will be higher than the temperature of sphere in the presence of an atmosphere containing greenhouse gases. If all your assumptions (implicit and explicit) are valid, your arguments seem to disprove Willis’s Elevator Speech.
However, if the contrary is true (i.e., if adding greenhouse gases to an atmosphere increases the surface temperature), then there has to be an error buried somewhere in your logic. I propose three sources of error. Each potential error source requires an amplification to your statement: “Since we know that net energy [rate] radiated to space is a function of temperature, we can say that Temperature surface(2) > Temperature surface(1).”
The amplifications I would like to add to your statement are, (a) the area of surface(1) is equal to the area of surface(2)–this is almost true by definition, (b) the radiation rate per unit area emissivity (i.e., proportionality constant) of surface(1) is the same as the radiation rate per unit area emissivity of surface(2)–if emissivity is a function of gas adjacent to the surface, this amplification is open to debate, and (c) in the presence of an atmosphere (gas) of any kind, the radiation rate per unit area of any surface has the same T^4 monotonically increasing temperature dependence as the radiation rate per unit area from a blackbody (or graybody)–I believe this is amplification is likely to be incorrect.
Consider these amplifications in isolation. First case, (a) the area of surface(1) is greater than the area of surface(2), (b) the emissivity (energy rate per unit area proportionality constant) is the same for both surfaces, and (c) for both surfaces the energy rate per unit area as a function of temperature is the same T^4 (monotonically increasing). In this case, surface(1) may be at a temperature less than surface(2) and still radiate energy at a rate greater than surface(2).
Second case, (a) the areas of surface(1) and surface(2) are the same, (b) surface(1) and surface(2) have different emissivities (i.e., different energy rate per unit area proportionality constants) where the emissivity of surface(1) is greater than the emissivity of surface(2), and (c) for both surfaces the rate energy is radiated per unit area is proportional to T^4 (monotonically increasing). In this case, surface(1) may be at a temperature less than surface(2) and still radiate energy at a rate greater than surface(2).
Third case, (a) the areas of surface(1) and surface(2) are the same, (b) the emissivity (energy rate per unit area proportionality constant) is the same for both surfaces, but (c) surface(1) radiates per unit area at a rate proportional to T^4 (monotonically increasing) whereas surface(2) radiates per unit area at a rate proportional to something less than T^4 (say T^3, which is also monotonically increasing). In this case, surface(1) may be at a temperature less than surface(2) and still radiate energy at a greater rate than surface(2).
By definition, we can assume surface(1) and surface(2) have the same area. Furthermore, we can assume that in a vacuum, both surface(1) and surface(2) have the same emissivity and a radiation rate per unit area dependence of T^4. However, it’s harder to argue that in the presence of an atmosphere the temperature dependence of the radiation rate per unit area will be T^4. As I understand it, the blackbody radiation dependence of T^4 applies to
“cavity radiation” emanating into a vacuum from a small hole in an enclosed volume whose inner walls are at a uniform temperature. The T^4 dependence does not apply to cavity radiation emanating into a gas or onto another surface. Similarly, the T^4 dependence does not apply to radiation from a surface in contact with either (a) another surface (e.g., radiation emanating from the outer surface of the smaller of two touching coincident spherical annuluses), or (b) a gas. Somewhere (I forget where) I’ve read or heard that the denser the gas, the greater the deviation from the T^4 rule. If true, there are two implications. First, to compare the radiation rate per unit area properties (and hence temperature) of a blackbody surface in a vacuum to the radiation rate per unit area properties of that blackbody surface surrounded by a gas requires careful treatment of radiation rate per unit area temperature dependence. Second, if in the presence of a gas the radiation rate per unit area temperature dependence is a function of the properties of the gas, then different combinations of gases can produce different surface temperatures. Thus, I believe an error that appears in almost all Earth temperature models is the blackbody (or graybody) radiation assumption of the Earth’s surface in the presence of an atmosphere.

Bart
January 15, 2012 1:51 pm

steven mosher says:
January 15, 2012 at 1:26 pm
Very interesting stuff.
Paul Dennis says:
January 15, 2012 at 1:29 pm
“I don’t understand your comment. Can you elucidate please? Thanks.”
The heat equation says that for equilibrium, the Laplacian must be zero. There are no solutions in spherical coordinates which are increasing with altitude. Hence, such a configuration is unstable.

Spiny Norman
January 15, 2012 1:53 pm

I will ask again … what are the properties of this proposed 100% transparent atmosphere? What is it made of? If its made of a known element, what is that element (or elements)? If its not, and its just a hypothetical substance, what are the properties of that substance that enable it to be 100% transparent (not just 99.99999…%, 100%). I would like to understand whether it has any known correlate in any possible world. If it doesn’t, what is it’s value as a hypothetical? Thanks.

Editor
January 15, 2012 1:53 pm

A maintenance elevator story FOR Willis’ QED
He already provided a simple enough argument (an express elevator story) but the following working-through of the history of a liquid planet dropped into a uniformly irradiated environment might help flesh it out a bit:
Assume the liquid is just like water, except that it does not freeze, and its gaseous form is not a GHG. Instead of water, call it fauxter. Assume that the incoming radiation levels are such that the SB equilibrium temperature of the planet Fauxter is below the boiling point of fauxter, and that when it pops into existence, Fauxter is entirely liquid and is colder than the SB equilibrium temperature.
When incoming radiation starts to strike Fauxter’s ocean, the ocean will begin to warm and some of the surface fauxter molecules will transition to vapor. Evaporation will cool the oceans as energy gets pumped into the atmosphere, but the overall effect will be warming. Both the oceans and the atmosphere will gain heat content, and the more the planet warms, the more readily the surface fauxter will transition to fauxter vapor, building the atmosphere.
In this initial phase, incoming radiation exceeds outgoing radiation. The difference is stored both in the rising heat content of the ocean and the atmosphere, and in the increased potential energy of the atmosphere as it gets lifted up through the planet’s gravity well.
Conduction should tend to bring the surface temperature of the ocean together with the near-surface atmospheric temperature. If they are brought fully together then the temperature above would lapse from the ocean surface temperature according to ideal gas law, decreasing with decreasing atmospheric pressure as altitude increases.
This seems to me to be the crux of the issue. The heat content of the atmosphere is all determined by the ocean surface, both through the warming of fauxter into fauxter vapor, and by heat conduction between ocean and atmosphere. If we assume no convection, then the temperature profile from the surface on up just follows the lapse rate, and it is the surface that determines the LEVEL of this profile. The temperature profile can be stepped up or stepped down but the level of the profile is driven from the bottom of the atmosphere, not the top.
This is why atmospheric pressure cannot warm the surface. The causality goes the other way, at least in this GHG-less-atmosphere example. It is surface heat that lifts the atmosphere in the first place and is responsible for the level of the temperature profile going up. That result of the surface temperature cannot in turn be the cause of the surface temperature. The push only goes one way. Atmospheric mass does determine the lapse rate, but not the level of the temperature profile.
Once the ocean surface temperature reaches the SB equilibrium temperature, the system does not gain or lose energy. Solar radiation will still pry fauxter vapor from the ocean, but an equal amount of fauxter should be phase transitioning back to liquid.
The upshot is that atmospheric pressure will not drive surface temperatures above the SB equilibrium surface temperature because the causality goes the other way. It is surface temperature, not atmospheric pressure, that determines the level of the atmospheric temperature profile.
You have now arrived at the Fawlty Towers penthouse. How’s the view?
(Again, for anyone who wants to see it, I put a link on my own blog to Han’s Jelbring’s 2003 paper, “The Greenhouse Effect as a function of atmospheric Mass,” so that Willis will not have to tolerate any link to Tallbloke’s Talkshop on “his” thread. Hopefully he will not again delete this link to my own blog. After all, I put it up at HIS suggestion. For someone who claims to be motivated by the need to fight censorship, Willis seems to have engaged in far more censorship than Tallbloke ever did. Still, very nice post Willis, and your marathon effort in responding to comments is much appreciated.)

January 15, 2012 1:55 pm

I just noticed the point of confusion about the gravitational effect.
It isn’t the process of compression that generates the heat.
It is the increase in atmospheric density at the surface caused by the gravitational compression that generates more collisional activity.That is what N & Z are referring to in their paper so any assertions that they are wrong on the basis of compression not being the cause are missing the point.

Bart
January 15, 2012 1:56 pm

Alan Wilkinson says:
January 15, 2012 at 1:36 pm
First, at higher altitudes the pressure is lower and there is less gas in each shell, therefore its total energy is diminished.
Pressure diminishes exponentially. Energy content would increase with the inverse square of the radius.
“Second, it is not unstable. An isothermal column is neutral to convection whereas a temperature gradient in either direction is not – being either stable or unstable according to direction.”
I think you are thinking of a flat geometry. Spherical distribution demands a particular direction for stability.
“However, as I noted above, the upper boundary of an isothermal atmospheric column is unphysical since it allows the gas to escape gravity.”
Good point.

Bart
January 15, 2012 2:02 pm

Alec Rawls says:
January 15, 2012 at 1:53 pm
“Once the ocean surface temperature reaches the SB equilibrium temperature…”
Will it do so? How do we know? What is the energy distribution? What is the spatial radiation distribution? Does it integrate to an equivalent SB temperature relationship for an ideal, perfectly smooth blackbody?
It is not a given. SB simply does not, in general, hold in such a situation.

Bart
January 15, 2012 2:08 pm

Bart says:
January 15, 2012 at 1:56 pm
Alan Wilkinson says:
January 15, 2012 at 1:36 pm
“Energy content would increase with the square of the radius.”
By which I mean, with constant pressure, but I have made the argument that the pressure and energy density cannot be made to match up due to the different rates of progression of pressure and elemental volume.

January 15, 2012 2:08 pm

“The upshot is that atmospheric pressure will not drive surface temperatures above the SB equilibrium surface temperature because the causality goes the other way. It is surface temperature, not atmospheric pressure, that determines the level of the atmospheric temperature profile.”
Do you then see a role for density of the atmosphere at the surface altering the rate of cooling of the surface and thus elevating the equilibrium temperature above S-B ?
Presumably the more fauxter that enters the atmosphere the greater the mass of the atmosphere, the greater the density at the surface due to gravitational compression and the higher the surface equilibrium temperature will become ?

Richard M
January 15, 2012 2:29 pm

steven mosher says
January 15, 2012 at 1:26 pm
some lecture notes for folks

Steve, are those the same calculations that go into the models? The problem is the models and reality don’t appear to match, which indicates something missing in the calculations. You can either believe the data or the theory?
For example, we have all these planets that don’t match these calculations, we have Miskolczi and his 230 observations that yield a constant optical depth. The point is the theory must be wrong when nature gives different results. Could it be the pot/lid hypothesis? I don’t know, I just try and follow the data and see where it leads me.

kbray in california
January 15, 2012 2:31 pm

So if I pump superheated nitrogen into a metal tank, the nitrogen will not release any heat to the metal walls via infra-red, but by physical contact alone?
Then the metal tank will be heated by the contact with the nitrogen and the tank will release heat to the environment via both infra-red and physical contact with the air surrounding the vessel.
I was not aware that some balls of glowing hot gas can not radiate heat via the infra-red.
It seems counter intuitive.
Is that correct?

beng
January 15, 2012 2:34 pm

****
Paul Dennis says:
January 15, 2012 at 12:55 pm
I think Willis’ model is correct, bar one aspect which is that of the dry adiabatic lapse rate. I think that in the model the atmosphere will rise to a uniform temperature which is that of the planetary surface. This atmosphere is unable to radiate energy because it is composed exclusively of non-GHG molecules. Thus at equilibrium all the energy is radiated from the planetary surface. There is no conduction of heat through the atmosphere since the surface and atmospheric column are all at the same temperature.
****
The earth’s dry lapse rate is caused by GHGs. Dry may mean w/o condensation (adiabatic), but the water-vapor, CO2, etc, are still present. The reason the upper atmosphere cools is because it can radiate w/GHGs, and is exposed to the cold of space. Air near the surface radiates too, but is insulated (and radiated back at) top & bottom by adjacent emitting/absorbing layers. A non-GHG atmosphere, as you stated, has an insignificant lapse-rate & eventually represents the avg surface temp thruout its entire column — it couldn’t be otherwise since non-GHG means it can only radiate heat away at insignificant levels.
Is the non-GHG world “warmer” at the surface than an airless world? It’s kinda apples & oranges, but the air temp right above the surface in the day would be hot indeed from conduction. There might be some wind & shallow mixing. But that air would also cool down at nite (from conduction) to near the bitter-cold surface temp & prb’ly form a strong but very shallow temp inversion. Bottom line is, the non-GHG world would be a cold place averaging well below freezing at earth’s orbit. Doesn’t matter what the surface pressure was — one bar or one-hundred.
I doubt there’s a “real” non-GHG planet out there — CO2, water vapor, methane, NO2, etc, seem to be common atmospheric components at least in our solar system.

January 15, 2012 2:47 pm

Willis & Robert Brown,
I think that the three of us agree that the emissivity of gaseous argon is not strictly zero, and we also agree that it is so small that treating it as zero can make no conceivable difference to your argument. Those (such as Anna V and Spiny Norman) who think that negligible emissivity will change your argument are naive.
Thepompousgit is of course right that in practice the effective emissivity of any real sample of argon would be dominated by various imperfections, such as the container (on earth) or passing dust (in space). But again this doesn’t change the argument.

Reed Coray
January 15, 2012 2:52 pm

Oops. In my haste to add my comment, I’m pretty sure I made a mistake. Specifically, in Ferd Berple’s logic, I forgot to include (a) conduciton between the surface and the atmosphere, and (b) more importantly the fact that radiation from the surface can go to both the atmosphere and space. So what I wrote above is likely to be partially correct or flat out wrong. I want to think some more.

blueice2hotsea
January 15, 2012 2:57 pm

Hi Willis –
Not an attempt at refutation, rather clarification.
Anna v and others have politely hinted that non-GHG’s are sources of radiative emission at wavelengths other than IR. Whether or not this is a significant source of cooling in your hypothetical world, it is still important to not confuse people.
From my own non-expert understanding of QED, there are infinite paths to electronic excitation. To me, that means nitrogen could obtain kinetic energy via surface collision and then acquire the additional (precise) energy (from whatever means) to undergo electronic excitation – a condition ripe for photon emission to space – and subsequent net surface cooling.
bi2hs

Joel Shore
January 15, 2012 2:59 pm

Robert Brown says:

This seems correct to me, although somebody (Tim F.?) asserted otherwise. Indeed, I suspect that the Earth is largely insensitive to changes in GHG concentrations, and might even operate the opposite way than expected in some cases. Once you are “opaque” you are opaque, and making it twice as opaque doesn’t really happen. I am actually curious as to whether anybody knows anything concrete about this, as it has bothered me for some time. I’d expect alterations in the outgoing radiation profile due to doubling CO_2 concentration to be, well, almost impossible to detect, as radiation of IR from the troposphere is going to still be radiation of IR from the troposphere. Is the troposphere going to move? Will the adiabatic lapse rate change? Why, exactly, is radiative balance going to change?

.
Robert: The issue is not a binary one of “Does the radiation from the surface get absorbed at all from the surface or does it not?” Rather, what you want to imagine is photons likely getting absorbed and subsequently emitted several times before the energy escapes the atmosphere. (In essence, a sort of random walk.) And, the question becomes at what altitude in the atmosphere the photon has a good probability of being able to successfully escape to space. It is at this “effective radiating level” where the average temperature of the atmosphere has to be ~255 K. Currently, that level is at ~5 km and hence the temperature at the Earth’s surface is obtained by extrapolating down from that level to the surface using the average tropospheric lapse rate, which is about 6.5 K per km: 255 K + (6.5 K / km)*(5 km) = 287.5 K.
What happens as you add greenhouse gases is that the effective radiating level increases while, to first order, the lapse rate (which is forced by convection to remain around the appropriate adiabatic lapse rate) does not change. So, as an example, let’s say you add enough greenhouse gases to increase the level to 6 km. In that case, what happens is that you have a radiative imbalance because the temperature at 6 km is colder than 255 K and hence the Earth + atmosphere is emitting less energy than is being absorbed from the sun. As a result, the Earth + atmosphere will warm until the new effective radiating level at 6 km is 255 K. And, the new temperature at the surface will now be 255 K + (6.5 K / km)*(6 km) = 294 K.
This is, of course, a simplified picture in many ways including:
(1) The level at which the radiation can successfully escape to space is in fact a strong function of frequency, so the “effective radiating level” is just an average value. To do quantitative calculations, you really need to use a line-by-line radiation transfer code.
(2) The lapse rate actually is expected to change a bit with warming: In the tropics, the appropriate lapse rate is generally the moist adiabatic lapse rate and this is a decreasing function of surface temperature. This effect is called the “lapse rate feedback” and it is a negative feedback that is included in all of the climate models. There are variations in the strength of this feedback from model to model; however, fortunately, because the physics involved is closely related to the physics of the water vapor feedback, models that have a stronger (negative) lapse rate feedback also have a stronger (positive) water vapor feedback and, hence, it turns out that the variability of the total of these two feedbacks from model to model is much smaller.
Nonetheless, to first order the picture you want to have in your mind is that the greenhouse gas concentration controls the level in the atmosphere at which the temperature has to be 255 K…and, in particular, that this level rises as the concentrations increase. The average temperature at the surface is then obtained by simply extrapolating using the average environmental lapse rate of ~6.5 K per km.

Paul Bahlin
January 15, 2012 3:06 pm

Perhaps it would be helpful to invent an even simpler model to develop some thinking.
What if you had a black body sphere, per Willis’s description, evenly exposed to radiation from every direction. It is infinitesimally thin but surrounded by some magic insulating material that is transparent to all radiation wavelengths, a solid. This magic insulation provides us with a black body surface that has radius B. The magic stuff has radius M.
By conduction, the magic stuff equilibrates to some constant ‘lapse rate’, really a temperature gradient that goes from Tb K at the surface, at distance B from the center to 3K at distance M from the center. The only transport mechanism is conduction so it seems that there must be a gradient like you would get from any insulation. Magic stuff has some energy content that implies, since no new energies are involved, that the black body has given up some of its energy to the magic stuff but the overall energy content, when equilibrated, is exactly the same.
I only pose this because, for me, it brings up the intriguing possibility that the incoming radius is impinging on a sphere of radius B but the outgoing energy might just be coming from some sphere of radius between B and M, we’ll call it X, since the volume of the contained energy available for exit is larger than the volume that was available for incoming.
If this is true then the outgoing flux value measured at X has to be less than the incoming flux value at B since there is more area to transit the same energy. Could this provide for a change, up or down on surface temperature?

Leonard Weinstein
January 15, 2012 3:09 pm

Willis,
I appreciate the great length of the replies and how some of the misunderstandings can make you feel crazy. You are doing a great job overall, even though I disagree on some individual points.
I have tried to respond to make you understand where N&Z and Jelbring went wrong. Making an elevator speech for N&Z and Jelbring is not necessary. Both make the erroneous assumption that somehow the adiabatic lapse rate determines a temperature. It does not. It only determines a gradient, and even that requires enough mixing to prevent conduction from reducing the lapse rate, and in the limit, approaching to near isothermal. The greenhouse gas (and or aerosols) are needed to raise the location of average outgoing radiation leaving to space (to match the level of absorbed solar radiation), so that a fixed absolute level of temperature is determined at that effective altitude, and the adiabatic lapse rate does the rest.
However, your elevator speech also fails when you state it is absorbed energy, radiated by the atmosphere back to the surface, that warms the surface in the presence of a greenhouse gas. In order to warm the surface, you have to have net heat transfer. For a warmer surface than atmosphere, the heat transfer will be from surface up. This is true if it is radiation or conduction combined with convection. It is true that energy can be transmitted from the atmosphere to the ground by radiation, but more energy is radiated up than down, and it is the NET energy transfer that is the heat transfer. The back radiation is not the cause of the ground heating above the non-greenhouse case, it is a result of the ground being warmer than otherwise as stated above, and the presence of a partial insulation of radiation.
This is in some ways similar to the way an insulation layer on an electrical heater of constant power slows heat transfer out, resulting in a hotter heater to get the power out. Back conduction is not heating the heater surface, it is only slowing the escape rate at the previous temperature. It is not energy going into the insulator and then partially conducting back to the surface, but it is causing the surface to accumulate extra temperature due to energy not being removed as fast, and the increase eventually raises surface temperature so the energy is removed fast enough. However, the atmosphere is different in other ways, as convection can still carry power out. That is where the lapse rate and set temperature at a particular altitude come in. The surface accumulates extra solar heat energy and convection and some radiation transfer carry the excess heat upwards until the adiabatic lapse rate is reestablished at the new level. This then determines the ground temperature as long as the lapse rate is the adiabatic (wet) lapse rate.

January 15, 2012 3:13 pm

Stark Dickflüssig said @ January 15, 2012 at 1:13 pm

LO! thepompousgit maketh a great clamouring, as the tinkling of brass upon January 15, 2012 at 12:51 pm
Stark Dickflüssig said @ January 15, 2012 at 12:24 pm
All objects are required to emit photons until they are at absolute zero.
And where in Resnick & Halliday (or other physics text) will I find this quote, or its equivalent?
Are you saying it’s untrue, or are you just too lazy to do a text search on the single largest collection of human knowledge in all of history?
http://lmgtfy.com/?q=objects+emit+photons+absolute+zero

Nope, I can’t see the word “untrue” in my response. I asked for a reference. Quantum events, such as the emission of photons, are probabilistic and that implies a gas molecule has some probability of absorbing/emitting outside of its characteristic emission lines, that probability is vanishingly small. In the limit (an infinite number of seconds away) then all molecules will have emitted all available photons. I see no mention of this in the first link you provided, merely the bald, unsupported statement.
I’m not sure what this has to do with Willis’s thought experiment.

Eric Barnes
January 15, 2012 3:19 pm

Willis Eschenbach says:
January 15, 2012 at 12:29 pm
Once again, you claim to understand Huffman.
EB> I think I’ve explained it better than you have explained the GE. Your elevator speech
EB> explains nothing. It’s a hypothesis without an experiment, or better yet, a hypothesis
EB> with a counterexample in the data of earth and venus as explained by Huffman.
Once again, you fail to explain it in a clear, concise manner as requested.
EB> You are being purposefully obtuse IMO. I won’t speculate as to why.
Once again, I won’t believe you understand it until you can explain it.
EB> The only thing Huffman (and I) have noticed is the correlation. You are discarding that
EB> data in favor of your speculation about GE theory. Once you or I get a hold of a reasonable
EB> sized planet that we can experiment with at our leisure we can settle our disagreement.
EB> Until then I’ll respectfully disagree with you.

Joel Shore
January 15, 2012 3:29 pm

Willis says:

The earth’s dry lapse rate is caused by GHGs.

It is NOT caused by GHGs, it exists whether the gases in question are GHGs or not. The dry adiabatic lapse rate is given by g / Cp, which is gravity divided by specific heat. There is NOTHING IN THERE about GHGs.

It is a little more confusing than that because the adiabatic lapse rate represents a stability limit on the actual lapse rate…That is, lapse rates greater than this are unstable to convection whereas lapse rates less than this are not. An atmosphere with no GHGs will not be able to cool significantly through radiative emission…and hence some, such as Roy Spencer for example, have argued that it will end up with an isothermal profile or a lapse rate less than the adiabatic one. (I have no strong opinion either way on this.)
However, your point is basically correct for our Earth that does have some GHGs: The troposphere is strongly warmed from below (by both solar radiation absorbed at the surface and GHG absorption) and cooled from above. As a result, the lapse rate would, in the absence of convection, exceed the adiabatic lapse rate and this sparks convection which reduces the lapse rate down to the appropriate adiabatic lapse rate. And, as I noted, to the first approximation, this average lapse rate does not change with the addition of GHGs (although to a better approximation, it is expected to decrease slightly).

January 15, 2012 3:56 pm

Willis simplified steady state model shows that the force of gravity on a non-“greenhouse” atmosphere has no effect on surface temperature and with no convection there is very little energy transfer by other than radiation. Think about the perfect gas law. (PV)/(nT)=constant. Pressure (P) is the result of the gravitational attraction between the mass of the planet and the mass of n molecules of gas. Now take two planets with the same mass but with different number of molecules of gas. P/n will be the same for both so there should be no change in T. Now what should we expect when we increase the input radiation. The surface temperature will increase and conduction will transfer some of that energy (not much) to the atmosphere. That slight increase in atmospheric temperature will increase the volume but not the pressure because there has been no change in n. The surface will soon radiate as much energy as it receives.

steveta_uk
January 15, 2012 4:04 pm

Willis, I’d like to congratulate you on providing, possibly unintentionally, one of the finest BS filter posts we’ve seen on this site.
In particular, there are a number of individuals who I previously read with interest who have exposed their shallow understanding of various physical mechanisms. I shall in future read their output with a deal more caution.
And thanks in particular to the interjections of some real experts.
And a final note to those who believe all matter must radiate until it reaches absolute zero. Considier a single N2 molecule out in space. Is it radiating? For that matter, does it have a temperature? If so, in which frame of reference is this temperature defined? Does it appear “hot” to one observer, and therefore appear to be radiating, while appearing “cold” to another observer in a difference frame of reference?

steveta_uk
January 15, 2012 4:10 pm

PS. Clearly my previous post is off topic, so I expect to be snipped. And deservedly so!

Bill Hunter
January 15, 2012 4:22 pm

Willis says:
“The atmospheric lapse rate is a result of the kinetic/potential tradeoff for all molecules. It does not require bulk convective motion for the lapse rate to exist. As Tim says, at equilibrium the atmosphere will be isentropic, rather than isothermal.”
Perhaps through most of the atmosphere.
But if the atmosphere is not losing energy to space via radiation it will be warmer than the average temperature.
The daily pulses of energy would push the system warmer because so little would be conducting down to the surface to be radiated away. Common air is one of the best insulators around.
Also anything that does conduct down would be largely replaced by convection within a few hours.
This would work exactly like that well insulated passive solar water heater system where the storage is placed higher than the collector and the water gets far warmer than average day where it is placed does. In fact it often gets warmer than the day ever gets because of no loss due to convection it gets as warm as you can make a greenhouse get (which is a lot warmer than most greenhouses because hardly anybody goes beyond dual glazing in greenhouses)
The atmosphere at the bottom at night you would have this tight little temperature gradient inversion layer. It would likely be measured in inches due to the lack of weather and non-existent convection.
We currently see these inversion layers at night where the surface cools faster than the atmosphere. When the sun comes up convection dominates because of the need to replenish heat lost by the atmosphere due to radiation which make up virtually all of the atmosphere cooling. At that time the gradient is the least steep and evaporation kicks to lower the gradient as well.
It seems clear that’s the case and the only question is a number for it as its all been sucked up into universal CO2 theories. If its the entire number then perhaps Jelbring is right because it would be consistent with Robert Brown’s comment: “Can gravity ever act as an energy source that releases heat in a gas? Sure. It is the primary source of heat in stars as they form, right up to where they “ignite””.
Gravity would not need to be a continuing source of heat as this heat would never be lost. It would just be there. Its also interesting how Jelbring has correlated this with mulitple planets based upon the masses of their atmospheres. Kind of reminds me of how climate science pooh poohed the mid 20th century warming and ocean oscillations until they could no longer hide from them. Akasofu’s and Easterbrook’s charts of the direction of the climate system made a lot more sense too in how they matched temperature oscillations in time with the PDO.
So I would like my question answered about what the surface actually is radiating rather than depending upon calculations dependent upon proxies to tell me. Seems most of the controversy here is in the idea that surface radiation is cranking all that gravity induced heat to space and gravity then has to replace it. I think not! I tend to think the theory would have to fail if it had to do that. And a reread of Jelbring has led me to believe he never claimed that.

c0h0nes
January 15, 2012 4:33 pm

The atmosphere would be in motion and not isothermal for the following reasons:
I don’t know what the topology of this imagined planet with a transparent atmosphere is, but I would have to assume eventually a non-smooth surface at some arbitrary scale. With planetary rotation and differences between polar and equatorial insolation, any atmospheric state would be unstable, and particulates would always be present in the atmosphere, if from nothing else, bombardment from other cosmic bodies. The largest source of particulates eventually becomes the planet surface. Thus the atmosphere would contain particulates and thus not remain uniformly transparent, and would be heated by the same process as the surface, and the suspended atmospheric particulates would radiate back to the surface, raising temperatures above the S-B temperature. And the atmosphere would be turbulent.
In effect the surface area of the planet is enlarged by suspending some of it in a third dimension above the planet. Because the incoming radiation is of a different, higher temperature blackbody spectrum from the reradiated IR, the opportunity for scattered radiation to be absorbed on a second or subsequent collision with either another particle or the surface is magnified, and unlike water vapour clouds, the albedo of dust, especially from extraterrestrial origin is low. Thus the likely equilibrium temperature of an optically and IR transparent atmosphere would be higher than a water dominated atmosphere such as found on Earth.
So even without a GHG, a “transparent” atmosphere would have the same effect, at some arbitrary particulate level.
Thus the elevator explanation is the same, except that “particulates” (I like “dust”!) replaces “GHGs” (or synonyms).

jjthom
January 15, 2012 4:38 pm

Absorption spectra of gases discussed in detail:
http://www.patarnott.com/atms749/pdf/GrantPettyDeathPhotons.pdf
If it doesn’t absorb it doesn’t radiate

Reed Coray
January 15, 2012 4:42 pm

I’m pretty sure I made a mistake analyzing Ferd Berple’s comment. That won’t, however, stop me rom trying to analyze Willis’s “Elevator Speech.” Willis said:
The poorly-named “greenhouse effect” works as follows:
• The surface of the earth emits energy in the form of thermal longwave radiation.
• Some of that energy is absorbed by greenhouse gases (GHGs) in the atmosphere.
• In turn, some of that absorbed energy is radiated by the atmosphere back to the surface.
• As a result of absorbing that energy from the atmosphere, the surface is warmer than it would be in the absence of the GHGs.

Although not explicitly stated, I infer from the third and fourth bullets that “the energy radiated by the atmosphere back to the surface” is the source of the increase in surface temperature. My question then is: “What stops the process–i.e., what stops the surface temperature from increasing without bound?”
For example, consider the surface temperature in the presence of an atmosphere that is greenhouse gas free. I assume the surface temperature will eventually stabilize at some temperature T0 > 0. Now add a greenhouse gas to that atmosphere. According to Willis’s elevator speech, (a) the surface will radiate energy (IR and non-IR), (b) some of the surface IR radiation will be absorbed by the greenhouse gases in the atmosphere, (c) the greenhouse gases will radiate some of their absorbed energy, (d) a portion (backradiation) of that “re-radiated” IR energy will reach the surface, and (e) the backradiation will increase the surface temperature.
OK, so far so good. For a moment, let’s assume that this produces a steady-state surface temperature T1 > T0. Applying this assumption, we now have a surface temperature of T1. Since T1 > T0 > 0, the surface will radiate energy (both IR and non-IR). As before, some of the Isurface radiated IR energy will be absorbed by the greenhouse gases in the atmosphere and a portion of that absorbed energy will be “backradiated” to the surface. If as occured when we first introduced greenhouse gases into the atmosphere, “greenhouse gas backradiation” raises the surface temperature from T0 to T1, why won’t greenhouse gas backradiation raise the surface from temperature T1 to T2? And once a surface temperature of T2 is reached, why won’t the process repeat raising the surface temperature to T2; to T3, etc?
Although there may be many ways to limit the surface temperature, I can only see one way. Specifically, if the amount of IR surface radiation eventually decreases to zero there will be no greenhouse absorption of surface radiated IR energy and no IR backradiation of IR energy radiated by the surface. This will occur if the surface temperature approaches infinity because as the surface temperature approaches infinity, the amount of blackbody radiation in the IR band decreases.
As long as IR backradiation increases the surface temperature, I don’t see how “greenhouse gas saturation” (i.e., the state where the greenhouse gases that exist in the atmosphere are absorbing all the outgoing radiation they can absorb) will stop the temperature increase. This is because backradiation doesn’t stop when greenhouse gases become saturated–i.e., some IR backradiation will exist for both saturated and unsaturated greenhouse gases. And we have assumed that backradiation increases the surface temperature If someone claims that backradiation does not necessarily raise the surface temperature, then (a) I either want to know under what conditions backradiation stops increasing the surface temperature, or (b) how can it be argued (by Willis or anyone) that backradiation for Earth-like greenhouse gas levels will raise the Earth surface temperature.

Surfer Dave
January 15, 2012 4:49 pm

I think that you have forgotten about things like frequency conversion (where something receives energy at one wavelength and reradiates at a different wave length which is what the earth does) and there is no mention of convection and the evaporation of the atmosphere into free space as a result of convective heat transfer. This boiling of the atmosphere and the Ideal Gas Law explain how a transparent atmosphere in a gravity field still provides a layer of insulation and also transports a part of the energy received through the kinetic motion of the molecules of the transparent gas boiling into space, changing the temperate gradient across the span from far space to the core of the planet so that the 0K point is moved to the top of the atmosphere. Think about it, by adding the atmosphere you have changed the diameter and density of the object under consideration. You need to think about the gradient from free space to the core of the planet, not just the thin film of the solid to air/space boundary. Think also of the boundary or extreme cases say for example a massive or extremely dense planet with a huge gravity well and therefore a very dense atmosphere.
Is the case of a pefectly transparent gas possible? I thought all elements and molecules absord/emit at some point in the spectrum and there can be no zero radiation from a gas except at 0K? The SB equation returns zero only for 0K.
What is the temperature of your array of suns? That means, what is the power spectrum density from those sources? Our sun radiates most in the ultra violet because of a surface temperature of 5000+C. That UV heats the surface of earth which radiates most in the IR because of a temp of around 300K. Both bodies radiate across the entire spectrum with different PSD so the spectrum of the incoming radiation is different to the spectrum of the outgoing.
Also, what is the geothermal heat flux in your thought experiment and for a real planet like earth, how does that flux vary over time, constantly in one direction or randomly as a result of cosmological factors? Don’t forget there are spots with measured geothermal heat flux up to 15Wm-2 in the USA.
And just to be provocative, why are you assuming the energy received at the surface is transmitted outward only? Surely energy is radiated and conducted inward as well? Over an infinite period of time, would not the core of the planet reach an infinite temperature in your simple model?

Surfer Dave
January 15, 2012 4:51 pm

Sorry, my bad, that last question should read: Over an infinite period of time the planet would not the core of the planet reach the temperature of your array of suns?

jjthom
January 15, 2012 4:59 pm

A brilliant site for the physics:
http://www.patarnott.com/atms749/
The presentations are clear – from First Course in Atmospheric Radiation: Second Edition, by Grant W. Petty.

Bart
January 15, 2012 5:02 pm

Willis Eschenbach says:
January 15, 2012 at 3:08 pm
“Not if they are at the same temperature there isn’t. And at equilibrium, there is no temperature differential across the surface.”
You can have a temperature equilibrium at the surface/atmospheric interface without being in thermodynamic equilibrium. There is no reason the temperature equilibrium is the same as it would be if you took the atmosphere away. In fact, I can assure you, it is not the same. The magnitude of the difference is the only valid point of argument.
We do not know that magnitude. And, we are not going to pin it down through “thought experiments” because the subject is, frankly, too complex for back-of-the-envelope calculations. We need data from experiments which duplicate the exact physical state of the system under study. As far as I can tell from my, admittedly limited, searches, there is no such data available.

u.k.(us)
January 15, 2012 5:12 pm

Argumentum ad nauseam
A giant thread, which in the end only produced enemies.
WUWT.

gbaikie
January 15, 2012 5:13 pm

“I don’t know if this answers your questions but I’m satisfied that Willis’ reductio ad absurbum approach provides a proof that gravitational effects cannot raise the temperature of the planetary surface.”
It proves that 240 watts of solar flux times 4 does not equal 960 watts per square meter.
240 watts of solar flux times a million also does not equal 960 watts per square meter.
Million sources of 240 watts can not cause temperature higher than -18 C.
The Moon [no greenhouse gases] has temperatures much higher than -18 C
Mars [which does have greenhouse gas, be recieves far less sunlight than the Moon or earth] also is heated during the to higher than -18 C and receives less than 1/2 the solar flux as the earth or the Moon gets.
Some may claim that the earth receives an average solar flux of 240 watts, but you can’t average
the solar flux and expect it to give correct answers.
The only thing averaging the energy from the sun, does is tell you how much something with 4 times the surface area can radiate.

Bart
January 15, 2012 5:16 pm

steveta_uk says:
January 15, 2012 at 4:04 pm
“And a final note to those who believe all matter must radiate until it reaches absolute zero. . Considier a single N2 molecule out in space. Is it radiating?”
In cold space, it will radiate until it reaches its ground state, which could take infinite time to reach. A macroscopic quantity of matter collectively in its ground state is, by definition, at absolute zero.
“For that matter, does it have a temperature?”
No. Temperature is a macroscopic measure whose definition breaks down at that level.

gbaikie
January 15, 2012 5:17 pm

“The only thing averaging the energy from the sun, does is tell you how much something with 4 times the surface area can radiate.”
And btw, the planet with million sun which provide 240 Watts per square meter, would be radiating
a million times more energy than earth.

gbaikie
January 15, 2012 5:33 pm

“Both make the erroneous assumption that somehow the adiabatic lapse rate determines a temperature. It does not.”
It does. Temperature meaning air temperature. Air temperature is always discussed as earth temperature. So corrected, adiabatic lapse rate determines air temperature.
The only with saying that is it’s too darn obvious.
Thought experiment:
Remove all oceans from earth.
Perhaps a less obvious thing other than the absence of water,
is you create a world with high plateau [the land masses]. Most the planet if removed
the ocean would have 3 km deeper atmosphere.
At the bottom of these once exist ocean floors, the air temperature would higher- due to the
adiabatic lapse rate.

January 15, 2012 5:44 pm

To summarise the story so far:
1) Willis accepts that his non GHG atmosphere will produce a dry adiabatic lapse rate with the warmest temperatures at the surface.
2) The warmth at the surface is NOT due to gravitational compression but gravity is responsible for placing the maximum density of non GHG molecules at trhe base of the atmospheric column.
3) That maximimum density causes the greatest number of molecular collisions to occur just above the surface.
4) Such collisions transfer energy by conduction and not radiation. Their effect is to convert incoming solar shortwave radiative energy into kinetic energy and to retain it longer until it can be released upward as outgoing longwave IR.
5) It is that delay that increases the surface temperature.
6) That delay is caused by gravity concentrating the maximum density of atmospheric molecules at the surface and the greater the density, the longer the delay and the higher the equilibrium temperature must become.

Bart
January 15, 2012 5:50 pm

Surfer Dave says:
January 15, 2012 at 4:51 pm
“Over an infinite period of time the planet would not the core of the planet reach the temperature of your array of suns?”
Since the equations of thermal equilibrium are the same as for the gravitational potential, I would guess that the temperature profile would settle out such that it decreased proportional to radius squared.

OzWizard
January 15, 2012 5:54 pm

Since Tallbloke first published the “Unified Theory of Climate” by N & Z, and reminded everyone about Hans Jellbring’s hypothesis, I have watched the melting of the inimitable Willis Eschenbach’s mind in fascination. I feel I must attempt to put out the fire in his head before it consumes him completely. I have enjoyed most of his earlier perspicacious writings and do not want to lose him.
I believe Willis’ approach to this theory is revealing: (a) it makes his head hurt; (b) he wants someone else to explain where his thinking is wrong.
Well, I would hate to see him self-destruct due “excess heat” build-up in his head so I offer the following (including my “elevator speech”) as an antidote to his dilemma. Be honest with yourself, Willis. The fact this alternative theory makes your head hurt is a sign of “impending change”; your paradigm is being altered and that is making you uncomfortable.
The biggest trouble with thought experiments, Willis, is that if you are not extremely careful, you can end up believing that things like M C Escher’s impossible “ascending-descending” stairway is physically possible. The main problem is that there is no “reality check” built into a thought experiment. That is why Einstein’s relativity theory led to the TWIN PARADOX.
You should be able to agree that paradoxes do not exist in REALITY, but only in the mind, generally as a result of false premises in the logical process which generates them as a part of their “logical conclusion”.
For me, the Greenhouse Gas Theory is in the same class as the Escher stairway (or the Twin Paradox of Relativity, or the Wave-Particle Duality of Planck and others). If you believe any one of these things is possible, and are prepared to defend it as “a reality”, you have already “lost the plot”. You have effectively surrendered your intellect to a smooth-talking con-man.
The false premise in the ‘GG theory’ is the postulated existence of the effect of ‘back radiation’ from a cold atmospheric trace gas, namely, that such a ‘cool’ gas can cause a ‘warmer’ surface to be raised to a higher temperature than it would otherwise be, without the presence of the ‘cool’ gas. If this does not seem to you to be the quintessential recipe for a perpetual motion device, then you have lost the essential critical faculty which defines a scientific mind.
As for the Atmospheric Temperature Enhancement (ATE; NTE) postulated by N & Z, here is my “elevator speech”:
·1. The existence of a dimensionless Thermal Expansion Coefficient of steel does not imply that “gravity cause steel to expand”.
·2. Likewise, the existence of a dimensionless ATE ‘factor’ does not imply that “gravity causes heating of the lower atmosphere”, in defiance of the 2nd Law of Thermodynamics.
·3. In both cases, the dimensionless ratio in question enables us to calculate easily what the effect of “heat input” will be on, in the first case, a bar of steel and, in the second case, a planetary atmosphere subjected to gravitational compression.
I hope that did not strain your attention span, Willis. From that point on, you should be able follow the logic. I’ll leave the pleasure of that process of discovery for you to enjoy at your own pace.

Bill Illis
January 15, 2012 6:16 pm

If the blackbody object had an atmosphere of the non-GHG hydrogen 695,000 kms thick, the blackbody object would heat up to about 10 million Kelvin, then hydrogen fusion would start and the blackbody object would then heat up to about 15 million Kelvin.

Bob Fernley-Jones
January 15, 2012 6:28 pm

Willis,
I’m a tad timorous to ask, but:
1) Didn’t you agree elsewhere that the moon does not exhibit the expected S-B behavior based on the so-called average or effective radiative temperature, spread over 4Pi*r^2? One reason for this is that there is an extreme hotspot directly under the sun which loses heat proportionally to the fourth power of T. (= very much more than elsewhere). In an Earth with transparent atmosphere the insolation in the solar hotspot would be practically identical, although some of the resultant heat would be conducted/convected/advected away. (BTW, even at hurricane wind-speeds, this would be a relatively slow process). Paraphrasing, I think you have claimed that it is impossible for the surface of a planet to radiate more than that of the theoretical value of the black body equivalent of the insolation.
Where that simplistic statement fails is that temporally and spatially, there is no sensible temperature on which to base your assertion, because of that naughty T^4. (and some other things)
~~~~~~~~~~~~~~~~~~~~~~~~~~~~~~~~~~~~~~~~~~~~~~~
Willis, if you are kind and sympathetic towards me on point 1), I may contemplate submitting point 2) and beyond, that you seem to have overlooked.

Bob Fernley-Jones
January 15, 2012 6:35 pm

Whoops,
Further my post just above, where I wrote:
Paraphrasing, I think you have claimed that it is impossible for the surface of a planet to radiate more than that of the theoretical value of the black body equivalent of the insolation.
I should have added, according to standard climate science: unless there are GHG’s present

KevinK
January 15, 2012 6:55 pm

Willis wrote;
“Let me give you an example to show what I mean. Here’s an elevator speech about the greenhouse effect:
The poorly-named “greenhouse effect” works as follows:
• The surface of the earth emits energy in the form of thermal longwave radiation.
• Some of that energy is absorbed by greenhouse gases (GHGs) in the atmosphere.
• In turn, some of that absorbed energy is radiated by the atmosphere back to the surface.
• As a result of absorbing that energy from the atmosphere, the surface is warmer than it would be in the absence of the GHGs.
OK, that’s my elevator speech about why the Earth is not a block of ice. Note that it is not just saying what is happening. It is saying how it is happening as well.”
Ok, here is your elevator speech with appropriate corrections;
Let me give you an example to show what I mean. Here’s an elevator speech about the greenhouse effect:
The poorly-named “greenhouse effect” works as follows:
• The surface of the earth emits energy in the form of thermal longwave radiation. AND THEN IT COOLS BY AN AMOUNT THAT CORRESPONDS TO THE ENERGY RADIATED (REFERENCE: FIRST LAW)
• Some of that energy is absorbed by greenhouse gases (GHGs) in the atmosphere. YES INDEED IT IS.
• In turn, some of that absorbed energy is radiated by the atmosphere back to the surface. AND THEN THE ATMOSPHERE COOLS BY AN AMOUNT THAT CORRESPONDS TO THE ENERGY RADIATED (REFERENCE: FIRST LAW)
• As a result of absorbing that energy from the atmosphere, the surface is warmer than it would be in the absence of the GHGs. NO NO NO NO…… the surface is simply warmed (and then cools) a second, third, fourth, etc. time as the energy makes a few small (and very speedy, i.e. at the speed of light) side trips through the system.
I’m sorry good sir, but your elevator ran off the tracks at your step #1.
The “GHE” only delays the flow of energy through the Sun/Earth/Atmospheres/Universe system by at most 10 to 100 milliseconds. Since this is far less than the period of One day (~86 million milliseconds) no “Extra Energy” is left over at the end of each day to cause a “Higher Equilibrium” to exist. OK, that’s my elevator speech.
Just a couple of other observations;
The S-B law (and it closely related relatives) are reasonably good at predicting the amplitude and/or spectral content of the radiation from a surface, but it is THEORETICAL, no real object yet observed agrees completely with the theory.
HOWEVER, and this is a REALLY BIG HOWEVER, it (the S-B law) says NOTHING about the temperature that a volume will reach when it absorbs radiation that happens to closely follow the S-B law. There are far more factors involved, like the thermal capacity and thermal diffusivity of the absorbing materials. And of course the volume of the respective materials MUST be considered. The hypothesis that the miniscule thermal capacities of the alleged GHGs are in fact “forcing” the MASSIVE thermal capacities of the Oceans present on the Earth into thermal equilibrium with said gases will be seen as LAUGHABLE in a few more years’ time.
Other than that, your post is quite interesting.
Cheers, Kevin.

Dr Burns
January 15, 2012 6:58 pm

Willis is correct for the hypothetical situation he describes. An IR transparent atmosphere cannot influence the surface temperature. However I feel this is misleading. In the real world, only 30% (on average) of the Earth’s surface is open to the sky. 70% is cloud cover. In this situation, the temperature of the radiating surfaces of clouds becomes important and this is influenced by the lapse rate of an IR transparent atmosphere. Cloud cover also reduces the effect of GHGs.
I’d also like to contratulate Willis for the great effort he has put into responses.

Ian H
January 15, 2012 7:01 pm

The issue lies with the exact definition of GHG and consequently the definition of what a non-GHG atmosphere would be like. You seem to be asserting that a non-GHG atmosphere cannot participate in any radiative processes – that the gases must be totally transparent to all frequencies. Under that definition your argument is sound. The energy budget of the entire planet must balance and with those restrictions the only way energy can enter/leave THE PLANET (many people are missing that unsubtle point) is by radiation at the surface which determines the temperature there. QED.
However where I must take issue with you is that I believe your definition of GHG to be too restrictive, non-standard and in fact incorrect. To you a GHG is simply any gas which is not completely transparent. So all gases are GHGs to some extent under your definition. I’d say that GHGs are gases which are transparent at the frequencies emitted by the sun but absorb the frequencies emitted by the planet. So energy has an easier time getting in than out. This aspect of the greenhouse effect was completely missing from your elevator description. If I were riding in that elevator with you I’d say “but surely the GHGs also absorb the sun’s energy coming in, cooling the planet”, and I’d claim to have a back of the envelope calculation showing that if all frequencies are absorbed equivalently, the two effects would balance. And I think you’d respond to that comment by modifying your description along the lines indicated above – that the frequencies emitted by the planet are absorbed more strongly than those emitted by the sun. However as a consequence you would no longer be able to assert that non-GHGs must be perfectly transparent.
To me a GHG is a gas which absorbs longwave radiation more strongly than shortwave radiation. A planet with a non-GHG atmosphere is therefore not one with a completely transparent atmosphere, but rather one where the atmosphere is equally transparent at all frequencies. So the atmosphere can indeed absorb and radiate, but must do so at all frequencies equally.
Under my definition even a perfectly opaque gas would be a non-GHG since it treats all frequencies equally (it absorbs them all). Let’s consider that extreme example for a minute because it is instructive. Under the conditions of a perfectly opaque atmosphere, the radiative balance occurs at the top of the atmosphere, not the bottom. So that is where the Boltzman radiative equilibrium temperature is located. The temperature at the surface is then determined from the temperature at the top of the atmosphere by the lapse rate which arises from purely convective processes and has nothing to do with radiative properties at all. As gases fall in the atmosphere compression heats them. As they rise they cool. So vertical mixing establishes a vertical temperature gradient and temperature at the surface is therefore hotter than the temperature at the top of the atmosphere. A thicker atmosphere in this model would make for a hotter surface. QED
Now imagine the atmosphere of such a world gradually becoming less opaque until it became totally transparent. At the end of that process we’d be back at your model with the Boltzmann temperature at the surface. But in between we’d be in a much more realistic complicated state with the Boltzmann temperature at an intermediate depth in the atmosphere and the surface somewhat hotter. How much hotter depends on many things – on the lapse rate (which depends on the amount of vertical mixing)- on the opacity – on the depth of the atmosphere. And this happens with at no stage any of the gases being “greenhouse” in the classical sense since we maintain the requirement that they absorb an equal fraction of the radiation at all frequencies throughout.

January 15, 2012 7:09 pm

I can improve on my previous summary thus:
1) Willis accepts that his non GHG atmosphere will produce a dry adiabatic lapse rate with the warmest temperature at the surface.
2) The warmth at the surface is NOT due to gravitational compression ( although a tiny fraction of it would be) but gravity IS responsible for placing the maximum density of non GHG molecules at the base of the atmospheric column.
3) That maximimum density causes the greatest number of molecular collisions to occur just above, or in contact with, the surface.
4) Such collisions transfer energy by conduction and not radiation. The surface converts incoming solar shortwave radiative energy into kinetic energy and the non GHG gases in contact with or in close proximity to the surface retain that kinetic energy by exchanging the energy via conduction between molecules of non GHGs until it can be released upward as outgoing longwave IR.
5) It is that delay that allows the surface temperature to rise as energy accumulates within the system and most particularly at or just above the surface.
6) The greater the density of the non GHG atmosphere, the more molecular collisions occur and the longer the delay the higher the equilibrium temperature must become. The increased density does NOT slow down the rate of conduction. Instead it increases the proportion of fast moving radiation that is retained for longer as slower moving conduction for an average net reduction in energy flow through the system.
[COMMENT: We cross-posted, Stephen, see my post above. Thanks, -w.]

January 15, 2012 7:29 pm

Willis, just seen your comments which suggest more clarity needed on my part so here goes again:
1) Willis knows that his non GHG atmosphere will produce a dry adiabatic lapse rate with the warmest temperature at the surface.I am assuming some movement via convection to achieve it.
2) The warmth at the surface is NOT due to gravitational compression ( although a tiny fraction of it would be) but gravity IS responsible for placing the maximum density of non GHG molecules at the base of the atmospheric column.
3) That maximimum density causes the greatest number of molecular collisions to occur just above, or in contact with, the surface.
4) Such collisions transfer energy between themselves by conduction and not radiation. The surface converts incoming solar shortwave radiative energy into kinetic energy and the non GHG gases in contact with or in close proximity to the surface retain that kinetic energy by exchanging the energy via conduction between themselves and between themselves and the surface until it can be released upward by the surface as outgoing longwave IR.
5) It is that delay that allows the surface temperature to rise as energy accumulates within the system and most particularly at or just above the surface.
6) The greater the density of the non GHG atmosphere, the more molecular collisions occur, the longer the delay and the higher the equilibrium temperature must become. The increased density does NOT slow down the rate of conduction. Instead it increases the proportion of fast moving radiation that is retained for longer as slower moving conduction for an average net reduction in energy flow through the system.
Any further comments ?

gbaikie
January 15, 2012 7:39 pm

“I’d say that GHGs are gases which are transparent at the frequencies emitted by the sun but absorb the frequencies emitted by the planet. ”
CO2, H20, O2, and O3 are not greenhouse gases acorrding to your definition:
http://en.wikipedia.org/wiki/File:Solar_Spectrum.png
From:
http://en.wikipedia.org/wiki/Sunlight

gbaikie
January 15, 2012 7:48 pm

4) Such collisions transfer energy by conduction and not radiation.
“No. At equilibrium, the atmosphere is quiet, with no energy going into or out of it. Collisions are exchanging energy, not transferring energy. It’s not going anywhere.”
Do you accept if it’s windy, one has transfer by conduction?
I disagree with4] since I would say it’s convection rather than conduction- with air, conduction is normally slow.
Fiberglass insulation indicates this. The fiberglass in term of volume is mostly air, the fiberglass inhibits the the air from convecting heat.

January 15, 2012 7:54 pm

I should add this:
7) At equilibrium we now have solar shortwave hitting the surface at 240W/m2 and IR longwave leaving the surface at 240W/m2 but additionally we now have a backed up pool of kinetic energy bouncing between air molecules at the surface and between those molercules and the surface giving the necessary temperature boost at the surface.

Myrrh
January 15, 2012 7:55 pm

But when the temperature of a perfect blackbody planet rises … the surface radiation of that planet must rise as well.
And because the atmosphere is transparent, this means that the planet is radiating to space more energy than it receives. This is an obvious violation of conservation of energy, so any theories proposing such a warming must be incorrect.
===
I can’t quite put my finger on this, but it feels as if there’s a disjunct somewhere.
Ah, ‘because the atmosphere is transparent, this means that the planet is radiating to space all the energy it receives..
You’ve not included any mechanism for why the planet is radiating more energy than it is receiving. The transparent atmosphere is not adding to it.

evilincandescentbulb
January 15, 2012 8:14 pm

w–> “They theorize that some kind of atmospheric/gravity interaction is what is warming the earth to about 30°C above the theoretical S-B temperature. They claim it doesn’t require GHGs. My proof shows that they are incorrect, that their claims violate the conservation of energy.”
You cannot have “proof” because you have no proof that whatever causes gravity does not also cause the additional warming — unless of course you have your own supercollider and found Higgs’ “goddamn particle.”

gbaikie
January 15, 2012 8:14 pm

“4) Such collisions transfer energy by conduction and not radiation. The surface converts incoming solar shortwave radiative energy into kinetic energy and the non GHG gases in contact with or in close proximity to the surface retain that kinetic energy by exchanging the energy via conduction between molecules of non GHGs until it can be released upward as outgoing longwave IR.”
If any conduction increases molecules velocity. And if one has a few million molecule which have increase their velocity, one call this an air packet. This air packet is constantly interchanging gas molecules but as a location.if it’s average molecular speed is higher than molecules above it, it less dense and has buoyancy, and rises [convection].
“5) It is that delay that allows the surface temperature to rise as energy accumulates within the system and most particularly at or just above the surface. ”
One can have heat inversions near surface, this inhibits convection- if you inhibit convection the surface can get warmer.
For there to be gain in heat capacity, the needs to cool the surface. Cooling the surface, causes lower temperature [a few degrees] and surface absorbs more solar energy and if cooler radiates less energy.
“6) The greater the density of the non GHG atmosphere, the more molecular collisions occur, the longer the delay and the higher the equilibrium temperature must become. The increased density does NOT slow down the rate of conduction. Instead it increases the proportion of fast moving radiation that is retained for longer as slower moving conduction for an average net reduction in energy flow through the system.”
The greater the density of the non GHG atmosphere, the more molecular collisions occur [with the surface- otherwise don’t see how it’s relevant].
” the longer the delay and the higher the equilibrium temperature must become.”
I don’t understand.
“The increased density does NOT slow down the rate of conduction.”
Yes. And I does increase amount conduction [higher potential of transferring energy per second].
“Instead it increases the proportion of fast moving radiation that is retained for longer as slower moving conduction for an average net reduction in energy flow through the system.”
No idea what this means. “fast moving radiation”? Radiation is speed of light.
Rate flow of heat?

Myrrh
January 15, 2012 8:14 pm

In other words, you’re the one saying it’s radiating more than it is receiving, but, the 4times is calculated on the temperature it has been raised to, not to the power imput that raised it to that temperature.
The disjunct is that you have to provide the real radiation imput in the other half of the equation.

evilincandescentbulb
January 15, 2012 8:22 pm

Since most of the surface of the Earth is water heat is lost through evaporation.

shawnhet
January 15, 2012 8:22 pm

As a suggestion to help keep WUWT the great website that it is, let me suggest that allowing a thread’s author to moderate that thread (at least when moderation is active) is a pretty bad idea. If we are going to have threads that snip commenters posts liberally, such snipping should be done by a third party. Even if the thread’s author is completely justified in snipping, if he snips himself, there is no way to avoid the *appearance* of censorship. (IMO, snipping doesn’t work at all, but if you’ve got to do it, try to do it in a way that seems fair to all concerned).
For the record, as far as I can tell and IMVHO, proponents of gravity-enhanced temperature effects cannot answer simple and *necessary* questions on their theories. (The main one being: how can the surface of a planet such as Earth emit more energy than the planet as a whole emits to space?)
Ultimately, though the proponents of the theory have the responsibility to answer questions relating to the theory (a point that the snipping does attempt to make). If someone really doesn’t want to answer a question, there is nothing that anyone can do here to make them do so.
Cheers, 🙂

JIsbert
January 15, 2012 8:23 pm

I think N&Z have not violated energy conservation. This is why:
N&Z take Penn State’s model which has a one layer atmosphere. The “greenhouse effect” of this model is created by heating this layer to its emission temperature through radiative transfer from the surface and then have it radiate towards space and the surface.
This is much like in your earlier post “The Steel Greenhouse” where a “greenhouse effect” is generated on a sphere inside a sphere. The back radiation causes the inner sphere to radiate at a higher temperature than the outer sphere to get the net energy transfer equal to the outside radiation.
By using this type of model the temperature at the surface (the inner sphere) can be higher than the emission layer (outer sphere) temperature without violating energy conservation.
Essential to this model is the absorption of LWIR by this atmosphere layer and changing its temperature in the process.
The dry air atmosphere changes this essential characteristic. No LWIR absorption hence none is radiated back or away from there. The entire energy balance happens on the surface (except possibly at the start of its timeline).
This fundamental change which makes the “dry air atmosphere model” a completely different model. Conclusions from this model do not apply to N&Z’s model.
Then they add convective heat transfer to the radiative transfer and take it from there. They correctly find that convection is far more efficient than radiation for transferring energy, which I believe has been mentioned before.
However there are problems with this paper:
In particular their conclusion that the temperature is a function of pressure. I find it difficult to believe that the earth lost half its atmospheric pressure in the last 50 million years given that the atmosphere has been around for a few billion years.

willb
January 15, 2012 8:24 pm

Willis Eschenbach says:
January 15, 2012 at 1:31 pm

No gas spontaneously liquifies at elevation.

Excepting maybe H2O, right? 🙂
But seriously, I’m still trying to wrap my mind around your gedanken experiment. I don’t think Triton or Pluto would have any problem spontaneously condensing Nitrogen gas out of their atmospheres. However, I see you’re talking about a planet with more Earth-like qualities. In that case, to get Nitrogen to condense, the (dry adiabatic) lapse rate would have to extend out to beyond 20-25km to get below its melting point. I would speculate this might be possible if the atmosphere were pure Nitrogen.
The point I was trying to make was that even without GHGs you can still have radiation spontaneously emitted at altitude from the atmosphere of a planet or moon, if not through atmospheric phase changes then perhaps through the creation of plasmas of charged particles, similar to the Earth’s ionosphere.
Thanks for the link to the Caballero paper. I think it’s an excellent resource.

Myrrh
January 15, 2012 8:28 pm

Also, if radiation in is raising temperature of something, the radiation out has to be added to the energy contained in the temperature, kinetic.
But anyway, the radiation out isn’t relating back to radiation in, but to the temp of the object heated.
If this thought experiment is to work out how more radiation can be emitted than the radiation in, of some energy budgets, then perhaps it’s simply that the radiation in has the wrong figures..

Bill Hunter
January 15, 2012 8:29 pm

Willis says:
“but cooling will occur over not geological but cosmological time frames or slower. Because as they cool, such transition-energy collisions get less and less probable.”
Would that slowing perchance be a 4th root function?

gbaikie
January 15, 2012 8:34 pm

“I don’t accept that it is windy on the hypothetical planet we are discussing. ”
In the million sun world? Yeah, no wind.
The surface could could only be -18 C if significant fraction of warming was caused by sunlight.
Something like Neptune could be much warmer but it doesn’t have a significant fraction of it’s heat caused by sunlight.

evilincandescentbulb
January 15, 2012 8:38 pm

–>”However there are problems with this paper:
In particular their conclusion that the temperature is a function of pressure. I find it difficult to believe that the earth lost half its atmospheric pressure in the last 50 million years given that the atmosphere has been around for a few billion years.”
When Earth is a snowball there is no snow, no rain and maybe… less atmospheric pressure.

gbaikie
January 15, 2012 8:42 pm

“I should add this:
7) At equilibrium we now have solar shortwave hitting the surface at 240W/m2 and IR longwave leaving the surface at 240W/m2 but additionally we now have a backed up pool of kinetic energy bouncing between air molecules at the surface and between those molercules and the surface giving the necessary temperature boost at the surface.”
Apparent you discussing the million sun world, and there, you would have million times 240 W/m2 leaving the surface

Reed Coray
January 15, 2012 8:42 pm

In reference to Willis Eschenbach says: January 15, 2012 at 6:11 pm
Willis, I will read (and will give serious thought) to your two referenced posts. I would like to start a dialog with you regarding radiation between spherical shell surfaces. However, I don’t believe “comments” to a guest post on WUWT is the proper forum for such a dialog. I have thought a lot about radiation between concentric, uniform-surface-temperature, spherical, blackbody/greybody shells where the innermost shell has a constant internal source of thermal energy and a vacuum exists between the shells. If I remember correctly, starting with Planck’s blackbody differential radiation law (differential solid angle, differential planar surface area, differential frequency interval), I derived the electromagnetic radiation rates between the outer surface of the inner shell and the inner surface of the outer shell as well as the temperatures of these surfaces. I have a word file of several megabytes that contains my derivations. That file is much too large for a comment on this blog. I would, however, like to exchange thoughts with you on a one-to-one basis. To do that, I need your e-mail address and you need my e-mail address. I am reluctant to put my e-mail address on an open forum such as WUWT. However, Anthony Watts has my e-mail address; and I give Anthony permission to give you my e-mail address privately. If you’re interested in exchanging e-mails with me, get my e-mail address from Anthony, send me an e-mail to establish communications, and I’ll be happy to begin a dialog. If you’re not interested, just respond to this comment with a short comment so indicating.
Thank you for your time,
Reed Coray.

anna v
January 15, 2012 8:45 pm

Willis Eschenbach says:
January 15, 2012 at 11:13 am
There is no matter that does not radiate electromagnetically when in bulk.
The radiation may be small, in the low part of the electromagnetic spectrum not measurable easily, but it is there.
One has to realize that all, ALL , observable interactions of matter that affect every day temperatures are finally, at the last contact, electromagnetic. Electromagnetic means photons as a bottom line.
Argon has an emission spectrum too.
In any case it is not necessary to excite distinct emission lines in order to radiate. There exists a continuous spectrum, also quantum mechanically, because all solutions of potential problems end up with n=infinity, l=infinity, m=infinity, i.e. there are very dense quantum mechanical states that can be excited by scatterings as the molecules and atoms move and interact with the surrounding field of other atoms but appear as continuum. Even for noble gases. That is the way the radiative field of black body radiation is also generated, not only with distinct lines..

evilincandescentbulb
January 15, 2012 9:17 pm

w–>Instead, it points out that the consequence of increased temperature (from whatever mechanism) is increased radiation. But increased radiation violates conservation of energy.
You are pointing out what has been said by others: to fear runaway global warming is to believe in the possibility of a perpetuum mobile.

Brian Macker
January 15, 2012 9:27 pm

Willis,
You are correct so far in every single regard except … You didn’t take into account a planet that is covered densely with girl scouts roasting marshmallows and an appreciable number of those morsels caught fire from holding them too close to the flames. Also you failed to mention that my favorite girl scout cookie is the samoa. Other than that you are spot on with your understanding of the physics involved, and have shown your opponents to be a bunch of ignoramuses, and none of this is above the level of high school honors physics and chemistry.

evilincandescentbulb
January 15, 2012 9:33 pm

Oh sure his understanding of physics may be spot on but AGW theory is a religious matter.

Spector
January 15, 2012 9:33 pm

I think Willis is right here. The only thing I might change is the word ‘characteristic’ for ‘theoretical’ because there is no ‘theoretical’ reason for that characteristic temperature to be found anywhere except on a body of uniform temperature. It is, in effect, the fourth root of the flat-area-weighted average of the fourth powers of all absolute temperatures on a body. For zero and one-hundred this average would be about 84.
A GHG-Free atmosphere is a hypothetical creature; however I understand that transparent, non-GHG gases do have a residual absorption and emission capability on a scale equivalent to the imperceptible non-transparency of these gases in their pure form. Some gases may have absorption/emission bands in the optical range and would require heating to a significant fraction of the surface temperature of the sun before their emissions would balance their absorption. Perhaps the stratosphere is our best example of such a regime and only the LWIR radiation by carbon dioxide in the mesosphere above allows heat absorbed by the stagnant air below to escape to outer space.

Bill Hunter
January 15, 2012 9:43 pm

Willis Eschenbach says:
January 15, 2012 at 8:57 pm
“Instead, it points out that the consequence of increased temperature (from whatever mechanism) is increased radiation.”
I realize the popular theory of radiation is it willy nilly flies in all directions with no particular destination, no connection with where its going. But I would point out thats pure theory with absolutely zero evidence of it being true. If for example, radiation (and light) only flows from warm objects to cold objects in ways similar to other energies like electricity, conduction, magnetism (positive negative charges instead of warm cold), etc. then what you say above would not be always true. People should be very careful about generalizing theories and making sweeping statements about them.
I have been thinking about Jelbring’s theory and there is a sense to it and it does seem to fit into some of these potential cracks of generalized theories. Thus they probably should not be discarded via a wave of an arm.
Dr. Brown talked about gravity creating heat in gas in big planets and stars. The gravity of these planets could hold an attraction to certain base levels of energy that prohibits its release and in a gaseous atmosphere that could be manifested by a level of heat. Only heat in excess of a certain level is actually emitted from an object to space (not empty space but to that near solid wall of objects in space no matter how far they are). Its hard to imagine electro magnetic energy being sent on an endless journey wandering its way through space curving around stars but never reaching a destination. After all we do know that gravity attracts light as the Einstein theory exposed.
Bottom line here is Jelbring said his model planet does not radiate, yet you can’t listen long enough so you can get on with the job of discarding it for violating a law for doing something it doesn’t do. It seems its getting a lot like Real Climate around here.

Bart
January 15, 2012 9:57 pm

OzWizard says:
January 15, 2012 at 5:54 pm
“That is why Einstein’s relativity theory led to the TWIN PARADOX.”
Now, this really should have been censored, Willis. I will not suggest… no I will not.
For anyone interested: Fermi-Walker transport explained the so-called “paradox” decades ago. The unequal effect of accelerated motion is confirmed in atom smashers daily. Thomas precession is another consequence of the effect which has been observed almost continuously since early in the last century. Velocity is relative. Acceleration is not.
Willis Eschenbach says:
January 15, 2012 at 6:28 pm
“I said because the planet is at thermodynamic equilibrium there is no exchange of heat across the interface, not the other way around as you have it.”

In a state of thermodynamic equilibrium, there are no net flows of matter or of energy, no phase changes, and no unbalanced potentials (or driving forces), within the system. A system that is in thermodynamic equilibrium experiences no changes when it is isolated from its surroundings.”

I want you to pay special attention to these words: No unbalanced potentials.
What is the system we are looking at? It is the entire planet/atmosphere system. Not the planet alone. When you take the planet alone, you are removing the balanced potential between it and the atmosphere. You are no longer dealing with a system in thermodynamic equilibrium. There is a temperature gradient with an abrupt discontinuity there. When you separate the planet and its atmosphere, you must account for the disappearance of that gradient.
The system is in thermodynamic equilibrium. Not PART of the system. THE SYSTEM.
As I said, you are doing the equivalent of drawing a free body diagram for a two-body system without the contact forces from the connected body.Or, of claiming two resistors in series are equivalent to only the resistance from the one closest to ground, because: 1) when you short the other one, you measure the supply voltage drop across the one remaining 2) total voltage cannot be greater than the supply 3) therefore, the other resistor can have no effect.
It is complete circulus in probando. You are saying there is no effect from the atmosphere, therefore there is no effect from the atmosphere.
“You can only “assure me” if you can explain how that might work to me. “
You’ve been given numerous explanations. For this thought experiment specifically: Temperature alone DOES NOT determine radiation from a non-Planckian energy distribution, and there is an avenue through the surface/atmosphere interface to develop a non-Planckian energy distribution. The atmosphere can absorb heat through conduction. It can dissipate that heat through the same conduction interface. There is no law, no physical principle, which says that the effect of that exchange has to be uniform across energy states.
“It’s not as complex as relativity.”
Relativity is not complex. It messes with our intuition, but that is not the same as saying it is complex.

Paul Dennis
January 15, 2012 10:20 pm

Willis,
You state:
“Finally, I think (sadly) that Dr. Roy is wrong about the eventual distribution at equilibrium being isothermal. That would put the maximum total energy (kinetic plus potential) at the top of the atmosphere. I see no reason why that uneven energy distribution would be favored over the even energy distribution reflected by the normal lapse rate. I see no reason why the normal lapse rate would not obtain.”
This statement seems to imply that the lapse rate is a result of the loss of kinetic energy with height in a gravitational field. Surely the lapse rate is determined by the adiabatic expansion of the atmosphere and requires the motion of air parcels. If you remove convection from the thought model (and I don’t see why convection will occur on your evenly heated sphere) you are simply describing the Loschmidt effect. This paradox is dealt with by Coombes and Laue, 1985, Am. J. Phys. A paradox concerning the temperature distribution of a gas in a gravitational field.
What you see as an uneven energy distribution is not the case. The temperature is determined by the average kinetic energy of the ensemble of molecules in a given volume element. Whilst both the number of molecules and kinetic energy of molecules decreases with height, the average kinetic energy remains the same and thus the temperature is constant with height. A similar proof has also been given by Roman, White and Velasco, 1996, Eur. J. Phys., On a paradox concerning the temperature distribution of an ideal gas in a gravitational field.
None of this affects your argument about gravity effects and surface temperature. However, there are some who believe the Loschmidt effect is the origin of the lapse rate and I believe they are wrong.
Finally, I’m amazed this thread has such legs. There again there are some who believe in perpetual motion!

wayne
January 15, 2012 10:33 pm

jjthom says:
January 15, 2012 at 4:59 pm
A brilliant site for the physics:
http://www.patarnott.com/atms749/
The presentations are clear – from First Course in Atmospheric Radiation: Second Edition, by Grant W. Petty.
— — —
jjthom, that is hugely appreciated and exactly at the right time. I sure needed that! Hope Willis and others a bit confused also read it. So… N2 does have rotational lines after all… just what I expected! Thanks again.

Alexander Harvey
January 15, 2012 11:00 pm

The thought experiment given by Willis is sound.
The question of whether the resultant atmosphere will conform to the DALR or become isothermal is not relevant to his argument.
There is a disagreement on that point but although it is not crucial to his argument it is interesting in its own right.
I think I can show that conformation to the DALR as opposed to the isothermal state is not mandated.
In his model the atmosphere is only coupled to the surface, that is its only boundary where energy transfer can take place, i.e. it is not coupled radiatively or by any other means, via its upper boundary with deep space
1) Should the atmosphere become isothermal, for any reason, a spontaneous move towards adopting the DALR would require a transfer of energy from the atmosphere in a way that promotes a temperature gradient. The atmosphere would then be transfering energy up that temperature gradient. That is a problem with repsect to the 2nd Law. This does not say that it must become isothermal but that conversion to the isothermal state is irreversible without doing work.
2) A DALR conforming atmosphere is open to a perpetual motion exploit in that the bottom and top of the atmosphere have the potential to be used as a thermal source and a thermal sink. In practice this could be achieved by constructing a heat engine using a gas with a differnt Cp to the atmosphere, as a working fluid, or by coupling using a rod with high thermal conductivity, but thermally insulated along its length, to bridge the distance. I suspect other methods could be devised. Work could be extracted and heat would be transfered in the manner promoting a move towards an isothermal distribution. A requirement for the temperature profile to adhere to the DALR would require that the energy be returned up the temperature gradient again problematic with regard to the 2nd Law.
3) Similarly any “substantive” move towards the isothermal state caused spontaneously due to thermal conduction/diffusion would be irreversible without doing work. Here substantive is to be understood as more than the inevitable random fluctuations.
None of those points says it must be isothermal but do argue that the isothermal state is preferred due to entropic considerations.
It is clear that the DALR is stable in the respect that any tendency to increase the slope of the profile away from the DALR will be conteracted by increasing convention and hence the thermal coupling. A tendency to decrease the slope of the profile away from the DALR will tend to inhibit convection reducing the coupling. If this progresses the profile will move towards the isothermal and ever greater stability as the atmosphere becomes incresingly stratified. This describes the DALR as being the maximum slope limiting condition. The isothermal profile being the other limiting condition.
None of this gives an indication of to what degree or over what timescale a move towards the isothermal would take place but it does suggest that a profile conforming to the DALR is not a thermodynamically preferred option.
Alex

JimF
January 15, 2012 11:00 pm

C says:
January 14, 2012 at 12:45 am
Thank you for that reference to Joseph Postma’s work. I went and read “The Model Atmospheric Greenhouse Effect”, found here:
http://www.tech-know.eu/uploads/The_Model_Atmosphere.pdf
Very informative. Basically, this says that the Radiative Greenhouse Effect model is completely wrong, and if you pick the wrong boundary conditions, then this kind of thinking occurs: “…On average day and night over the planetary surface, the Earth receives about 240 W/m2 of energy from the sun. The theoretical S-B temperature for this amount of radiation (if it were evenly distributed) is about -18°C, well below freezing….” (This is not intended to criticize Willis; his words simply met my needs, and do in fact represent the “consensus” thinking. Postma shows how these numbers come about, and gives a strong case as to why they are wrong).
One should read this and consider what I think are valid points regarding the physical model we’re dealing with, and see how this might lead one into making calculations that are silly (like the numbers quoted above). Dr. Brown, who recently started looking at Earth’s baseline black-body model – “a damn hard problem” – would surely find it of interest.
This ongoing discussion is really interesting, because it addresses the fundamental basis of the radiative greenhouse effect, which Postma says is garbage. I’m just learning as I go. I think Willis’ “elevator speech” approach is a good one that we all can adopt to use.

lrshultis
January 15, 2012 11:14 pm

Willis:
This is a simplified view of the GHG-free atmosphere with solar input according to my understanding.
Some solar energy is transferred to the Ar, O2, and N2 by absorption at various frequencies, mainly high in the atmosphere, and by momenta transfer by scattering of photons. The remaining solar energy is absorbed by the local surfaces giving them local temperatures (not necessarily the same as the local weather temperature of the air several feet above the surface). Some of that surface kinetic energy is radiated to space as IR. Some more is conducted to the atmosphere causing it to do work by expansion decreasing its density and convecting to higher altitude. Gravity then causes cooler dense air to fall to the surface and doing work on it equivalent to the work done by the surface heating by increasing its density and temperature. A lapse rate is established by gravity as pressure*volume/ mole decreases and thus temperature decreases with increased height. The amount of surface heat to the atmosphere determines what the equilibrium lapse rate will be. Thus the kinetic energy transfer which causes convection does work against gravity and does not need to be radiated to space.
With GHGs the lapse rate will change due to the increase in atmospheric temperature due to the transfer of energy from GHG molecules by transferring kinetic energy ( due to absorption of IR from the local surfaces) from the GHG molecules to the Ar, O2, and N2 atoms and molecules.
Larry

JIsbert
Reply to  Willis Eschenbach
January 16, 2012 9:52 am

Willis,
I think N&Z are getting a bit carried away with their conclusions, which is also reflected in their abstract.
Their model is a 2 layer model (surface + emission layer). Heat is transferred from the surface to the emission layer by radiation only (in the Penn State model) and by radiation + convection in their “unified” model.
They show solutions for the equations describing either model (Fig. 2 and 3). And their finding is that their “unified” model carries the day, i.e fits with the atmospheres of many solar system planets and moons.
The important point here is that their model requires the 2nd layer. They never explore what influence the composition of the atmosphere has in creating this situation.
Your “dry air model” takes away this condition and changes the model into a 1 layer model (the surface only). This change in model makes the different conclusions not applicable to their case.
Since they do not explore what mechanisms this 2nd layer requires and how the composition of the atmosphere might change their case, their claim that the composition does not matter is just that – an unexplored and thus unproven claim.
Cheers,
JI

January 16, 2012 12:00 am

I’d like to hear Willis’s further comments on the last version of my post which was as follows:
1) Willis knows that his non GHG atmosphere will produce a dry adiabatic lapse rate with the warmest temperature at the surface.I am assuming some movement via convection to achieve it.
2) The warmth at the surface is NOT due to gravitational compression ( although a tiny fraction of it would be) but gravity IS responsible for placing the maximum density of non GHG molecules at the base of the atmospheric column.
3) That maximium density causes the greatest number of molecular collisions to occur just above, or in contact with, the surface.
4) Such collisions transfer energy between themselves by conduction and not radiation. The surface converts incoming solar shortwave radiative energy into kinetic energy and the non GHG gases in contact with or in close proximity to the surface retain that kinetic energy by exchanging the energy via conduction between themselves and between themselves and the surface until it can be released upward by the surface as outgoing longwave IR.
5) It is that delay that allows the surface temperature to rise as energy accumulates within the system and most particularly at or just above the surface.
6) The greater the density of the non GHG atmosphere, the more molecular collisions occur, the longer the delay and the higher the equilibrium temperature must become. The increased density does NOT slow down the rate of conduction. Instead it increases the proportion of radiation that is retained for longer within the system in the form of slower moving conduction leading to an average net reduction in energy flow through the system at any given level of input.
7) At equilibrium we now have solar shortwave hitting the surface at 240W/m2 and IR longwave leaving the surface at 240W/m2 but additionally we now have a backed up pool of kinetic energy bouncing between air molecules at the surface and between those molecules and the surface giving the necessary temperature boost at the surface.
It sounds somewhat like a pressure bulge in a hosepipe when there is a weakness in the wall which then stretches under pressure of increasing throughput. It comes and goes with the speed of flow. Water for the hose, solar energy for the non GHG atmosphere. In the non GHG atmosphere it is the pressure and temperature at the surface which changes in response to changes in solar input and atmospheric density.
I don’t see how one could accept a slowing down of the energy throughput without some sign of higher temperature somewhere in the system and by virtue of the dry adiabatic lapse rate it has to be in the air where it contacts the solid surface.

Bart
January 16, 2012 12:10 am

Willis Eschenbach says:
January 15, 2012 at 10:55 pm
shawnhet says:
January 15, 2012 at 8:22 pm
“The main one being: how can the surface of a planet such as Earth emit more energy than the planet as a whole emits to space?”
And, when did you stop beating your wife?
You have to show it is necessary that more energy be emitted by the planet for non-GHG heating to work before discounting the idea on that basis. SB does not demonstrate it. SB is not a fundamental law – it has loopholes. And, the widespread neglect of these loopholes is just plain sloppy.

PhilC
January 16, 2012 12:21 am

I’m interested in Reed Coray’s question and I had a look at the links you gave him, and I think that there is something missing from the elevator speech for the “Greenhouse Effect”. That is that Infra Red absorption can raise the temperature of the atmosphere higher than that of the radiating surface.
This apparent breaking of the 2nd law of thermodynamics is possible because the energy is transmitted from the surface to the atmosphere as radiation, not heat. IR absorbing molecules in the atmosphere lose energy to the atmosphere as heat, by collision with other molecules.
Because this disturbs the radiation budget of the planet, and energy can only escape the planet by radiation, this can only be restored by an increase in temperature of the surface, until the equilibrium set the the Stefan-Bolzmann curve is restored. There is downward welling energy transfer, but it’s mostly heat rather than radiation. All the IR photons do eventually escape, but they may be absorbed and re-emitted many times in the atmosphere on the way.
This effect of energy (not heat) being transmitted from a cooler to a hotter body is not unique to the atmosphere. The Sun’s corona is another instance. It has a temperature of a million K because of energy transmitted from a body of 6000 K (the surface of the Sun).
It seems to me that is how the atmospheric “Greenhouse Effect” actually works.

RobB
January 16, 2012 12:27 am

I asked this further up the thread, and this is a change to the model that Willis proposed. But, does anyone know the state of an otherwise identical model if the atmosphere did emit radiation at some rate? I think this is a very good analogy to a GHG free Earth. The surface conducts/convects energy into the atmosphere but any IR emitted goes straight out to space. The atmosphere itself emits radiation at some rate, either up or down, down hits the ground and up goes to space, so the radiation physics are very simple still in this case. The down radiation heats the surface which either gets emitted as IR into space or warms the atmospere by conduction.
Any idea that the surface air temperature is in this slightly altered model?

Editor
January 16, 2012 12:33 am

Willis
For what it’s worth I was unhappy with the initial banning of Joel by tall bloke and said so. Equally the amount of snipping here is prodigious, surely the first time in climate history that anything has actually genuinely been ‘unprecedented’
However what has become clear is that no one has managed to make the elevator speech you asked for even though there are plenty of people here who could,if such a device were possible to explain the theories. That leaves a bystander such as me fairly sure that such a speech is not possible. The theories that have been postulated are radical and important and it would be interesting to rerun this thread in a months time when hopefully less heat and more light can be shone on the subject by people who have had more time to think about it
All the best
Tonyb

Martin A
January 16, 2012 12:34 am

Willis – I can see no fault in your argument by contradiction.
For your model, you said “we add an atmosphere to the planet, a transparent GHG-free atmosphere.”
Its surprising how many commenters insist that the atmosphere in your model will radiate nonetheless – ie they insist that the atmosphere is NOT a transparent GHG-free atmosphere, despite your explicitly stating the assumption that it is.
However –
” How many comments like mine didn’t even make it past moderation?
My dear, there are no comments like yours, so the answer is none.”
Am I alone in finding this reply sexist, condescending and unworthy of W.E. ?

gbaikie
January 16, 2012 12:37 am

shawnhet says:
“The main one being: how can the surface of a planet such as Earth emit more energy than the planet as a whole emits to space?”
Not sure of question being asked.
how can surface of earth emit more energy than entire earth emits to space?
So obviously surface of earth is part of earth. So you asking does surface emit energy which doesn’t go into space, and how could these surface emission [not going into space be greater.
Otherwise It makes no sense.
Don’t what has do anything, but see if I can answer.
We could assume you mean by earth emits, that this different then earth reflecting sunlight.
Or we can assume anything come from the direction of the earth [includes reflected sunlight and anything else].
If we address the latter [which should the greatest amount], how much energy comes from the direction of earth. We have forest fires and geothermal heat but the big number is the sunlight- so we focus of that number. Wiki says “The Earth receives 174 petawatts (PW) of incoming solar radiation (insolation) at the upper atmosphere.”.
So question is how can surface emit more energy than 174 petawatts?
174 petawatts is 174 10^15 watts or 1.74 10^17 joules per second.
Now, what is surface. Do we mean top 1 mm of land or water which meets air of atmosphere,
do mean the troposphere. Or first couple miles of land and/or ocean.
No idea.
Anyways, the surface area of earth is 510 million square kilometers, or 5.1 10^14 square meters, and this approximation of surface area probably is what is meant. So, let’s see, 1740 10^14
divided by 5.1 10^14 is 374 joules per second. So question is are 374 Joules per sq meter being emitted or transfered somewhere other than space per second.
Do powerlines count as surface? Humans emit about 100 joules per second.
Well atmosphere is warmed [requires energy to emit] and cooled [requires energy to emit].
Oh so is it this cooling and warming that you mean?
The atmosphere is 5.1 x 10^18 kg. Nah, let’s do 10 tonnes per square a one C degree change in 10 tonnes of air is 1.0 kJ/kg.K so 10,000 times 1000 is 10 million joules. Say 10 C per nite
And 12 hours times 3600 is 43200 seconds. 100 million divided by 43200 is 2314 joules per second per square meter. 2314 is 6 times more than 374 Joules.
And of course it warms up during day, and so has same scale of joules in energy is being emitted.
So guess answer is yeah.
Dew which is 1 mm thick per square meter is how much energy involved? so meter is 100 cm square, so 10,000 square cm. And mm is 1/10 of cm, so 1000 cubic cm or 1000 grams. And Latent heat of evaporation – 2,270 kJ/kg. So 2.7 million joules. That’s not much.
How about campfire? it warms air, and air molecules warm other air molecules and they each emit energy to other molecules and they do this zillions of times.
Or actually even if air stays same temperature one of the zillions molecules are exchanging energy every fraction of nanosecound- it and all the rest of them are constantly averaging the molecular speed.
But the exchange velocity or exchange of heat could be said not to be what is called the energy emitting.
Perhaps you mean only photons. I don’t know number photons. Sun is emitting a lot of them.
And standard greenhouse theory is all about photons. CO2 absorbs photons, and emits photons.
Is CO2 absorption and emission of photon what you asking? Consider the theory on this, seems shortage of discussion on how photons are involved.
I wonder many photons to a watt:
What is the energy in a single photon of light at 500nm?
a single photon is 0.039756 × 10-17 J
Or one joule/watt equals 25.15 × 10^17 photons
http://reefkeeping.com/issues/2006-02/sj/index.php
Every wavelength has different energy, but ignoring that
Sunlight: 1.74 10^17 joules times 25.15 × 10^17 photons
So 4.4 10^34 protons per second from the Sun.
So, don’t know how many photons a CO2 or H2O molecule can absorb and emit per second.
Ok so “As the frequency of the radiation increases (wavelength gets shorter), the amount of energy in each photon increases”
Which means the hotter something the less photons used per joule. double 500nm to 1000 nm
and per watt it’s twice the photon [or so it seems to this idiot].
So since Sun is hotter, it’s energy uses less photons, and colder objects to emit same energy need more photons.
So original question was:
“The main one being: how can the surface of a planet such as Earth emit more energy than the planet as a whole emits to space?”
So since earth is colder it will need to emit more photons to equal the energy.
Every grain of sand is emitting photons. Does every atom in rock emit photons?
I would guess, since there more joules of energy on earth then is delivered by the Sun in a second [or nanosecond] and any thing isn’t mostly facing the sky and gases are suppose absorb and emit some of these photons going towards space and things are warming and cooling, that is how.

Julian Braggins
January 16, 2012 12:42 am

Willis has proved his point, that under his specified conditions, multiple suns, no IR sensitivity of atmosphere, etc. the temperature would not exceed S-B level.
This does not disprove N&Z’s theory as these very conditions prohibit the mechanism of the gravity heat engine of atmospheres, convection.
Surface warming of the atmosphere would cause convection. Heat produced by a falling atmosphere that has lost some of that heat by radiation at height will will cause a higher temperature at the surface than the average of the atmosphere, as long as there is a differential in heating to prevent equilibrium, and that there is a day/night cycle sufficiently rapid to enable a reset of conditions for the heat engine cycle.
This is not to say that the temperature will necessarily exceed S-B, but in the case of the moon it seems the average temperatures can exceed this level.

gbaikie
January 16, 2012 1:09 am

“Willis Eschenbach says:
January 15, 2012 at 11:50 pm
gbaikie says:
January 15, 2012 at 8:34 pm
“I don’t accept that it is windy on the hypothetical planet we are discussing. ”
In the million sun world? Yeah, no wind.
You sure you understand how this whole “thought experiment” deal works?”
Yeah you think million suns uniformly across the sky of a planet which each has solar flux of 240 Watts per square meter. Is the a same as one sun with solar flux of 1300 watts per square meter.
I don’t think if you had a planet which extremely high “geothermal” heat of 240 watts per square meter it would cause the planet to get above -18 C. I don’t a greenhouse effect which adding 240 watts per square would increase the planet above -18 C.
So a million suns, high geothermal heat, super greenhouse gases and still only -18 C
And you want someone to explain how the N&Z theory will increase it the surface temperature above -18 C
Of course with million suns and billion or more years you would get a “geothermal” heat of 240 watts per square meter. So the only remaining point is why would greenhouse gas add any heat?

Paul Dennis
January 16, 2012 1:22 am

Willis,
thank you for your considered response. I can only quote from Coombes and Laue, 1985:
“Question: If a vertical column of air which is adiabatically enclosed is in thermal equilibrium, is the temperature the same throughout the column or is there a temperature gradient along the direction of the gravitational field? – Let us assume an idealized model in which there are no interactions between air molecules and no collisions once the equilibrium distribution has been established. The resolution of the paradox below is unrelated to this assumption. Energy conservation mplies that the kinetic energy (KE) of a molecule decreases with its height z above the Earth’s surface according to the formula:
KE = (KE)o – mgz
where m is the mass of the molecule and g is the acceleration due to gravity. Now, the two conflicting answers to the above question are as follows.
(1) There is no temperature gradient because a system which is in thermal equilibrium has the same temperature throughout.
(2) The temperature decreases as one goes up the column for the following reasons. (a) Since Eq (1) implies that every molecule loses kinetic energy as it moves upward, the average kinetic energy of all molecules decreases with height. (b) Temperature is proportional to the average molecular kinetic energy. Combining (a) and (b) establishes statement (2) above.
Which of the statements (1) or (2) is correct? Where lies the mistake? Note that the argument is short and that it is hard to find anything wrong with any of the statements. In fact, the second arguments seem so convincing that one begins to doubt the validity of state,ent (1) in the presence of a gravitational field. We recommend that the students be made to wait from one period to the next before the solution is given. We also recommend that the reader of this Note spend some time carefully thinking about the problem before continuing.
The answer is that statement (1) is right and statement (2) wrong. Statement (2) is wrong because the conclusion in statement 2(a) is wrong. The seemingly paradoxical truth is that the average kinetic energy of all molecules does not decrease with height even though the kinetic energy of each individual molecule does decrease with height”
Willis, I urge you to read Coombes and Laue and then the other paper I referred to upthread to read the proofs of these assertions. Of course they are counter to your elegant description above and suggest the atmosphere will be isothermal.

January 16, 2012 1:34 am

Has everyone here been treating radiative and conductive energy as interchangeable ?
I don’t think one can do that for an equilibrium scenario as proposed by Willis’s model because any leakage from one to the other will disturb the equilibrium.
The consequence is that the surface temperature has to rise to a level that can sustain both processes in parallel and independently in order to maintain equilibrium.
The S – B equation deals with radiation alone and not conduction so if the energy allocated to the conductive exchange is kept separate as it must be at equilibrium then the S – B radiative equations will still be complied with even though the surface is actually at a higher temperature.
Some examples:
i) A planet with no atmosphere.
Radiation is all there is and it comes straight in and goes straight out as per the S – B equation.
ii) A planet with the same gravity and atmospheric mass as the Earth but comprised of non GHGs.
The atmosphere diverts 150W/m2 to the surface/atmosphere conductive energy exchange. That energy is removed from the energy needed to sustain radiative balance so the surface cools. Over time energy builds up in the system again so that the S – B radiative balance has been restored but the energy required for the continuing conductive exchange is permanently denied to the radiative process. The surface therefore has to rise to a higher equilibrium temperature to sustain both processes but the S – B equation remains valid because it is blind to any energy engaged in the separate conductive energy exchange. There will still be 240W/m2 coming in and 240W/m2 going out as regards radiative energy but the surface will be at a temperature that supplies both the radiative 240/m2 going out AND its part of the energy exchange with the atmosphere at 150W/m2. It has to be so because if the surface supplies less than 150W/m2 to the air on an ongoing basis there will be no equilibrium. It matters not that the surface gets that 150W/m2 straight back again. That is the nature of an equilibrium.
iii) A planet with the same gravity as the Earth but double the atmospheric mass.
The atmosphere diverts 300W/m2 to the conductive energy exchange with 240W/m2 still allocated to the radiative exchange and a correspondingly raised surface temperature.
The error people have been making is to suggest that there is no need for a surface/air energy exchange once the air temperature reaches the surface temperature (or vice versa).That would imply that the conductive energy could be reallocated back to the radiative exchange but that is not correct.
The energy taken away from the radiative process in favour of conduction is lost to the radiative process permanently or for so long as there is an atmosphere with any mass at all. Yet both tranches of energy contribute to the surface temperature needed to maintain equilibrium.
It isn’t that the S – B equations are breached, merely that they relate only to the radiative portion of the total energy exchange.
Any flaws ?

OzWizard
January 16, 2012 1:36 am

Apologies for my “long-windedness”, Willis. Thanks for reading the ‘uninteresting’ part of my comment. Let me take your advice to be brief (Well, as brief as a 68 year old engineer metrologist can be).
Re my atttitude (in one sentence): I like what you have done in the past – have enjoyed almost every one of your previous posts – but the above post is one of your worst (IMO).
My objective: I want to help ease the pain of your “hurting head”.
My “elevator speech” re the Near-surface Atmospheric Temperature Enhancement (ATE, or NTE) postulated by N & Z (at p.6 of “Unified Theory of Climate”):
• The existence of a dimensionless Thermal Expansion Coefficient of steel does not imply that “gravity cause steel to expand”.
• Likewise, the existence of a dimensionless ATE ‘factor’ does not imply that “gravity causes heating of the lower atmosphere”, in defiance of the 2nd Law of Thermodynamics.
• These dimensionless ratios enable simple calculation of what the effect of “heat input” will be: in the first case, on a bar of steel; in the second case, on a planetary atmosphere subjected to gravitational compression.
• Don’t overstretch this simile (or is it an analogy?) – it is only a guide to how to read N & Z without causing “head pain”.
From that point on, you should be able follow their logic. I’ll leave the pleasure of that process of discovery for you to enjoy at your own pace (unless you think I can help further).
My (unasked-for) advice: Be very, very wary (dare I say, skeptical) of “thought experiments”!!! They have not a reliable substitute for REALITY.

PhilC
January 16, 2012 1:42 am

<>
The second law applies specifically to entropy, and ISTM entropy increases even if thermal equilibrium is disturbed. I did search for “entropy” in your post, I could only find it in the comments.
I did also supply a real physical example of energy transfer from a cooler to a hotter object – Sun’s surface to Corona. Here too entropy increases as thermal equilibrium is disturbed. The thermal energy of the Corona isn’t internally generated, it’s supplied by the Sun, AIUI as radiation which is converted to heat.
Phil

OzWizard
January 16, 2012 1:51 am

Ooops! Change, “They have not …” to, “They are not …” in the last sentence of my previous comment.

Myrrh
January 16, 2012 1:59 am

Whoa, back up. There is generally no special thermal radiation that occurs at phase changes. Why would a phase change cause thermal radiation(not the release of latent heat that simply warms the surrounding air, but thermal radiation)?
? The release of latent heat can be by conduction, convection or radiation – if by radiation then the latent heat IS thermal radiation, thermal infrared. Thermal infrared is heat energy, thermal energy, on the move.

OzWizard
January 16, 2012 2:12 am

Hello Bart (January 15, 2012 at 9:57 pm)
Your concept of ‘explaining’ a physical paradox mathematically – using the same ‘relativity’ theory which gives rise to it – does not resolve the paradox. You do agree that paradoxes cannot exist in REALITY, don’t you?

Bob Fernley-Jones
January 16, 2012 2:18 am

Willis Eschenbach @ January 15, 7:26 pm
I don’t know why I bother, but in response to my January 15, 6:28 pm & 6:35 pm, raising point 1), you wrote, with my bold added:

Bob, if I understand you, Dennis Wingo talked about this upthread, you might have missed it. I said:
Read the head post FIRST, Dennis, and then comment. See the multiple suns in the picture. Read the part about how the surface temperature is even everywhere?
So I [Willis] am in mystery as to what a hotspot on earth has to do with this discussion. We’re discussing an evenly heated planet, specifically to avoid all those ugly T^4 complications.

Oh Willis, I’m totally distressed, even after already smacking my wrist before this. It did not fully occur to me that your “thought experiment” of an ABSOLUTELY INCREDIBLY TOTALLY IMPOSSIBLE planet banned any evolutionary discussion that might extend into rationality in a more realistic model.
Zero value in raising my points 2) and beyond then?

Alexander Harvey
January 16, 2012 2:49 am

Hi Willis,
I am pleased that you apprecieted the thought (and time) that went into my post, as so I yours.
Where I clearly have an issue is with your point 3) I quote in full:
“3. Even if the energy were partitioned in such a manner, what would prevent it from re-partitioning itself in the normal manner? The molecules are constantly colliding with each other, and exchanging energy in a constant averaging process. What is it that you think would prevent the energy from moving from where it is concentrated and spreading itself out evenly? ”
I cannot see (in your model) how the DALR can be attained from the isothermal distribution without heat being transfered from a region of lower temperature to one of a higher temperature wihout doing work, due to the 2nd law. I do not see anyway to avoid this argument or anyway for your system to spontaneously perform the required work. I also fail to see how in the isothermal case I that convection can occur without work being applied to the system.
That said, I must make my strongest answer in your favour, The DALR seems to represent the profile with the lowest energy. To make what I say clear: concentrating the kinetic energy lower down seems to cause the atmosphere as a whole to contract lowering the centre of mass. I have given this some consideration and think that is correct, if speculative. If that be the case the question is which consideration wins out. Equipartion of the total energy (and my speculative minimisation of it) which is favoured by the DALR, or maximising the entropy which is favoured by the isothermal.
In general I can make more arguments favouring your case than mine but I am sure you can see my concern that anything suggesting an issue with the 2nd Law, if true, must trump other arguments.
There are puzzles where the increase in entropy wins out over energetic considerations such as why the CO2 doesn’t all sink to the bottom of the atmosphere or the case of evaporative cooling. If you or anyone know of a treatment that considers diffusion (and or conduction) in a fluid subject to a gravitational field that might settle things.
Now I am not going to push this because it is irrelevant to the thrust of this thread, as I noted preciously, and I am not absolutely certain. However precisely because it is irrelevant I might suggest that you consider not nailing colours to the mast over this unless you are very certain. Roy Spenser does support isothermal and I would advise that you take it up with him directly as he has more at stake in this than I do.
I am content to agree to disagree and make no further comment, but please feel free to do otherwise. I am not here to rain on your parade.
Best Wishes
Alex

Julian Braggins
January 16, 2012 2:56 am

On re-reading N&Z, I cannot say that I agree that with a completely transparent atmosphere the heat of compression, which is what they are saying is ATE, the alternative to GHE, would remain. The surface would radiate to S-B equilibrium temperature.
As all their examples have atmospheres that contain some GHG’s they are on shaky ground by stating that their ATE is independent of them, however closely the planets’ temperatures align with pressure calculations alone.
My view is that for work to be done by insolation there must be radiation to cool the upper atmosphere to complete the cycle. As with the hydrological cycle that enables Hydro- Electricity.
This does not negate their premise that pressure is the dominant factor in ATE/ GHE, just that it is not the sole one, and we see from Earths GHG’s, a very small proportion can change the dynamics of atmosphere substantially.

Brian Macker
January 16, 2012 2:58 am

Julian Braggins says:
January 16, 2012 at 12:42 am
Willis has proved his point, that under his specified conditions, multiple suns, no IR sensitivity of atmosphere, etc. the temperature would not exceed S-B level.
This does not disprove N&Z’s theory as these very conditions prohibit the mechanism of the gravity heat engine of atmospheres, convection.

No, convection isn’t prohibited. It just is not going to occur at equilibrium. If you were to perturb the temperature profile in any way that is not symmetric then convection would occur.

January 16, 2012 3:02 am

Willis said:
“Ah, I see the problem here. At equilibrium, the lowest level of the atmosphere is the exact same temperature as the surface. As a result, there is no flow of heat either into or out of the atmosphere.
The incoming solar energy flows into the surface and is radiated away again constantly. But no, the energy flow is not delayed or retained or slowed down by the atmosphere, because there is no heat flow between the atmosphere and the surface in either direction.”
That really is the problem so see my post at 1.34 am which anticipated that very point.
In fact there is a constant conductive energy exchange between surface and atmosphere as long as there is an atmosphere.
It is exactly the same process of delaying energy loss as is attributed to the radiative greenhouse effect but in this case it is caused by atmospheric density increasing collisional activity.
The molecules in the air don’t just stop dead in their tracks when the point of equilibrium is reached. That is what you are proposing.
For there to be a zero exchange the earlier process would have to be reversed. If you recall, the addition of the atmosphere first reduced the surface temperatue by diverting energy to conduction but over time the surface temperature recovered.
To get back to a zero exchange that process has to be reversed via a warming of the surface as the energy in the conductive process is recovered by the surface then a return back to the S – B temperature over time.
That reversal cannot take place while the atmosphere remains in place so there is a continuing conductive exchange between atmosphere and space and, finally, I think I can say that you have been wrong all along.

Myrrh
January 16, 2012 3:03 am

Willis Eschenbach says:
January 16, 2012 at 1:35 am
The incoming solar energy flows into the surface and is radiated away again constantly. But no, the energy flow is not delayed or retained or slowed down by the atmosphere, because there is no heat flow between the atmosphere and the surface in either direction.
Why isn’t there a delay? As the atmosphere slows down radiation direct from the Sun because the atmosphere has density. The oceans slow this down even more. Off hand I don’t recall the figure, 14 times as much or something, but anyway, even more than the atmosphere, because it is denser than the atmosphere. It’s still travelling damned fast, just not as fast as it would be travelling through a vacuum.

J. Radefahrt (Ger)
January 16, 2012 3:03 am

Would you please consider that a transparent atmosphere has no clouds at all and therefore no cloud albedo? So the assumed 240W/m² ISR can’t be correct I think.
Otherwise you have to consider the clouds for the OLR as well, what of course would make the atmosphere limited transparent.
Not to do so is, from my point of view, implausible.

Capo
January 16, 2012 3:13 am

Tallbloke:
“No Willis, before we can move on to a discussion of the science, you need to acknowledge that Hans Jelbring defined his model planet as one which does not radiate to space,…”
That means that Jelbrings model is not “based on first principles of physics” in contradiction to what Jelbring says. Every body with T>0 radiates, so where’s the sense in this assumption?
If this is one of the assumptions, the conclusion would not belong to real, but a phantasy world.
Next time you introduce a model planet without conservation of energy?

January 16, 2012 3:45 am

I read up to comment 137, then by the time I came back we were up to 832, so apologies if this has already been pointed out.
Here’s a thing in all these discussions about Greenhouse effects, and how much greenhouse gasses increase the temperature of the planet. How much warming would there be in a world that had an atmosphere, no greenhouse gasses, but with water covering 70% of the surface? How much heating would the presence of water in its different states, have?
Only after this has been ascertained, can we attribute any “extra” heating to greenhouse gasses. The current meme is that without GHG our planet would be 33K cooler, but that includes the effect that water has on the atmosphere. This should be stripped out, and then we can see what GHG’s really add. If the current estimate of 90% of the warming is attributable to water vapour is true, then that means that the prescence of GHGs are actually adding 3K, not 33K.
How much the water is adding is important, because if it is 95% only 1.5K can be said to be attributable to GHGs. Yet I hear of no paper that definitively states that water on its own adds X amount of heating.
This point needs to be hammered home. Lumping in water with the other GHGs is done to deceive people into thinking that GHGs (which most people think of as CO2 and methane) are alone responsible for all of the warming.

Alexander Harvey
January 16, 2012 3:46 am

I find several aspects of the Jelbring paper perplexing:
“[The globe] G and the atmosphere (AT) are surrounded by a concentric, tight, black spherical shell with a surface area (S).”
Why “black” and in what sense? Given the context one might infer that it means an emissivity of unity. No mention of whether the globe (G) is black is made.
“The energy content in the model atmosphere is fixed and constant since no energy can enter or leave the closed space. Nature will redistribute the contained atmospheric energy (using both convective and radiative processes) until each molecule, in an average sense, will have the same total energy. In this situation the atmosphere has reached energetic equilibrium. The crucial question is what temperature difference (GE) will exist between A and S?”
Here there is a clear indication that radiative processes are involved. From this I infer that along with the shell (S) at least one other component of the system is radiatively active. This seems to be problematic.
If the the shell (S) and the globe(G) both have non zero emissivities the difference between their temperatures must result in a net radiative flow from (G) to (S). Given that this temperature difference is meant to occur at the equillibrium state some mechanism must be transfering energy from (S) back to (G) up a temperature gradient which is problematic with respect to the 2nd Law.
On the other hand, if it is the atmosphere that is radiatively active then I must infer that it contains radiatively active gasses aka GHGs, which would be odd given the thrust of the paper. At any rate any radiative coupling inside the atmosphere would result in a net radiative flux in the direction of increasing height (lower temperatures). This is prone to the same 2nd Law problem as above.
Either (or both together) of these radiate couplings act to drive the system towards equalising temperatures and hence away from the DALR. This leads to the somewhat purplexing state of affairs that the GHE as described in the paper would be weakened by the addition of GHGs to the atmosphere.
I will note strongly that I do not see that the same issue would not true of the model used by Willis as his is not constrained by the total energenic isolation of the Jelbring model and GHGs would then radiate to deep space which tends to ensure adherence to the DALR.
Now this is pretty much by the bye as I do not see that Jelbring states the precise nature of the radiative processes that are mentioned in the quoted passege above. This is an issue as it leaves much for the readers to infer as best pleases them.
Alex

Brian Macker
January 16, 2012 3:53 am

Hail Core says:
January 14, 2012 at 1:40 am
I have talked with one engineer who is working infrared warmer manufacturer company. They have tried to warm air with infrared warmer and they have never managed to do so. They have even set tens of radiators to work at same time (hundreds kilowatts), no measurable results in air temperature. So the claims that infrared radiation warms atmosphere can’t be correct, so the main question is, what is absorption when you deal with gases (or air).

Why would anyone expect to get any significant amount of radiative heating of the amount of air one could fit in a heater. How could they possibly have isolated the much larger effect one would expect from conduction from the hot surfaces to the air? You obviously do not understand that the opacity to infrared of air increases with distance through the air, and that conduction efficiency depends upon proximity. If their chamber was the size of the earth the proportions would be quite different and most of the heating would be by radiation, not conduction with the sides of the chamber.
Negligible effect at a certain size compared to conduction at that scale does not equate to no effect.
To show yourself that opacity increases with distance try looking through a pane of glass as we normally do, then compare that with looking through one of the edges through the entire width. Aquarium glass appears clear when we look through the flat surface of around one quarter inch. Looking through a full eighteen inches edgewise and it is green. Same thing can be done with a glass of colored water. The deeper the water the darker the color.
The number of posters here who are ignorant of basic physics is astounding. Not only is it obvious you can’t understand the gibberish that Willis is criticizing in these obviously incorrect papers but you don’t understand how to basic everyday physics.

gbaikie
January 16, 2012 4:07 am

“The answer is that statement (1) is right and statement (2) wrong. Statement (2) is wrong because the conclusion in statement 2(a) is wrong. The seemingly paradoxical truth is that the average kinetic energy of all molecules does not decrease with height even though the kinetic energy of each individual molecule does decrease with height””
Hmm.
It’s not a paradox if you include buoyancy of air packets.
If you had sole a molecule on a gravity body as it bounced higher it would lose kinetic energy- and gain it back as it fell.
If didn’t lose kinetic energy- it would simply escape the gravity of planet [you would have no atmospheres anywhere].
If you include air packets and buoyancy then packets could gain altitude and not
“cost” velocity or kinetic energy- other packets or other molecules are supporting this buoyancy.
A boat isn’t using up energy by floating on the water, but for boat to float it does need the water to be there.
Air packets are somewhat similar to a bubble of air in water. particularly when you consider that water can have air dissolved in it. Though differences densities with air and water are greater- so it’s dissimilar in that respect and I guess also dissimilar because of the high surface tension of water.

Myrrh
January 16, 2012 4:23 am

Willis – I found something by Tyndall the other day which I’m a bit reluctant to introduce because there’s this strange idea around which claims that shortwave visible heats land and oceans …
Tyndall, who knew the difference between light and heat and did much work with heat which he knew was the invisible thermal infrared, made the observation that:

TYNDALL, JOHN.
On the Absorption and Radiation of Heat by Gases and Vapours, and on the Physical Connexion of Radiation, Absorption and Conduction
“The solar heat possesses… the power of crossing an atmosphere; but, when the heat is absorbed by the planet, it is so changed in quality that the rays emanating from the planet cannot get with the same freedom back into space. Thus, the atmosphere admits of the entrance of the solar heat, but checks its exit; and the result is a tendency to accumulate heat at the surface of the planet.” -Tyndall
What I think this is. The heat energy, thermal energy, direct from the Sun to Earth is powerful because it is uni-directional – it comes in at terrific speed direct from the Sun to Earth in straight lines, only taking 8 minutes to reach us. But, what heat energy, (thermal infrared, thermal radiation) gets emitted from a hot Earth’s surface is constrained by two things, coming from a lower temperature than the Sun it doesn’t come with the same power, it is ‘diffuse’ by contact with the atmosphere because the atmosphere diffuses it. It doesn’t have the same freedom to exit because it isn’t powerful enough against the density of the atmosphere.
What the actual mechanics of that are will involve knowledge of the optics of surfaces and reflection and transmission and that on a molecular/electron level. But, the Sun must then be radiating out an awful lot of heat energy if radiating 4times its temperature.
This thermal energy on the move direct from the Sun is also more powerful because of its size difference to visible light, the difference between non-thermal near infrared and thermal infrared is between microscopic and pin head size – that’s were the bigger punch, power, also comes from to get through the atmosphere to us. So that, coupled with the great unidirectional speed of greatly concentrated huge amount from much hotter source gives it the power to cross the atmosphere to Earth, but, that coming from a lower temperature (the Earth radiates out thermal energy around the same wavelength we do, around 10 microns) not having the same concentrated unidirectional greater amount from much greater heat source, is weaker, can’t travel as far through the atmosphere because the density is stopping it doing so. Size here hindering travel not aiding it as the greater surface of weak pin head size is going to have more pressure points on it from the atmosphere, a ton per square foot at the surface, and this will hinder its progress in exit, diffusing it.
So the “The solar heat possesses… the power of crossing an atmosphere; but, when the heat is absorbed by the planet, it is so changed in quality that the rays emanating from the planet cannot get with the same freedom back into space. Thus, the atmosphere admits of the entrance of the solar heat, but checks its exit; and the result is a tendency to accumulate heat at the surface of the planet.” -Tyndall
Is explained. The concentrated unidirectional powerful heat energy from the Sun must be similar to that of an incandescant light bulb in 4times? Aroung 95% of the energy radiating out from an incandescant bulb is thermal infrared, heat, the remaining 5% is visible light. Same sort of thing with heating steel.
A billet of steel: http://m.plantengineering.com/index.php?id=2831&tx_ttnews%5Btt_news%5D=33209&cHash=db4db9479b
“Not as widely recognized is the fact that incandescent objects emit a tremendous amount of invisible infrared radiation. For example, the radiance of a steel billet at 1500 F is 100,000 times greater in the infrared spectrum than in the visible spectrum.”
Also, (and really, to those reading this, the following can’t be understood if you think that the heat we feel from the Sun comes from visible light, it doesn’t, we can’t feel heat from visible light, the heat we feel from the Sun is thermal infrared – look up hydroponics and get a grip on why they take out thermal infrared from their lights to produce the visible required for photosynthesis without burning up the plants. To feed them with light energy required for the the chemical change to sugars, chemical energy isn’t creating heat, it’s creating sugars.)
So.
Do we have the figures right for the amount of thermal infrared, actual heat, reaching us from the Sun, as a percentage of heat to light in such objects suggests this is underestimated.

Martin A
January 16, 2012 4:41 am

… Are you issued a Taser in case some sexist starts getting obstreperous? Instead of “Police Constable”, does the “PC” on your funny hat mean “Politically Correct”?
Yes very funny indeed.

wayne
January 16, 2012 4:48 am

wayne: “jjthom, that is hugely appreciated and exactly at the right time. I sure needed that! Hope Willis and (others a bit confused) also read it.”
Willis: Listen, you unpleasant person. If you think I am confused, QUOTE MY WORDS that show my confusion. Your nasty throw-away insults do nothing but reveal the paucity of your knowledge and the depth of your self-doubt.
That is how it was meant. So please, don’t be unpleasant yourself. I thought you would like the book. It has some interesting statements on the question whether nitrogen radiates at all, not at all, or up in the air.

don penman
January 16, 2012 5:02 am

Just a thought ,I think the atmosphere would be stretched and compressed by the gravitational pull of the moon and the sun as the earth moves around its orbit and this would heat the atmosphere above its equilibrium temperature with the solid earth and we would not be able to assume that the atmosphere was warmed entirely by the solid earth.

JJThoms
January 16, 2012 5:07 am

wayne says: January 15, 2012 at 10:33 pm
jjthom, that is hugely appreciated and exactly at the right time. I sure needed that! Hope Willis and others a bit confused also read it. So… N2 does have rotational lines after all… just what I expected! Thanks again.
=======
You need to read the plots and text. O2 N2 etc have mainly rotational “oscillation” this means that the wavelengths absorbed and radiated are in the microwave frequencies. NOT IR e.g. O2 radiates at 2.5mm wavelength CO2 at 4.5 um and 16 um
If you look at the ppt for chapt6 slide 9 provides a satellite down and ground up spectrum. This shows that the satellite see reduction in level when the ground sees increase. Something in the air is turning these wavelengths from outbound to inbound.
This CAN ONLY be GHGs.
Gravity has NO term for wavelength filtering!
From Arnott’s blurb on gaseous IR transfer.
DANGER:
“Water vapor is a very strong ‘greenhouse’ gas. CO2 may very well change the distribution and amount of water vapor, so is both a greenhouse gas, and a lever arm that affects water vapor content. Recall that saturation vapor pressure of water vapor is an exponential function of temperature — a little temperature increase can be important for water vapor.
Looking at the effects of CO2 change in isolation of the bigger picture is dangerous.”

Tim Folkerts
January 16, 2012 5:50 am

There are a lot of interesting side-issues, but (as Willis has to keep repeating) they really don’t belong in this thread. Things like “does N2 emit/absorb IR?” or “what is the ‘dry adiabatic lapse rate’?” or “can photons go from cool places to warm places without violating the laws of thermodynamics”?
Without some sort of FAQ, these issues will get brought up over and over in any similar discussion by people who have not really thought through the concepts. The same fire has to be put out over and over. With an FAQ, Willis wouldn’t have to snip so many slightly off-topic comments, but could instead direct people to the appropriate place to learn and discuss those ideas.
Otherwise, we might as well start calling Willis “Sisyphus” (I am sure he is tarting to feel that way already). And with that, I think I will go work on other never-ending tasks and leave this thread.

Joel Shore
January 16, 2012 5:55 am

JIsbert says:

N&Z take Penn State’s model which has a one layer atmosphere. The “greenhouse effect” of this model is created by heating this layer to its emission temperature through radiative transfer from the surface and then have it radiate towards space and the surface.
….
Then they add convective heat transfer to the radiative transfer and take it from there. They correctly find that convection is far more efficient than radiation for transferring energy, which I believe has been mentioned before.

It is very important to understand how they add convection: They add it INCORRECTLY. They even tell you this when they say, “Equation (4) dramatically alters the solution to Eq. (3) by collapsing the difference between Ts, Ta and Te and virtually erasing the GHE (Fig. 3).” However, we know in the real world, convection does not drive the atmosphere to an isothermal temperature profile with height; it only drives it as far as the adiabatic lapse rate profile. This is because lapse rates less steep than the adiabatic one are stable and convection is suppressed.
Hence, what they have shown is that IF convection could drive the atmospheric temperature profile to an isothermal profile then the greenhouse effect would be erased. This is in fact well-known. In particular, Ray Pierrehumbert’s book “Principles of Planetary Climate” emphasizes the fact that the temperature at the effective radiating level has to be lower than the surface temperature in order for there to be a greenhouse effect.
So, in the real world, what convection does is to decrease the magnitude of the greenhouse effect (on Earth from something like 60 K, as I vaguely recall it would be in the absence of convection, to the actual ~33 K). However, it cannot eliminate it precisely because convection can only decrease the lapse rate so far.

Leonard Weinstein
January 16, 2012 5:59 am

Willis,
I really don’t mean to nitpick, but your last three statements in your elevator speech were:
• Some of that energy is absorbed by greenhouse gases (GHGs) in the atmosphere.
• In turn, some of that absorbed energy is radiated by the atmosphere back to the surface.
• As a result of absorbing that energy from the atmosphere, the surface is warmer than it would be in the absence of the GHGs.
This implies that back radiation results in the surface being warmer. The word “that” in the third of the three statements has to refer to the back radiation in the second of the three statements. That may not have been what you intended to say, but it is exactly your quote. I would agree with you completely if you had simply left the word “that” out of the last statement, or left the second of the three statements completely out (it is true but irrelevant and leads to a wrong emphasis). What my comment on the heater with insulation showed, was that the heated atmosphere is an effect, not cause. We can argue what words mean, but I am only using what you said.

Paul Bahlin
January 16, 2012 5:59 am

If you had a giant loss free fan mounted, say 5 km, above a black body under the influence of no external radiation such that it blew upwards, it would create a flow of gas (pick your own) that ascended, cooled (solely due to expansion and some due to radiation if you pick such a gas), descended and then heated, no?
So the energy input to the fan is first converted to potential energy going up, then kinetic energy on the way down, which is converted to heat as it compresses, then radiation as it is absorbed and emitted by the black body surface. Energy is conserved as all the energy into my loss free fan is converted to IR.
Is this what would happen in THIS thought experiment?

Joel Shore
January 16, 2012 6:12 am

Willis Eschenbach says:

This statement seems to imply that the lapse rate is a result of the loss of kinetic energy with height in a gravitational field. Surely the lapse rate is determined by the adiabatic expansion of the atmosphere and requires the motion of air parcels.

No, it’s the rate of loss of kinetic energy as potential energy increases, as your first statement says.

These two different interpretations of what the adiabatic lapse rate is due to have been bugging me for a while: Is it due to the work done by expansion of a parcel of gas as it moves upward or is it due to the change in gravitational potential energy…or are they two sides of the same coin?
I finally worked through it last night and the answer I got is that it is due to both. I.e., there are contributions due to both the fact that the gravitational potential energy decreases with height and the fact that the parcel of gas does work on the gas around it by expanding.

That’s why the dry adiabatic lapse rate contains the gravity term, because it depends on gravity, not on the motion of bulk air parcels. Certainly, if the initial condition of a column of dry air is not at equilibrium it will overturn until the temperature profile of the DALR is restored. But that bulk movement is not what creates the DALR.

The DALR is not an equilibrium condition. The condition for hydrostatic equilibrium is a different equation. (It determines the relationship between pressure and density with height.) The DALR is a condition for stability, i.e., lapse rates steeper than the adiabatic lapse rate are unstable to convection and lapse rates less steep than the adiabatic lapse rate are stable and convection is suppressed. This is, of course, why we see lapse rates less than the adiabatic lapse rate in parts of our atmosphere (such as the stratosphere).

Joules Verne
January 16, 2012 7:02 am

Stephen Wilde says:
January 16, 2012 at 3:02 am
“In fact there is a constant conductive energy exchange between surface and atmosphere as long as there is an atmosphere.”
No, Stephen. Willis happens to be right at least in the isolated case of saying there is no energy entering or leaving a transparent atmosphere when the temperature at the lowest level of the atmosphere is the same temperature as the surface.
The problem for Willis is that he doesn’t seem to understand that all matter with a temperature above absolute zero radiates. So that transparent atmosphere is constantly losing a small amount of energy to space via radiation and this is replaced by conduction from the surface.
And it is that conduction from the surface which establishes the temperature gradient from surface to space. Gravity can only raise the temperature while it is reducing the volume of a gas. The volume of the atmosphere is not being reduced. Gravity and pressure are in equilibrium. What gravity does is create a pressure gradient and a temperature gradient follows it. It cannot raise the surface temperature above the S-B temperature. It just redistributes the heat capacity making it greater at the surface and lesser at TOA but has no effect at all on total energy in the column. Nikolov et al seem to believe gravity somehow increases the total energy in the column and it simply does no such thing.

Andy West
January 16, 2012 7:16 am

hi Willis, you snipped my post of 5.06am on 14th, for not beig an elevator pitch, when in fact it is a 3 sentence pitch on the temperature profile of your ideal planet. It may be whollly wrong, but WUWT? Sorry i didn`t notice earlier, i am travelling in scandanavia and nly accessing by kindle. I have no dog in the fight, but just liked your challengeenough to have a go.

Bill Hunter
January 16, 2012 7:21 am

Capo says:
January 16, 2012 at 3:13 am
“Tallbloke:
“No Willis, before we can move on to a discussion of the science, you need to acknowledge that Hans Jelbring defined his model planet as one which does not radiate to space,…”
That means that Jelbrings model is not “based on first principles of physics” in contradiction to what Jelbring says. Every body with T>0 radiates, so where’s the sense in this assumption?
If this is one of the assumptions, the conclusion would not belong to real, but a phantasy world.”
The outer shell of Jelbring’s object has T=0, . . . .I think. What is going on inside of the object is an extension of the hypothesis that every body with T>0 radiates. The deal here could be there is some level of opaqueness of any gas whether it be full frequency or partial frequency may not matter. I hypothesized above a GHG-free atmosphere and found it to be warmer than the average radiation the surface of the planet receives. . . .not unlike the greenhouse formula suggests. But a GHG-free atmosphere may be impossible and so-called greenhouse gases may not matter and what you see from space is simply just a mix of the signature of different frequencies of transparency so that any “shell” TOA with a T>0 will radiate in unique ways.
Your comment I think is good but there is no evidence it is correct.
It seems a lot of stuff is colder on its outer shell than it is internally and it radiates according to its outershell and not at the temperature of the nuclear fusion thingy temperature in its core. Is that a violation of first principle physics?

January 16, 2012 7:31 am

I don’t agree Joules.
Willis doesn’t realise that all matter vibrates with kinetic energy if it is above absolute zero so that there can never be a zero exchange between surface and atmosphere. There can be a zero NET exchange but his view of the consequences of that is incorrect.
Even in a non GHG atmosphere there is a constant exchange of conductive energy between the surface and atmosphere.
N & Z do not say gravity is the direct cause. They say gravity is involved but the source of the heat is additional collisional activity when more densely packed molecules are irradiated under pressure.

steveta_uk
January 16, 2012 7:38 am

January 16, 2012 at 1:34 am
Has everyone here been treating radiative and conductive energy as interchangeable ?

The surface therefore has to rise to a higher equilibrium temperature to sustain both processes but the S – B equation remains valid because it is blind to any energy engaged in the separate conductive energy exchange. There will still be 240W/m2 coming in and 240W/m2 going out as regards radiative energy but the surface will be at a temperature that supplies both the radiative 240/m2 going out AND its part of the energy exchange with the atmosphere at 150W/m2. It has to be so because if the surface supplies less than 150W/m2 to the air on an ongoing basis there will be no equilibrium. It matters not that the surface gets that 150W/m2 straight back again. That is the nature of an equilibrium./150W

Wow – I’d completely failed to understand that there are two kinds of temperature – that the part that follows the S-B law IS JUST THE RADIATIVE PART OF THE TEMPERATURE! Meanwhile, the CONDUCTIVE part of the temperature doesn’t do any radiating at all!
I’d strongly suggest you do some reading on physics and dimensional analysis, and try to understand the different between energy and force, because you are seriously confused about this stuff, and keep repeating exactly the same errors in the odd belief that somehow we’re going to end up understanding you.

wayne
January 16, 2012 7:44 am

JJThoms, thanks for your viewpoint. I was not looking for IR, just ANY radiation from this post’s non-GHG atmosphere and a reference and that chaper and anna v provided.

Hail Core
January 16, 2012 7:46 am

Is this correct? http://tallbloke.wordpress.com/2010/07/27/nasif-nahle-nails-the-radiative-physics-of-co2/ Considering the data obtained by many researchers on this matter, the total emissivity of the carbon dioxide is low. It is 0.0017.
This value is very important for calculating the amount of energy that the carbon dioxide absorbs and emits each second. Given the specific heat capacity of the carbon dioxide at its current density and temperature, which is of the order of ~871 J/Kg K, the carbon dioxide is not the cause of any change of the Earth’s climate.

January 16, 2012 7:56 am

“What two forces are acting for particles in any orbit? I only count one — gravity!”
Plus escape energy makes two.
In fact ANY energy in a molecule causes movement that counts as a second force interacting with gravity.
Are you getting lost in semantic detail and losing the bigger picture ?

No, I’m not getting lost at all.
Newton’s Second Law is (popular version):
F = ma = m dv/dt
On the left hand, one requires a second agent (Newton’s Third Law) acting on the mass m, exerting an actual force of nature. There are four known forces of nature — gravity, the weak nuclear force, electromagnetism, and the strong nuclear force. The nuclear forces are very short range and completely irrelevant to the cooling or heating save insofar as they conspired to create a heavy charged nucleus for atoms. Gravity is enormously weak but is ubiquitous — it is the force that binds big collections of mass together, planets and stars and galaxies and clouds of interstellar gas. Note well that it requires an entire planet to create a force acting on us that a thin surface layer of electromagnetic forces in the molecules of the chair I’m sitting in (further structured by quantum theory and the Pauli exclusion principle) to oppose it so that I don’t fall down towards the center. Basically we “are” electromagnetism — electromagnetic forces mediate the binding of electrons to nuclei to make atoms, bind atoms together to make molecules, bind molecules together to make us, and along the way mediate light and sound and thought, not to mention the computers and network that make us part of a collective consensual superorganism as we exchange information.
Note well that there are four forces and “escape energy” is not one of them. In fact, calling an energy a force is the kind of mistake that I spend a whole lot of time bopping undergraduates upside the head for. They are not the same. They don’t even have the same units. Conflating the two leads one only to error.
Note that the error is a deep error, a really serious error. It isn’t just a matter of speaking differently. It is a fundamental lack of understanding. You cannot add or subtract an energy and a force. It is, quite literally, akin to adding a length to a volume, or a length to a time. Imagine a carpenter trying to add three seconds to the length of a one meter board section he is trying to cut. What the hell does that mean?
Energy arises from taking Newton’s Second Law — the dynamical principle for one of the versions of classical physics — and making it time independent. The calculus is:
F = m dv/dt = m dv/dx dx/dt (chain rule) = m v dv/dx (because v = dx/dt).
Thus
F dx = m v dv
and integrating between matching end points in the units of both sides:
\Delta W = \int F dx = \int m v dv = \Delta 1/2 m v^2 = \Delta K
That is, “The work done by the total force acting on an object equals the change in its kinetic energy”, where the latter is defined as K = 1/2 m v^2.
This is the “Work-Kinetic Energy Theorem” in one dimension. With a bit of work (all of which you can read and follow in my online book) you can make this three dimensional. If one applies it to “special” forces (where the laws of nature are in general special) such that the work done between two points by the force is independent of path, such forces are called conservative forces and one can transform the WKE theorem into the Law of Conservation of Mechanical Energy by defining the potential energy to be the negative of the work done by a conservative force. For Newton’s Law of Gravitation
F_r = – GM m/r^2
and Coulomb’s Law (the electrostatic force law, part of Maxwell’s Equations) one can do the integrals for the potential energy once and for all:
U = – G M m /r
for gravity (for example).
Consider, now, a particle in orbit. Only gravitational forces act on it. It has some total mechanical energy (at any instant in time) that is constant:
E = – G M_1 m/r + 1/2 m v^2
In general, for any bound orbit, E < 0. It is a bit involved to transform the kinetic energy into a rotational part plus a radial part, because the latter is generally written in terms of the angular momentum which is a constant of the motion for radial forces (which exert no torque), but it is in my book if you want to learn how. As one adds energy to an orbiting particle (by doing work on it with another force of nature) with a constant angular momentum, one makes the orbit increasingly eccentric, moving from circular to elliptical to parabolic — the first orbit that can actually escape from any radius to infinity.
The condition for the parabolic orbit is:
E(escape) = – G M1 m/r + 1/2 mv^2 = 0
In this orbit, one can find the particle at infinity where U is zero with zero velocity, consistently. The actual trajectory is in fact a parabola, an “infinitely stretched ellipse” in a certain sense. Energies greater than zero lead to hyperbolic orbits and escape even faster. Note that all of the allowed orbits are conic sections — this was a marvellous geometric consequence of the 1/r^2 force law (and is experimentally encoded in Kepler’s Laws). All of this stuff actually drove the Enlightenment and is a wonderful story of human achievement and risk and transformation from a primitive myth-bound magical culture to the scientific and technological society we enjoy today.
From this fairly complete algebraic exposition, you can see that under no circumstances can one call “Escape Energy” a force, or argue that it has anything at all to do with a particle escaping. A particle escapes if the orbit it happens to be in happens to have enough energy to escape, when the only force acting is gravitation and when its trajectory happens to not intersect other objects along the way. There is no “balance” involved. In a radial reduction one can speak of a “balance” between the angular momentum barrier L^2/2mr^2 (which is not a potential energy or force, but which behaves like one in the reduced coordinates of the problem) but you would have to work pretty hard to learn enough to understand that, and in the end it changes nothing because it is not a force, it is just the angular part of the kinetic energy.
This explains 1) in my previous reply in complete detail. To explain 2) is a bit beyond my vacation-day means, as deriving the Maxwell-Boltzmann equation is a pain in the ass and requires that you know a lot of things that you clearly don’t know. I am therefore going to take the lazy way out and just post a link here:
http://en.wikipedia.org/wiki/Maxwell%E2%80%93Boltzmann_distribution
You will note that the key element in its derivation is the Boltzmann distribution. This distribution describes the maximum entropy/maximum ignorance assignment of probabilities of finding a system in any given state with any given energy, as a function of temperature, and in some sense defines the temperature (although there are other definitions, all consistent with this one). You will also please pay careful attention to the figure associated with “distribution of speed”.
As noted above, escape energy is E = 0. For r = R_E and M1 = M_E (the surface of the Earth) solving the escape energy condition yields a speed of around 11.2 km/sec. Objects launched with this speed in a vacuum will go away and never come back. If one goes over to the right (imagine the figure extending out to 15 km/sec on the right) one can draw a line at 11.2 km/sec. All molecules in the MB distribution of temperatures with a speed greater than this line that happen to be on a trajectory that gets “out of the atmosphere” without a collision with another molecule go away. This is how the Earth loses atmosphere, a continuously happening process. There are always molecules that collide just right to have sufficient velocity to escape.
The same general process, BTW, is one way of understanding how the cooling of gases outside of their resonant channels can occur. At any finite temperature some of the collisions of molecules in the gas have enough energy to excite an electron from the ground state to where it spontaneously emits a photon. That photon can then escape, removing a quantum of energy. Even though the average energy of collision may be far too weak to excite an electron, there are always collisions in the MB tails going on, and they slowly bleed energy out of the system.
Now, are other forces involved in getting molecules out of the atmosphere? Sure. Collisions between molecules that keep them in rough local thermal equilibrium. These are basically short range electromagnetic interactions from a classical point of view, although the short range repulsion involved gets a big contribution from Pauli and quantum properties of electrons as fermions. Molecules absorb and emit photons via their electromagnetic coupling with the ambient electromagnetic field, which may or may not be describable by a “temperature” at all (in general not, actually — blackbody radiation is the “Maxwell-Boltzmann” curve for electromagnetic energy or a “gas” of photons, and the radiation field near the Earth pretty much never has the right spectrum and is never in equilibrium.
The point, however, is you have to learn a bunch of stuff before you can discuss this problem intelligently where intelligently is just “not using terms in completely wrong, conceptually wrong, ways in discussion”. Ultimately, of course, one wants to have worked through all of the derivations of and relations between the important equations, and worked on it until one understands the whole thing conceptually. Otherwise you will be inventing forces that aren’t, claiming processes that violate the laws of thermodynamics or laws of nature or both, elevating irrelevant processes to importance and leaving out important processes as irrelevant.
One last point, to Anna, if she is still reading. “Transparent” was the term I was addressing. A solid object is not transparent where it emits, it is opaque. An object does not absorb at all where it is transparent. I do believe that you probably understand this, but you are saying it completely wrong, and in a way that is bound to lead to substantial confusion in a discussion of the radiative properties of atmosphere that is indeed transparent in some bands and opaque in others.
There is little to no thermal radiation from a layer of atmosphere that is optically “thin”, that is to say, transparent, in some band of frequencies. The thermal radiation comes from a layer that is optically thick, thick enough to be opaque. So you should be very careful in how you use the term “transparent” when discussing radiation. Willis’ point is that a transparent atmosphere does not directly act to enable cooling. It has to be able to thermally radiate energy and thermal radiation does not (significantly) occur where the emissivity/absorptivity is low (transparent), it occurs where it is high (opaque). This is what confounds elevator summaries of N&Z (at least to me, still, yet) — you cannot have convection heating something on the bottom unless you also remove the heat at the top. The only way to actually remove the heat at the top AFAICT is radiation, and blackbody radiation from the cold top of an opaque atmospheric layer is the greenhouse effect. If you don’t cool up there, you never generate the masses of cooler denser air that displace the warmer less dense air underneath to create the buoyancy force or “lift” that drives convection. Convection requires a hot reservoir (where heat is added) and a cold reservoir (where it is removed) — it cannot occur between two adiabatic energy trapping surfaces.
In fact, the second law of thermodynamics says precisely this same thing. You cannot get work out of a system (including convective work) any more efficiently than you can get it out of an ideal heat engine running between two thermal reservoirs. If those reservoirs are at the same temperature, the ideal efficiency is zero and no steady state motion is possible. One of many, many reasons that denying that the so-called and misnamed “Greenhouse Gas Effect” exists at all is enormously silly. That really is being a “denier”; denying that the laws of nature and the laws of probability are correct is the act of a crank, not a serious person, unless and until they are prepared to present a sufficiently complete and consistent theory to replace, well, all of physics. And even that won’t help you with the laws of statistics that underlie statistical mechanics and the second law.
The logic supporting the GHE as a critical aspect of the overall non-equilibrium processes that describe energy flow into “the Earth” and thence “out of the Earth” is impeccable; based on simple physics and directly supported by the observed spectrum of radiation from the Earth. Denying that it occurs, given the measurements, is literally denying the evidence of our own eyes (in space).
What is at issue is not “The atmospheric effect is a key component of energy flow that determines the warming of the Earth relative to an identical Earth without an atmosphere or with a perfectly transparent dry atmosphere”. That is simply a true statement. What is at issue are things like how this effect is functional on the underlying parameters, both in oversimplified models and in the actual chaotic, heat transport laden, albedo altering, inhomogeneous processes that describe our real atmosphere as opposed to an idealized one dimensional model atmosphere. In particular the feedback/sensitivity and role of the Sun, in particular the relative sizes of natural oscillations compared to responses to increased CO_2.
And even this isn’t an adequate general explanation — understanding the albedo of clouds as a problem in multiple scattering being a good example. Even if all the multiple scattering collisions with photons are elastic and the cloud itself absorbs no energy (other than trapping optical energy in caged motion) as one makes the cloud thicker and thicker, less and less radiation comes through because the random walk of an enemble of bouncing photons carries more and more of them back to the surface (where they become part of the diffuse reflected light. The thicker the cloud, the darker underneath and the whiter on top even without any actual heating of the cloud. And of course, clouds do warm in the sun. The air itself warms in the sun. Everything gets warm in the sun. It’s one of the things that makes clouds so interesting and such a wild card in cooling models.

January 16, 2012 8:05 am

Joe, I saw your remark in passing, but the thread is now too long for me to even catch up on in less than a day (and I can’t even FIND it in a quick pass through), and there are a million remarks along the way I would like to make about Bad Physics in other places. I only saw the first paragraph in passing looking for ANOTHER reply I wanted to comment on.
If I recall correctly what you were starting to say — and I probably am not — the way I describe the atmospheric effect completely omits any reference to upwelling or downwelling radiation, and is completely independent of the details of how heat gets in. Given an influx of heat energy that must be lost to keep a planet in “dynamic equilibrium”, the integral of the outgoing flux has to equal dQ/dt in, on average. If the flux is in a BB spectrum at temperature T, you have the ideal case. If it is split up into surface and atmosphere (by any heat-transfer mechanisms) and emitted at a cooler temperature for part of the frequency spectrum, this reduces the net flux UNLESS the temperature (and hence flux) increases at other frequencies. If the surface is warmer than the atmosphere this warms the surface. If it is cooler, it cools the surface (it works the other way, in other words, just as well — I don’t care WHICH of the two bands is the warmer or cooler one.
The details of transfer are irrelevant to the effect and its connection to the measured emission spectrum.
rgb

steveta_uk
January 16, 2012 8:07 am

At last! I’ve found an error in something from Robert Brown!
He said “one can find the particle at infinity where U is zero with zero velocity”.
Of course, this is wrong – if one of your particles has gone to infinity (or beyond) there’s no way you’re going to find it again!

JimOfCP
January 16, 2012 8:18 am

I gave this analogy at Tallbloke’s site and have expounded a bit here:
If you turn on an air compressor and let the tank fill, the tank gets hot. This compression is no different from compression due to gravity. But if you wait a while, the tank cools off. The air pressure is somewhat less after cooling, but still considerably higher than one atmosphere. If one allowed air to accumulate on an airless Earth in, say, the period of a week; the air at the bottom would likewise get hot. But like the compressor tank, it would radiate away the heat, cooling in the process.
Once the air inside the compressor has cooled, the molecules in the compressor will be hitting each other more frequently than the air outside it also. Yet, the compressor and the higher pressure air in it are the same temperature as the air outside the compressor. Also, the air in the compressor is more dense than the air outside it, just as the air in the lower atmosphere is denser than the air above it.
Anything happening at the atmosphere/Earth interface should also be happening on the inner surface of the compressor.

January 16, 2012 8:22 am

Thanks to Robert Brown for the effort put into that. I will pay close attention to it but in my mind the issue has moved on a bit because I can now see that the effect of gravity is not to cause any heating in itself (except a miniscule portion) but rather to reorder mass so that other processes generate heat energy.
In the current context it is not gravity directly that heats the non GHG atmosphere but the extra collisional activity that occurs when mass is irradiated whilst under pressure.Gravity only sets up the scenario in which that can happen.
Then the important issue would seem to be the role of conduction as compared to the roile of radiation.
This whole issue is a multidisciplinary one and no non specialist can go into every component as deeply as has already been done by many here within their own specialisms.

January 16, 2012 8:29 am

“Wow – I’d completely failed to understand that there are two kinds of temperature – that the part that follows the S-B law IS JUST THE RADIATIVE PART OF THE TEMPERATURE! Meanwhile, the CONDUCTIVE part of the temperature doesn’t do any radiating at all!”
I think you did the jumbling up there.There is no radiative part of the temperature or conductive part of the temperature. That would be nonsense and I did not say it.
There are two kinds of energy transfer for current purposes. Radiation and conduction. The temperature is the result of the two processes combined.Thus radiation and convection can each contribute seperately to the final temperature.
It is the mass in a non GHG atmosphere that doesn’t do any radiating (except perhaps a trace that can be ignored for present purposes).
If energy is held by mass that doesn’t radiate then it can only be transferred by conduction. Convection may occur later but only after the energy has been conducted from surface to non GHG molecules.

January 16, 2012 8:51 am

“Once the air inside the compressor has cooled, the molecules in the compressor will be hitting each other more frequently than the air outside it also. Yet, the compressor and the higher pressure air in it are the same temperature as the air outside the compressor. Also, the air in the compressor is more dense than the air outside it, just as the air in the lower atmosphere is denser than the air above it.”
You now need to irradiate with an energy source such as the sun. The denser mass should gain a higher temperature for the same energy input.The greater density and more collisions allows that mass to hold on to more of the solar irradiation in kinetic form hence the higher temperature.

steveta_uk
January 16, 2012 9:10 am

“The denser mass should gain a higher temperature for the same energy input.”
Sorry, again you’ve either got your dimensions wrong, and don’t mean “energy”, or else this is simply inverted. Same energy applied to higher mass results in a smaller temperature increase.
Exactly like heating two pints of water in a kettle for one minute, compared to heating one pint for one minute in the same kettle. The greater mass will increase in temperature less. Density is irrelevant.

Crispin in Waterloo
January 16, 2012 9:40 am

Willis: You wrote in reply to Crosspatch that he must repeat [and keep in mind] a GHG-free atmosphere. It seems to me there is a basic gap in the concept. A gas that does not (meaningfully) intercept/absorb LWIR does no mean it cannot emit such radiation or that it will not cool be emissions of electromagnetic radiation at all. In other words the definition of ‘GHG’ seems to be based on its ability to efficiently absorb certain wavelengths but not all, and that it primarily emits in the same range. It is an unreasonable definition for our purposes. There are materials that do not emit IR as efficiently, or shall we say ‘preferentially’, as well as other materials. But emit they do. So right from the start there is a problem conceptually about the emission of energy in the form of photons, reducing temperature being limited.
Surely this is true: all objects emit IR if they are heated enough. Is it correct to say there is such a thing as an atmosphere that will not emit any IR? I find that unbelievable. Not absorb IR, that is quite different, if you picked carefully.
Create a black planet in space, heat it from within. Give it a very thick atmosphere of gases that have no meaningful absorption of IR. The atmosphere will insulate the plane to some extent, or is it more true to say that if that atmosphere does not absorb IR it will not insulate the planet? When you want to put a gas between the panes of a double-glazed window, do you put in an IR absorbing gas, or on that does not absorb IR at all? The purpose is insulation so do we put in CO2 or Argon? We use Argon because it is such a lousy transmitter of heat. Conclusion, a non-GHG atmosphere can be a good insulator. The base of the atmosphere will be hot from conduction. The temperature will not be S-B in equilibrium state.
The heat conduction coefficient of air or argon varies with density (really, viscosity) so gravity has an effect. http://www.boulder.nist.gov/div838/theory/refprop/NAO.PDF see formula 4.
The insulation is more efficient for any given planet if there is a) more if it, and b) if it is made of gases that are poor thermal conductors that are also dense (argon being one example). Gravity (leading to gas density) matters. That paper explains how density (and viscosity) alter the thermal characteristics of a gas. The GHG-free atmosphere that does not insulate does not exist because it would also have to have no ability to conduct heat. Even Hydrogen has some heat conduction capacity. Convection is a heat transport mechanism that merely complicates the understanding.

steveta_uk
January 16, 2012 9:54 am

“The GHG-free atmosphere that does not insulate does not exist because it would also have to have no ability to conduct heat.”
Conduct to where? If this is a non-radiating (or neglibably radiating to be picky) gas, then being conductive is irrelevant, as the is nowhere for the heat to be conducted to. Above is free space, non-conductive, below is a radiating surface.

hotrod (Larry L)
January 16, 2012 9:55 am

The conditions imposed by Willis:
ie. an atmosphere that has no GHG, is inherently impossible in a real world. In the context that he is defining such an atmosphere as an atmosphere that cannot radiate ANY energy in the IR, he is also implicitly not allowing radiation of any form of energy in any other part of the radiation spectrum. This is not possible in a real atmosphere.
Therefore the proof is also impossible.
He is ignoring that gases can lose energy by radiating it at other frequencies. It may have to go through complex transition trains to result in emissions in the visible, microwave, UV, or xray or some other higher frequency but eventually the energy will be radiated away to space. If nothing else real atmospheres contain dust and other solids in the form of particles so small that they do not settle out thanks to constant mixing from weather. Even if the atmosphere was composed of a single gas that in and of itself could not radiate energy at any frequency (microwave to IR) than the entrained dust in that gaseous atmosphere would still radiate away energy in the IR when it was mixed high in the atmosphere.
Like engineering problems that introduce impossible limitations like “infinitely rigid”, or “without mass”, the model might be useful to isolate a single concept but it does not and will not accurately represent the real world physics of the actual object.
In a real atmosphere the temperature profile is determined by the lapse rate. That lapse rate will determine the temperature of the actual radiation surface of the planet, which due to convection will be far above the physical surface. That radiation surface may or may not consist of gas molecules, since real atmospheres contain entrained dust and other complex molecules that do radiate energy to space.
Larry

kwik
January 16, 2012 9:58 am

Okay, I repeated my Elevator speech, with Willis’ comments, and my answers to Willis.
IMO it was not snarky.
I had “kwik says”, “willis says” and “kwik answers” so one could see the sequence of interchange.
Willis snipped the whole thing, so I will leave the discussion.

January 16, 2012 10:11 am

“Same energy applied to higher mass results in a smaller temperature increase”
And if the higher mass is pressurised as compared to the lower mass ?

Bart
January 16, 2012 10:13 am

Willis Eschenbach says:
January 16, 2012 at 1:22 am
“Am I alone in finding your assumption patronizing, sexist, condescending, and unworthy of Martin A?”
No, you aren’t. But, the puritanical streak runs deep in America, and resurfaces periodically to leech the fun out of everything.
Paul Dennis says:
January 16, 2012 at 1:22 am
“The answer is that statement (1) is right and statement (2) wrong. Statement (2) is wrong because the conclusion in statement 2(a) is wrong. “
The answer is that statement (1) is wrong because it is impossible for a spherical geometry. You are claiming there is no lapse rate. I do not think you will find many people who agree.
“The seemingly paradoxical truth is that the average kinetic energy of all molecules does not decrease with height even though the kinetic energy of each individual molecule does decrease with height”
Cannot at the same rate and satisfy the fact that there is no source and no sink in the atmosphere (Laplacian = 0). You must take the geometry into account.
OzWizard says:
January 16, 2012 at 2:12 am
“You do agree that paradoxes cannot exist in REALITY, don’t you?”
There is no paradox. The twin who accelerated away slowed his rate of time relative to the twin he accelerated away from. When he decelerated to get back, he matched his rate of time back to the twin on the ground, but by then, his twin was already older. That’s all there is to it. This is no conjecture. The experimental confirmation of the effect is mountainous.
Stephen Wilde says:
January 16, 2012 at 3:02 am
“That reversal cannot take place while the atmosphere remains in place so there is a continuing conductive exchange between atmosphere and space and, finally, I think I can say that you have been wrong all along.”
It is a dynamic equilibrium. And, it has to be taken account of in the “free body” decomposition. Willis refuses to recognize that, so he is indeed wrong. The only question is, how significantly wrong is he?
Joules Verne says:
January 16, 2012 at 7:02 am
“No, Stephen. Willis happens to be right at least in the isolated case of saying there is no energy entering or leaving a transparent atmosphere when the temperature at the lowest level of the atmosphere is the same temperature as the surface.”
No, Joules, Stephen is right here. There is an interface, and a balance which exists as long as the surface and atmosphere is in contact. Take away the atmosphere, and there must be a reaction.
“It cannot raise the surface temperature above the S-B temperature.”
You are once again repeating the mistake of reversing Kirchoff’s law. SB does not limit temperature, it limits radiation.

Bart
January 16, 2012 10:18 am

steveta_uk says:
January 16, 2012 at 9:10 am
Sorry, again you’ve either got your dimensions wrong, and don’t mean “energy”, or else this is simply inverted. Same energy applied to higher mass results in a smaller temperature increase.”
Try: The denser mass should gain a higher temperature for the same energy flux input. Eventually, the flux balances, but not before it has left behind a deeper deposit of energy.

don penman
January 16, 2012 10:26 am

steveta_uk
It is not the same energy being given to the denser atmosphere and the less denser atmosphere as you say but it is a constant flow of energy that is being given to both by the Sun or the surface,in the end the denser atmosphere will have more energy than the less dense atmosphere.

January 16, 2012 10:45 am

Right Willis, you agree that there is a dynamic equilibrium between surface and atmosphere so that there is a continuing conductive energy exchange between surface and atmosphere at equilibrium.
Previously I think you said that once equilibrium was reached all such exchanges would stop.
Now lets take the next step.
The atmosphere takes energy from the surface via conduction to its maximum capability. What N & Z seem to be suggesting is that such capability is set by pressure and density so as to allow greater collisional activity at greater densities.
Once the atmosphere has taken what it can then that energy is removed from the in/out radiative flux and has to be made up via a cooler surface radiating less out until the S – B balance is restored.
But meanwhile the energy in the conductive exchange between surface and atmosphere hasn’t gone away. All that energy is still present in the air above the surface and for there to be a dynamic equilibrium the surface has to give up as much conductive energy to the air as it receives from the air.
So the surface has to be warm enough to supply BOTH the outgoing radiation to satisfy S – B as regards the radiative energy exchange AND the upward conductive energy to balance what is coming back down from the air.
That sounds like it requires a higher surface temperature than one would expect from the S – B equations.
Or not ? Over to you.

January 16, 2012 10:50 am

Whoops, Bart has reformatted his post and now I see that it is he who supports the dynamic equilibrium point rather than you, Willis.
Anyhow despite that see if you can still follow my logic.

Bart
January 16, 2012 10:50 am

Bart says:
January 16, 2012 at 10:18 am
steveta_uk says:
January 16, 2012 at 9:10 am
“Try: The denser mass should gain a higher temperature for the same energy flux input. Eventually, the flux balances, but not before it has left behind a deeper deposit of energy.”
Let’s try again: The denser mass should gain more heat energy for the same energy flux input. Eventually, the flux balances, but not before it has left behind a deeper deposit of energy. This creates a different boundary condition at the surface/atmosphere interface than would otherwise be the case.
I haven’t read the full exchange, so I’m not taking sides. Merely trying to facilitate the discussion.

Martin A
January 16, 2012 10:52 am

Bart says:
January 16, 2012 at 10:13 am
Willis Eschenbach says:
January 16, 2012 at 1:22 am
“Am I alone in finding your assumption patronizing, sexist, condescending, and unworthy of Martin A?”
No, you aren’t. But, the puritanical streak runs deep in America, and resurfaces periodically to leech the fun out of everything.
============================
It is the standard response of a bully when challenged on their bullying to reply:
– It was just a joke.
– The person raised no objection, so what is it to you?
– *If* what I said caused offense, then *you have my apologies*
A real apology is along the lines of “I am sorry; I should not have said that.”

Bart
January 16, 2012 11:02 am

Well, I’ve got to get back to making a living. It is distressing that one camp never wants to take into account the boundary conditions. In every solution to PDEs, boundary conditions are key. It amazes me that so few get it. You really have to wonder what they are teaching in the schools these days.
But, it is futile for me to keep harping on it. If I haven’t swayed any minds yet, I’m not going to. Finis.

shawnhet
January 16, 2012 11:03 am

Willis Eschenbach says:
January 15, 2012 at 10:55 pm
“Shawn, thanks for your thoughts. As you say, despite the fact that I said what I was going to snip, and that I was justified in the snipping, there’s no way to avoid the appearance of censorship.
And since that is one of the several points that I was trying to make by snipping off-topic posts, you are the winner. For your prize, you get to go explain that to Tallbloke.”
Yes, I totally agree. I did not twig that this was part of a larger strategy though.
“Can’t make folks follow requests, that’s for sure. I requested elevator speeches on the Jelbring or N&Z hypotheses, or falsifications of my proof. I tried to snip posts that were off-topic or didn’t answer my request. All people wanted to do was something else, anything else. It’s like herding cats. No, like herding feral cats. No amount of snippage would help. But I suspected that going in. I figured to snip the worst ones pour encourage les autres. But the number of worst ones was overwhelming.”
Personally, I think that the reason that no one can provide simple explanations of how this sort of effect works is that no one has such an explanation(Sorry to be blunt folks, but there it is). From a scientific POV, that’s really the end of it. Maybe at some point, someone will be able to explain this supposed effect, at which point, we can revisit the idea but until then, there’s not much that can be done.
“I figure if I can establish a tradition of judging people’s understanding of a topic by whether they can explain the theory in a clear, concise manner … then who cares if I’m popular? I’m tired of vague handwaving. Boil it down to the elevator speech, then boil it again to half that size, and give us the simplest, clearest explanation possible.”
I agree here completely. this is always a good goal to strive for, but especially, when communicating on the internet where everyone has a different background and breadth of knowledge. I have a hard time not chuckling when someone says I could explain something, but because you are too rude/have already made up your mind/are a member of a vast conspiracy.
Cheers, 🙂

Jim Petrie
January 16, 2012 11:03 am

The entire warmist case depends on positive feedback – on some mysterious tipping point which will trigger a climate catastrophe.
But the immediate feedback is negative – a small rise in temperature gives a big rise in radiative heat loss.
Increasing CO2 increases the rate of plant growth, so CO2 levels will eventually stabilize at a higher level than today – but at a level which is still entirely safe.
Increased water vapor increases clouds. This by and large will have a cooling effect.
Positive feedback is a dangerous and expensive myth

shawnhet
January 16, 2012 11:13 am

Bart says:
January 16, 2012 at 12:10 am
“”The main one being: how can the surface of a planet such as Earth emit more energy than the planet as a whole emits to space?”
And, when did you stop beating your wife?
You have to show it is necessary that more energy be emitted by the planet for non-GHG heating to work before discounting the idea on that basis. SB does not demonstrate it. SB is not a fundamental law – it has loopholes. And, the widespread neglect of these loopholes is just plain sloppy.”
Actually, it is fine with me you want to argue that SB does not apply (for whatever reason). My question is based on what we can observe (ie that the surface emits on average ~390W/m2 while the planet as a whole emits ~240W/m2). You can either dispute that observation or try and explain it. SB on its own appears to explain the radiation of the planet as a whole, but not the radiation at the surface. The currently constituted GHE explains both.
If you have another theory, that’s fine, but I can’t read your mind. I don’t understand how you think it would work at all.
Cheers, 🙂

Phil.
January 16, 2012 11:20 am

Paul Dennis says:
January 14, 2012 at 11:27 pm
Your redcutio ad absurbum argument is elegant and shows conclusively that in there can be no gravitational effect. End of as far as I’m concerned.

Agreed as I said above.
What is an interesting physics question is in your model what will the temperature distribution in the non-GHG atmosphere? I think it might be constant throughout with no convection and therefore no adiabatic lapse rate. The atmosphere in this model is effectively a gas column in an adiabatically bound state, heat conduction throughout the column will lead to an equilibrium of constant temperature distribution. I think Roy Spencer may have indicated this to be the case as well in his earlier comment.
I disagree because the system will be unstable to any perturbation so you’ll end up with the dry lapse rate.
Thank you for an elegant disproof of gravitational theorems.
Agreed.

January 16, 2012 11:22 am

Well, Willis, that was a bit over the top given that I’ve already explained that I made an error in attribution due to a change in the formatting of Bart’s post.
I think that your outburst shows a very nasty underlying attitude to the extent that I don’t want to engage with you any further.

JimOfCP
January 16, 2012 11:27 am

Stephen Wilde says:
January 16, 2012 at 8:51 am
“You now need to irradiate with an energy source such as the sun. The denser mass should gain a higher temperature for the same energy input.The greater density and more collisions allows that mass to hold on to more of the solar irradiation in kinetic form hence the higher temperature.”
First, unless the wall of the compressor is at absolute zero, it is radiating in the infrared.
Second, you have to specify the gas. Nitrogen and oxygen are not dipolar and therefore do not interact with infrared radiation. CO2 and H2O are polar and do interact.
So assuming the air inside and outside the compressor wall contain the same % of CO2, the air inside and outside will have the same temperature. Otherwise, the inside would be perpetually hotter than the outside and you could get work done for free, a clear violation of thermodynamics.
http://www.elmhurst.edu/~chm/vchembook/globalwarmA5.html

January 16, 2012 11:31 am

“First, unless the wall of the compressor is at absolute zero, it is radiating in the infrared.”
Have you never heard of conduction ?
This thread is a total waste of my time.

Joel Shore
January 16, 2012 11:33 am

I said:

I.e., there are contributions due to both the fact that the gravitational potential energy decreases with height and the fact that the parcel of gas does work on the gas around it by expanding.

Of course, that sentence should have said that the gravitational potential energy INCREASES with height.

shawnhet
January 16, 2012 11:38 am

gbaikie says:
January 16, 2012 at 12:37 am
““The main one being: how can the surface of a planet such as Earth emit more energy than the planet as a whole emits to space?”
So since earth is colder it will need to emit more photons to equal the energy.
Every grain of sand is emitting photons. Does every atom in rock emit photons?
I would guess, since there more joules of energy on earth then is delivered by the Sun in a second [or nanosecond] and any thing isn’t mostly facing the sky and gases are suppose absorb and emit some of these photons going towards space and things are warming and cooling, that is how.”
I’m sorry to say that I do not understand your post. My question was referring to the fact that the Earth as a whole emits ~240W/m2 while the surface emits~390W/m2. From the POV of my question, it doesn’t matter whether the 390w/m2 comes from the sun or a giant bonfire or gravitational collapse of the planet or microscopic amounts of pressure induced nuclear fusion or whatever. In the case of a Nitrogen atmosphere if the surface emits 390W/m2, then the planet as a whole should based on current theory emit 390W/m2. Since this is not what we observe in the real world, we can either explain what is different about the actual atmosphere compared to a nitrogen one of the same mass and pressure or we can explain how a nitrogen atmosphere would actually produce the difference in emission btw the surface and the planet as a whole. If we can’t do either, then we do not have a useful hypothesis IMO.
Cheers, 🙂

January 16, 2012 12:06 pm

“or we can explain how a nitrogen atmosphere would actually produce the difference in emission btw the surface and the planet as a whole.”
By storing solar energy in kinetic form (originally acquired by conduction from the surface) and then engaging in a dynamic conductive energy interchange with the surface.
So 240Wm2 comes to the surface from the sun and the surface radiates out 240W/m2 to space to achieve radiative balance.
Meanwhile the surface conducts 150W/m2 to the atmosphere and receives the same in return to achieve balance in the conductive energy exchange.
So the surface actually receives 240 from the sun + 150 from the atmosphere = 390
And radiates 240 to space plus conducting 150 to the atmosphere = 390
So there we have the answer and all in a non GHG environment.

Phil.
January 16, 2012 12:06 pm

Bill Hunter says:
January 16, 2012 at 7:21 am
Capo says:
January 16, 2012 at 3:13 am
“Tallbloke:
“No Willis, before we can move on to a discussion of the science, you need to acknowledge that Hans Jelbring defined his model planet as one which does not radiate to space,…”
That means that Jelbrings model is not “based on first principles of physics” in contradiction to what Jelbring says. Every body with T>0 radiates, so where’s the sense in this assumption?
If this is one of the assumptions, the conclusion would not belong to real, but a phantasy world.”

Jelbring’s model is nonsense which bears no relationship to reality of any kind.
His model involves a planet surrounded by an atmosphere which is in turn contained by a non-transparent shell at a density altitude of 0.1atm.
Neither the shell nor the planet is allowed to radiate, J seems to believe that this means the shell is black. Also the shell and planet surface have negligible thermal mass.
The atmosphere is heated by the planet by conduction and convection and so achieves the dry lapse rate between the planet and the shell. (Presumably the planet has an internal source of heat).
He defines this lapse rate as the ‘Greenhouse effect’ and because this is dependent on g therefore the real and totally unrelated Greenhouse effect must be too! The guy is completely out to lunch, he believes that no physical laws are violated in the construction of this model!

Crispin in Waterloo
January 16, 2012 12:10 pm

I was going to add a comment here that given the possibility that the atmosphere moves to an equilibrium state faster than the components of it change, it can be viewed as being in a quasistatic state. There are arguments that it is always in a state of non-equlibrium but I think that is simplistic. Bejan argues that it is self-regulating, always on the verge of being chaotic, because there is water in the atmosphere. My view is it is always self-stabilising to the point that it is quasistatic.
But, an important point was raised (or missed) when steveta_uk says:
January 16, 2012 at 9:54 am
>“The GHG-free atmosphere that does not insulate does not exist because it would also have to have no ability to conduct heat.”
>Conduct to where? If this is a non-radiating (or neglibably radiating to be picky) gas, then being conductive is irrelevant, as the is nowhere for the heat to be conducted to. Above is free space, non-conductive, below is a radiating surface.
This misses what I perhaps could have emphasized more: just because a gas ignores passing IR radiation does not mean it cannot be heated by conduction, and ALSO it does not mean it will not emit photons (which was called ‘negligably radiating’). In other words, a planet with no IR absorbing gases (by definition, all are non-GHG’s), still emits IR from those gases and energy at other wavelengths. You cannot find a gas that will not radiate any energy at all at any frequency no matter how high the temperature. Thus GHG’s have to be defined as those which do not absorb IR.
At some stage of excitement (input of energy) all gases will emit energy by radiation in some form.
As a simple example, take a polished silver globe and heat it. In an inert atmosphere it will not oxidise and the colour will remain the same; the emissivity remains pretty constant too. As the temperature rises, the wavelength range of emissions changes until the thing is glowing bright white at visible frequencies. IR is not the only way to emit energy. IR is not a ‘special kind of radiation’ it is just an arbitrary frequency range, a portion of the whole spectrum. A GHG-free atmosphere will still emit IR and other frequencies from the molecules in the atmosphere if it is first warmed by conduction from the heated black surface. Because there is some value of insulation provided by all gases, there being no superconducting room temperature gases, the black surface of our mental picture planet will heat the lower atmosphere. That heat will accumulate until either the temperature rise of the surface is hot enough to vent through IR alone or the system goes into radiative equlibrium with the atmosphere serving as a portion of whole radiative source. The latter is the only possible (real) solution.
Part of the problem for many contributors is confusing heat with temperature. The black body surface does not increase immediately to ‘full temperature” until the conductive, non-GHG atmosphere has been heated first. By then, the atmosphere has started radiating energy in all directions, just like GHG’s, some of it IR. To repeat, just because a gas does not absorb IR radiation does not mean it will not emit IR and other radiation.

Phil.
January 16, 2012 12:26 pm

Stephen Wilde says:
January 15, 2012 at 7:29 pm
Willis, just seen your comments which suggest more clarity needed on my part so here goes again:
1) Willis knows that his non GHG atmosphere will produce a dry adiabatic lapse rate with the warmest temperature at the surface.I am assuming some movement via convection to achieve it.
2) The warmth at the surface is NOT due to gravitational compression ( although a tiny fraction of it would be) but gravity IS responsible for placing the maximum density of non GHG molecules at the base of the atmospheric column.
3) That maximimum density causes the greatest number of molecular collisions to occur just above, or in contact with, the surface.
4) Such collisions transfer energy between themselves by conduction and not radiation. The surface converts incoming solar shortwave radiative energy into kinetic energy and the non GHG gases in contact with or in close proximity to the surface retain that kinetic energy by exchanging the energy via conduction between themselves and between themselves and the surface until it can be released upward by the surface as outgoing longwave IR.

No you have this wrong, the gas molecules transfer energy both to and from the surface by collisions, at thermodynamic equilibrium the net transfer is 0 and the surface temperature is constant as is the radiational exchange with space (equal to the radiation in the absence of the atmosphere).
5) It is that delay that allows the surface temperature to rise as energy accumulates within the system and most particularly at or just above the surface.
Such a delay is the time required to reach equilibrium.
6) The greater the density of the non GHG atmosphere, the more molecular collisions occur, the longer the delay and the higher the equilibrium temperature must become.
More collisions occur transferring energy in both directions, as we know for a gas obeying the ideal gas laws the thermal conductivity of gases is independent of pressure!
The increased density does NOT slow down the rate of conduction. Instead it increases the proportion of fast moving radiation that is retained for longer as slower moving conduction for an average net reduction in energy flow through the system.
This is frankly gobbledygook, and bears no resemblance to any physics I am aware of!

January 16, 2012 12:29 pm

While we’re working on our elevator pitches, I would love to see one that describes the step-by-step linkages of how the atmosphere contributes 33C to the Earth’s surface average temperature. Let’s not just say blocked, trapped and/or stored. Blocked by what? How? Stored by what? How? Stored by what? How?

kbray in california
January 16, 2012 12:39 pm

This thread reminds me of the “thought experiment” in which you give a group of monkeys some typewriters, paper, and infinite time…. eventually one of them types something meaningful.
Snipping the chaff makes 800+ posts a lot easier to wade through.
Some people here need to take a banana break.

Phil.
January 16, 2012 12:43 pm

anna v says:
January 15, 2012 at 8:45 pm
Willis Eschenbach says:
January 15, 2012 at 11:13 am
There is no matter that does not radiate electromagnetically when in bulk.
The radiation may be small, in the low part of the electromagnetic spectrum not measurable easily, but it is there.
One has to realize that all, ALL , observable interactions of matter that affect every day temperatures are finally, at the last contact, electromagnetic. Electromagnetic means photons as a bottom line.
Argon has an emission spectrum too.

Yes indeed it does, an electronic transition, you apparently didn’t read your source carefully Anna, the spectrum is a discharge spectrum which required the passage of an electric current through the Argon. In a planetary atmosphere this would require a permanent electrical discharge through the atmosphere, there is no absorption of visible or IR light between those energy levels in the atmosphere because the excited electronic states are not populated in that case.

Phil.
January 16, 2012 12:47 pm

Crispin in Waterloo says:
January 16, 2012 at 12:10 pm
To repeat, just because a gas does not absorb IR radiation does not mean it will not emit IR and other radiation.

Repetition of a fallacy doesn’t make it true, the Physical Law that contradicts your statement is known as Kirchoff’s law. Emissivity at a wavelength is equal to the absorptivity at that wavelength.

January 16, 2012 12:51 pm

“Your not quoting me in this particular instance is not an “error”. An error is spilling milk. We all do that.”
If you check my earlier contributions you will see that I always (or almost always) complied with your request to be quoted.
That was the first time that I can recall when I failed to do so.
Your behaviour is unacceptable.

shawnhet
January 16, 2012 12:59 pm

Stephen Wilde says:
January 16, 2012 at 12:06 pm
“So 240Wm2 comes to the surface from the sun and the surface radiates out 240W/m2 to space to achieve radiative balance.
Meanwhile the surface conducts 150W/m2 to the atmosphere and receives the same in return to achieve balance in the conductive energy exchange.
So the surface actually receives 240 from the sun + 150 from the atmosphere = 390
And radiates 240 to space plus conducting 150 to the atmosphere = 390″
My first question here is if the surface receives a total of 390W/m2, how and why does it only emit only 240 to space? IOW, how do you know that a nitrogen atmosphere will donate the 150W/m2 to the surface? How do you calculate that amount to be 150W/m2?
Secondly, does your answer change at all if we assume that their are some GH gases in the atmosphere?
Cheers, 🙂

Bill Hunter
January 16, 2012 1:01 pm

Phil. says:
January 16, 2012 at 12:06 pm
“Jelbring’s model is nonsense which bears no relationship to reality of any kind.
His model involves a planet surrounded by an atmosphere which is in turn contained by a non-transparent shell at a density altitude of 0.1atm.
Neither the shell nor the planet is allowed to radiate, J seems to believe that this means the shell is black. Also the shell and planet surface have negligible thermal mass.
The atmosphere is heated by the planet by conduction and convection and so achieves the dry lapse rate between the planet and the shell. (Presumably the planet has an internal source of heat).
He defines this lapse rate as the ‘Greenhouse effect’ and because this is dependent on g therefore the real and totally unrelated Greenhouse effect must be too! The guy is completely out to lunch, he believes that no physical laws are violated in the construction of this model!”
I guess it is correct he believes no physical laws are violated. Couldn’t the internal source of heat be of the sort that heats the gas giants before they ignite from getting too big and therefore too hot? Dr Brown here described such a process if I read him right.
Seems off the top to be related to the argument about whether the atmosphere would be isothermal or isentropic. Clearly convection would cease once it became isentropic at that point you would have no conduction because the kinetic energy of every molecule would be the same. So you are kind of stuck with a temperature gradient that doesn’t conduct. Kind of a unique property of gases and a mushy concept of temperature.
I guess the only way to make Jelbring to go away is really energetic arm waving so as to avoid headaches.

JimOfCP
January 16, 2012 1:11 pm

Stephen Wilde says:
Have you never heard of conduction ?
This thread is a total waste of my time.
Stephen, the gas in the compressor, the compressor wall (a metal) is a great conductor, and the gas outside the compressor all conduct. I never said energy was exchanged ONLY by radiative mechanisms. There is no reason to throw a hissy fit.

Bart
January 16, 2012 1:27 pm

shawnhet says:
January 16, 2012 at 11:13 am
“My question is based on what we can observe (ie that the surface emits on average ~390W/m2 while the planet as a whole emits ~240W/m2). “
We do not, in fact, observe that at all. We observe temperature at the surface (and, I haven’t even pointed out that, that temperature is actually not of the surface, but of the air a little above the surface). The 390 W/m^2 figure is a derived quantity from SB which, as I have been explaining over and over and over and over and over and over and over and over, is not dispositive.

January 16, 2012 1:42 pm

“some wacky position of yours”
Such as the equilibrium between surface and atmosphere being dynamic ?
The more wacky proposal is that it is not dynamic such that once equilibrium is reached there is no further energy exchange. No net exchange maybe but no exchange at all ? Hardly.
The thing is that you depend on it being non dynamic for your proposition to hold. If it is non dynamic then the conductive part of the energy exchange can be ignored, the numbers no longer balance and you can claim that a non GHG atmosphere doesn’t have a warming effect similar to the radiative greenhouse effect.
If it is indeed dynamic then the numbers do balance and your proposition fails. No wonder I make you emotional.
As regards Robert Brown I have no problem with him schooling me. The issue there was my inability to match his terminology. As an experienced professional his use of terms is highly specialised such that my words in a more colloquial form were not adequate to get my meaning across.
In the end it didn’t matter because it turned out that the issues he focused on were not directly relevant to the wider scenario.Orbital mechanics is not a critical component of climate science.

January 16, 2012 1:44 pm

JimOfCP says:
January 16, 2012 at 1:11 pm
Sorry, Jim. It was directed at Willis rather than you.

Bart
January 16, 2012 1:46 pm

Crispin in Waterloo says:
January 16, 2012 at 12:10 pm
“To repeat, just because a gas does not absorb IR radiation does not mean it will not emit IR and other radiation.”
You should also add that higher frequency emissions radiate far greater energy per photon. An ultraviolet photon carries three orders of magnitude more energy than an IR photon.

Bart
January 16, 2012 1:48 pm

“An ultraviolet photon carries three orders of magnitude more energy than an IR photon.”
And, on that note, here.

Bart
January 16, 2012 1:55 pm

And, so, stream of consciousness here, we must consider that Willis’ fictitious planet, which he has artificially constrained from normal dissipative mechanisms, is likely roiled by lightening storms, which puts a real kink into all the hypotheticals.

January 16, 2012 1:56 pm

“IOW, how do you know that a nitrogen atmosphere will donate the 150W/m2 to the surface? How do you calculate that amount to be 150W/m2?”
I was setting up an example that fits Earthly figures just to show that it is possible.
Also Willis’s model assumed equilibrium so it has to be a dynamic exchange between surface and atmosphere otherwise a disequilibrium would be introduced. It would not then be possible to account for the difference between energy in and the temperature of the surface (which is the situation Willis would prefer because it suits his proposition).
In my view for an equilibrium situation radiative in must balance radiative out and conductive up must balance conductive down with no leakage either way.
All that must follow from basic physics so it beats me why Willis thinks anything I’ve proposed violates accepted physics.
“Secondly, does your answer change at all if we assume that their are some GH gases in the atmosphere?”
Yes it does because GHGs allow radiation out of the atmosphere which Willis’s model does not. Whether the net effect of GHGs is cooling or warming is contentious in some quarters. For reasons I have stated elsewhere I think the effect is neutral but they do introduce a change in the rate of energy flow through the system which does have a minor climate consequence.

Bart
January 16, 2012 2:00 pm

…lightning…

January 16, 2012 2:01 pm

“The 390 W/m^2 figure is a derived quantity from SB which, as I have been explaining over and over and over and over and over and over and over and over, is not dispositive”
Yes, that is how I understood it.
In reality the figures for the radiative and conductive energy transfers in a non GHG model could be different from the ones I used but they would have to balance in the way I described.

shawnhet
January 16, 2012 2:01 pm

Bart:”We do not, in fact, observe that at all. We observe temperature at the surface (and, I haven’t even pointed out that, that temperature is actually not of the surface, but of the air a little above the surface). The 390 W/m^2 figure is a derived quantity from SB which, as I have been explaining over and over and over and over and over and over and over and over, is not dispositive.”
Ok, let’s unpack this a bit. Is it your contention that an infrared camera (ie a camera that forms an image using infrared radiation) will show the same picture of a balloon at the surface(after cooling to ambient) temperature as of that same balloon will at a height of 3km?
Cheers, 🙂

January 16, 2012 2:08 pm

Bart says:
January 16, 2012 at 2:00 pm
…lightning…
Possible 🙂
More likely a miniscule shift in the surface pressure distribution moving the permanent climate zones latitudinally by a mile or so. But don’t get me started on all that or Willis will shout at me again.

Martin A
January 16, 2012 2:18 pm

Willis Eschenbach says:
January 16, 2012 at 11:17 am
One more thing: (…) You are one sick puppy, bro’, much sicker than we can help you with here. Go see a professional, your obsession with righting imagined offenses suffered by people you’ve never met must be painful to live with.
w.
======================
Gosh! I seem to have touched a nerve somewhere!

shawnhet
January 16, 2012 2:23 pm

Stephen Wilde says:
January 16, 2012 at 1:56 pm
““IOW, how do you know that a nitrogen atmosphere will donate the 150W/m2 to the surface? How do you calculate that amount to be 150W/m2?”
I was setting up an example that fits Earthly figures just to show that it is possible.”
I understand this, but the fact that something may be *possible* does not at all establish that it actually takes place. Respectfully, to be engaging in science requires that you give sufficient detail to be able to tell *by using measurements of the real world* whether or not something that *might be* happening *actually is* happening.
If you are at all interested in convincing others to your POV, you need to focus on using real world measurements to demonstrate your point. In theory, absolutely anything may be possible, but only a small fraction of those possibilties can actually be uniquely demonstrated as being true using measurements of the real world.
Cheers, 🙂

Bart
January 16, 2012 2:26 pm

shawnhet says:
January 16, 2012 at 2:01 pm
“Is it your contention that an infrared camera (ie a camera that forms an image using infrared radiation) will show the same picture of a balloon at the surface(after cooling to ambient) temperature as of that same balloon will at a height of 3km?”
No, that is not my contention at all. Actually, I have no contention, merely a question, viz: How do we know the full spectrum and radiation pattern of surface emissions, so that we may arrive at a truly full accounting of emitted power?
I have pointed out two ways in which I can see offhand that estimates of surface emissivity could be corrupted: 1) measurement of the partial spectrum and bootstrap estimation of the full spectrum using SB 2) spatial narrowing of the emissions pattern from surface roughness and absorptive obstacles.
To nail this down, we would need to take carefully controlled, full spectrum measurements across a truly representative sample of surface conditions with a narrow FOV sensor at multiple angular offsets to the surface normal. I am not aware of such data being available. If you are, by all means, share it.

January 16, 2012 2:32 pm

“I understand this, but the fact that something may be *possible* does not at all establish that it actually takes place”
That is a two edged assertion.
This is a discussion blog, not an engagement in science despite the presence of some fine specialists.
Given that the physical principles that I am expounding on are pretty basic I think it is for those who support an alarmist position to do the proving.
I’d like to see established evidence for Willis’s non dynamic equilibrium with zero energy exchange between surface and atmosphere once equilibrium temperature is reached.
Without that his proposition is dead in the water in my humble opinion.

Bart
January 16, 2012 2:33 pm

shawnhet says:
January 16, 2012 at 2:23 pm
“Respectfully, to be engaging in science requires that you give sufficient detail to be able to tell *by using measurements of the real world* whether or not something that *might be* happening *actually is* happening.”
What you are saying is that scientists in general have no obligation to consider tracking down leads which might tend to detract from their favored hypothesis unless evidence is presented fully formed and airtight by another party. Furthermore, that lack of utterly compelling evidence so presented otherwise elevates their hypothesis to fact.
This is the antithesis of the scientific method. A true scientist puts considerable effort into attempting to falsify his own hypothesis, and only calls it fact when he has exhausted all reasonable efforts at doing so.

January 16, 2012 2:36 pm

Here’s something interesting to consider…the top of Mount Everest gets more incoming solar radiation than, pick a place, the Mojave Desert? I guess there’s less CO2 radiation broadcasting to the top of that mountain, so it’s colder.

Trick
January 16, 2012 2:46 pm

If Higley7 (way) above at 1/13 10:27 et. al. after that didn’t include the required add-in elevator speech for transparent GHG-free (same as non-GHG) atmosphere thermal conduction, then I will (simple but may possibly be simpler & hope this effectively formats):
The non-GHG Pressure-induced Thermal Enhancement (PET) works as follows:
• The surface of a planet has a temperature from Standard Planetary Gray Body energy.
• Some of that energy conducts to non-GHG atmosphere adjusting to equilibrium temperature with near-surface non-GHG atmosphere above the equilibrium temperature at TOA.
• Due to planet’s permanent gravity retaining & pressurizing any non-GHG atmosphere, its kinetic energy at surface is permanently higher than at surface of reduced pressure&volume atmosphere or no atmosphere SPBG.
• As a result of permanently exchanging kinetic energy from the near-surface conductive non-GHG atmosphere, planet’s surface would be permanently warmer than it would be in the complete absence of atmosphere. This works with earth & many near earth planets.
Willis’ quote:
“Now suppose we add an atmosphere to the (theoretical S-B temperature) planet, a transparent GHG-free atmosphere. If the theories of N&K and Jelbring are correct, the temperature of the planet will rise.
But when the temperature of a perfect blackbody planet rises … the surface radiation of that planet must rise as well.
And because the atmosphere is transparent, this means that the planet is radiating to space more energy than it receives. This is an obvious violation of conservation of energy, so any theories proposing such a warming must be incorrect.”
No mention of thermal conduction in those words. As others have written in this mega-thread, Willis’ surface radiation proof is too simple, it violates conservation of energy test as it is incomplete w/o thermal conduction from non-GHG PET which can resolve the conservation of energy (2nd Law) conundrum.
The non-GHG atmosphere SPGB surface will not radiate to space more energy than it receives since it will be cooled just enough by thermal conduction and thermal radiation to equilibrium temp. higher than theoretical S-B temperature due to the PET added surface KE of non-GHG atmosphere & gravity. Non-GHG TOA can achieve equilibrium temperature to space from thermal radiation.
Adding the GHGs for a real Earth atmosphere means we can now add Kirchoff’s specific frequency: emit, absorb. And the IPCC. And all the math needed to describe how many Kelvins thermal conduction and thermal radiation each add (by different, independent & yet to be blogged on physics) to avg. Earth temp. above SPBG only. Now do your thing. Oh, BTW:
——————————————————
Genghis says: 1/13 11:14PM
“The non greenhouse gas atmosphere is a perfect conduction insulator to space, it can’t radiate its heat out.
This doesn’t make sense. I would have written:
“The non greenhouse gas atmosphere is a perfect conduction insulator to space, it can’t conduct its heat out.”
The non-GHG TOA is matter and has a temperature. Therefore it can radiate (or absorb) heat to obtain temp. equilibrium with ~3K space as all matter emits thermal radiation above 0K.

George E. Smith;
January 16, 2012 2:57 pm

“””””
Tim Folkerts says:
January 14, 2012 at 8:05 pm
Ed Fix mistakenly thinks: “Both, however, emit their own S-B radiation in a spectrum determined only by their temperature.”
Sorry Ed, but the ability to emit IR is the same as the ability to absorb IR. If this were not true, it would be easy to violate the laws of thermodynamics. Google “Kirchhoff’s law of thermal radiation” (for example http://en.wikipedia.org/wiki/Kirchhoff%27s_law_of_thermal_radiation) for more info. “””””
Tim,
Since YOU cited this Wiki reference to refute Ed Fix’s position; perhaps you would consider READING the wiki reference that you gave, and then make use of what they say, in your argument.
I just read YOUR wiki reference, purely out of curiosity; not necessity, and even your reference says that Kirchoff’s law requires equality off emission and absorption, WHEN THE SYSTEM IS IN EQUILIBRIUM. That means that the material in the system is in thermal equilibrium at a single Temperature, and in equilibrium with the radiation field.
Earth’s atmosphere is NEVER in thermal equilibrium, nor is it in equilibrium with the EM radiation field. The oceans are continually absorbing electromagnetic radiation, nearly all EM radiation that falls on it in fact. Yet the oceans are NOT a source of white light with a spectrum matching the incoming sunlight.
Problem is that the oceans are not in a state of equilibrium with the radiation field.
Kirchoff’s Law relating to emission and absorption HAS NO APPLICABILITY TO ANY PART OF ATMOSPHERIC PHYSICS, INCLUDING CLIMATE.
This thread is interminably bogged down with “discussions” and I hesitate to use that term, of matters that are unrelated in any way to the beginning subject matter, which apparently is some gravitational effect on climate.
As for the “Stefan-Boltzmann Equation or Formula”, that everyone is bandying around, like popcorn; they are talking about the STEFAN-BOLTZMANN CONSTANT, which is a fundamental CONSTANT of Physics; not something to be “junked” as Myrrh insists NASA did.
Moreover, it has a precisely known value:-
S-B Const, sigma = (pi)^2k^4 / 60h^3c^2 = 5.67051E-8 W / m^2K^4
It can also be obtained as the definite integral of the Planck Radiation Law for the Spectral Radiant Emittance of a BLACK BODY, from zero to infinity wavelength.
L = (2hc^2.lambda^-5) / (exp (hc/lambda.kT) -1)
A BLACK BODY is a non-existent ideal theoretical object, that has no defined properties other than it absorbs any and all electromagnetic radiation that falls on it.
The law can also be stated in terms of frequency as :-
L = (2hf^3c^-2) / (exp (hf/kT) – 1)
In particular, a black body has no specified atomic or molecular structure of any kind, nor any quantized energy levels of electrons or anything else. It is entirely a construct of the Statistical Mechanics of Classical Physics. No quantum Mechanics is required in the derivation of the Planck Law. The derivation is based on the application of the equipartition principle; not to the kinetic degrees of freedom of particles, as was used to investigate the heat capacity and specific heats of gases and solids; but rather an equipartition of energy among all of the frequencies of a continuum spectrum (not quantized frequencies)
Planck simply required that the energy assigned to any frequency be an integral number of “quanta” of energy at that frequency, where the size of the quantum is E = h f.
So the energy at some frequency is some integral number of times the quantum of energy AT THAT FREQUENCY.
This meant that at higher frequencies, the number of quanta at that frequency must diminish, compared to the number of quanta at some lower frequency, that complied with the (roughly) equipartition of the total energy among all frequencies.
The earlier attempts by Sir James Jeans, and Lord Raleigh, had allowed ANY (equipartitioned) continuous value of energy to exist at any frequency, which resulted in the energy at high frequencies increasing without limit.
So Planck did not specify the basic physical mechanism by which electromagnetic radiation could be emitted; just the statistical mechanics of it. It was the work of Heinrich Hertz and James Clark Maxwell that showed how electromagnetic waves could be propagated from a finite length conductor carrying a VARYING electric current.
Simplest way to think of it is to think of the electric and magnetic fields around the conductor rising and falling in sympathy with the increasing, and decreasing currents.If you think of it as the field expanding and contracting, which is not quite accurate, but easier to understand, then one can appreciate, that the velocity of that expansion and contraction is limited (to the velocity of light), so if you try to vary the current too rapidly or make the path longer, the fields end up not being able to collapse back completely on the wire (antenna), and a closed loop is formed in the field, which can then propagate away (at c ). It’s not unlike how soap bubbles disconnect from your soapy wire , when you move it faster than the velocity of sound in the medium, can move the soap film.
Well that’s probably simple enough for the lay folks to follow, and the PhDs can just live with it, or offer their own version.
It helps somewhat, if one is a Ham Radio fan (I’m not) or if one studied Radio-Physics at some University. (one of MY majors).
In any case, the varying electric current flowing over a non zero distance (antenna length) is all that is required to radiate EM waves. The physical antenna is not necessary, if you can simply have a flow of electric charge. And it is trivial to see that a varying electric current is identical to the acceleration of electric charge. Anna v was not quite accurate in saying you need charges moving in some ambient electric field; a free electron that is accelerating will radiate. Now I will give Anna back her gold star in that pedantically, you can’t have an accelerating electric charge UNLESS you have an electric field to kick it along (or slow it down)or you have a magnetic field to steer it around in a circle, which is a continuous centripetal acceleration.
Now you see, Anna v is a particle Physicist, who understands accelerators; so she knows that stuff, and it was nice to see her chime into some of the messes up above.
So the long and the short of it is that gases can and do absorb and emit a thermal continuum radiation spectrum, that is spectrally governed (spectrum envelope)by the Planck formula, and the direct source of that emission is the acceleration of the electric charges in atoms or molecules THAT OCCURS WHILE THEY ARE IN COLLISIONS WITH EACH OTHER.
Those collisions are a direct consequence of the TEMPERATURE. It is the collisions that are the reason for the radiation not the kinetic energies of the particles. A single molecule or atom in free flight and not colliding with anything emits no radiation; but then absent COLLISIONS it also has NO TEMPERATURE.
Now at ordinary atmosphere Temperatures and pressures, the velocities are modest, and the collision rates are too, so the amount of energy radiated per collision is not much per molecule.
Phil has several times referred to the relative importance of such collision induced emissions (from gases) and the molecular resonance lines and bands of the GHG molecules, that have a sizeable electric dipole moment when disturbed from their geometrically symmetrical ground state for CO2 for example. H2O of course has a permanent dipole moment, which is why it is THE premier green house gas.

Joel Shore
January 16, 2012 3:00 pm

Ken Coffman says:

Here’s something interesting to consider…the top of Mount Everest gets more incoming solar radiation than, pick a place, the Mojave Desert? I guess there’s less CO2 radiation broadcasting to the top of that mountain, so it’s colder.

It would probably make skeptics look a little better in the eyes of scientists if some of them, like you, did not engage in silly “strawman” arguments. There is no controversy about the fact that there is a lapse rate in the troposphere (although apparently it is something that Nikolov and Zeller chose to completely ignore when they added convection to the simple radiative model of the greenhouse effect such that it drove the temperature to an isothermal state with altitude).
However, the existence of the lapse rate does not allow you to disobey energy conservation. In the absence of an IR-absorbing atmosphere, the fact that the Earth+atmosphere must radiate back into space the same power it receives from the sun places a constraint on what the SURFACE TEMPERATURE will be. It is only when the atmosphere absorbs IR radiation and hence the effective radiating level is somewhere up in the atmosphere that the lapse rate comes in to help determine what the surface temperature will be.

Phil.
January 16, 2012 3:35 pm

Bill Hunter says:
January 16, 2012 at 1:01 pm
Phil. says:
January 16, 2012 at 12:06 pm
“Jelbring’s model is nonsense which bears no relationship to reality of any kind.
His model involves a planet surrounded by an atmosphere which is in turn contained by a non-transparent shell at a density altitude of 0.1atm.
Neither the shell nor the planet is allowed to radiate, J seems to believe that this means the shell is black. Also the shell and planet surface have negligible thermal mass.
The atmosphere is heated by the planet by conduction and convection and so achieves the dry lapse rate between the planet and the shell. (Presumably the planet has an internal source of heat).
He defines this lapse rate as the ‘Greenhouse effect’ and because this is dependent on g therefore the real and totally unrelated Greenhouse effect must be too! The guy is completely out to lunch, he believes that no physical laws are violated in the construction of this model!”
I guess it is correct he believes no physical laws are violated. Couldn’t the internal source of heat be of the sort that heats the gas giants before they ignite from getting too big and therefore too hot? Dr Brown here described such a process if I read him right.

What he believes is not the point, the internal source of heat is not the issue.
A solid surface at a non-zero temperature but which doesn’t radiate i.e. emissivity of 0?
Just how this is supposed to be a model for a real planet is a mystery since real surface emissivities are far closer to 1. Never mind the lid on the atmosphere which also has an emissivity of zero and has negligible thermal mass compared with the atmosphere!
Then he decides because he has no radiation involved in the model to call the temperature lapse rate the ‘greenhouse effect’, talk about a strawman.

shawnhet
January 16, 2012 3:45 pm

Bart:”No, that is not my contention at all. Actually, I have no contention, merely a question, viz: How do we know the full spectrum and radiation pattern of surface emissions, so that we may arrive at a truly full accounting of emitted power?”
Well, it seems to me that one way to check this is to do the sorts of tests I outlined last time which is to compare the IR emissions at different temperatures. Personally, I doubt that there is any substantial effect here, as this would be easy to measure and would make anyone who finds contradictory evidence pretty famous.
““Respectfully, to be engaging in science requires that you give sufficient detail to be able to tell *by using measurements of the real world* whether or not something that *might be* happening *actually is* happening.”
What you are saying is that scientists in general have no obligation to consider tracking down leads which might tend to detract from their favored hypothesis unless evidence is presented fully formed and airtight by another party. Furthermore, that lack of utterly compelling evidence so presented otherwise elevates their hypothesis to fact.”
I agree with you to the extent that everyone should be open to new possibilities, but treating possibilities as hypotheses requires some evidentiary basis. I don’t think anyone is asking for airtight and fully formed evidence, but some evidence is necessary IMO. It is one thing to buy and infrared camera and check radiation vs. temp levels at various times and temperatures, collect evidence that shows SB to be invalid and go from there. It is an entirely different thing to say that SB might be wrong, so we should throw all theories that reference it out the window. SO far, no one has provided any disconfirming evidence of SB while there is plenty (IMO) of confirming evidence(such as photos taken by infrared cameras) of it.
Stephen Wilde:”Given that the physical principles that I am expounding on are pretty basic I think it is for those who support an alarmist position to do the proving.”
The point is you can expound on all the physical principles you want but you can’t show how these expositions uniquely support what happens in the real world. OTOH, the GHE does make predictions about the real world (such as a difference in the emission of radiation from the surface and the planet as a whole) that seem to do this. The mere fact that it is possible that some vague non-GHE process could also act in the same way *does not* put these two theories on the same footing. Here’s a challenge for you: design an experiment that given a particular set of observations would prove you right and the GHE wrong.
Cheers, 🙂

Phil.
January 16, 2012 3:51 pm

Stephen Wilde says:
January 16, 2012 at 12:06 pm
“or we can explain how a nitrogen atmosphere would actually produce the difference in emission btw the surface and the planet as a whole.”
By storing solar energy in kinetic form (originally acquired by conduction from the surface) and then engaging in a dynamic conductive energy interchange with the surface.
So 240Wm2 comes to the surface from the sun and the surface radiates out 240W/m2 to space to achieve radiative balance.
Meanwhile the surface conducts 150W/m2 to the atmosphere and receives the same in return to achieve balance in the conductive energy exchange.
So the surface actually receives 240 from the sun + 150 from the atmosphere = 390
And radiates 240 to space plus conducting 150 to the atmosphere = 390

So the surface temperature is equal to the temperature of the adjacent atmosphere, and there is no net heat transfer from the surface to the atmosphere. I.e. Thermodynamic equilibrium.

jae
January 16, 2012 3:53 pm

Willis: uptheresomewhere you referred to Einstein’s thought experiments. Now, one of the things Einstein’s work emphasized is that experimental evidence is necessary before any theory becomes acceptable. But you seem to be avoiding the most important part of the N&Z and Jelbring’s (and Huffman’s) treatises; i.e., they provide empirical evidence for their “pet theories.” You do not.
Can’t post on this thread anymore; too long/slow.

don penman
January 16, 2012 3:55 pm

Willis, I prefer Roy Spencer’s review of N&K’s theory to your attempt to prove their theory wrong using a thought experiment.It is clear that you have no respect for them .What did they do to you?
So you are entitled to your opinion on the merit of their theory but please don’t try to bully others into accepting your view.

DeWitt Payne
January 16, 2012 3:58 pm

Hail Core says:
January 16, 2012 at 7:46 am
Is this correct? http://tallbloke.wordpress.com/2010/07/27/nasif-nahle-nails-the-radiative-physics-of-co2/ Considering the data obtained by many researchers on this matter, the total emissivity of the carbon dioxide is low. It is 0.0017.

No, it’s not correct. That emissivity is only valid for a path length of 1 cm. Given the source, I’m not at all sure that it’s correct even for a 1 cm path, but I’m not willing to pay Spectralcalc for a one month subscription to prove it.

DeWitt Payne
January 16, 2012 4:08 pm

Robert Brown says:
January 15, 2012 at 8:26 am
To avoid confusion, I’m NOT stating that the GHE is a chemical reaction, that is simply an analogy. What happens is the GHGs become well mixed in the atmospheric profile due to pressure and heat. Hence, the GH effective radiating altitude gets set very high even with low concentrations. Because the atmospheric profile is changed very little by adding additional GHGs, that altitude does not increase if things like additional CO2 are added. It is that height that determines the overall GHE.
This seems correct to me, although somebody (Tim F.?) asserted otherwise. Indeed, I suspect that the Earth is largely insensitive to changes in GHG concentrations, and might even operate the opposite way than expected in some cases. Once you are “opaque” you are opaque, and making it twice as opaque doesn’t really happen.

CO2 has lots of absorption lines. The further you go from the center of the band, the weaker the lines. The center of the band may be opaque, but the wings aren’t. When you increase the concentration, you are increasing absorption in the wings. Science of Doom has lots more detail, specifically in this article. Caballero is a good start on radiative transfer, but you should really get something that goes into more detail like Grant Petty, A First Course in Atmospheric Radiation, 2nd Edition, Sundog Publishing. Available direct from the publisher at a discount from retail.

jae
January 16, 2012 4:29 pm

Heard in an elevator:
1.) The relationship between temperature , pressure, and volume is given by the ideal gas law, which can be put in the form: T=PV/R. Lets use R = 0.0820 L atm/K mol.
2.) If we have one liter of a gas at one atmosphere, the T must be increased by 1/0.0820 = 12.2 K
3.) True, not 33 K, but not 0, either.
4.) Cannot have a gas at 1 atmosphere at absolute zero, no?

DeWitt Payne
January 16, 2012 4:37 pm

Paul Dennis says:
January 15, 2012 at 12:30 am
Willis,
I don’t think I agree with you here and am with Roy Spencer on this point. I think the the lapse rate does require convection.

I agree with you and Alexander Harvey. I would add one condition, though, local thermal equilibrium. Once the altitude gets high enough and pressure low enough that LTE no longer applies, all bets are off. For a planetary atmosphere similar in size to the Earth’s atmosphere, that would be somewhere between 30 and 100 km altitude. Since the thought experiment defines the surface as isothermal, convection will stop when the lapse rate reaches the adiabatic rate. After that point, there is only conduction. Conduction is driven only by the temperature gradient and, as long as LTE holds, cares nothing about gravity or gravitational potential energy. If that weren’t so, the partial pressure of CO2 at the surface would be higher than it is and the volumetric mixing ratio would decline with altitude, or as someone else put it, all the CO2 would fall to the bottom of the atmosphere.
Bart,
You will need to do more than make an assertion about the Laplacian in spherical geometry to prove that an isothermal atmosphere is unstable. The atmosphere is a very thin shell around the planet and the planar approximation is quite good.

Martin A
January 16, 2012 4:41 pm

Willis Eschenbach says:
January 16, 2012 at 3:05 pm
(…)
In other words you’ve been acting like an arrogant dickwad since you opened your mouth on the subject … and now you are surprised that I am offended when you insult me and you act like a total douche?
Yeah, you touched a nerve, d’uh. You are an irritating jerkwagon who goes out of his way to pompously and patronizingly lecture people he’s never met about their supposed moral failings.
(…)
==============
It must be really awful to be so readily provoked to such a level of anger.

Bart
January 16, 2012 4:54 pm

Willis Eschenbach says:
January 16, 2012 at 3:16 pm
“If any mechanism increases the surface temperature with a transparent GHG-free atmosphere, and the surface energy states settle into the same configuration that they would have in the absence of the atmosphere, then the surface will be radiating more to space than it is absorbing, and that is a violation of conservation of energy.”
Fixed that for you. Do you yet see that you have been begging the question all along?
shawnhet says:
January 16, 2012 at 3:45 pm
“Personally, I doubt that there is any substantial effect here, as this would be easy to measure and would make anyone who finds contradictory evidence pretty famous.”
Well, hell, let’s just end scientific inquiry altogether, as there’s nothing left to discover. If it hadn’t been, lots of people could be famous. Since they aren’t, there’s nothing left to discover. QED.
Conduction and convection of heat from the Earth’s surface are enormous. To blithely assume that has no effect when there is an avenue open for it to do so makes no sense to me.
“It is an entirely different thing to say that SB might be wrong, so we should throw all theories that reference it out the window.”
Nobody is saying SB is wrong, merely that it has not been demonstrated to be entirely applicable to the problem at hand.
Phil. says:
January 16, 2012 at 3:51 pm
“So the surface temperature is equal to the temperature of the adjacent atmosphere, and there is no net heat transfer from the surface to the atmosphere. I.e. Thermodynamic equilibrium.”
(Facepalm) This is so basic.
Not of the surface stripped of the atmosphere. The surface/atmosphere system is in equilibrium, not the surface alone. Just as you cannot break out the pieces of a mechanical linkage in a free body diagram without including the contact forces, you cannot ignore the temperature gradient at the interface when you separate the parts of the system.

Bart
January 16, 2012 5:01 pm

Bart says:
January 16, 2012 at 4:54 pm
shawnhet says:
January 16, 2012 at 3:45 pm
“Nobody is saying SB is wrong, merely that it has not been demonstrated to be entirely applicable to the problem at hand.”
Or, at least, reasonably applicable.
“Well, it seems to me that one way to check this is to do the sorts of tests I outlined last time which is to compare the IR emissions at different temperatures.”
Nope. You need to measure a reasonably continuous spectrum to determine its shape, and you have to integrate the power over the entire hemisphere, not just over one spot.

Bart
January 16, 2012 5:09 pm

“You need to measure a reasonably continuous spectrum to determine its shape, and you have to integrate the power over the entire hemisphere, not just over one spot.”
And, it has to be across representative environments: calm and choppy seas, wet and dry land in mountains and plains and everything in between, calm and windy deserts, etc… And, these have to be weighted in the final calculation to approximate all the different surface states of the Earth on a typical day.

Myrrh
January 16, 2012 6:22 pm

Willis Eschenbach says:
January 16, 2012 at 10:19 am
Myrrh says:
January 16, 2012 at 1:59 am
Willis: Whoa, back up. There is generally no special thermal radiation that occurs at phase changes. Why would a phase change cause thermal radiation(not the release of latent heat that simply warms the surrounding air, but thermal radiation)?
Me: ? The release of latent heat can be by conduction, convection or radiation – if by radiation then the latent heat IS thermal radiation, thermal infrared. Thermal infrared is heat energy, thermal energy, on the move.
Willis: Myrrh, you are claiming that;
1. The imaginary planet described in the head post has nitrogen that for some reason will liquify in the atmosphere. I don’t know why it would do that, but that is your claim.
2. You also claim that the latent heat from the liquification of nitrogen is emitted as thermal radiation.
Bro’, you desperately need some citations for those claims. I think both of those things are fantasies. Latent heat is called latent heat and not latent radiation for a reason. It comes out as heat. Since what we’re talking about is nitrogen liquifying at something like -170C or wherever nitrogen liquifies, the amount of radiation will be microscopic.
===========
Quote my words Willis!!
You may well be talking about it with someone, but I’ve said nothing about nitrogen liquefying or claimed that it’s in the head post’s atmosphere…, at any temperature. 🙂
My post was referring to your exact words as I quoted them. Nothing more.
If it wasn’t becoming so irritating it would be still be funny, the same mistake which has been promoted as a fisicsfiction meme by those pushing AGW – thinking heat is something different from thermal radiation.. That’s why so many here can’t tell the difference between heat and light.
So, to quote your words again exactly, I’m now referring to what you said here:
“Latent heat is called latent heat and not latent radiation for a reason. It comes out as heat.”
If that latent heat is released in a vacuum where it must be radiated out, you and your ilk think it isn’t heat!
What is it then?? A few squiggly lines?
If you would all just stick with basic physics about this you wouldn’t get so confused.
Latent heat is called latent heat because it is the heat released or absorbed because of a phase change. It’s the latent that’s descriptive here, about which process, form, state the heat is in. Heat is heat, how it’s transmitted doesn’t change it. Conduction, convection and radiation are likewise such descriptions. The heat is the same.
So, to quote your words again exactly, I’m referring to what you said: “Latent heat is called latent heat and not latent radiation for a reason. It comes out as heat.”
Is gobbledegook.
What the heck is “latent radiation”? (p.s. while waiting to post this I looked it up and the only reference to such a thing is in those who’ve had chemotherapy treatment, which then later may cause complications etc.)
Here, for anyone interested to learn something about HEAT:

http://thermalenergy.org/
Thermal Energy Explained
What is thermal energy ?
Thermal Energy: A specialized term that refers to the part of the internal energy of a system which is the total present kinetic energy resulting from the random movements of atoms and molecules.
The ultimate source of thermal energy available to mankind is the sun, the huge thermo-nuclear furnace that supplies the earth with the heat and light that are essential to life. The nuclear fusion in the sun increases the sun’s thermal energy. Once the thermal energy leaves the sun (in the form of radiation) it is called heat. Heat is thermal energy in transfer. Thermal energy is part of the overall internal energy of a system.
At a more basic level, thermal energy comes form the movement of atoms and molecules in matter. It is a form of kinetic energy produced from the random movements of those molecules. Thermal energy of a system can be increased or decreased.
When you put your hand over a hot stove you can feel the heat. You are feeling thermal energy in transfer. The atoms and molecules in the metal of the burner are moving very rapidly because the electrical energy from the wall outlet has increased the thermal energy in the burner. We all know what happens when we rub our hands together. Our mechanical energy increases the thermal energy content of the atoms in our hands and skin. We then feel the consequence of this – heat. [Link]Laws of Thermodynamics

Italics as used in the piece.
Further:

http://thermalenergy.org/heattransfer.php
Heat Transfer
Thermal energy and heat are often confused. Rightly so because they are physically the same thing. Heat is always the thermal energy of some system. Using the word heat helps physicists to make a distinction relative to the system they are talking about.

Heat: Term used to describe the transfer of thermal energy between two thermodynamic systems at different temperatures.

Take a small piece of ice out of your fridge and hold it in your hand. The thermal energy content of your hand is higher then the thermal energy content of the ice cube.
The atoms that comprise your hand are moving more rapidly then the atoms that make up the ice cube. Therefore, there will be a transfer of thermal energy from your hand to the ice cube. While this thermal energy is in transfer, it is called heat. This will cause the atoms in the ice cube to speed up while the atoms in your hand slow down.
The increase in speed of the ice cube atoms changes the state of water from solid to liquid. This transfer of thermal energy will continue until an equilibrium is reached between your hand, the ice (now water), and the air in the room.
When you put your hand over a hot stove you can feel the heat. You are feeling thermal energy in transfer. The atoms and molecules in the metal of the burner are moving very rapidly because the electrical energy from the wall outlet has increased the thermal energy in the burner. We all know what happens when we rub our hands together. Our mechanical energy increases the thermal energy content of the atoms in our hands and skin. We then feel the consequence of this – heat

So, heat is the same thing in all of this! It is the kinetic energy of some system in the random movement of atoms and molecules. Heat is the thermal energy as in the Sun, it is the thermal energy leaving the Sun in the form of radiation. These are not different kinds of heat, they are different descriptions about heat, just like latent is a description about heat. The HEAT they are describing is the SAME THING.
“Thermal energy and heat are often confused. Rightly so because they are physically the same thing
Thermal means of heat, that’s a description of the energy. The energy described is that which is heat.
Kinetic means of or due to motion, it’s a description of the energy Kinetic energy is thermal energy is heat in the total of the random movement of atoms and molecules.
“Young just called it energy. Lord Kelvin (1824-1907) added the adjective “kinetic” to separate it from “potential energy”, which was named by William Rankine (1820-1872) in 1853.
http://physics.info/energy-kinetic/
Same page this is what it says about latent heat:
“Heat absorbed or released as the result of a phase change is called latent heat. There is no temperature change during a phase change, thus there is no change in the kinetic energy of the particles in the material. The energy released comes from the potential energy stored in the bonds between the particles.”
So, Heat in the form of radiation is thermal infrared. Thermal infrared is the thermal energy of the Sun which has left the Sun and is on the move, it is the HEAT of the Sun on the move.
Visible Light is not Heat.
Visible, Light, is not the thermal energy of the Sun, is not the total kinetic energy of the Sun, it is not heat of the Sun on the move. If it was, it would be called Heat.
The visible light is being created by the heat in the Sun, it’s an effect of heat.
Thermal infrared is the thermal energy of the Sun which has left the Sun and is on the move in the form of radiation. The thermal energy of the Sun is Heat. Therefore, thermal infrared which is thermal radiation is HEAT ON THE MOVE.
Heat, thermal infrared, thermal energy in the form of radiation, is what you can feel radiating out to you from a stove even if the stove isn’t hot enough to produce visible light; the heat you feel radiating out to you from a hot pavement; the heat you feel from the Sun. It is invisible.
You cannot feel visible Light.
Apologies for the italic gone haywire in my last post, it’s something to do with using blockquotes, but I haven’t yet fathomed it. I was going to offer to repost it to make it easier to read.

Dr Burns
January 16, 2012 6:27 pm

>>Thanks, Dr. B. But misleading? I have shown that there is no way for an IR transparent atmosphere to affect the surface temperature. What is misleading about that?
Yes, GHG’s exclusively set the surface temperature in your model. However, it is misleading because your model exacerbates the apparent impact of GHG’s in the real world. The 70% of the Earth’s surface with cloud cover, does not behave according to your model. It might be useful to model the area covered by cloud in a comparable fashion to your naked sphere. It does not seem to me that modelling cloud as a thin shell is adequate, because the surface temperature becomes the same as the cloud temperature. I’d be most interested to see you take the next step and present an enhanced model.

Bill Illis
January 16, 2012 6:43 pm

There are all kinds of Potential Energies that can turn into actual thermal energy.
Gravitational Potential Energy can turn into thermal energy when a object with mass enters into or falls through a gravity field.
Nuclear Potential Energy can turn into massive thermal energy when mass is converted into energy.
Chemical Potential Energy, Electric Potential Energy.
The universe is not only made up of radiation transfers between objects.
In fact, the only reason the solar forcing exists is because a great deal of mass gained thermal energy falling into the gravity field of the Sun (gravitational potential energy) and then Nuclear potential energy took over and now the Sun is putting out 63,200,000 watts/m2 of solar radiation towards us.
All kinds of strange things that we wouldn’t expect happen in real quantum physics.

NoIdea
January 16, 2012 6:55 pm

” SO far, no one has provided any disconfirming evidence of SB while there is plenty (IMO) of confirming evidence(such as photos taken by infrared cameras) of it.”
Maybe use Miles Mathis’ derivation of SB? Might be some entertainment value in that…
http://milesmathis.com/stefan.html

gbaikie
January 16, 2012 7:05 pm

“I’m sorry to say that I do not understand your post. My question was referring to the fact that the Earth as a whole emits ~240W/m2 while the surface emits~390W/m2. From the POV of my question, it doesn’t matter whether the 390w/m2 comes from the sun or a giant bonfire or gravitational collapse of the planet or microscopic amounts of pressure induced nuclear fusion or whatever. In the case of a Nitrogen atmosphere if the surface emits 390W/m2, then the planet as a whole should based on current theory emit 390W/m2. Since this is not what we observe in the real world, we can either explain what is different about the actual atmosphere compared to a nitrogen one of the same mass and pressure or we can explain how a nitrogen atmosphere would actually produce the difference in emission btw the surface and the planet as a whole. If we can’t do either, then we do not have a useful hypothesis IMO.”
Well the surface of earth doesn’t emit 390 w/m/2, nor does it radiate 240 W/m/2 at top of atmosphere. These are models.
The surface emits varying amounts of energy- depending on where and when.
15 C or 288 K can emit 390 watts. My temperature outside is not 15 C.
Now, suppose I lived somewhere where it was 15 C, and suppose it was night time and my driveway was 15 C and remained around 15 C for an hour, Do you think it would be producing 390 watts of energy per second per square meter- or emit 3600 times 390 watts per hour?
What is the difference in term power produced between my 15 C driveway and if my floor, walls and ceiling were also about 15 C?
Could I not say that my walls are kept warm due to air temperature? Can not also say the driveway is also kept warm from air temperature?

Bart
January 16, 2012 7:23 pm

DeWitt Payne says:
January 16, 2012 at 4:08 pm
“Science of Doom has lots more detail, specifically in this article.”
DeWitt – You appear to have commented there a lot, and to be fairly knowledgeable, so maybe you can answer my question about this article. I fear the thread there has gone stale, and I doubt I can get an answer as nobody will be looking any longer.
As can be seen in Figure 4, the emissivity drops off precipitously as the observation angle approaches 90 deg. Not surprising, of course, but SB is calculated assuming a completely clear field of view, i.e., the surface is completely smooth, so the emissions are roughly uniform over the overarching hemisphere. Experience tells me that a smooth and polished black panel will reflect a lot more light than one with a rough surface. I would expect the same would be the case with emissions, which is, after all, essentially 1/2 of the reflection process.
And, Figure 4 appears to back this impression up. So, it seems the proper equivalent emissivity for a body with a rough surface should, in fact, be scaled by the area under these curves.
When I integrate, for example, the function in Figure 4a over the hemisphere, I get something like 0.8 as the total effective emissivity. If I assumed that the Earth were all ocean with this as the effective emissivity, that takes it down from equivalent radiated energy of 390 W/m^2 to 312 W/m^2.
Now, if I suppose that these were pretty calm waters, and that the lowered effective emissivity were more on the order of maybe 0.7 worldwide, that would get me down to 273 W/m^2. Only 33 W/m^2 to go to reach the incoming 240 W/m^2. Actually, if I take the incoming to be distributed across a sphere 100 km higher than the surface, that becomes 248 W/m^2 (ratio of radii squared), so that takes me down to 25 W/m^2 to account for.
I suspect GHG warming does account for at least some of the budget, so give or take a little here or there, and there are other things to consider, but I do not want to detract from my point here. The point is, the putative GHG warming effect could be, at the very least, a lot less than commonly believed.
So, the question is, have climate folks actually factored the losses due to surface roughness into their estimates? Or, have they perhaps grossly overestimated total emissions of non-ideal bodies?
There are other considerations, e.g., does the morphology of emissions spectra change with observation angle as well? Experience with optical gratings suggests to me that the spacing of interfering bumps and crags could be important in determining which frequency components get out, and which get reabsorbed. And, so on. But, perhaps this will do to go on for now.

Ned Nikolov
January 16, 2012 7:29 pm

Dear Willis Eschenbach ,
Karl Zeller and I have been working on preparing a reply to your questions, which have also been asked by a number of other people. Our reply will be in two parts. Part 1 will become available by Wed (Jan 18). Part 2 will arrive a week or so later … Part 1 focuses on the magnitude of the GH effect, as this topic seems to be the crux for most people including yourself. You will learn (and see physical proof of) that our atmosphere actually raises the surface temperature not by 33K, but by well over 100K! Now, think what are the implications of this fact alone for the current GH theory? Do you know what’s the total amount of GH gases in the atmosphere as a percent of total atmospheric mass? How likely is that handful of gases can raise the near-surface temperature by 133K through ‘back radiation’?
About the ‘elevator speech’ – that was given in our first paper! However, you apparently did not get it. So, it will take far more explanation to convey the basic idea, which we will try to do in Part 2 of our reply.
In the mean time, if I may offer an advice, please try to find some good old textbook on classical thermodynamics preferably published before 1985, and read about the implications of the Gas Law, specifically what is the role of pressure in isobaric processes. That would be a time well spent for you!
Thank you.
– Ned

Bart
January 16, 2012 7:31 pm

“So, it seems the proper equivalent emissivity for a body with a rough surface should, in fact, be scaled by the area under these curves.”
Scaled by the sine of the observation angle to get the correct incremental area on the hemisphere.

Anton Eagle
January 16, 2012 7:54 pm

Willis,
The more I think on it… the more I think that your clear IR transparent atmosphere actually cools the surface of your theoretical planet.
Consider….
Although N2 and O2 are transparent in the IR spectrum… the DO still radiate if at a temperature above 0 kelvin. Granted, they don’t radiate in the IR spectrum… but energy out is still energy out… regardless of what frequency it occurs.
So, your atmosphere acquires heat energy from the surface via conduction, and propagates it throughout the atmosphere via convection, and finally radiates it out to space via SB radiation. Albeit, again, not at IR frequencies.
Both your surface, and your atmosphere are now radiating.
Thus, because of conservation of energy, the surface must now be radiating less than it would without the atmosphere. This is understandable since it gave up some of its energy to the atmosphere.
Another way to look at it is that the non-GH gasses in the atmosphere are sucking energy from the surface… cooling it… much in the same way that sweat sucks heat from our skin (via phase change).
I don’t know if this helps anyone’s argument.
One thing is clear though… focusing on only what’s going on in the IR spectrum causes one to miss what’s really going on. We need to focus on radiative energy transfer… regardless of the frequency. Although N2 and O2 are poor absorbers in the IR spectrum and thus poor IR radiators, they are certainly not poor radiators across all frequencies.
Also, I contend that my theoretical bottle of N2 or O2 gas would loss temperature pretty much as rapidly as any other gas… again just not radiating in the IR spectrum.
-Anton

Trick
January 16, 2012 8:04 pm

Willis Eschenbach says in response 1/16 3:16pm (fine turnaround time with all these conversations & I will risk not restating my standing elevator speech):
“If any mechanism increases the surface temperature with a transparent GHG-free atmosphere, the surface will be radiating more to space than it is absorbing, and that is a violation of conservation of energy.”
Right, to preserve conservation of energy, the surface temperature with any mechanism ADDED to the black body by Willis (a transparent GHG-free atmosphere) (or any added by me) means the planet surface CANNOT be thermally radiating to space more than it is absorbing, nature needs a rescue that must be provided by any mechanism added to BB. Willis added this particular second mechanism to the planetary black body, not me. I have not added any mechanism.
This particular second mechanism added by Willis brings into the nature of the system thermal conduction which did not operate in Willis’ one mechanism black body thermally radiating to space. This particular two mechanism surface temperature is now both thermally radiating AND thermally conducting then radiating to space just right to preserve energy conservation, nature is rescued. These two mechanisms are from Willis. No two mechanism system energy conservation violation does or need arise.
Details:
Top post starts w/one mechanism: planetary black body with gravity thermally radiating to space; no thermal conduction operates.
Willis’ top quote verbatim: “Let me call the “theoretical S-B temperature” the temperature that an evenly heated blackbody planet in outer space would have for a given level of incoming radiation in W/m2.”
Planet equilibrium temp. T established on physical details. No violation.
———————————————
Any mechanism is added by Willis: in particular a transparent GHG-free atmosphere second mechanism is added to the same one mechanism planetary black body.
Willis top quote verbatim: “Now suppose we add an atmosphere to the planet, a transparent GHG-free atmosphere.”
Two mechanisms now: 1) a planetary black body surface thermally radiates to space + 2) transparent GHG-free atmosphere thermally conducts and then thermally radiates to space at TOA.
As Willis quote above states: this planet surface cannot be radiating more to space than it is absorbing so it doesn’t (i.e. the BB surface radiates exactly what it receives in the presence of Willis’ added thermal conduction which comes to nature’s rescue).
Same planet has equilibrium temp. near surface established higher T + delta T due to second mechanism added by Willis: the ideal gas KE from a transparent GHG-free atmosphere introduced in presence of gravity so nature reveals operation of gravity, the ideal gas law AND thermal conduction to us – all at once, & at least ideally, permanently w/sun & no loss of mass in that atmosphere mechanism. No energy conservation violation thus need arise, energy in = energy out as long as sun & gravity lasts with two mechanisms at equilibrium T + delta T.
Fun & interesting discussion Willis et. al., though Willis appears to me might be growing weary from the size of it. Just my observation from reading a blog, could be wrong.
PS: Apologize about cap.s used, I am not screaming – just don’t know how to emphasize w/underline or bold on this site.

anna v
January 16, 2012 9:05 pm

Phil. :
January 16, 2012 at 12:43 pm
Absorption and emission are symmetric conditions quantum mechanically. If a line exists it can be excited, meaning it can absorb the same line. The ambient electromagnetic spectrum is a continuum as the black body radiation shows (frequency is 1/wavelength). The high tail of the kinetic energy distribution always has enough energy to excite and ionize some of the gas during a collision , though it would be quite rare.
Argon, as all molecules will also have the continuum radiation arising from the collisions distorting its field and creating continuum levels that can absorb and emit radiation.
There is no physical bulk matter that will not radiate as a black body.

jimmi_the_dalek
January 16, 2012 9:20 pm

There seem to be a lot of people who do not understand the concept of a thought experiment. It does not have to be physically plausible! Especially when the point is that it is part of a proof by contradiction, as the whole point of such a proof is to show that an impossible conclusion has resulted and therefore the starting point must have been incorrect.
So those saying that Willis’s model should be improved, e.g. because all substances interact with electromagnetic radiation, and so must absorb at some frequency, even an inert gas like Argon, are correct, but have missed the point completely. The completely transparent atmosphere is not Willis’s invention – it is implicit in the models he is criticising. These models claim that all of the GHE can be explained without any mention at all of absorption or emission of radiation by the atmosphere. It follows that their theories should be true for an atmosphere which is completely transparent. Perhaps the authors of these theories do not realise that they have built this into their model, but they have. So it is legitimate to ask how a completely transparent atmosphere would behave. Willis has given the answer, and shown that the ‘it’s all due to gravity’ models are nonsense.
PS to Trick above – a transparent atmosphere does not radiate anywhere, even at TOA.

shawnhet
January 16, 2012 10:15 pm

Bart says:
January 16, 2012 at 4:54 pm
““Personally, I doubt that there is any substantial effect here, as this would be easy to measure and would make anyone who finds contradictory evidence pretty famous.”
Well, hell, let’s just end scientific inquiry altogether, as there’s nothing left to discover. If it hadn’t been, lots of people could be famous. Since they aren’t, there’s nothing left to discover. QED.”
Here’s a reality check for you: Can you point to a **single** piece of evidence that supports your position? I submit that every day there are thousands if not millions of tests of the SB relationship. You want to say that those tests don’t mean anything because they don’t meet your (self-imposed) standards. I submit that there is no reason to believe that more tests will only find what has already been found. I’m quite sure that the theory of gravity hasn’t been tested in all the different environments you think SB needs to be tested, but most folks are pretty confident in that theory even so. Why?: because no one has been able to produce any evidence of real world occurences that contradict the theory.
If you are really interested in scientific inquiry into the SB relationship – here is a pretty good place to start IMO:
http://www.cce.ufes.br/jair/web/stefBoltz.pdf
Cheers, 🙂

Crispin in Waterloo
January 16, 2012 10:52 pm

@Bart
I see you understand my major point: A non-GHG atmosphere will emit energy, apart from the black body surface. The total energy entering will not all leave from the surface as IR. Part will radiate from the atmosphere because it will heat up.

>>To repeat, just because a gas does not absorb IR radiation does not mean it will not emit IR and other radiation.
>Repetition of a fallacy doesn’t make it true, the Physical Law that contradicts your statement is known as Kirchoff’s law. Emissivity at a wavelength is equal to the absorptivity at that wavelength.
I refer you to the explanation given at length above by George E Smith. I have no need to repeat its truths.
Anna V also states very clearly the point I was trying to make. It is incorrect to say that a black body planet with a GHG-free atmosphere must emit all its energy by IR radiation from the surface, and equally untrue that the atmosphere will not increase in temperature, the same regarding the absence of thermals and wind.
With regards to Willis’ investigation, the heavier and the deeper the atmosphere, the greater the insulating effect (with nod to the argument that gases vary in conduction coefficients with pressure). If gravity were to increase, the efficiency of conduction from the surface to the atmosphere would increase (directly) in efficiency and the proportion emitted from the non-GHG atmosphere into space would increase as the molecular collisions would be more frequent and more enegetic.
There is no such thing as a non-GHG atmosphere that does not emit radiation so the thought experiment is not going far. I agree with Willis’ earlier calculation that the surface might well be hotter if all GHG’s were removed and the ‘non-GHG’s’ retained because evaporated/condensed water is an efficient working (heat transport) fluid and superb IR radiator.
Regards
A radio HAM

jimmi_the_dalek
January 16, 2012 11:08 pm

Crispin,
“There is no such thing as a non-GHG atmosphere that does not emit radiation so the thought experiment is not going far.”
On the contrary, the thought experiment shows precisely what it is supposed to, namely that a model in which the radiative properties of the atmosphere do not exist, cannot be correct. So, instead of criticising Willis’s thought experiment, you should be asking Nikolov and Zeller why they omitted these factors.

Bart
January 16, 2012 11:09 pm

shawnhet says:
January 16, 2012 at 10:15 pm
“I submit that every day there are thousands if not millions of tests of the SB relationship.”
I have not questioned the SB relationship at all. I have questioned the shallow manner in which it is being applied. The difference between us appears to be, I know how it is derived, and the conditions upon which its successful application depends. You, however… apparently not so much.

shawnhet
January 16, 2012 11:12 pm

gbaikie says:
January 16, 2012 at 7:05 pm
“Could I not say that my walls are kept warm due to air temperature? Can not also say the driveway is also kept warm from air temperature?”
The driveway can be warmed by the air, but it can only do so by transferring heat into it. In the absence of a GHE it cannot increase the amount of heat in the Earth system at any given time. OTOH, a GHE can increase the amount of heat in the Earth system at any given time by causing some IR radiation to be absorbed and re-emitted multiple times before the radiation finally gets high enough to radiate to space.
Cheers, 🙂

Bart
January 16, 2012 11:23 pm

DeWitt Payne says:
January 16, 2012 at 4:37 pm
“You will need to do more than make an assertion about the Laplacian in spherical geometry to prove that an isothermal atmosphere is unstable. The atmosphere is a very thin shell around the planet and the planar approximation is quite good.”
No, I really don’t. Unstable is unstable. It does not matter if it is fast or slow, eventually the unstable state will find a way to transition to a stable one. And, the discussion is about a system in “steady state”.

John Mason
January 16, 2012 11:24 pm

In physics – all topics require being aware of the frame of reference
With that in mind here is an ‘elevator scenario’
1st scenario – no atmosphere
Ok, the Sun just turned on:
Our black body starts to heat up
as the temp increases the black body emits energy back out
equilibrium is achieved
life is happy
2nd scenario – add an atmosphere
Ok, the Sun just turned on
Our black body starts to heat up
There is a delay in the heat getting back to space because of the atmosphere.
as the temp increases the black body which includes this atmosphere emits energy
equilibrium is achieved
life is happy
But – note the delay. If you agree this start up scenario introduces a delay in how soon equilibrium is achieved then you’ll recognize there is a lapse rate and delay in the conduction of heat back to space caused by the atmosphere.
That’s my attempt at a simple elevator type thought.
The rest is more musings on the above.
The total frame of reference will make both scenarios the same looking at the whole planetary system. But if you narrow the frame of reference to just the surface of the planet where there is always a delay taking place you’ll see a local rise in temps near the surface and above this as you head towards space a cooler area. The net of the whole system is the same but there is a differential in temps in the atmosphere depending on what altitude you measure the temps.
ie: Warmer by the surface, cooler as you approach space.
If there is a delay in the energy getting into space as the planet is first being heated by the sun creating a warmer and cooler zone in the atmosphere, then this difference will persist.
If there is no delay in the initial energy getting into space as the planet is first being heated by the sun then your original premise is correct.
The greater the pressure of the atmosphere the greater this initial delay. Also the greater the local frame of reference of temperature will be from low altitudes to high altitudes, though the system average temp – surface and air will be at the constant for a planet with no atmosphere.
So, is there a delay in the initial heating of a black body if there is an atmosphere? Does the presence of said atmosphere create a gradient of measurable temps that are highest near the surface and get cooler as you move towards space? Yet the cooler temps as you go higher do not emit as much as the surface does and conduction is the major player here in both the delay and the local apparent increase in temps near the surface.
As any pilot knows the surface of the earth is too warm and as you go up it’s too cold. But the average atmospheric temperature plus surface radiation will be the same as a black body with no atmosphere. It simply has to be. Also the cold area offsets the warm area and as an earlier poster says this equilibrium point is about 5k up. So just measure in the atmosphere where the expected S-B temp is and you’ll have warming below and cooling above netting to no change. But if the local frame of reference is not the whole system but just a small area by the surface, since above 5k is cooler, that below 5k area must be warmer or equilibrium will not be at the expected value for the whole system.
Lapse rate, delays, high and low temps as altitude changes in an atmosphere are what the N-Z theory seems to be about. Hopefully the above helps to potentially describe where they are coming from.

GabrielHBay
January 17, 2012 12:14 am

I can see this thread is slowing to a snails’ pace. Thank goodness. Time for an elevator speech on my humble observations of the exhausting few days of following the entertainment. Since it is an elevator speech, I shall not be mentioning names. You know who you are. Nor will I get technical. We have had enough of that.
Here goes:
1. The majority, but certainly not all, of the posters seem seriously confused about the difference between the concepts of temperature and heat (or energy).
2. A great many posters are seriously hazy as to how adiabatic pressure vs temperature works in an atmosphere, particularly about the ‘need’ for convection for it to work. Think isothermal vs isentropic
3. The confusion about radiation (or not) from transparent atmosphere at above 0K is epic. The adamant to-the-death defense of the stance of the ‘not’ was a sight to behold, especially but not confined to the main poster.
4. The pervasive and abysmal ignorance on just the above three points so dominated the discourse that the thread is probably 10x as long as it needs to be. 50% of the remainder was wasted on personal and unbecoming bitchiness that would give my youngest daughters’ school classmates a run for their money. (And that is saying a lot)
5. One comes away with a feeling of disbelief that so many individual posters who clearly believe they know a lot and can swamp us with technicality still err on basic issues.
My sincere thanks to the few voices in the wilderness who get the basics right and who thereby refreshed my own very rusty memory. I also stand deliciously refreshed in my scepticism of majority opinion out there.
Gabriel van den Bergh, signing off… (phew)

anna v
January 17, 2012 12:43 am

jimmi_the_dalek says:
January 16, 2012 at 9:20 pm
There seem to be a lot of people who do not understand the concept of a thought experiment. It does not have to be physically plausible!
A thought experiment must not contradict existing data. It is a form of a “theoretical model”. They must not contradict physical data. If they do, they are science fiction, not physics models. Particularly if they are used to a proof by contradiction (Reductio ad absurdum). Conservation of energy is a basic physical law.
PS to Trick above – a transparent atmosphere does not radiate anywhere, even at TOA.
only in science fiction.

J. Radefahrt (Ger)
January 17, 2012 2:13 am

I miss one point in this discussion: reflexion. Every electromagnetic wave -and IR is such an EM-wave- can be reflected, and we know that it is happen because of such things as radio, especially longwave, fata morganas etc. Each boundary layer, especially with relatively large discrepancies between the strata, is capable to reflect radiation.
All radiative interacting substances act with three relevant parameters:
– absorption
– transmission
– reflexion
Actually, none of those is 0. Therefore it is necessary to know the ratio between reflection and absorption.
If one knows the transmission one can answer the question: how much of the downwelling radiation is reflected? This is relatively easy to find out – as far as we have the transmission ratio (see below).
Idealized one can assume that absorbed energy is re-emitted upwards and downwards with the same ratio, means 50% each. Considering that reflected goes 100% down and absorbed energy goes 50% down resp. 50% up, one has to take the difference between up and down radiation and gets the amount of reflected energy.
To find out the effects of GHG we have to consider that it needs 2 times of the absorbed energy to equal the reflected energy. This leads to the following:
If
– 2x absorbed > reflected ==> warmer
– 2x absorbed = reflected ==> no difference
– 2x absorbed cooler
To calculate the amounts of transmitted energy and therewith the ratio contains a problem. It is quite difficult to number the actual amount because it contains not only the atmospheric window (40W/m²) but also the unsaturated absorption bands of GHG (at least difficult for me, perhaps someone can help).
The higher the transmitted energy the lower the absorbed and re-emitted part of energy and with this the higher the part will be reflected.

J. Radefahrt (Ger)
January 17, 2012 2:19 am

– 2x absorbed cooler
Actually it has to be:
2x absorbed cooler.
I would be apreciated if someone would consider this and change it.
Thanks.

Spector
January 17, 2012 2:19 am

It appears that many people have been confused about the fact that most greenhouse effect calculations are based on average energy flow rates–usually measured in watts per square meter. Technically, power in watts represents the energy flow rate in joules per second. These calculations are based on steady-state conservation of radiant energy requirements. To make these values palpable, they are often represented by the uniform temperature that the Stefan-Boltzmann formula says would produce that same average energy flow from a presumed level, flat surface on the Earth.
Conservation of energy says that the Earth must radiate over its whole surface the same energy flow that it is absorbing (not reflecting) from the disk of intercepted solar radiation. This is an average escaping LWIR energy observed at the top of the atmosphere and it is independent of any greenhouse effect in the atmosphere below.
The nominal value for this required flow is usually stated as 239 or 240 W/m². If the surface were emitting energy at a higher average rate, say 396 W/m², then something must be extracting 156 W/m² from the *radiant* energy flow going up in the atmosphere and returning it back to the ground. Only greenhouse gases can perform that function on a continuous basis.

J. Radefahrt (Ger)
January 17, 2012 2:24 am

– 2x absorbed ‹ reflected ==> cooler
Sorry again.

Willis Eschenbach
January 17, 2012 2:37 am

Paul Dennis says:
January 16, 2012 at 1:22 am

Willis,
thank you for your considered response. I can only quote from Coombes and Laue, 1985:

“Question: If a vertical column of air which is adiabatically enclosed is in thermal equilibrium, is the temperature the same throughout the column or is there a temperature gradient along the direction of the gravitational field? – Let us assume an idealized model in which there are no interactions between air molecules and no collisions once the equilibrium distribution has been established. The resolution of the paradox below is unrelated to this assumption. Energy conservation mplies that the kinetic energy (KE) of a molecule decreases with its height z above the Earth’s surface according to the formula:
KE = (KE)o – mgz
where m is the mass of the molecule and g is the acceleration due to gravity. Now, the two conflicting answers to the above question are as follows.

Thanks, Paul. I’ll have to think about that some more. The oddest statement is that

The seemingly paradoxical truth is that the average kinetic energy of all molecules does not decrease with height even though the kinetic energy of each individual molecule does decrease with height”

Seems paradoxical to me, all right. I’ll have to ponder some more.
Fortunately, it makes no difference to my proof above.
Many thanks,
w.

Bill Hunter
January 17, 2012 2:49 am

jimmi_the_dalek says:
“The completely transparent atmosphere is not Willis’s invention – it is implicit in the models he is criticising. These models claim that all of the GHE can be explained without any mention at all of absorption or emission of radiation by the atmosphere. It follows that their theories should be true for an atmosphere which is completely transparent. Perhaps the authors of these theories do not realise that they have built this into their model, but they have. So it is legitimate to ask how a completely transparent atmosphere would behave. Willis has given the answer, and shown that the ‘it’s all due to gravity’ models are nonsense.”
Nobody does any reading it appears. In a nutshell Willis claims that if the bottom of the atmosphere is warm the surface will emit and since any emission would be in excess of incoming its a violation of the law of conservation of energy.
But if you read Jelbring he says the model receives no external source of radiation and it radiates nothing. “The energy content of the model atmosphere is fixed and constant since no energy can enter or leave the closed space. Nature will redistribute the contained atmospheric energy (using both convective and radiative processes) until each molecule will have the same total energy”
Basically the entire thing is a planet in a bottle with no external interaction and no misappropriation of energy. and in fact it does mention absorption or emission of radiation in the atmosphere, just not from external sources.or to external sources. Instead the only thing allowed is a redistribution of energy already in the bottle. Absolutely nothing you say about Jelbring’s model even approaches corresponding to the description of it in the paper.
The only thing going on here is a mass reading comprehension problem.

Bill Hunter
January 17, 2012 3:23 am

Specter says:
“The nominal value for this required flow is usually stated as 239 or 240 W/m². If the surface were emitting energy at a higher average rate, say 396 W/m², then something must be extracting 156 W/m² from the *radiant* energy flow going up in the atmosphere and returning it back to the ground. Only greenhouse gases can perform that function on a continuous basis.”
If you check modtran and UofChicago you will find that at 288k upward radiation by the surface in the absence of greenhouse gases is 347w/m2. Thats a function of the surface albedo. Thats a substantial drop. The way numbers get thrown around in the climate game, myself as an accountant gets really antsy. I see lots of proxies being used and virtually nothing to validate them. Stevenson screen proxies for land surface emissions. 3 foot deep water temperature readings for radiating and evaporating surface films. A weather station network never designed to do this job. A bunch of conspirators massaging the data. Completely ignoring the fact the surface film is pumping out 102 watts in a convection conveyor belt. Gee that couldn’t affect the temperature of the radiating surface. . . .or could it?
I have never ever seen anything in my life like it. And I did some pretty extensive restructuring work with collapsed businesses. I have never seen the gall of some people threatening to erase data while speaking with peer pals to avoid the public inspecting it . . . .then to have it actually disappear! I can’t even imagine how anybody could have faith in any number out there.
Seems to me one heckuva lot of money is being thrown away on a lot of stuf that simply doesn’t need to be done. Perhaps we need better ideas of the surface radiating temperatures and maybe we need it globally. But it seems to me the Cloud 9 guys have run far out in front on this deal from the perspective of understanding the basic underlying principles.

jimmi_the_dalek
January 17, 2012 3:26 am

Anna V,
What was the sentence in my post after the one you quoted? Go read it again please.

Joel Shore
January 17, 2012 3:42 am

Ned Nikolov says:

Part 1 focuses on the magnitude of the GH effect, as this topic seems to be the crux for most people including yourself. You will learn (and see physical proof of) that our atmosphere actually raises the surface temperature not by 33K, but by well over 100K! Now, think what are the implications of this fact alone for the current GH theory? Do you know what’s the total amount of GH gases in the atmosphere as a percent of total atmospheric mass? How likely is that handful of gases can raise the near-surface temperature by 133K through ‘back radiation’?

Ned: We have already discussed this; your reply sounds like it will just pretend that all the discussions we’ve had about this don’t exist. 100K of your 33K is just due to the fact that a direct average of T produces very different results for the same value of emitted power depending on the distribution of the temperature across the surface. Nobody but you is claiming that the radiative greenhouse effect raises average temperatures by 133 K. What it does is raise surface emissions from ~240 W/m^2 to ~390 W/m^2. The 390 W/m^2 on the current Earth corresponds to an average temperature of ~288 K as the current temperature distribution is uniform enough that assuming it completely uniform is not a bad approximation.
In the absence of the greenhouse effect, the 240 W/m^2 emissions from the surface would correspond to an average surface temperature of ~255 K or less. A planet without a greenhouse effect will probably have a somewhat less uniform temperature distribution but probably not enough to lower the average temperature that much from 255 K. And, even if it does, this is really not the interesting part of the effect: It is probably best to characterize the temperature of the planet by the fourth root of the average of T^4 instead.
I hope your replies to comments shows that you have learned something from the criticisms of your theory but from what you have been saying so far, I am not at all optimistic that it will.

gbaikie
January 17, 2012 5:01 am

“Could I not say that my walls are kept warm due to air temperature? Can not also say the driveway is also kept warm from air temperature?”
The driveway can be warmed by the air, but it can only do so by transferring heat into it.
And air cools as a result.
And this only way any atmospheric gas can cool down.
At night air cools. So air must have warmed surface to cool.
There is about 10 tons of air above every square meter of the earth surface.
When the air at surface cools 1 C, then roughly all the air cools by 1 C.
One has varying lapse rates with changing condition, but air cools at lower
elevation it also cools in higher elevation.
The specific heat of air is 1.005 kJ/kg per K
So 10,000 kg times 1.005 per lowering of 1 C is 10 million watts seconds.
In the absence of a GHE it cannot increase the amount of heat in the Earth system at any given time.
The temperature of air depends upon it’s velocity. Most air is transparent to any photon CO2 might emit, and such photons would not increase the velocity of gas molecules of air.
“OTOH, a GHE can increase the amount of heat in the Earth system at any given time by causing some IR radiation to be absorbed and re-emitted multiple times before the radiation finally gets high enough to radiate to space.”
If GHE gases are interchanging photons many times this has no affect in term of heat in regards to the surface or gases, the only possible significant is if a photon were to be absorbed by surface.
The general “greenhouse theory” seems to be that photon would first start from the surface and possibly be absorbed by CO2 and possible return to the surface or possibly be emitted into space.
That CO2 holds or interchanges photons has no significance in terms of temperature [the gas molecules aren’t caused to move quicker]. And the amount heating cause by photons re-emitted back to the surface, doesn’t seem like much to me- perhaps someone would like to quantify this amount, I could be enlightened.
But regardless of this, the air cools at night, the cooling can only occur by warming the surface, and there is much more energy stored in velocity of air molecule than most land surfaces [which normally cool during each 24 hr night time period- the heat in say, 4″ of dirt, sand, or concrete].

gbaikie
January 17, 2012 5:29 am

“So let us assume that we have the airless perfectly evenly heated blackbody planet that I spoke of above, evenly surrounded by a sphere of mini-suns. The temperature of this theoretical planet is, of course, the theoretical S-B temperature.
Now suppose we add an atmosphere to the planet, a transparent GHG-free atmosphere. If the theories of N&K and Jelbring are correct, the temperature of the planet will rise.
But when the temperature of a perfect blackbody planet rises … the surface radiation of that planet must rise as well.”
The amount energy from the sun is around 1300 watts.
The term: theoretical S-B temperature for earth, I assume is 240 watts per square meter averaged over entire surface.
For our thought experiment what is the theoretical S-B temperature the chosen planet?
Is it 240 watts? Or is the incoming energy 240 [earth receiving 1300 watts]
If latter can we assume that the theoretical S-B temperature is 240 watt [include choice of Bond albedo and time it by that by 240 Watts] and divide by 4. In which case the theoretical S-B temperature is less than 60 watt per meter per sun?
Or is it 1300 watts and one must have earth’s Bond albedo?
Or did you have something else in mind?

Andre
January 17, 2012 5:48 am

Sorry for being late, but there is an essential oversight, apples and oranges. The surface indeed can’t violate stephan boltzman but the inert non-emitting atmosphere can. And secondly the average surface temperatue on earth is not the actual surface temperature. One is the contact temperature of the soil, while the other surface “temperature” is the temperature of the atmosphere on 1.50 meter height in the thermometer house. When the surface cools due to emission during the night, it cools only the lower boundary layer of the atmosphere, if nothing else happens the heavier cooler gas stays put , and a very strong surface inversion emerges. In an iglo for instance the air at the entrance hole can easy be -20C, while the air at the ceiling is +10 +20C, just 1.5 meter higher.
For more cooling of the atmosphere wind and turbulence have to replace the boundary layer and iin case of a radiative atmosphere the emitting lower atmosphere will heat up the surface/actually decreases the cooling rate of the surface while cooling itself. But in a non radiative atmosphere, the majority of the gas stays as warm as it got in day time due to convection. Hence the thermometer house at 1.5 meters can be warmer than the surface.

dr.bill
January 17, 2012 5:59 am

Shore
I have no idea what you look like, but I picture you with a very robust bone structure and heavily developed exo-skeletal musculature (something like this). Otherwise, I don’t see how you could carry that massive ego around with you…
/dr.bill

AnonyMoose
January 17, 2012 5:59 am

Thanks to the continued discussion, a misunderstanding on my part has become clear. I had thought that the gravity was only being credited with creating a thick atmosphere, and the thicker atmosphere is all that was associated with warmer temperatures. So I thought that the paper was only finding a correlation between the density of the atmosphere and its temperature. I had missed that gravity was being credited with continuously creating heat.

DavidA
January 17, 2012 6:01 am

I was about to argue that the Earth example contradicts Willis’ proof in that the surface temperature can be higher without violating the 2nd law: our surface temperature is higher than S-B though outgoing equals incoming (small variation when catch up is being played for changing forcings).
The difference is the atmosphere above comes into play when it disrupts the outgoing radiation from the surface (no detail – elevator is moving!).
The non-GHG atmosphere won’t disrupt outgoing from the surface so if the surface temperature is higher than the S-B temperature then the outgoing radiation should be greater than incoming – which is a big problem.
I can understand Willis taking the stance he has; there are too many ill explained kook theories doing the rounds on the skeptic side which are quickly embraced as the next ‘final nail’ by the unlearned. That first draft of the Dragon Slayer paper was embarrassing in its claimed 2nd law violation.

January 17, 2012 6:25 am

dr. bill,
I always pictured joel like this.☺

Trick
January 17, 2012 6:26 am

jimmi_the_dalek says 1/16 9:20:
“PS to Trick above – a transparent atmosphere does not radiate anywhere, even at TOA.”
Jimmi – The transparent GHG-free atmosphere is still made of matter so will thermally radiate/absorb w/space ~3K at TOA. All matter with temperature above absolute zero emits thermal radiation.
Any atmosphere above absolute zero can no longer thermally conduct to space at TOA.

Crispin in Waterloo
January 17, 2012 6:36 am

jimmi_the_dalek says:
January 16, 2012 at 11:08 pm
Crispin,
“There is no such thing as a non-GHG atmosphere that does not emit radiation so the thought experiment is not going far.”
On the contrary, the thought experiment shows precisely what it is supposed to, namely that a model in which the radiative properties of the atmosphere do not exist, cannot be correct. So, instead of criticising Willis’s thought experiment, you should be asking Nikolov and Zeller why they omitted these factors.
++++++++++++
Oh, I have no problem imagining impossible atmospheres. It is of course pointless to discuss them because one could simply plonk down additional impossiblities such as changing other physical laws or making Miss Piggy the Secretary of the UN.
BTW thanks Anna V, again.
I am not sure it is entirely Willis’ thought experiment though. I understood he was looking at root of claims made by others. What has emerged after a good deal of discussion is that the postulated atmosphere either could not exist or needs to be restated so that it could exist with different definitions, for example, that is does not absorb IR but can emit radiated energy.
If the idea persists that there is such a thing as a non-GHG gas atmosphere, where non-GHG means ‘not absorbing or radiating or re-radiating energy’ then it is contrary to nature so investigation of black body surface radiation under different gravity-pressure scenarios leads nowhere.
I found the exercise very beneficial, however. For me it lays to rest forever the idea that only certain gases labelled GHG’s heat the Earth. I already knew there is no such thing as a ‘greenhouse effect’ per se but that has a very different explanation. The assertion that the Earth’s atmosphere is primarily heated by CO2 and would be in a permanently frozen state in its absence is bunk. Ergo, the suggestion that building windmills to replace coal-fired power plants will in any way dampen temperature fluctuations upward is sad, unless you are in the windmill business. Et cetera.

dr.bill
January 17, 2012 7:00 am

re Smokey, January 17, 2012 at 6:25 am :
I don’t want to be disagreeable, Smokey, but that’s my picture of “Phil.dot” you’re parading around there. ☺
/dr.bill

January 17, 2012 7:08 am

dr. bill,
They’re interchangeable, no?

dr.bill
January 17, 2012 7:19 am

re Smokey, January 17, 2012 at 6:25 am :

They’re interchangeable, no?

Well, they do seem to arrive and leave together, and they’re pretty much equally obnoxious, but I don’t think Phil.dot is the brighter of the two bulbs. ☺☺
/dr.bill

January 17, 2012 8:04 am

Willis says: “I hold it can be proven that there is no possible mechanism involving gravity and the atmosphere that can raise the temperature of a planet with a transparent GHG-free atmosphere above the theoretical S-B temperature.”
That’s a valid statement but it does not invalidate the crux of what N and Z are telling us.
On a planet with spatially variable temperature the warmed atmosphere moving from equator to pole returns energy to the surface. Similarly the ocean gains energy in a warm location and releases it in a cooler location. It’s a feedback loop that raises average surface temperature due to lateral energy transport. The average radiation from the surface will reflect this feedback loop.
All atmospheres possess particulates and molecules that mediate the return of radiant energy to the surface and it appears (in terms of a lack of surface warming after 1998) that the partial pressure or relative concentration of these receptor molecules (above a tiny minimum threshold) is irrelevant. I instance the efficacy of ozone in reversing the decline of temperature with altitude at the tropopause when it is present at about 1ppm.
The denser it is, the greater is the energy that the atmosphere accumulates and the more effective it is in transferring energy. The average temperature as indicated by a thermometer will reflect that. If there is no atmosphere, there is no warmth at two meters and no redistribution.
Given the necessary minimum level of greenhouse gas to facilitate the transfer it is the bulk density of the atmosphere that determines the ‘average’ surface temperature.
That is the novelty and value of the Nicolov and Zeller presentation.
But, none of this is has any great relevance since it is the cloud cover that mediates the incidence of solar radiation changing within the scale of a human lifetime, and this is also a point that N and Z make.
Willis, I want to make a few comments on the manner in which this discussion has been handled.
As for the snips the role of the teacher is to keep the discussion on track and so I see the snips as necessary and desirable. The only pity is that you had to do it yourself. The nice thing was that you indicated where the snips had occurred. That does not seem to happen at Real Climate.
As for the tone of your replies: Well, when you say ‘Good for you’, its disparaging and a teacher who seeks to encourage would not say that. It tends to alienate people. It is useful to discourage the real time wasters and it’s perhaps not so big a deal in the adult world and on a blog like this where some delight in being insulting while adding nothing to the discussion. These types need firm handling because they are thoroughly disrupting. You can mix it with the best of them but a better policy might be to have a third party insist that the comment be cleaned up before its posted.
This has been a very successful post. I’ve read nearly all the comments and learned a lot. And you are a good teacher.
There is nothing quite like a good hypothetical.

DeWitt Payne
January 17, 2012 8:17 am

Bart,

When I integrate, for example, the function in Figure 4a over the hemisphere, I get something like 0.8 as the total effective emissivity. If I assumed that the Earth were all ocean with this as the effective emissivity, that takes it down from equivalent radiated energy of 390 W/m^2 to 312 W/m^2.

Then you’re doing the integration incorrectly. The emissivity doesn’t drop below 0.9 until you get to an angle between 65-70 degrees. 90% of the area of the hemisphere is covered by a solid angle of 130 degrees. The emissivity graph doesn’t start until 20 degrees. So conservatively the emissivity between -65 and 65 degrees is greater than 0.96. That means the total emissivity is at least 0.86 if the emissivity were zero for angles outside that range. It’s not. Plot the full emissivity curve as a function of sine(θ) and then integrate.

DeWitt Payne
January 17, 2012 8:23 am

Bart,

The heat equation says that for equilibrium, the Laplacian must be zero. There are no solutions in spherical coordinates which are increasing with altitude. Hence, such a configuration is unstable.

That’s the Laplacian of the temperature, i.e. the kinetic energy, not the total energy. The Laplacian of an isothermal atmosphere is indeed zero. If the temperature were increasing with altitude, you would be correct. Heat would flow downward driven by the temperature gradient. The same thing is true if the temperature decreases with altitude, heat flows upward. There is no thermal equilibrium if there is a difference in temperature. And that’s temperature, not potential temperature.

Leonard Weinstein
January 17, 2012 8:27 am

Anna & others that think a non-greenhouse gas emits thermal radiation:
Yes there is some radiation even from non-greenhouse gases, but it is so small compared to the ground radiation and greenhouse gas radiation that it can be considered zero for the analysis. Keep in mind that a room full of air has a finite probability to all move into one end of the room and suffocate you. I don’t think that would make you worry. When probabilities many orders of magnitude smaller than important details are invoked, you are really grasping at straws.

January 17, 2012 8:38 am

Thanks to the continued discussion, a misunderstanding on my part has become clear. I had thought that the gravity was only being credited with creating a thick atmosphere, and the thicker atmosphere is all that was associated with warmer temperatures. So I thought that the paper was only finding a correlation between the density of the atmosphere and its temperature. I had missed that gravity was being credited with continuously creating heat.
Which leads, incidentally, to a twenty second demonstration that the system violates the e.g. heat engine or refrigerator statement of the second law of thermodynamics, because gravity, in fact, does not create heat. It can only help move heat created some other way around. In fact, the assertion that it can spontaneously polarize temperatures and maintain that polarization without the input of actual heat is a violation of the refrigerator statement — it doesn’t need help! No cyclic process can have the sole effect of moving heat from a cold reservoir to a hot reservoir. End of story.
For the umpty-zillionth time — in a static atmospheric profile (one where the density variation is fixed) gravity is doing no net work on the system. In an atmosphere in thermal equilibrium the temperature of the atmosphere is constant. A lapse rate only exists because the atmosphere is not in equilbrium, and there is differential bulk transport of heat away from the surface that maintains disequilibrium. If the upper atmosphere is not permitted to cool via radiation, this heat transport continues until there is no lapse rate for a superconducting/non-rotating Earth, or it drives lateral transport of heat from warmer regions to cooler ones to increase the net cooling of the planet relative to the initial condition of no transport but yes differential heating. This transport cools the tropics, warms the poles, and produces net warming on average. This transport is the only mechanism I can see so far that affects the actual average temperature, and that not relative to an ideal black body but because it drives the non-ideal (but overall cooler) rotating Earth absorber towards the warmer ideal blockbody by enabling not superconduction of heat but relatively weak transport of heat.
Willis’ thought experiment is dead on the money. Given a superconducting ideal absorber Earth (the baseline for most discussions of GHG warming) and a perfectly transparent non-GHG atmosphere, the atmosphere is completely irrelevant if you block or ignore lateral transport of heat — it warms to a uniform temperature from top to bottom that matches the surface temperature underneath and is as irrelevant to gain or loss otherwise as the other non-radiating matter in thermal contact with the surface just beneath the surface. There is no lapse rate in such an atmosphere. It cannot gain or lose heat anywhere but at the point of contact with the surface, and internally it shares heat until it is driven to a constant temperature.
As for people who are asserting that gravitational potential energy is somehow equivalent to heat, as if a falling body heats up as it moves — OMG. Please, take a short course in thermodynamics. Get a friggin’ physics book and read the thermo chapters. Learn the difference between reversible and irreversible processes. Just FYI, the difference between “heat” and “internal energy” is entropy. Heat flow into a system is always accompanied by the exchange of entropy. This connects it with the Second Law (entropy statement) via Clausius’ Theorem.
To put it in a way anybody can understand, if you drop a rock from rest it doesn’t change temperature at all as it falls, but when it inelastically collides with the ground at the bottom some of its initial potential energy relative to the ground is converted to heat – nearly all of it for a rock much smaller than the Earth. But this conversion is a one way thing. It creates a hot spot where it hits, and the heat then spreads out into the entire system, warming it until it is at a uniform temperature again. It is then not “impossible”, but more or less infinitely improbable that this disorganized kinetic energy in the internal energy of all of the molecules will combine “just right” to kick the rock back up into the sky to its original height.
When you understand that statement, you will begin to understand entropy and why heat will always spontaneously flow from warmer to cooler given a channel between them capable of carrying energy, and why one requires a “Maxwell’s Demon” that can decrease entropy, sort out faster molecules and slower molecules so that detailed balance is not maintained in order to create a hot and cold separation. It isn’t even as likely as the separation of black and white dyed salt grains into nicely separated layers as you shake a container — a lot more atoms, a lot less likely (although on the other hand, nearly zero equals nearly zero on all scales).
In fact, read:
http://en.wikipedia.org/wiki/Maxwell%27s_demon
and learn. It is even relevant to human consciousness and quantum theory, as measurement processes always involve irreversible entropy increases, something that leads to several silly paradoxes and nonphysical “processes” (e.g. “wavefunction collapse”) in the discussion of open quantum systems.
rgb

shawnhet
January 17, 2012 10:32 am

Bart says:
January 16, 2012 at 11:09 pm
“I have not questioned the SB relationship at all. I have questioned the shallow manner in which it is being applied. The difference between us appears to be, I know how it is derived, and the conditions upon which its successful application depends. You, however… apparently not so much.”
So show me an instance where the correct (IYO) application gives the answer you are expecting under your hypothesis. Again, you are the one claiming that the evidence underlying the current view of the radiative state of the *entire* atmosphere is faulty. If this were true, it should be relatively easy to find some evidence of it. The atmosphere is a very big place.
Cheers, 🙂

Spector
January 17, 2012 10:44 am

RE: Bill Hunter:(January 17, 2012 at 3:23 am)
“Specter says:
“’The nominal value for this required flow is usually stated as 239 or 240 W/m². If the surface were emitting energy at a higher average rate, say 396 W/m², then something must be extracting 156 W/m² from the *radiant* energy flow going up in the atmosphere and returning it back to the ground. Only greenhouse gases can perform that function on a continuous basis.’
“If you check modtran and UofChicago you will find that at 288k upward radiation by the surface in the absence of greenhouse gases is 347w/m2. Thats a function of the surface albedo. Thats a substantial drop. The way numbers get thrown around in the climate game, myself as an accountant gets really antsy. I see lots of proxies being used and virtually nothing to validate them.”

In this case the situation is very simple: If the Earth is hot enough to have a global average surface radiant energy flow of 396 W/m², then something in the atmosphere must capture an average 156 W/m² from this radiant flow before it gets away. That is because the average energy actually flowing from the whole planet must be the nominal 240 W/m² required by conservation of energy.
There is no reference to temperature. There is no reference here to MODTRAN. The 396 value comes from the Trenberth diagram. Again, no reference to temperature is required as this is a simplified energy balance equation. It is not a temperature balance equation. No atmospheric absorption of solar energy is considered here.
The standard value of 240 W/m² assumes a 30 percent reflection of incoming solar energy and 100 percent LWIR surface emissivity.

Bill Hunter
January 17, 2012 10:49 am

Dr Brown says: “it warms to a uniform temperature from top to bottom that matches the surface temperature underneath and is as irrelevant to gain or loss otherwise as the other non-radiating matter in thermal contact with the surface just beneath the surface. There is no lapse rate in such an atmosphere. It cannot gain or lose heat anywhere but at the point of contact with the surface, and internally it shares heat until it is driven to a constant temperature”
But when you introduce a rotating planet the surface is always changing temperature. That combined with the fact in a passive solar water heating system with well-insulated storage above collectors you can create a natural convective loop that warms the water in the tank to well above the average temperature of the surface over periods of days. In fact because outside the system convection exists the water system gets warmer than even the warmest days.
But if you start stripping insulation off of the water storage the temperature you can obtain goes down in this system. You can boost the heat by greenhousing the collectors and eliminate immediate heat loss but some folks just lay out black pipe and get comfortable shower temperatures in the system well above the ambient temperature outside at the peak of the day with a much more economical system.
The system comprises of black copper piping arrayed into a collector. The pipe is insulated as it runs up the hill to the storage tank that is also well insulated. Heat loss at night is primarily through the insulation as even with water conductive heat down the pipe is rather minimal (and water is about 24 times more conductive than air). So that as an analogy it seems logical that the following is true for a non-radiating atmosphere in our world: 1) it would be a lot warmer than the average temperature everywhere with the warmth of the tropics overwhelming the poles; 2) At near equilibrium (true equilibrium would never be reached) Convection would be minimized though not completely eliminated due to the pulsing of ground temperatures as the sun comes up and goes down (convection would be minimized because of the very low heat losses in the system, the surface would get very cold at night but as a system very little; 3) conduction losses to the ground would be measurable but rather insignificant to overall warming that was generated. 4 )as I effectively strip off insulation in the storage vessel (the atmosphere) and introduce radiating gases to the atmosphere the atmosphere would cool in proportion to how much the radiation capabilities of the atmosphere goes up. 5) by adding another .04% of a radiating gas that is .125 efficient I am going to very slightly cool the system. 6) that the moist lapse rate is lower than the dry lapse rate is purely and moreso a function of latent heat transport, otherwise as purely a radiator it would be increasing the lapse rate; and 7) by assumption that the 30 degree greenhouse effect we have today is the residuals in the system from not making the atmosphere’s thermal conductance to space as conductive as the ground. After all these passive water heating systems work in Alaska too at least when the sun is shining..
So Dr Brown I have read your posts with fascination, though a lot of it was over my head. Where have I gone wrong above? This is relevant to me in viewing Jelbring’s model as I have to break through intuition that arises from simple analogy models like that above.
.

jimmi_the_dalek
January 17, 2012 11:04 am

Trick,
When you (and many others) say “The transparent GHG-free atmosphere is still made of matter so will thermally radiate/absorb w/space ~3K at TOA. All matter with temperature above absolute zero emits thermal radiation.”
the problem is not your physics but your use of the English language. The word “transparent” means “does not absorb”. What you should be saying, in my opinion, is not “transparent gases absorb radiation”, but “there is no such thing as an absolutely 100% transparent gas”. Which is fine, but irrelevant. What you should be asking, as I see it, is whether Nikolov et. al (or Hoffman), by claiming that the GHE can be deduced purely from gravitational considerations, have made an assumption which is equivalent to assuming transparency, because if they have, then they have built a non physical component into their theory.

Trick
January 17, 2012 11:06 am

Robert Brown says at 1/17 8:38am:
“If the upper atmosphere is not permitted to cool via radiation, this heat transport continues until there is no lapse rate…”
Willis 2nd mechanism doesn’t specify “upper atmosphere is not permitted to cool via radiation” which is unnatural. If Willis’ 2nd mechanism upper atmosphere is made of matter (N2, O2) above zero absolute temperature initially by conduction from the BB surface (& it is by his specification) then his upper atmosphere can radiate heat away to space as all matter > 0K can do & that energy radiating away is equal to incoming energy, thus there is a lapse rate in real nature & energy conservation.
See Willis top quote verbatim: “Now suppose we add an atmosphere to the planet, a transparent GHG-free atmosphere.”
Robert Brown also says:
“…(transparent non-GHG) atmosphere warms to a uniform temperature from top to bottom that matches the surface temperature underneath…”
Again, no lapse rate is not a property of natural transparent non-GHG atmosphere real matter &”no lapse rate” is not specified in Willis’ 2nd mechanism.
Willis’ 2nd mechanism planetary has planetary BB at surface T – delta T plus transparent non-GHG atmosphere in the presence of gravity arrives at T + delta T by conduction from the surface. All matter (O2 and N2 included) above absolute zero emits thermal radiation to adjust Willis’ 2nd mechanism to one observing conservation of energy.
This means Robert Brown must agree with N&Z for lapse rate in Willis’ 2nd mechanism which will be warmer near the surface (T + delta T) due to conduction + non-GHG atmospheric matter’s radiation than the thermally radiating only Willis’ 1st mechanism planetary BB at surface T.
This delta T comes from conduction & gravity causes that by acting on atmosphere with ideal gas law (if shut off gravity then get cooler near surface and eventually go back to 1st mechanism, no atmosphere and no delta T). This means Willis’ proof which doesn’t consider conduction, is not of nature and does violate energy conservation as Willis’ writes. Willis needs to include thermal conduction in his 2nd mechanism for a valid Q.E.D, if there is one – I don’t see it.
Summary: Willis’ 2nd mechanism w/radiation at TOA will emit same as Willis 1st mechanism at surface, no energy conservation violation. Sun energy in = sun energy out both 1st & 2nd mechanisms. But through conduction, gravity & the ideal gas law Willis 2nd mechanism atmosphere near the surface will be at T + delta T and the BB surface will be at T – delta T.
N&Z put a number to delta T in a 3rd mechanism that excludes radiation only for simplicity, Robert Brown may disagree with the number. Tell us why when you have studied up on it.
Note: CO2 (which doesn’t exist in Willis’ 2nd mechanism) especially has a huge peak to emit/absorb thermal radiation in the IR band so that’s even another mechanism, the GHGs.
Sorry, wordy: this is no elevator speech since mine for N&Z stands above at 1/16 2:46pm.

Phil.
January 17, 2012 11:17 am

anna v says:
January 16, 2012 at 9:05 pm
Phil. :
January 16, 2012 at 12:43 pm
Absorption and emission are symmetric conditions quantum mechanically. If a line exists it can be excited, meaning it can absorb the same line. The ambient electromagnetic spectrum is a continuum as the black body radiation shows (frequency is 1/wavelength).

OK so far.
The high tail of the kinetic energy distribution always has enough energy to excite and ionize some of the gas during a collision , though it would be quite rare.
That would be the understatement of the century! Taking your example of Argon, although the result would be about the same for N and O:
If we calculate the Boltzmann distribution for 300K and approximate the energy needed to ionize Argon as 15.8eV then the fraction having energy above that threshold is about 10^-139.
I.e. the chance of it have happened once between the birth of Christ and now in our atmosphere is less than 10^-100, quite rare indeed!
Argon, as all molecules will also have the continuum radiation arising from the collisions distorting its field and creating continuum levels that can absorb and emit radiation.
There is no physical bulk matter that will not radiate as a black body.

I assume you mean a grey body? As I’ve pointed out here many times the emission of N2 is many orders of magnitude less than CO2 and H2O in the Earth’s atmosphere, as is the collisional emission. Anna you claim to have a physics education, you should know better than to post this nonsense, it’s the sort of thing expected from Smokey!

Phil.
January 17, 2012 11:23 am

Leonard Weinstein says:
January 17, 2012 at 8:27 am
Anna & others that think a non-greenhouse gas emits thermal radiation:
Yes there is some radiation even from non-greenhouse gases, but it is so small compared to the ground radiation and greenhouse gas radiation that it can be considered zero for the analysis. Keep in mind that a room full of air has a finite probability to all move into one end of the room and suffocate you. I don’t think that would make you worry. When probabilities many orders of magnitude smaller than important details are invoked, you are really grasping at straws.

Welcome to the club Leonard, it’s a game of whack-a-mole, but the moles keep coming back up!

Bart
January 17, 2012 11:35 am

DeWitt Payne says:
January 17, 2012 at 8:17 am
“Then you’re doing the integration incorrectly. The emissivity doesn’t drop below 0.9 until you get to an angle between 65-70 degrees. 90% of the area of the hemisphere is covered by a solid angle of 130 degrees.”
You have gotten your angular coordinates reversed. In these plots, the observation angle is at zero degrees when you are at the top of the hemisphere. The incremental area ratio is sin(theta)*d_theta, where theta is the observation angle. The integral from zero to 65 degrees is 1 – cos(65) = 0.58, so 58% of the area is in what you call the “solid angle of 130 degrees”, by which I assume you mean the area of the sphere bounded by its intersection with a 130 degree cone.
DeWitt Payne says:
January 17, 2012 at 8:23 am
“That’s the Laplacian of the temperature, i.e. the kinetic energy, not the total energy.”
The total energy includes the potential energy of the gravity well, AND the orbital energy of the individual particles which, when they are not colliding with one another, are in orbit about the center of the spherical mass of the planet. This energy is a constant, therefore it can be neglected.
The atmosphere is not homogeneous, because of the change in density. But, the density is decreasing exponentially, while incremental volume of progressively higher altitude shells is increasing with radius squared. No matter how you scale it, you cannot ratio out an exponential function with a quadratic factor.
“The same thing is true if the temperature decreases with altitude, heat flows upward.”
Yes, that is something which has been mostly neglected in the discussion. Heat is flowing upward due to the conductive interface with the surface. I touched on the here and the next two subsequent posts.
So, what happens to that upward heat? It is fairly clear that it will accumulate until significant ionization occurs, which allows for electrical discharge and ionized air glow, releasing bundles of extremely high energy photons in the UV range (recall that UV photons are on average three orders of magnitude more energetic than IR photons, so it doesn’t take a lot to balance things).
This is the problem with the “thought experiment” process. It can help you familiarize yourself with concepts, but it is not a real experiment and, if you’ve neglected something important like the electrical characteristics of matter, your conclusions may bear little relationship to reality.
In the end, all these random discharges will make the system effectively chaotic, so there really is no equilibrium of any kind ever established. Which leads us to:
“There is no thermal equilibrium if there is a difference in temperature.”
Bingo!

Joel Shore
January 17, 2012 11:39 am

dr bill says:

Shore
I have no idea what you look like, but I picture you with a very robust bone structure and heavily developed exo-skeletal musculature (something like this). Otherwise, I don’t see how you could carry that massive ego around with you…

See here: http://wattsupwiththat.com/2012/01/17/thanks-and-apologies/#comment-867434

Bart
January 17, 2012 11:57 am

Phil. says:
January 17, 2012 at 11:23 am
Leonard Weinstein says:
January 17, 2012 at 8:27 am
“Yes there is some radiation even from non-greenhouse gases, but it is so small compared to the ground radiation and greenhouse gas radiation that it can be considered zero for the analysis.”
Prove it. UV radiation is 1,000 times more energetic on average than IR, so even if it is 0.1% of the total, it is just as great an energy mover.
It is likely that, to the extent that GHGs dissipate energy from the atmosphere, they are only displacing what would otherwise be radiated away from other processes. So, even if you can argue that, on Earth, there is no other significant radiation, it does not provide information for what would happen with the GHGs removed, i.e., if their effect would be compensated.

anna v
January 17, 2012 12:04 pm

Phil.
January 17, 2012 at 11:17 am
You are being offensive, but I will by pass it.
No radiation means no radiation, if the rate is small, it will take longer to cool, but cool it will.. Physical matter radiates as a grey body ( and if you had read my earlier comments you would see that yes, I mean emissivity times black body) and cannot be used as an argument in a proof by contradiction.
I am amazed that people can think that bulk matter composed of atoms and molecules at a temperature T will not radiate.

Bart
January 17, 2012 12:13 pm

Robert Brown says:
January 17, 2012 at 8:38 am
“…it warms to a uniform temperature from top to bottom”
Impossible in a spherical distribution. See my comments elsewhere.
“If the upper atmosphere is not permitted to cool via radiation, this heat transport continues until there is no lapse rate for a superconducting/non-rotating Earth, or it drives lateral transport of heat from warmer regions to cooler ones to increase the net cooling of the planet relative to the initial condition of no transport but yes differential heating.”
Given that it is impossible for there not to be a lapse rate, this rings of the old “irresistable force meeting the immovable object” conundrum. It is a signal that something is physically untenable in the thought experiment.
I have found an answer: ionization of the upper reaches of the atmosphere can produce the electrical discharge needed to move the heat energy away.

shawnhet
January 17, 2012 12:36 pm

Bart:”You have gotten your angular coordinates reversed. In these plots, the observation angle is at zero degrees when you are at the top of the hemisphere. The incremental area ratio is sin(theta)*d_theta, where theta is the observation angle. The integral from zero to 65 degrees is 1 – cos(65) = 0.58, so 58% of the area is in what you call the “solid angle of 130 degrees”, by which I assume you mean the area of the sphere bounded by its intersection with a 130 degree cone.”
I could be wrong here but I believe the plots you are referring to from Science of Doom are referring to the angle and changes of a satellite rotating around the Earth and Dewitt is referring to the emissivity changes following from the Earth’s movement around the Sun. It seems obvious to me because the satellite is much closer to the Earth much greater changes in the relevant angles are possible. OTOH, I may be missing the point entirely 😉
Cheers, 🙂

jimmi_the_dalek
January 17, 2012 12:56 pm

Anna V and Bart
“cannot be used as an argument in a proof by contradiction”
“It is a signal that something is physically untenable in the thought experiment.”
Of course there is something wrong with the thought experiment. That is the whole point!
General structure of the argument
1. start with a postulate
2. follow said postulate to its logical conclusion
3. check conclusion against known physics
and now
a) either the conclusion agrees with known physics, which is actually the uninteresting case because it does not even show that the initial postulate is correct, only that it is not obviously wrong
or b) the conclusion is incompatible with known physics, which is the interesting case, because it does show that something is wrong with the starting point.
So in this case the assumption that the atmosphere is completely 100% transparent leads to nonsense. So that must be wrong. Now you have to decide whether Willis’s assertion that the gravity only model, neglecting radiation entirely, is equivalent to claiming that the atmosphere is transparent. Well, is it?

Editor
January 17, 2012 1:48 pm

Willis, re N&Z – you can only get energy from gravity once. If N&Z are correct, then I’ve got a nice gravity power plant for you: http://www.peswiki.com/index.php/Directory:_Hidro_–_Water_Pressures_Energy_Conversion_%28WAPEC%29
NB. We’re talking ‘elevator argument’ so don’t get picky with the word “get” above.

Bart
January 17, 2012 1:59 pm

shawnhet says:
January 17, 2012 at 12:36 pm
“I could be wrong here…”
You are. The experiment was on the ground. Read up on it at the links provided in previous posts.
jimmi_the_dalek says:
January 17, 2012 at 12:56 pm
“Well, is it?”
I resolved the problem. The atmosphere will conduct heat to the top, where it will accumulate until it activates a release mechanism. There really is no other physical possibility.

DeWitt Payne
January 17, 2012 2:15 pm

Bart,
You’re right and I was wrong about the area. But the emission varies by cosine(θ) as well because the effective emitting area goes to zero at 90 degrees from the zenith. So 82% of the total energy of emission at an emissivity of 1 is collected in a solid angle of 65 degrees from the zenith (that was another mistake). I still don’t see how you get an emissivity of 0.8 for the ocean surface.

So, what happens to that upward heat? It is fairly clear that it will accumulate until significant ionization occurs, which allows for electrical discharge and ionized air glow, releasing bundles of extremely high energy photons in the UV range (recall that UV photons are on average three orders of magnitude more energetic than IR photons, so it doesn’t take a lot to balance things).

That’s a bizarre contention. It’s an obvious violation of the Second Law. The temperature of the upper atmosphere under the conditions described by Willis cannot exceed the temperature of the surface. You get ionization in the upper atmosphere of the Earth because of absorption of ultraviolet radiation from the sun under conditions where the mean free path of an ion or molecule is measured in kilometers, not from heat transfer from the surface.

…it warms to a uniform temperature from top to bottom
Impossible in a spherical distribution. See my comments elsewhere.

Again, not impossible at all. The Laplacian of a constant is zero regardless of the coordinate system and in the heat equation one takes the Laplacian of the temperature, not the total energy. The temperature isn’t increasing with altitude, it’s constant.

John A. Fleming
January 17, 2012 2:34 pm

I didn’t see it explained like this, so I’ll try.
Since insolation is uniform, surface temperature is uniform, there will be no convection.
Since atmosphere is thermal longwave transparent, it neither absorbs or emits in the IR. The atoms do emit and absorb photons, just not at IR.
A constant-temperature everywhere atmosphere is suddenly introduced, and the temperature is lower than surface temperature. The atmosphere will be heated by conduction. The surface temperature will cool, until the atmosphere at the surface reaches surface temperature equilibrium. The increased-thermal energy atmosphere atoms will diffuse, raising the surface pressure. The surface area of the outer-edge of the atmosphere will increase. The total emission of non-IR photons by the atmosphere will be increased over the initial state.
A simpler initial condition is the atmosphere everywhere at surface temperature. The atmosphere will then begin radiating photons at the air/space boundary at its preferred frequencies. By diffusion, the surface temperature will have to cool to keep energy balance.
A non-GHG atmosphere then cools a planet’s surface temperature, by transforming outgoing IR radiation to other frequencies. In/out energy remains in balance.
This result does not extrapolate to non-uniformly heated, partially GHG atmosphere.

Bart
January 17, 2012 2:41 pm

DeWitt Payne says:
January 17, 2012 at 2:15 pm
“So 82% of the total energy of emission at an emissivity of 1 is collected in a solid angle of 65 degrees from the zenith (that was another mistake).
82% is the total integral. I made it 80% because the four plots were variable in how they rolled off, and I figure that 2% is within experimental error.
“It’s an obvious violation of the Second Law. The temperature of the upper atmosphere under the conditions described by Willis cannot exceed the temperature of the surface.”
In fact, I have already told you the temperature gradient has to be downward with altitude. But, what is the temperature of the surface? That is what is to be determined in the first place, so you are in a vicious circle.
“You get ionization in the upper atmosphere of the Earth because of absorption of ultraviolet radiation from the sun under conditions where the mean free path of an ion or molecule is measured in kilometers, not from heat transfer from the surface.”
On Earth, because it does not get hot enough, because we have GHGs to cool us off.
“The temperature isn’t increasing with altitude, it’s constant.”
Impossible. I have already explained. Go back and review.

shawnhet
January 17, 2012 2:43 pm

Bart says:
January 17, 2012 at 1:59 pm
shawnhet says:
““I could be wrong here…”
You are. The experiment was on the ground. Read up on it at the links provided in previous posts.”
Ok, 😉 I was wrong about it being a satellite in space, (it was from a fixed oilrig), but my overall point is still valid IMO. You can’t compare how the emissivity changes in this particular setup to how the emissivity changes for the Earth rotating around the Earth.
Cheers, 🙂

Bart
January 17, 2012 2:44 pm

I have already told you the temperature gradient has to be downward with upward altitude, in case there is confusion in what I mean.

Myrrh
January 17, 2012 2:44 pm

Willis says: Let me give you an example to show what I mean. Here’s an elevator speech about the greenhouse effect:
The poorly-named “greenhouse effect” works as follows:
• The surface of the earth emits energy in the form of thermal longwave radiation.
• Some of that energy is absorbed by greenhouse gases (GHGs) in the atmosphere.
• In turn, some of that absorbed energy is radiated by the atmosphere back to the surface.
• As a result of absorbing that energy from the atmosphere, the surface is warmer than it would be in the absence of the GHGs.
OK, that’s my elevator speech about why the Earth is not a block of ice. Note that it is not just saying what is happening. It is saying how it is happening as well.
===
Dear Willis,
An elevator speech for you
First build elevator
Press button to call elevator
Enter
Choose level on which to exit by pressing corresponding button
Take a ride
When elevator stops and doors open on chosen floor, exit
And take your stupid greenhouse effect design with you.
The problem with your elevator speech is that the design you’re describing is a house built on a hole in shifting sand because you’re using concepts based on figures which have been deliberately manipulated to lead the unwary through a labyrinth of ever more nonsensical physical properties and processes to believe the design you’ve described as if it is fact, but which relates not in any part to the real physical world around us, it is a description of a fictional effect in a fictional world. You’ve been had. That you believe it doesn’t mean we should follow you in your delusion, but we should call you on it even at the risk of being at the receiving end of your sharp though sometimes witty tongue, because you produce no proof to back up that design. You produce no proof because there is none, because the greenhouse effect you give was designed purely to confuse, to con, not to enlighten us about the real physical world around us. Prove it. Show us your working out.
Here’s why you won’t be able to, and why no one else claiming this greenhouse effect represents the real physics of the world around us has EVER given any proof of the basic premise of your design or any of its parts, it begins with the figures you use of Earth’s temperatures as Latour summarised here:

GHG Theory 33C Effect Whatchamacallit
Pierre R Latour, PhD, Houston, January 15, 2012
“GHG Theory was invented to explain a so-called 33C atmospheric greenhouse gas global warming effect. In 1981 James Hanson, stated the average thermal T at Earth’s surface is 15C (ok) and Earth radiates to space at -18C (ok). Then he declared the difference 15 – (-18) = 33C (arithmetic ok) is the famous greenhouse gas effect. This is not ok because there is no physics to connect these two dissimilar numbers. The 33C are whatchamacallits. This greenhouse gas effect does not exist.”

Note that well, there is no physics ever given to explain the 33°C difference between these two temperatures. You will not find it because it doesn’t exist and so you and your ilk can never produce it when asked, no matter how many times you and your ilk pushing this junk science fiction have been requested to produce it. Show us the hypothesis!
The whatchamacallits are sleights of hand. Watch carefully to see how Hansen fools you.
The figures Latour OK’s are bog standard industry figures, but there’s one missing. It’s importance in this con will be seen once the descriptions of what each relates to is known and the missing re-instated, as follows:
Earth with atmosphere as we have it now: 15°C
Earth without any atmosphere at all: -18°C
Earth with atmosphere but with no water: 67°C
As I’m sure you spotted immediately on reading this, Hansen has taken out the Water Cycle from these standard figures. The main greenhouse gas water vapour through the water cycle reduces the temperature of the Earth with our atmosphere by 52°C from the temperature the Earth would be without it, so bringing it down to the 15°C.
There is no Water Cycle in the AGW Science Fiction Incs KT97 and ilk’s energy budget.
And thus, no way can Hansen find any real world physics to account for the 33°C increase from -18°C to 15°C, except by making absurd claims about the properties of carbon dioxide and mangling real physics processes.
From this, from this sleight of hand and from this alone, we have it pushed down our throats that there is some woolly blanket affect of ‘greenhouse gas warming’ by a huge 33°C created by a trace gas, essential for all life and now called a poison, back-radiating to warm the surface, or, trapping heat in the atmosphere stopping heat escaping, and, the threat of worse to come, because the con says the heat will build up further as this trace gas carbon dioxide accumulates – properties and processes impossible for the real gas carbon dioxide which is anyway not 99.96% of the atmosphere. Gosh, such a supermolecule to raise the Earth’s temperature 33°C and make an insulating blanket out of nothing.
Instead of an explanation and proof we get obfuscation and ad homs for daring to question the fictional fisics because there never is any actual hypothesis produced capable of explaining how this extraordinary greenhouse effect is possible, instead the sleight of hand avoids producing it by misdirection, by insisting it is all based on ‘already proven well-known science’, but never fetches it.
Whatever mangling of physical properties and processes can be brought into the mix to support this unsubstantiated claimed ‘greenhouse effect’ suffices for AGWCon’s common purpose – to confuse the real world physics so that people like you can give it credibility by claiming it real, because you’ve never properly investigated it yourself, have you? Or maybe you have, and all you are is like minded with the con artists promoting it and only pretending to sceptic leanings, we’ve recently had such a pretend skeptic exposed. So which are you Willis? Too full of yourself to properly investigate or in the pay of big oil and the bwankers…?
Without the Water Cycle the Earth would be 67°C, water vapour is the main greenhouse gas, it cools the Earth, think deserts. Carbon Dioxide is fully coupled to the Water Cycle. THEREFORE, GREENHOUSE GASES COOL THE EARTH.
You’re pushing the opposite moronic claim and because you have no physics to show for your preferred version you use the idiotic claims about properties and processes which have been deliberately created to back up this sleight of hand, such as back-radiation heating the surface, regardless how many times it’s been explained by applied scientists that such a thing can’t happen because it leads to perpetual motion. That you can no longer appreciate the difference between Heat and Light, if you ever did, because the physics of radiation has been so thoroughly mangled, is just one effect of the contrived fisics this con comes up with to back it up which believers are encouraged to mindlessly repeat. That carbon dioxide can accumulate in the atmosphere defying gravity another example, and that achieved by eliminating gravity as the AGWCon had eliminated the Water Cycle, here simply by calling the non-condensable gases of our atmosphere the imaginary ideal..
Real Carbon dioxide is fully part of the Water Cycle, all pure clean rain is carbonic acid, therefore, carbon dioxide is fully part of the COOLING role of the main greenhouse gas water vapour, a real world physics process driven by the Sun.
Let me put that all together, my elevator speech:
Earth’s Real Greenhouse Effect
Earth with atmosphere as we have it now: 15°C
Earth without any atmosphere at all: -18°C
Earth with atmosphere but with no water: 67°C
The Water Cycle cools the Earth by 52°C from the temperature it would be of 67°C without water.
Water vapour is the main greenhouse gas, it COOLS the Earth, think deserts.
Carbon Dioxide is fully coupled to the Water Cycle.
Real Carbon dioxide is fully part of the Water Cycle, all pure clean rain is carbonic acid, therefore, carbon dioxide is fully part of the COOLING role of the main greenhouse gas water vapour, a real world physics process driven by the Sun.
THEREFORE, GREENHOUSE GASES COOL THE EARTH.
So come on Willis – give us what Hansen has not given us – the “greenhouse effect” hypothesis –
show us the physics of carbon dioxide accounting for the massive 33°C rise in the difference between -18°C and 15°C.
Latour: “This is not ok because there is no physics to connect these two dissimilar numbers. The 33C are whatchamacallits. This greenhouse gas effect does not exist.”
whatchamacallits = non sequiturs.
Concise Oxford Dictionary (COD) non sequitur n. Conclusion that does not logically follow from the premises. [L, = it does not follow]
The resulting claim of the greenhouse effect is conveniently described in the entry preceding:
nonsense Absurd or meaningless words or ideas,
Best effort now Willis, show it’s a working hypothesis and not a whatchamacallit.
Myrrh

George E. Smith;
January 17, 2012 2:47 pm

“”””” Leonard Weinstein says:
January 17, 2012 at 8:27 am
Anna & others that think a non-greenhouse gas emits thermal radiation:
Yes there is some radiation even from non-greenhouse gases, but it is so small compared to the ground radiation and greenhouse gas radiation that it can be considered zero for the analysis.
“”””” Anna & others “””””
AKA; Anna and George E. Smith
Well the competing postulates are:-
#1 Stated repeatedly in all sorts of well known Textbooks on Physics. “EVERY object at a Temperature above zero Kelvins, emits “Thermal Radiation”, that being electromagnetic radiation that is emitted solely as a consequence of the Temperature of that object”.
#2 Stated almost solely by “Climate Scientists” in relation to earth’s atmosphere:- “EVERY object EXCEPT GASES at a Temperature above zero Kelvins, emits “Thermal Radiation”, that being electromagnetic radiation that is emitted solely as a consequence of the Temperature of that object”.
Unrelated to the above, but also often cited, with relatively little dispute:- “Some gases having more than two atoms per molecule CAN and DO emit and absorb Electromagnetic Radiation often in the Long Wavelength Infra-red region. These emissions/absorptions are ALWAYS at specific frequencies or bands of frequencies, that correspond to various electron energylevels that are peculiar to each such so-called greenhouse gas species, and are NOT continuum spectra characterized ONLY by the Temperature of the material.”
Now when I went to school, EVERY meant EVERY; sans NOTHING if you will.
Gases either emit and absorb thermal EM radiation or they do not. The spectral envelope of such EM radiation is given by the spectrum for a black body at the Same Temperature.
And as Anna said (in effect) nothing is nothing,, not something small.
The radiations from GHGs are a consequence of quantum mechanics, and the electronic structure of the molecule.
Black body Thermal radiation is NOT a consequence of quantum mechanics, it is a resultof ordinary classical physics, and requires no knowledge of any structure of the “particles” that emit and absorb such radiation.
Phil’s very low probability of ionising Argon at 15.8 eV from atmospheric thermal energies, is not challenged; but then you don’t need to ionise argon to get it to emit or absorb radiation, unless it is the specific atomic lines that you want to excite, not thermal.
And yes any individual atom or molecule emits or absorbs very little radiation, and solids contain many more atoms/molecules per unit volume, and undergo many more collisions than in gases; but the emission energy per collision is not greatly different.
Well I prefer to have nothing be nothing; not something small.
Nobody I know of has said that thermal emission from the atmosphere is the major driver of earth’s climate; but either it is there or it is not. Can’t have it both ways. I’m in the same boat as Anna.

Bart
January 17, 2012 2:53 pm

If there is no release of the heat building up in the upper atmosphere, then the heat is going to accumulate until the atmospheric particles reach escape velocity and leave the planet behind.
It is putting the cart before the horse in deciding that this hypothetical planet is going to retain its atmosphere in the steady state, you know.

Bart
January 17, 2012 2:57 pm

Let’s also not forget stimulated emission as another route through which highly excited particles may emit.

Daniel Kozub
January 17, 2012 3:11 pm

Willis, this is such an easy solution.
The surfaces of those one million suns heating your planet are all emitting more energy than they are receiving from the other 999,999. They all have a transparent atmosphere. GRAVITY keeps them together and maintains the proper pressure for the fusion reactions.
Your “planet” in this scenario must be made of helium and hydrogen and is undergoing fusion due to its GRAVITY.
Or, your planet is magic.

Bart
January 17, 2012 3:12 pm

DeWitt Payne says:
January 17, 2012 at 2:15 pm
“But the emission varies by cosine(θ) as well because the effective emitting area goes to zero at 90 degrees from the zenith.”
I see your point, but in fact, it doesn’t quite work that way. What we’re looking at is the FOV of a measuring sensor. Assume the spot it is looking at is a circle when you are directly above. As it looks to the side, the footprint becomes an ellipse with an increasingly elongated major axis. Think of the area illuminated by a flashlight shining on the ground as you swing it back and forth.
So, the cosine loss due to the projection decreases, but the actual area increases. I’d have to work out the ray tracing to actually determine the ultimate balance, but I’d guess the two effects should more or less cancel.

michael hart
January 17, 2012 3:13 pm

Bart,
I’d been wondering why nobody had mentioned ‘evaporation’ of the atmosphere. Considering a thermodynamic equilibrium, might we expect the gases to be lost to deep space? Is the atmosphere only really retained by kinetic limitations as to how fast it can escape?

Bart
January 17, 2012 3:20 pm

Willis Eschenbach says:
January 17, 2012 at 3:10 pm
“Thanks, Bart. I’m not clear what the “configuration of the surface energy state” means to you.”
Planckian morphology and uniform outward distribution pertaining to an infinitely smooth surface. The steady state energy and spatial distribution could easily be significantly different from these.

Bart
January 17, 2012 3:25 pm

michael hart says:
January 17, 2012 at 3:13 pm
“Is the atmosphere only really retained by kinetic limitations as to how fast it can escape?”
Yes, it was one of those forehead slapping moments. There is nothing in this thought experiment which prevents the atmospheric molecules from achieving escape velocity.

Daniel Kozub
January 17, 2012 3:27 pm

Solution #2 for Willis–
You may lose your hippie card for this. You basically described the planet as a hydrogen bomb.
So with that in mind, the planet is made out of uranium. The atmosphere is liquid tritium (IR transparent). GRAVITY and the inability to irradiate IR radiation keeps the tritium at absolute zero. It conductively cools the surface of the planet to near zero K. The surface irradiates inward (less than half of the absorbed energy from the 1 million suns) until it reaches criticality.

Bart
January 17, 2012 3:29 pm

So, let me update my elevator one last time:
A) Radiation enters the system
B) it is absorbed by the surface, which fluoresces in the IR
C) heat builds up at the interface between the surface and the atmosphere until its apparent temperature suggests an exceedance of the incoming radiance based on the SB relationship
D) in fact, though, the distribution of surface energy states is non-Planckian and/or not omnidirectional so the entire integrated spatially integrated energy flux is actually not as great as expected
E) the heat from the interface conducts through the atmosphere, establishing a temperature gradient as must exist in a spherically distributed atmosphere
F) the transferred heat accumulates in the atmosphere until:
1) highly energetic emissions are stimulated, which balances the energy fluxes all around in the same way GHGs would in the standard “greenhouse” theory
OR
2) the heat accumulates until the atmosphere achieves escape velocity and vanishes.
G) if greenhouse gasses which can radiate in the IR are added to the atmosphere, it will result in a net cooler atmosphere than otherwise would be the case, because there is now a lower energy outlet for heat to escape.

Bart
January 17, 2012 3:31 pm

“…integrated spatially integrated…”
Strike out the first “integrated”.

Daniel Kozub
January 17, 2012 3:39 pm

Bart, you are correct about the atmosphere escaping for this hypothetical planet as the energy builds. Since the premise includes having an atmosphere, gravity must keep it from escaping. PV=nRT If the pressure increases and the volume and mass remain constant, the temperature increases.
And that’s #3 Willis.

Bart
January 17, 2012 3:41 pm

This is a real “Eureka!” moment for me. The so-called “greenhouse gases” do not heat the atmosphere, they cool it. Now that is really something. How could so many smart people be so wrong? Eh, well, it happens. I’ve seen it happen before, and no doubt, I will again.

Bart
January 17, 2012 3:43 pm

Daniel Kozub says:
January 17, 2012 at 3:39 pm
“Since the premise includes having an atmosphere, gravity must keep it from escaping.”
Which means the gravity has to be so powerful that it will hold it until it reaches a point where it can discharge all the accumulated heat.

January 17, 2012 3:53 pm

[snip – sorry, I’m not going to tolerate that, feel free to rephrase and re-submit sans the accusations – Anthony]

January 17, 2012 4:03 pm

[snip – sorry, I’m not going to tolerate that, feel free to rephrase and re-submit sans the accusations – Anthony]
Couldn’t you just snip the last sentence and repost it. I didn’t retain a copy ?
Anyway, it was far less offensive than the things he has said to and about me.
REPLY: I’m sorry, but I didn’t save a copy either. It is the risk you run. Feel free to resubmit. – Anthony

Daniel Kozub
January 17, 2012 4:04 pm

Way to go Bart! Yes, GHGs cool the atmosphere. Otherwise, boom! And they heat non-GHGs through convection. When they reach a high-enough energy state, they start cooling too. And they can also heat those GHGs some more which then cool some more.
The atmosphere is a massively negatively-coupled system. It was about 10 years ago that I surmised that the planet won’t blow up. That’s when I stopped getting my global warming information from wikipedia.

David
January 17, 2012 4:15 pm

Elevator speech for Willis’s parameters, with a zero gas atmosphere, verses a non GHG atmosphere.
At its most basic only two things can effect the heat content of any system in a radiative balance. Either a change in the input, or a change in the “residence time” of some aspect of those energies within the system. (David’s Law) ++NOTE
The addition of a non GHG increases the residence time of the specific heat at the surface, thereby increasing the specific heat above the S-B equation.
There done. Now to add a few further basics for a longer elevator ride.
++ note, adding an atmosphere changes the system to a different system with increased volume requiring more energy to have an equal T, which is naturally attained over time when the atmosphere molecules reach their specific heat with the surface per the second law, which here , just like with radiation, is net flow from higher to lower.
A zero atmosphere surface cools strictly through radiation, which is very fast. The specific heat of the BB surface reaches a certain level determined by the S-B law, at which point the radiating temperature, radiates the heat away to match the insolation.
The existence of an atmosphere adds a second method of cooling the surface. That method is conduction.
Now the surface has two methods of cooling. These methods are not additive, (they do not accelerate the cooling) They are subtractive. (Now less of the specific heat is radiating from the surface, as some of the specific heat is now conducting.)
The dry adiabatic lapse rate is g / Cp, where g is gravity and Cp is the specific heat of the atmosphere The lapse rate may be constant, but it is a constant VARIATION, which appears to be predicated on g and Cp. (IE, the greater the gravity in an otherwise equal atmospheric content of non GHG, the higher the specific heat content of the atmosphere) What does “specific heat emanate from? If specific heat, which is “heat capacity per unit mass of a material”, then the more mass per volume, the greater specific heat per volume. Therefore an atmosphere of denser mass, (caused by either more atmosphere same gravity, or more gravity same atmosphere) will have a higher specific heat content then a thinner atmosphere. The lapse rate will be the same in all atmospheres, just the starting point or temperature will be different. The individual non GHG molecules do not vibrate at a higher specific heat, but due to the fact that there is more per M2 the T is raised. This is just the opposite of some molecules at the top of the atmosphere which, subject to very high energy SWR, vibrate at a very high T, but the actual air T is low because there are so few per M2.
So, with greater bottom of atmosphere residence time of conducted specific heat, (both down dwelling and upwelling), due to increased mass per volume as a result of more atmosphere or gravity, the flow from the surface is delayed due to the decreased gradient between the surface and the atmosphere, plus there is backflow, or “back conduction” to the surface, thereby increasing the specific heat above the S-B equation.
Conduction, just like radiation from a GHG flows both ways. Some of this conducted heat flows back to the surface, and slows the flow of heat from the surface, raising the temperature of the surface above the S-B law. And the densest air being at the surface (due to gravity) that air insulates the surface for longest, thus raising the surface temperature, effectively delaying the loss of energy to space so that energy builds up in the atmosphere until a new higher temperature equilibrium is attained.
Both this and GHG do the same thing, increase the residence time of energy in the earth, ocean atmosphere system allowing additional accumulation of energy relative to S-B and the earths albedo. Either both are breaches of the Laws of Thermodynmics or neither are.

hotrod (larry L)
January 17, 2012 4:20 pm

Bart says:
January 17, 2012 at 3:25 pm
michael hart says:
January 17, 2012 at 3:13 pm
“Is the atmosphere only really retained by kinetic limitations as to how fast it can escape?”
Yes, it was one of those forehead slapping moments. There is nothing in this thought experiment which prevents the atmospheric molecules from achieving escape velocity.

Yes even at relatively cool temperatures, there is a small probability that a few atoms will through energy transfer of collisions achieve escape velocity. In doing so they carry energy out of the system and are a form of energy transfer.
As the mathematician in the movie Jurassic park said, “life will find a way” in the thought experiment Willis proposes “physics will find a way” to lose the excess energy. IR radiation is not the only way to lose energy, there is a full spectrum of electromagnetic radiation ranging from long wave radio up to gamma ray frequencies that allow atoms/molecules to lose energy.
There are also kinetic transfers of energy, and one of those paths will be increased evaporation of the atmosphere as random atoms achieve escape velocity.
Energy can be radiated as visible light and radio waved due to static charge build ups in a mixing atmosphere, or by florescent glow as our atmosphere does on the night side as oxygen and nitrogen molecules emit energy as light that was absorbed during the day. In a dusty atmosphere you also have static charge accumulation due to dust impacts (ever see the St. Elmo’s fire from a helicopter rotor when filmed by a night vision camera?). Any electrical charge motion will result in currents and magnetic field fluctuations which will radio in the radio spectrum. There are many many ways for a planet to lose energy without a single photon of IR being emitted.

Bart
January 17, 2012 4:22 pm

Willis Eschenbach says:
January 17, 2012 at 4:00 pm
“At equilibrium, as you stated, the temperature is indeed uniform.”
Not possible. With no heat sink in the upper atmosphere (hence no adiabatic lapse rate), the temperature profile must satisfy the steady state heat equation, which says the Laplacian has to be zero. The solution of this is
T = A + B/r
T is temperature, r is the radius, and A and B are constants. At infinity, T must be zero, hence
T = B/r

Bart
January 17, 2012 4:28 pm

Daniel Kozub says:
January 17, 2012 at 4:04 pm
“Way to go Bart! Yes, GHGs cool the atmosphere. Otherwise, boom!”
I feel like I just emerged from a tunnel into the light. Yes, boom! It’s all so obvious. What a fiasco. I knew it was bad, but this?!

January 17, 2012 4:29 pm

I think Willis has inadvertently described a model that if complied with cannot be used to invalidate his proposition.
i) In order for there to be an energy imbalance requiring GHGs to correct it Willis proposes a static equilibrium between surface and atmosphere with a zero energy exchange between the two at equilibrium. That is highly unrealistic given the kinetic energy present in the air and on the surface. In reality there will be a substantial dynamic interchange. If values are given for that interchange then the apparent energy imbalance disappears without needing GHGs at all.
ii) Willis proposes a uniform surface and multiple suns but the consequence of that uniformity is an absence of atmospheric circulation such that from conduction over time the non GHG atmosphere will heat up indefinitely until it is blown off into space. In reality a non GHG atmosphere on a rotating uneven sphere under a single sun will have a hugely powerful atmospheric circulation between the day and night sides as the night side sucks energy out of the air by conduction as fast as the day side releases energy to the air by conduction.
So, regrettably, it is not possible to prepare an elevator speech in response to Willis’s proposition whilst complying with the parameters he has set.

dr.bill
January 17, 2012 4:29 pm

re Joel Shore, January 17, 2012 at 11:39 am :
Joel: It says a lot about you that you couldn’t ignore even that minor bit of levity, and the comment you linked to does nothing but confirm my original premise.
Having an argument with the world and trying to shout them all down simply doesn’t advance your cause, and neither does an appeal to your training and credentials. In the past week and a half, I’ve interviewed six people in an attempt to fill two vacant positions in my Department. They all had PhD’s, and looked great on paper, but four of them weren’t fit to put in front of an undergraduate Mechanics class, so being credentialed doesn’t always mean much.
A couple of years ago, in a comment I posted here, I inadvertently came off sounding almost as patronizing as you generally do, and was quickly put in my place (by the inestimable Smokey, as it happens). He was right to do so. My credentials might be real, and I might know what I’m talking about on some subjects, but in every group of people of any kind, half of them are in the bottom half, and just yelling at people and insulting them isn’t going to persuade them that you’re in the other half.
There’s no shortage of idiots posting all kinds of nonsense all over the Net. We all eventually figure out who they are, and then just ignore them. If you truly want people to read what you write, instead of skipping over anything with your name at the top, you might think of not trying to emulate the screaming idiots.
You’re giving scientists (and Physicists in particular) a bad name.
/dr.bill

January 17, 2012 4:32 pm

David says:
January 17, 2012 at 4:15 pm
Well said David,
Very similar to my analysis.

Anything is possible
January 17, 2012 4:45 pm

illis Eschenbach says:
January 17, 2012 at 3:55 pm
Ned Nikolov says:
January 16, 2012 at 7:29 pm
“In any case, I fear that your “Part 1″ may be of little use. I know that a GHG-free atmosphere can reduce the losses that would occur in its absence, we have the moon as an excellent example.
I’m not particularly interested in further discussion of the greenhouse effect. I want to know how your effect would heat the surface to well above the theoretical S-B temperature without greenhouse gases.
Which means that I have to wait for “Part 2″ for you to reveal the secret … any chance you could post Part 2 first?”
__________________________________________________________________________
Willis, with respect, part 1 (which i have just had the pleasure of reading) is absolutely critical to the N+Z theory, because it proposes that the traditional method of calculating the theoretical S-B temperature is incorrect. That changes everything. IMHO, it is a “must read”, and doesn’t mention the “g”-word, once!.

January 17, 2012 4:46 pm

“At equilibrium, as you stated, the temperature is indeed uniform. I was totally wrong to state it followed the dry adiabatic lapse rate.”
Depends on whether there is an external energy supply.
No energy supply and the column will equalise to the same temperature throughout but with more molecules at the base compared to the top due to gravity. So there will be greater energy per unit volume at the bottom as compared to the top but the same temperature throughout.
Then add the energy supply and since there are more molecules per unit volume at the bottom they will absorb proportionately more of the incoming energy per unit volume and become warmer than the molecules where the energy per unit volume is lower.
That is what creates the temperature gradient as far as I can tell.
It is back to what I tried to tell you before. The denser gases convert a higher proportion of the radiation flowing through them into kinetic energy for a higher temperature and the denser the gas the longer that energy remains in kinetic form. Whilst in kinetic form the transmission of energy through the system is slower than it would have been if it had simply hit a bare surface and been converted immediately into outgoing IR.
The slower the transmiission through the system the higher the equilibrium temperature of the system.
Exactly as for the radiative greenhouse effect but caused by mass/density/pressure and so involving the entire mass of the atmosphere including non GHGs.

Bart
January 17, 2012 4:53 pm

Willis Eschenbach says:
January 17, 2012 at 4:39 pm
“Argon can absorb and radiate high energy only. At normal temperatures, it does not radiate at all in the IR. Additionally, at normal temperatures there’s not enough energy for it to emit high-energy radiation.”
That’s the problem, Willis. In your thought experiment, you have no outlet for radiation of the heat conducted to the atmosphere. As a result, your atmosphere heats, and heats, and heats.
Eventually, if the atmosphere hangs around, all that stored up energy is going to find a way out, until balance is achieved. Until that balance is achieved, there is no equilibrium, so SB doesn’t hold. You cannot be radiating out more than you get in without experiencing cooling, but the energy that would normally be going into radiation is going into heating up the atmosphere through the conductive interface.
This is a much more exotic situation than you realized when you came up with the thought experiment. Your normal intuition based on what happens on Earth does not apply.

Spector
January 17, 2012 5:04 pm

I think we should keep in mind that emission and absorption are two sides of the same coin; one becomes the other on time reversal. This is why those photons most likely to be emitted by a carbon dioxide molecule are also those most likely to be absorbed by another CO2 molecule. If an atmosphere has a method for absorbing photons, then it has a method for emitting those same photons.
If a theoretical non-greenhouse gas has a optical wavelength absorption band, then it will both absorb solar energy and emit terrestrial energy by this mechanism. Such a gas would continue to absorb heat from the sun until it reaches a temperature sufficient to excite emission of the same energy flow back to outer space or until this atmosphere becomes so hot that it all escapes to outer space.
The problem with the gravitational heating theory is that it does not provide any mechanism to continuously extract or reflect an average 156 W/m² from a hypothetical average 396 W/m² radiant energy flow of a typical GHG warmed surface so that only the allowed average 240 W/m² actually escapes to outer space. As these are average energy calculations, any reference to surface temperatures or temperature variations are irrelevant. These averages apply over time as well as space.

DeWitt Payne
January 17, 2012 5:30 pm

Bart,
If I calculate absorptivity = 1-reflectivity according to the Fresnel equations using an index of refraction for water of 1.333, I can indeed calculate an average emissivity of 0.82. But the emission to the surrounding hemisphere decreases with cosine(θ). The effect of increasing area is not sufficient to cancel this effect. When I correct the emission for emissivity and integrate over the hemisphere, the effective emissivity is 0.93.
Willis now agrees that the atmosphere will be isothermal. Heat can only accumulate in the upper atmosphere until the atmosphere is isothermal. It won’t conduct after that. You can’t have a constant reduction of temperature with altitude and also have continuous heat accumulation. Heat accumulation requires an increase in temperature.
As for ionization, oxygen has the lowest ionization energy of the three major components of the atmosphere at 1314 kJ/mole. For oxygen to be 0.1% ionized thermally requires a temperature of 22,900K or interaction with a 90nm wavelength photon for an individual molecule. 90nm photons don’t penetrate very far and the molecules won’t stay ionized unless the mean free path is long, i.e. higher than 100 km. Your glow discharge and/or lightning is absurd.

DeWitt Payne
January 17, 2012 5:51 pm

Bart,

Not possible. With no heat sink in the upper atmosphere (hence no adiabatic lapse rate), the temperature profile must satisfy the steady state heat equation, which says the Laplacian has to be zero. The solution of this is
T = A + B/r

The Laplacian of the temperature field can be zero if the temperature is constant with respect to r, θ and φ, i.e. no gradient. But unless B = 0 in your solution, there is a gradient in r and the Laplacian of T can’t be zero. If B is zero, then A isn’t zero, but is equal to the surface temperature.

hotrod (larry L)
January 17, 2012 5:52 pm

AnonyMoose says:
January 17, 2012 at 5:59 am

I had missed that gravity was being credited with continuously creating heat.

I think that is a total misreading of the theory. It is not saying gravity is continuously creating heat. Any more than someone thinks a compressed spring constantly produces energy. It only stores energy that already is in the system.
It is saying that an atmosphere in a gravity field due to ideal gas laws must develop a temperature gradient due to its lapse rate. The lower levels of the atmosphere will be cooler than the upper layer of the atmosphere. Therefore when the upper levels of the atmosphere reach a temperature where they lose heat at the same rate as additional heat is added to the system (note I did not say lose heat by IR radiation) then the lower levels of the atmosphere will be warmer.
To use your example, lets suppose the atmosphere cannot radiate a single photon in the IR. That does not preclude other mechanisms of losing energy. What those other mechanisms are does not matter as long as there is some method to lose energy. The atmosphere will do what ever it has to in order to reach those conditions to loose the excess energy.
Lets further suppose that all paths of radiation energy loss are closed. No loss of energy by visible light radiation due to florescence, no electrical discharge or electron emission or gamma ray emission or radio wave all those paths are closed. What would happen?
Simple — the atmosphere would continue to heat up but it would still have a lapse rate At some point high above the surface the temperature would rise high enough so that the atoms can achieve escape velocity (remember that due to this thermal expansion they are very high above the planet so gravity is reduced). At that point with all electromagnetic paths closed the atmosphere would still lose heat to interstellar space by mass transport as gases (like the solar wind) continuously blew off into space.
The theory does not speculate on the exact mechanism of energy loss, only that in a real atmosphere above a planet with a gravity field there must be a temperature gradient with altitude (we commonly call the lapse rate) due to the ideal gas laws.
If you can propose a condition where that is not the case, then you are invalidating the ideal gas laws, or your model is physically impossible.
That gradient, then forces the lower levels of the atmosphere to be warmer than the effective radiation temperature (or other heat loss mechanism) that eventually achieves thermal balance with the incoming radiation.
At least that is my take on it.

hotrod (larry L)
January 17, 2012 6:00 pm

Correction — typo:
The lower levels of the atmosphere will be cooler than the upper layer of the atmosphere.
should read:
The lower levels of the atmosphere will be warmer than the upper layer of the atmosphere.
Larry

jjthoms
January 17, 2012 6:16 pm

Bart says: January 17, 2012 at 4:53 pm
That’s the problem, Willis. In your thought experiment, you have no outlet for radiation of the heat conducted to the atmosphere. As a result, your atmosphere heats, and heats, and heats.
============
A non GHG neither emits nor receives energy by absorption
A non GHG can only heat by conduction
A Non GHG Will pass downward solar radiation and upward LW radiation without hindrance and without heating
The surface is heated by the solar radiation.
It is cooled by LW radiation. when incoming radiation = out going radiation thermal equilibrium will result. This equality only occurs at one temperature for fixed incoming radiation and surface albedo.
The non-GHG will be heated by the surface by conduction only – initially cooling the surface. There is nowhere for the heat to go – non-GHGs do not radiate and there is nothing to conduct to in space.
Eventually the non-GHG will reach the same temperature as the planet surface and heating by conduction will cease. The non-ghg will get no hotter than the surface.
The surface will now be surrounded by hot conductive gas which does not radiate but also does not stop surface radiation escaping (i.e. it does not act as a blanket retaining the surface heat,
So we still have the same radiation arriving at the surface and the same leaving. The albedo is the same. The temperature at the surface therefore is the same with and without Non GHGs

Bart
January 17, 2012 6:22 pm

Spector says:
January 17, 2012 at 5:04 pm
The problem with the gravitational heating theory is that it does not provide any mechanism to continuously extract or reflect an average 156 W/m² from a hypothetical average 396 W/m² radiant energy flow of a typical GHG warmed surface so that only the allowed average 240 W/m² actually escapes to outer space.”
What that means is not that the GHGs heated the surface above what it otherwise would have been. Rather, it means it is maintaining it that much cooler below what it otherwise would have been.
DeWitt Payne says:
January 17, 2012 at 5:30 pm
“The effect of increasing area is not sufficient to cancel this effect.”
In fact, it increases faster. Do the calculations.
When I correct the emission for emissivity and integrate over the hemisphere, the effective emissivity is 0.93.”
Still a lot less than 0.99. And, this is in calm seas. When you do the right calculation, it will be even worse.
Willis now agrees that the atmosphere will be isothermal.”
…So?
“You can’t have a constant reduction of temperature with altitude and also have continuous heat accumulation.”
You appear to be mixing fluxes in space and in time.
“Your glow discharge and/or lightning is absurd.”
On planet Earth. Not on Willis’ thought experiment world. You haven’t been keeping up. I suggest you read my latest contributions to the discussion.
Willis Eschenbach says:
January 17, 2012 at 5:47 pm
The point of starting from the uniformly heated condition is that there is no warmer arrangement for a given radiation level.”
Nope. Been there, done that. See last comment
here.
DeWitt Payne says:
January 17, 2012 at 5:51 pm
“But unless B = 0 in your solution, there is a gradient in r and the Laplacian of T can’t be zero.”
Aye yi yi.
Willis Eschenbach says:
January 17, 2012 at 5:55 pm
“It’s also what the textbooks say.”
The textbooks are looking at flat geometry. This is a spherical distribution.
“As to your further explanation that ended with T = B/r, where r is the radius, you’ll have to explain that.”
Come ON, you guys. Surely, you’ve solved the LaPlace equation somewhere along the way in your studies? Voltage from a point charge? Gravitational potential from a point mass? Ring a bell?
Willis Eschenbach says:
January 17, 2012 at 6:12 pm
“So your contention is that the surface will warm the atmosphere until it is much warmer than the surface?”
How do you know how warm the surface is? It is continually warming, too. There is no equilibrium until the atmosphere finds an outlet for its stored heat. At that point, you will find everything balances.

hotrod (larry L)
January 17, 2012 6:23 pm

If gravity could do that, if gravity alone could split the atmosphere into warm and cool, then we could get work out of the temperature difference … forever. With no input of energy.
Would be nice …
w.

It does on earth we call that work thunderstorms and winds.
Larry

gbaikie
January 17, 2012 6:32 pm

“Erl Happ says:
January 17, 2012 at 8:04 am
Willis says: “I hold it can be proven that there is no possible mechanism involving gravity and the atmosphere that can raise the temperature of a planet with a transparent GHG-free atmosphere above the theoretical S-B temperature.”
That’s a valid statement but it does not invalidate the crux of what N and Z are telling us.”
No it’s not a valid statement.
Main thing proven is the “theoretical S-B temperature” is wrong.
To be clear is hasn’t proven that T cubed times .0000000567 equals watts per square meter is incorrect.
But rather T cubed times .0000000567 divided by 4 which is what Willis is saying is “theoretical S-B temperature” is wrong.
If earth had million star which at earth distance has solar flux of 240 watts per square meter. To determine temperature of planet you do not cube 255 and times this by .0000000567 and divide it by 4. If you did this the “theoretical S-B temperature” would 60 watts per square meter. Which is
180 K. therefore the “theoretical S-B temperature” would indicate that million sun would heat a planet to 180 K. Whereas “logical sense” would suggest it’s 255 K- because sunlight is coming from all directions and you don’t need to divide by 4. But if you do this “even though you don’t need to” you get the wrong answer.
In addition you get wrong answer if subtract Bond albedo to determine planet’s temperature. It’s possible a highly reflective surface with planet with million sun would same temperature [and may be warmer].
In other words regardless of color or if planet was liquid mercury and/or cover with clouds the temperature will be around 255 K

blueice2hotsea
January 17, 2012 6:46 pm

Hi Willis –

[Argon] only absorbs or emits in one way, through electron transitions, and that takes a whole bunch of energy to force one of those, and it doesn’t happen in the IR.

Thinking in terms of average kinetic energy also makes ice sublimation at -20 C sound impossible. Yet, individual frozen water molecules are melting, reaching the boiling point and escaping as water vapor.

Because argon is a monoatomic gas, the atoms of argon have no mechanical way to absorb or emit energy through bending or stretching. There is no physical way for the atoms to absorb or emit energy, an atom of argon is a little ball and the energy simple can’t interact with it except in high-energy electron transitions.

Argon can absorb energy via collision (conduction). True, it won’t undergo electronic excitation until it has surpassed a certain high threshold. However, a millions suns might suffice, no?
Even in the condition of isothermal equilibrium, with no difference in temperature between planet and atmosphere, and no difference in temperature within the atmosphere in the vertical or lateral profile, there are still vast differences in kinetic energy between individual particles.
Enormous numbers of unpredictable collisions and interactions with random quantum vacuum fluctuations, etc. will eventually deal low temperature argon a royal-flush, the acquisition of the necessary additional energy for electronic excitation and upon relaxation to a lower state, the emission of a high energy photon. IMO, this continuing energy loss would set up a (possibly weak) convective circulation of surface heat removal resulting in lowered surface emission temperature.
This story is not complete, but a single counter-example. For more, perhaps Anna V. will help out.
bi2hs

Bart
January 17, 2012 7:05 pm

All of the questions people might have for me have been answered in earlier posts. There’s really no doubt about it. So-called GHGs cool the atmosphere, just like the astrophysicists say they will. If I were asked to choose between believing an astrophysicist or a climate scientist, I’m pretty sure where I’d place my bets.
For people still having problems, I suggest you read the preceding posts carefully until the light bulb goes off for you, too. I’m calling it a night.
A few reminders and notes to ponder:
Kirchoff’s law says SB places an upper bound on energy radiated given temperature. That means it places a lower bound on temperature given energy. SB does not hold until you are at equilibrium.
If one insists on believing that GHGs which radiate in the IR elevate surface temperatures, how much higher would the temperature be elevated by a GHG radiating in the UV?
The gradient of the function 1/r, where r = sqrt(x^2+y^2+z^2), in Cartesian coordinates is the vector (x/r,y/r,z/r). What is the divergence? I.e., what is d(x/r)/dx + d(y/r)/dy + d(z/r)/dz? Hint: ZERO.
Murray Gell-Mann is quoted as saying of physics: “Everything not forbidden is compulsory.” It’s a very useful saying which can help guide you to truth.

gbaikie
January 17, 2012 7:09 pm

“hotrod (larry L) says:
January 17, 2012 at 5:52 pm
AnonyMoose says:
January 17, 2012 at 5:59 am

I had missed that gravity was being credited with continuously creating heat.
I think that is a total misreading of the theory. It is not saying gravity is continuously creating heat. Any more than someone thinks a compressed spring constantly produces energy. It only stores energy that already is in the system.
It is saying that an atmosphere in a gravity field due to ideal gas laws must develop a temperature gradient due to its lapse rate. The lower levels of the atmosphere will be cooler than the upper layer of the atmosphere. Therefore when the upper levels of the atmosphere reach a temperature where they lose heat at the same rate as additional heat is added to the system (note I did not say lose heat by IR radiation) then the lower levels of the atmosphere will be warmer.
To use your example, lets suppose the atmosphere cannot radiate a single photon in the IR. That does not preclude other mechanisms of losing energy. What those other mechanisms are does not matter as long as there is some method to lose energy. The atmosphere will do what ever it has to in order to reach those conditions to loose the excess energy.
Lets further suppose that all paths of radiation energy loss are closed. No loss of energy by visible light radiation due to florescence, no electrical discharge or electron emission or gamma ray emission or radio wave all those paths are closed. What would happen?”
You same situation as a million suns.
The sun is only capable of heat an object to a certain temperature.
This temperature is somewhere near the highest temperatures on lunar surface.
Around 120 C [390 K]. 390 cubed times .0000000567 is 1311.7.
If Earth had a million suns it’s surface temperature would be 120 C.
Though it should noted that at higher elevations it would be cooler than 120 C.
Mt everest [8850 meters high] would be cooler- if assume 6 C per 1000 meters
it would be around 67 C. Even with a million suns, the oceans would not boil, but Ocean evaporation would be quite high.
If you don’t have million suns and rather as you say not heat could leave earth, the difference
between that and million sun is it would take longer to heat the entire planet to 120 C- a million sun may do this in 1000 years and the one sun might take a million years.
Getting to 80 C would pretty quick but nearer it gets to 120 C the slower the warming

Bart
January 17, 2012 7:10 pm

“The gradient of the function 1/r, where r = sqrt(x^2+y^2+z^2), in Cartesian coordinates is the vector (x/r,y/r,z/r)
Oops. That’s the gradient of r. The gradient of 1/r is -1/r^2 times that. And, the divergence to calculate is d(x/r^3)/dx + d(y/r^3)/dy + d(z/r^3)/dz = 0.

jjthoms
January 17, 2012 7:14 pm

from – A First Course in Atmospheric Radiation: Second Edition, by Grant W. Petty
most modes of energy storage at the molecular level and smaller are quantized. That is to say, a given molecule cannot have just any vibrational energy, but rather only one of a discrete set of energy levels E o , El, , Eco permitted by the laws of quantum mechanics applied to that particular molecule. The same principle applies to other modes of energy storage, such as that associated with molecular rotation and electron excitation. Only the translational kinetic energy of molecules and other unbound particles is unquantized.
At ordinary atmospheric temperatures, collisions between molecules are very rarely energetic enough to kick electrons into excited states. Those few that do briefly find themselves in an excited state, for whatever reason, usually give up their energy again in the course of subsequent collisions without emitting a photon.
There are therefore extremely few electrons found naturally in excited states and even fewer opportunities for the spontaneous emission of photons due to electronic transitions back to the ground state. This fact is of course consistent with the sharp fall-off in thermal emission at the short wavelength end of the Planck function.
When electron orbits in atmospheric molecules are found in an excited state, it is usually because of the absorption of an incoming photon with the right energy, usually one of solar origin. As for rotational and vibrational transitions, we therefore expect to find discrete absorption lines associated with each allowed transition to higher electronic orbital states
=====
O2 has absorption up to 250nm (uv end) In the earths atmosphere this is mainly removed by )3 etc. There is a further rotation exitation in the 10s of GHz region. (this is not relevant with the solar spectrum)

Deborah
January 17, 2012 7:46 pm

I’m a lawyer and read this website so that I can speak to issues from a different perspective than that of most of my peers. I really need that elevator speech. So, could you all stop sniping and make an attempt to attain that goal. Thanking you in advance for your cooperation – as we say in the legal profession.

otsar
January 17, 2012 8:39 pm

Hello Willis,
I posted this a while back, but it must have gone into the chronosynclastic infundibulum. I have not read the mentioned papers, and have not read all of the comments. I may be repeating what has already been posted.
1. I agree with you that the embodiment of the planet with many suns and a transparent atmosphere will be at the S-B temperature. This includes the surface and the gases.
2. Let us suppose that the suns can be turned up in brightness by a step DELTA_BRIGHTNESS. The surface and gas column will equilibrate to the new S-B temperature in a time T_heat by conduction from the surface to the gases then convection to the rest of the column.
3. Let us now turn the suns back to their original brightness. The surface will eventually go back to the original S-B temperature and the gas column will cool to the new S-B temperature by conduction. Convection will be inoperative. Time T_cool will be much longer than T_heat, but will eventually reach the original S-B temperature.
4. Let us now turn the suns brightness up and down by a step DELTA_BRIGHTNESS such that the cycle time is shorter than T_cool. The atmospheric column temperature will approach the upper S-B surface temperature as a limit. The surface temperature will approach the lower S-B temperature as a limit. The biggest temperature difference will be between the surface and the upper atmosphere.
5. For the electronically minded this can be modeled by a diode (convection) with a resistor in parallel (back conduction) feeding a capacitor (energy storage in the column). The diode (one way valve) is produced by the lack of downward convection, since the gas column does not radiate.
6. What would it take to reverse the scenario (diode)? If we placed the surface in the upper atmosphere, there would be only downward convection of cooled gases. The gas column would approach the lower S-B temperature as a limit.
7. Yes this is a matter of some gravity. Without gravity there would be no convection, etc.
Please feel free to pick the explanations apart. Why do I get the funny feeling that the audience is being trifled with, especially in light of the title of the post, and all of the odd behaviour?

gbaikie
January 17, 2012 9:03 pm

“I’m a lawyer and read this website so that I can speak to issues from a different perspective than that of most of my peers. I really need that elevator speech. So, could you all stop sniping and make an attempt to attain that goal. Thanking you in advance for your cooperation – as we say in the legal profession.”
Ok.
If you had a planet which surrounded by a million [any number from a couple dozen to infinity] suns and each sun radiates 240 watts per square meter. The temperature of that planet will warm until it’s -18 C.
When you take the average amount of sunlight, that reaches earth remove 30% due to reflection
and divide the energy by 4; you also get the number 240 watts per square meter.
Summary the distance from the sun determines maximum temperature that the sun can heat any surface.
Climatic “scientist” divide the power of sunlight by 4 because the earth intercepts the sunlight as a disk, and 4 times this area is the entire surface of the earth. They are quite aware that sunlight doesn’t shine on night side of earth, but wish to consider that sunlight does warm the entire surface at one time.
This lack any sense but reason has little to do with what people want to believe,

ZP
January 17, 2012 9:18 pm

Willis Eschenbach says:
January 17, 2012 at 5:35 pm
Come back with the absorption lines for argon in the normal IR range.
I say it absorbs none. Zero. It only absorbs or emits in one way, through electron transitions, and that takes a whole bunch of energy to force one of those, and it doesn’t happen in the IR. Because argon is a monoatomic gas, the atoms of argon have no mechanical way to absorb or emit energy through bending or stretching. There is no physical way for the atoms to absorb or emit energy, an atom of argon is a little ball and the energy simple can’t interact with it except in high-energy electron transitions.

Willis, you are correct that argon cannot have bending or stretching modes of vibration since it is a monoatomic gas. It can only have electronic transitions; the energy of which depends on the orbitals that the electrons are jumping between. However, you are overestimating the energy associated with most transitions, especially those occurring between n > 3 and n = 3. This overestimation leads you to the incorrect conclusion that argon does not absorb/emit IR radiation.
Consider the Bohr model of the hydrogen atom. There are a number of emission line series that are attributed to the excitation/relaxation of the electron. Each series is named for the person who discovered it. The lines series are observed in the UV (Lyman), the visible (Balmer), the IR (Paschen, Brackett, Pfund, and Humphreys), etc. The actual wavelength of light observed is related to how far apart the two levels that the electron is jumping between are. The farther apart, the more energy required/emitted, and hence the shorter the wavelength. Thus, the Lyman series is attributed to transitions from n > 1 to n = 1. The Balmer series is attributed to transitions from n > 2 to n = 2. Longer wavelengths are associated with transitions between the higher energy levels since these levels are even closer together. Hence, the Paschen series, which is the first series occurring in the IR spectral region, is associated with transitions from n > 3 to n = 3.
Argon is a quite a bit more complex than hydrogen. However, the simplistic Bohr model still provides a good way to begin thinking about the possible electronic transitions that will be observed. The valence (outermost) electrons in a ground state Ar atom are located in the third shell (n = 3). These electrons can be excited to the higher unoccupied orbitals (n > 3), which leads to expectation that a Paschen type absorption/emission series in the IR is probable. Indeed, this series was first reported by Paschen in the early 1900s. I would recommend that you search through this thread. Anna V had provided a link to the absorption/emission spectra for argon, which includes the IR lines that you are interested in learning more about.

Trick
January 17, 2012 10:16 pm

Jimmi says at 1/17 11:04am:
“The word “transparent” means “does not absorb”
Right Jimmi. The transparent GHG-free atmosphere does not absorb any thermal radiation emitted from Willis’ 2nd mechanism black body surface. That thermal radiation goes right thru the transparent GHG-free atmosphere and allows Willis’ planetary black body one of the at least two physical means to cool to space and achieve equilibrium non-infinite temp.s.
In my view, Willis’ 2nd mechanism GHG-free atmosphere absorbs thru thermal conduction at his planetary black body surface interface to his atmosphere. This allows Willis’ planetary black body the second physical means to cool to the atmosphere which is matter (O, N) and this matter eventually can emit the balance of thermal radiation to space – this is the missing part which enables Willis to write a proof by contradiction that is incomplete & as consequence then violates energy conservation.
Willis leaves out of his proof by contradiction thermal conduction at his 2nd mechanism surface, ideal gas physics, and gravity so the proof is incomplete & finds a violation of energy conservation which can be resolved to no violation once these physics are employed. Nature is rescued and there could be other rescue teams.
As a consequence of the above two physical means, the same total energy from the sun is radiated back to space as in Willis’ 1st mechanism (BB only), 2nd mechanism energy is conserved, 2nd law is not violated, KE+PE = constant. This should be easy to see except for the million details & the myriad other mechanisms many others are discussing past Willis’ 2. Blogs can be inefficient. But like Waiting for Godot Act ll: “In the meantime, let us try and converse calmly, since we are incapable of keeping silent.”
I do not see where N&Z have a non-physical component assuming transparency, only see Willis 2nd mechanism incomplete proof by contradiction so far. But the play is in progress.

Bart
January 17, 2012 11:01 pm

Deborah says:
January 17, 2012 at 7:46 pm
” I really need that elevator speech. So, could you all stop sniping and make an attempt to attain that goal.”
Done. From my perspective, which is the right one.

Bart
January 17, 2012 11:21 pm

jjthoms says:
January 17, 2012 at 6:16 pm
“It is cooled by LW radiation. when incoming radiation = out going radiation thermal equilibrium will result. This equality only occurs at one temperature for fixed incoming radiation and surface albedo.”
So, so utterly and completely wrong. There is never a vanishing thermal gradient in the atmosphere, so long as energy cannot escape it. The temperature is of the form B/r, where B is a constant, which is a decreasing function of radius. So, heat will flow continuously up into the atmosphere until it can be released, either by high energy radiation, or “boiling” away of the atmosphere.
“There is nowhere for the heat to go – non-GHGs do not radiate and there is nothing to conduct to in space.”
Exactly. Which is why it will continue to build.
“Eventually the non-GHG will reach the same temperature as the planet surface and heating by conduction will cease.”
Nope. The surface will always be hotter, because B/r is a decreasing function of the radius. The gradient always exists, ergo conduction never stops. So there is never an equilibrium, and SB does not hold.
“The non-ghg will get no hotter than the surface.”
The non-ghg atmosphere will get hotter and hotter until there is some radiation or, if sufficient radiation does not exist to prevent the molecules from attaining escape velocity, it boils away.
GHGs short circuit the dynamic. They provide a heat sink which takes the accumulating heat energy out of the system. Without GHGs, we would not get colder. We would get hotter.

Bart
January 17, 2012 11:34 pm

“The temperature is of the form B/r, where B is a constant, which is a decreasing function of radius.”
Actually, I originally derived B under the assumption that the time rate of change of temperature had stopped. Since the persistence of a gradient says that hasn’t, then the heat equation PDE can be solved through separation of variables and B becomes an exponential function of time. So, the actual temperature profile is going to become
T = T(0,0)*exp(lambda*t)*(r_surf/r)
T(0,0) is the temperature of the surface at time zero, t is time, lambda is the exponential rate, r_surf is the radius of the planet, and r is the radius.
This form will persist until either the atmosphere starts radiating, or it boils away.

Bart
January 17, 2012 11:44 pm

Sorry if neglecting that little detail threw anyone off. We EEs are used to dealing with phasors and Helmholtz equations and the time varying character of the quantities is simply understood. It just didn’t occur to me to say, “oh, and you have to multiply this spatial function by the time dependent amplitude function.”

Bart
January 18, 2012 2:15 am

So, the actual temperature profile is going to become…”
Good thing I actually started thinking about the amplitude function, because the solution is actually not that elementary anymore, since the Laplacian of the spatial part of the temperature function is now equal to the constant lambda times the spatial temperature function. The solution is going to be of the form
T = T(0,0)*exp(lambda*t)*F(r)
where F(r) is a modified Bessel function scaled to equal one at r = r_surf. Everything still holds mutatis mutandis, The temperature function is still monotonically decreasing with altitude. I’ll work out the details later, or anyone reading this can if they like.

January 18, 2012 4:27 am

“Without GHGs, we would not get colder. We would get hotter.”
Agreed as regards Willis’s model with a uniform flat surface and multiple suns for even irradiation.
However a rotating uneven sphere under a single sun would have vigorous convection due to temperature differences between the day and night sides.
That should stabilise the temperature at some point from rapid surface radiation to space on the night side. The cold ground would suck conductive energy out of the fast moving air above.

JJThoms
January 18, 2012 5:06 am

Bart says: January 17, 2012 at 11:21 pm
“The non-ghg will get no hotter than the surface.”
The non-ghg atmosphere will get hotter and hotter until there is some radiation or, if sufficient radiation does not exist to prevent the molecules from attaining escape velocity, it boils away.
=======
The non-GHG is in contact with the surface heat will conduct either way – planet hotter by conduction will heat the gas: gas hotter the planet will get hotter via conduction. The gas can get neither hotter or cooler other than by conduction to the planet surface
In your mind you obviously have something heating the gas to a greater temperatuere than the surface – care to explain what this is. Where is the additional energy coming from?
If what you say is correct then we can use a heat engine to extract power from the temperature difference = free energy – and I’m sure you can see that is wrong?

beng
January 18, 2012 7:26 am

Willis, sorry if I’m beating a dead horse. The adiabatic lapse-rate formula is derived from an ideal gas process — compression/expansion. The reason the earth’s lapse-rate is near this is because it is constantly convecting up & down via Hadley cells.
Thought experiment — take your steel-shell greenhouse w/an atmosphere (non-GHG or not) underneath & between shell & surface w/the heat source at the shell instead of the surface, I don’t see how convection would occur in this case, and the temp profile of the air would be nearly isothermal underneath the shell regardless of gravity, specific heat, etc. I agree that it would be difficult to completely suppress convection in a surface-warmed real case — uneven surface heating would start at least some, and some lapse rate would develop.
IMO, the lapse-rate formula assumes convection is already occuring. Fire away.

shawnhet
January 18, 2012 8:33 am

Bart:”C) heat builds up at the interface between the surface and the atmosphere until its apparent temperature suggests an exceedance of the incoming radiance based on the SB relationship”
Just curious whether over the last couple of days you were able to come up with any evidence for this conjecture.
Cheers, 🙂

Bart
January 18, 2012 9:38 am

JJThoms says:
January 18, 2012 at 5:06 am
“In your mind you obviously have something heating the gas to a greater temperatuere than the surface – care to explain what this is.”
No. The surface is getting hotter, too. There is no steady state equilibrium anywhere in the system until the atmosphere is able to find an outlet for the energy building up in it. So, SB does not give you any information about surface temperature. Stefan-Boltzmann is a steady state theory. When you reach steady state, when the atmosphere has either found a heat sink in the form of radiative transfer or, barring that, boiling off into space, then, and only then will SB give you the lower bound on temperature.
“If what you say is correct then we can use a heat engine to extract power from the temperature difference = free energy – and I’m sure you can see that is wrong?”
You are hung up on this idea that there is an equilibrium somewhere. There isn’t. You can extract power from a non-equilibrated system.
shawnhet says:
January 18, 2012 at 8:33 am
“Just curious whether over the last couple of days you were able to come up with any evidence for this conjecture.”
Yes, quite a bit. A) As regards apparent versus real temperature, see my exchanges with DeWitt Payne. B) As regards the SB relationship, since there are no IR emitters in the atmosphere, temperatures are going to continue building continuously. See 1st reply to JJThoms above.

Bart
January 18, 2012 9:57 am

beng says:
January 18, 2012 at 7:26 am
“The reason the earth’s lapse-rate is near this is because it is constantly convecting up & down via Hadley cells.”
And, that is enabled by a heat sink above in the form of so-called greenhouse gasses, which can release energy in the IR, reversing the convection. Without that heat sink, there is no adiabatic lapse rate. Instead, the environmental lapse rate is determined by the temperature profile, which is a modified Bessel function qualitatively similar to a 1/r characteristic, where r is the radius from the Earth center.

January 18, 2012 10:52 am

Willis, if I may be so bold to ask, can you please tell me more about the amount of suns when you state “We might imagine that there are thousands of mini-suns in a sphere around the planet, so the surface heating is perfectly even.”
With the number of them fixed, perhaps I can work out how far away they are and how brightly they shine?
Would these mini-suns be much closer than one big sun? Could they have less mass? When the atmosphere is added to the planet it will gain mass and perhaps tug all the thousands of mini-suns closer? Would this be a case of gravity induced planetary heating with no laws broken?
NoIdea
PS I am not the NoIdea who had posted before on this thread.

shawnhet
January 18, 2012 11:03 am

Bart says:
January 18, 2012 at 9:38 am
““Just curious whether over the last couple of days you were able to come up with any evidence for this conjecture.”
Yes, quite a bit. A) As regards apparent versus real temperature, see my exchanges with DeWitt Payne. B) As regards the SB relationship, since there are no IR emitters in the atmosphere, temperatures are going to continue building continuously. See 1st reply to JJThoms above.”
Let’s see if I can be a bit more focussed. You seem to be hanging your hat on the following statement from the Niclos 2004 (where you apparently get your plots from).
“For larger angles, the effect of double or multiple reflections on the sea surface produces discrepancies between measured and theoretical SSEs, and more complex models should be used to get accurate SSE values”.
You are claiming, I believe, that the measured SSEs are in fact accurate and you do not, in fact, have to account for the effects of double or multiple reflections. I, personally, maintain that the experimental setup here is not analogous to that of the movement of the Earth through space, but even if I am wrong, you are left with the conundrum that you are *estimating* the drop off of emittance as the angle approaches 90 degrees (it is not measured). You, thusly, have at least two specific things that you need to demonstrate #1. that how you estimate emittance will drop off is actually how it does drop off and #2. that multiple reflection is not a problem for measuring emittance.
Cheers, 🙂

Bart
January 18, 2012 11:42 am

shawnhet says:
January 18, 2012 at 11:03 am
1) So, you’re saying we have no accurate measurements? Fine with me.
2) Events have passed this whole line of argument by, and I really don’t want to dwell on it anymore – the exact emissivity of the Earth’s surface is a tangential issue. The surface of the Earth is not hotter than it would be without IR radiating gasses (IRRGs, because GHG is an even worse misnomer than commonly though). Without IRRGs , the surface would be a lot hotter.
The IRRGs are what provide the heat sink to arrest the climb in temperatures. At steady state, it is no accident that they happen to intercept enough surface radiation to establish radiative balance – that is the essence of balance. But, the steady state conditions do not establish cause, any more than the fact that the FOV of the sky being clear establishes that there is a giant holding the Earth on his shoulders.
The proof is that the temperature gradient never goes away, due to the spherical distribution of the atmosphere. As a result, without a radiative heat sink in the atmosphere, there can be no steady state.

Joel Shore
January 18, 2012 11:47 am

beng says:

Willis, sorry if I’m beating a dead horse. The adiabatic lapse-rate formula is derived from an ideal gas process — compression/expansion. The reason the earth’s lapse-rate is near this is because it is constantly convecting up & down via Hadley cells.

IMO, the lapse-rate formula assumes convection is already occuring. Fire away.

You are right that when there is convection, the lapse rate is going to be close to the adiabatic lapse rate but you have the cause-and-effect partly backwards: Lapse rates less steep than the adiabatic lapse rate (such as in the stratosphere) are stable and hence convection is suppressed. Lapse rates steeper than the adiabatic lapse rate are unstable to convection and hence convection occurs, transporting heat upward until the lapse rate is equal to the adiabatic lapse rate.

Bart
January 18, 2012 12:11 pm

I wish I could provide the links to the Wikipedia articles but, of course, today that is not possible. In an atmosphere with no convection (the thought experiment world), the PDE governing the temperature is
dT/dt = alpha*del^2(T)
where alpha is the conductivity parameter and “d” is actually a partial differential operator. To solve this equation, we set T = T1(t)*T2(r), where T1 is wholly a function of t, and T2 is wholly a function of r. This leads to
(dT1/dt)/T1 = alpha*del^2(T2)/T2
where the “d’s” are now total differential operators. Since the left side is wholly a function of time, and the right wholly a function of r, they must both equal a constant, call it lambda.
Then, the solution of dT1/dt = lambda*T1 is elementary, T1 = T1(0)*exp(lambda*t). The solution of
del^2(T2) = (lambda/alpha)*T2
is a modified and adjusted zeroth order Bessel function of the second kind, which is qualitatively similar to a constant divided by radius.
Thus, as I discussed here, the full solution is
T = T(0,0)*exp(lambda*t)*F(r)
There is always a gradient downhill in the radial direction. Thus, there is always heat flow into the higher altitudes. This heat flow will continue until it stops, either by high energy radiation if the atmosphere allows, or boiling away of the atmosphere.

shawnhet
January 18, 2012 12:46 pm

Bart says:
January 18, 2012 at 11:42 am
“1) So, you’re saying we have no accurate measurements? Fine with me.”
No, I’m saying that your own reference puts specific limits on the accuracy of the measurements it makes. If you are going to ignore what your own reference says, you should provide some foundation for doing so. There are plenty of other references for the emissivity of water (see from the original SOD you referenced). Unfortunately, they all disagree with you on the emissivity of water.
“2) Events have passed this whole line of argument by, and I really don’t want to dwell on it anymore – the exact emissivity of the Earth’s surface is a tangential issue. The surface of the Earth is not hotter than it would be without IR radiating gasses (IRRGs, because GHG is an even worse misnomer than commonly though). Without IRRGs , the surface would be a lot hotter.”
Well, then you should revisit your elevator speech, I suppose. These issues follow directly from what you are talking about in your item C. I will wait and see what the new version looks like 😉
Cheers, 🙂

Bart
January 18, 2012 1:28 pm

shawnhet says:
January 18, 2012 at 12:46 pm
You are harping on items which are inconsequential at this juncture. But, OK, here is Bart’s elevator speech specifically tailored for shawnhet:
A) Radiation enters the system
B) it is absorbed by the surface, which fluoresces in the IR
C) heat builds up at the interface between the surface and the atmosphere until its apparent temperature suggests an exceedance of the incoming radiance based on the SB relationship
D) But, SB does not hold because the system is not in equilibrium. The heat that would be radiated away in steady state is, instead, being conducted to the atmosphere. SB will not hold until the system has reached an equilibrium!!!
E) the heat from the interface conducts through the atmosphere, establishing a temperature gradient as must exist in a spherically distributed atmosphere
F) the transferred heat accumulates in the atmosphere until:
1) highly energetic emissions are stimulated, which balances the energy fluxes all around in the same way GHGs would in the standard “greenhouse” theory
OR
2) the heat accumulates until the atmosphere achieves escape velocity and vanishes.
G) if greenhouse gasses which can radiate in the IR are added to the atmosphere, it will result in a net cooler atmosphere than otherwise would be the case, because there is now a lower energy outlet for heat to escape.

Bart
January 18, 2012 1:31 pm

I wonder if I stand a whelk’s chance in a supernova of wrestling this particular bone away from shawnhet.

Bart
January 18, 2012 1:38 pm

My apologies, shawnhet. You are right that the ES was not clear on point D. I was frustrated that you were not picking up on it, but people who see the original ES only without the subsequent discussion might be confused. Thank you for your comments.

Bart
January 18, 2012 1:56 pm

The alpha parameter is actually called “thermal diffusivity”, and it is inversely proportional to density. So, as the altitude increases, alpha will grow larger. So, even the Bessel function solution is not precise, and will only hold approximately in the lower atmosphere and before significant thermal expansion has taken place. A precise global solution would have to take all of these factors into account.
But, there is no chance of steady state because the steady state solution is T = B/r for a constant B, and since there is a gradient, there cannot be a steady state, and that creates a contradiction. So, the conclusion remains: there is always a thermal gradient pushing heat continuously into the atmosphere, and it will not stop until either there is some kind of radiative release, or the atmosphere flees.

shawnhet
January 18, 2012 2:07 pm

Bart:”D) But, SB does not hold because the system is not in equilibrium. The heat that would be radiated away in steady state is, instead, being conducted to the atmosphere. SB will not hold until the system has reached an equilibrium!!!”
I guess where I am coming from is that regardless of whether the system is in equilibrium if the energy from a surface of temperature 15C is conducting away from that surface without radiating, then the emissivity will appear to be less than if it is radiating as per SB. As such, you will still need to demonstrate some apparent emissivity that is in line with your numbers. The surface will either emit the energy it receives as radiation or it will not, but either way it will have consequences on what we measure the emissivity to be. If your hypothesis is valid, you should thusly be able to point to some evidence that (apparent) emissivity tracks with your hypothesis.
Cheers, 🙂

Bart
January 18, 2012 2:31 pm

shawnhet says:
January 18, 2012 at 2:07 pm
“…regardless of whether the system is in equilibrium if the energy from a surface of temperature 15C is conducting away from that surface without radiating, then the emissivity will appear to be less than if it is radiating as per SB.”
Stefan-Boltzmann is fundamentally a steady state theory. Emissivity is fundamentally a steady state quantity. It does not even make any sense to have the words “conducting” and “emissivity” in the same sentence. You absolutely must be in an equilibrium condition, or at the very least in something that can be plausibly called a quasi-equilibrium condition, or all bets are off.
Once you actually reach equilibrium, then and only then can you invoke SB. Let’s suppose the atmosphere absorbs at wavelengths higher than IR, but low enough that the air molecules do not achieve escape velocity. When the planet reaches a temperature such that ample amounts of that radiation are being intercepted, then an equilibrium is possible.
When that equilibrium is established, you would as easily be able to interpret it the same way you do the lower energy IR case: the above-IR-absorbing gas is intercepting the outward radiation of the surface, and the incoming flux matches the outgoing at TOA. But, in fact, what has happened is that the above-IR-absorbing gas has provided a heat sink to prevent the atmosphere from warming more.

Bart
January 18, 2012 2:34 pm

That is, the incoming flux matches the outgoing at TOA, and you could claim that the above-IR-absorbing gas has heated the surface above what it would be without an atmosphere. But, in fact, what has happened is that the above-IR-absorbing gas has provided a heat sink to prevent the atmosphere and the surface from warming even more.

Bart
January 18, 2012 2:36 pm

You see, the two interpretations are entirely equivalent. The steady state environment does not give you enough information to choose between the two.
But, because we know that a temperature gradient will always exist when the system is not in equilibrium, we know that the latter interpretation, that the above-IR-absorbing gas has provided a heat sink to prevent the atmosphere and the surface from warming even more, is the correct one.

DeWitt Payne
January 18, 2012 2:49 pm

Bart,
You’re neglecting the obvious trivial solution of:
dT/dt = alpha*del^2(T)
which is:
dT/dt = 0 , i.e. thermal equilibrium.
Then del^2(T) = 0 so T(r) = constant for all r. That constant would be the surface temperature. Your general solution only applies if dT/dt ≠ 0. Also, if alpha*del^2(T) ≠ 0, then T is varying with time and there isn’t thermal equilibrium.

shawnhet
January 18, 2012 3:04 pm

Bart:”Stefan-Boltzmann is fundamentally a steady state theory. Emissivity is fundamentally a steady state quantity. It does not even make any sense to have the words “conducting” and “emissivity” in the same sentence. You absolutely must be in an equilibrium condition, or at the very least in something that can be plausibly called a quasi-equilibrium condition, or all bets are off”.
I disagree. A substance can both radiate and conduct heat at the same time( it makes sense to talk about both concepts together because objects in the real world can do them together). If you perform the Niclos test on 15C water, it will appear to have an emissivity of some amount, regardless of whether or not it is in equilibrium or not. A system that has a temperature, radiates and conducts heat will always have an apparent emissivity. If it is not in equilibrium, then it may not have a *stable* (ie constant) emissivity.
Cheers, 🙂

Bart
January 18, 2012 3:09 pm

I want to bring up another point of evidence another commenter made in another thread which I found curious, but saw no resolution at the time. I do not remember the commenter’s name or where to find the post, so my apologies to him.
His question was, if the greenhouse effect on Earth raises the temperature level above -18C, but at -18C, the main greenhouse gas, water vapor, is in frozen state, how did we ever warm up?
My hypothesis here, as you can see, resolves this dilemma. It was always heating up, and the water vaporized, and stopped the rise.

DeWitt Payne
January 18, 2012 3:24 pm

Bart,

Come ON, you guys. Surely, you’ve solved the LaPlace equation somewhere along the way in your studies? Voltage from a point charge? Gravitational potential from a point mass? Ring a bell?

But this is heat flow in a medium, not the electric field from a point charge or the gravitational field from a point mass. The boundary condition that temperature has to decay smoothly to zero as r increases doesn’t apply. In fact, it leads to an obvious violation of the Second Law, temperature increasing without limit. I’ve also restricted the solution to atmospheric pressure high enough that local thermal equilibrium applies, i.e. to ~30km for a planet with a surface g = 9.81 m/sec², a surface pressure of 1013 hPa and a composition of a symmetric diatomic molecule with an effective molecular weight of 29 amu. The temperature above that point doesn’t matter as 99% of the atmosphere is below 30 km.
You keep saying that SB and by extension the Planck function, only applies when there is thermal equilibrium. That also isn’t true. The necessary condition is that local thermal equilibrium applies. That is always true for solids and liquids at normal temperatures at all times, i.e. not things like Bose-Einstein condensates, and is true for gases (Planck function times the emissivity at the emitting wavelength) as long as energy transfer by collision is much more probable than energy transfer by radiation. That also means the kinetic energy of the gas molecules must obey Maxwell-Boltzmann statistics. If SB only rarely applied, IR thermometers wouldn’t work as they rely on surface emitting according to the grey body version of SB (emissivity < 1).
Oh, and wind velocity roughening the sea surface can increase emissivity as well as lower it. It depends on the velocity.

jjthoms
January 18, 2012 5:23 pm

Bart says: January 18, 2012 at 1:28 pm
A) Radiation enters the system
B) it is absorbed by the surface, which fluoresces in the IR
C) heat builds up at the interface between the surface and the atmosphere until its apparent temperature suggests an exceedance of the incoming radiance based on the SB relationship
============
Solids above 0K radiate energy dependent on their temperature and their emissivity.
The sb curve just shows where the radiation peaks in the spectra.
It is not dependent on equilibrium – the body just radiates little at low temperatures and much at high temperature – instantly.
So if the planet warms then it radiates more – it cannot not radiate more. if 200w/sqm are received then the body cannot radiate more than this.
In your case you suggest that it gets hotter but does not radiate more – can you explain this please?
If a an instant change in radiation of gigawatts occurs the gas will still be heated only by surface. The surface will slowly heat from its current temp at a rate dependent on the thermal mass and the energy input. The non-ghg will only follow this temp it can never exceed the sb temperature. if this pulse disappears the surface temperature will start to cool.
If you chande its emmitance to 0 then it could reach the temperature of the suns. unfortunately if its emmitance is zero its reflectance is 100% so in fact it will neither heat nor cool

Bart
January 18, 2012 5:30 pm

DeWitt Payne says:
January 18, 2012 at 2:49 pm
“You’re neglecting the obvious trivial solution of:
dT/dt = alpha*del^2(T)
which is:
dT/dt = 0 , i.e. thermal equilibrium.”

Doesn’t work. T must go to zero at infinite radius.

Bart
January 18, 2012 5:40 pm

DeWitt Payne says:
January 18, 2012 at 3:24 pm
“…it leads to an obvious violation of the Second Law, temperature increasing without limit.”
No. Temperature will increase until it either A) excites the emission of higher frequency photons or B) the particles achieve escape velocity, and the atmosphere wafts away into space.
“The necessary condition is that local thermal equilibrium applies.”
There is no thermal equilibrium of any kind.
jjthoms says:
January 18, 2012 at 5:23 pm
“It is not dependent on equilibrium – the body just radiates little at low temperatures and much at high temperature – instantly.”
You guys really need to read up on SB. A surface will only radiate heat to the degree it cannot dissipate it otherwise.

The essential nature of black body (or Planckian) radiation is that it is the radiation resulting from thermal equilibrium. Thermal equilibrium is characterized by the absence of any net flow of energy. Just as a material body is characterized by a particular temperature and energy distribution (e.g. the Boltzmann distribution) when it is in thermal equilibrium, so too, the electromagnetic field may be thought of as a photon gas, characterized by a particular temperature and energy distribution (expressed by Planck’s law) when it is in thermal equilibrium.

Bill Illis
January 18, 2012 5:46 pm

An N2 atmosphere will grab heat energy off the surface faster than the relaxation IR emission time of the surface molecules. The time between collisions of surface atmosphere with solid surface material is just 0.00000000015 seconds which is many times shorter than the relaxation IR emission rate of the surface at SB temperature.
In other words, the atmosphere near the ground will heat up faster than the surface can emit in the IR. The energy accumulating in the atmosphere will be conducted up through the atmosphere until an equilibrium is reached between atmospheric collisional energy exchange at the surface and the amount that is being conducted up through the atmosphere.
One would to crunch the numbers to see if there is literally enough time difference between the two so that more energy ends up being stored in the atmosphere than is being stored at the surface accounting for both the SB temperature and the collision energy exchange rate.
But if the N2 atmosphere always grabs energy off the surface faster than the relaxation time, then there is NO IR emission. Now it can’t always be faster, there is always some really fast IR emission, but these amounts are small compared to the overall numbers.
In other words, the surface HAS to be warmer because it is has BOTH solar radiation and N2 atmosphere energy exchange. It will heat up until IR emission becomes very fast. In other words, many times hotter than the SB temperature.
Time and distance are important considerations in this debate but they are almost always ignored.

Bart
January 18, 2012 6:03 pm

DeWitt Payne says:
January 18, 2012 at 3:24 pm
“The boundary condition that temperature has to decay smoothly to zero as r increases doesn’t apply.”
At some point, the atmosphere effectively ends. So, you could argue that the concept of temperature simply breaks down at the ragged edge as the atmosphere reaches a particular density. But…
The edge of the atmosphere is going to be defined by the point at which the greater part of molecular velocities exceeds the escape velocity, which is proportional to the square root of the gravitational potential. If the temperature has to be uniform to, and is limited by, that distance, then you are imposing a uniform temperature limit which depends on gravity.
Out of the frying pan and into the fire. In disagreeing with me, you find yourself having to agree with N&Z!

DeWitt Payne
January 18, 2012 7:47 pm

Bart,

Doesn’t work. T must go to zero at infinite radius.

Wrong. The energy density must go to zero at infinite radius. And it does. For a gravitationally bound exponential atmosphere the total energy density in J/m3 goes to zero exponentially with altitude. Here’s a plot of altitude vs the the logarithm of total of gravitational plus kinetic energy/m3 calculated using 1005 J/kg K for the heat capacity of air at a constant temperature of 255 K at all altitudes. Note that the pressure at 100 km is 0.00188mbar so 99.9998% of the atmosphere is below 100 km. The US 1976 standard atmosphere has the temperature at 100 km equal to 195.1 K, not all that far from 255 K.

The edge of the atmosphere is going to be defined by the point at which the greater part of molecular velocities exceeds the escape velocity, which is proportional to the square root of the gravitational potential. If the temperature has to be uniform to, and is limited by, that distance, then you are imposing a uniform temperature limit which depends on gravity.

At 100 km altitude, the escape velocity has decreased by all of 87 m/s to 11,093 m/s. For nitrogen, the RMS velocity at 255 K is 476.6 m/s. The altitude at which the escape velocity is 476.6 m/s is 3.5E6 km.

There is no thermal equilibrium of any kind.

Wrong again. You’re simply asserting that with no proof whatsoever. You need to read up on the concept of Local Thermal Equilibrium and Kirchhoff’s Law. If LTE didn’t apply in the atmosphere, then it wouldn’t be possible to accurately calculate IR absorption and emission spectra. But it does and you can. If LTE didn’t apply to solids and liquids, it would be possible to construct perpetual motion machines of the second kind where the free energy is extracted from a single reservoir at one temperature.

DeWitt Payne
January 18, 2012 8:07 pm

Bill Illis,

An N2 atmosphere will grab heat energy off the surface faster than the relaxation IR emission time of the surface molecules. The time between collisions of surface atmosphere with solid surface material is just 0.00000000015 seconds which is many times shorter than the relaxation IR emission rate of the surface at SB temperature.

Here we go again. The emission rate is determined solely by the number of excited atoms or molecules and the decay rate. In the case of an excited molecule, that’s the Einstein A21 coefficient. The half life is approximately the inverse of the decay rate. But a molecule in an excited state can emit at any time. There are lots of molecules even if only a few percent are excited at any time. A solid or liquid surface has many more surface sites capable of emitting and absorbing radiation. At local thermal equilibrium the number of molecules or surface states in the excited state is constant. Local thermal equilibrium isn’t something rare. Most things are in LTE. Also, most of those collisions are elastic and don’t involve energy exchange so the actual lifetime of an excited state is longer than a nanosecond. But LTE requires that the collisional lifetime is orders of magnitude shorter than the radiative lifetime. See http://noconsensus.wordpress.com/2010/08/17/molecular-radiation-and-collisional-lifetime/ . As I remember, I’ve pointed this out to you before.
The only way the surface can heat to a temperature hotter than a black body is if the emissivity for long wavelength radiation is less than the absorptivity for short wavelength radiation. This isn’t true for most things that cover the surface of the Earth like water and dirt.

Bart
January 18, 2012 11:10 pm

DeWitt Payne says:
January 18, 2012 at 7:47 pm
“The energy density must go to zero at infinite radius.”
Of course the energy density goes to zero at infinite radius. The particle density goes to zero at infinite radius. But, the temperature also has to decline to the background radiation temperature of the cosmos, which is as close to 0K as makes little difference (actually, our hypothetical planet doesn’t have to inhabit this universe, so I’ll just proclaim it 0K).
“At 100 km altitude, the escape velocity has decreased by all of 87 m/s to 11,093 m/s. For nitrogen, the RMS velocity at 255 K is 476.6 m/s.”
That is the RMS deviation from orbital velocity. If that is the highest point you reach on an Earth orbit, you could be going as fast as 7844 m/s (circular orbit) +/- 477 m/s 1-sigma.
But, we’re not talking about Earth here, you know. Go ahead and specify that the hypothetical non-GHG planet has an Earth-like mass, if you like. But, then, it is still not required to have anything like the same temperatures.
That’s my whole point. It’s not going to have the same temperatures. If it has no other mechanism to radiate energy away, then eventually you could reach 2550K, or 25500K. You’re sure to start losing your atmosphere, then.
Doesn’t sound plausible? Of course it doesn’t! But, that is because we have an implausible atmosphere on a thought planet that doesn’t even radiate in the ultraviolet or beyond!
But, it still has a mechanism to prevent “temperature increasing without limit”, which was your objection, and which has been satisfied.
“If LTE didn’t apply in the atmosphere, then it wouldn’t be possible to accurately calculate IR absorption and emission spectra.”
In Earth’s atmosphere. Fine. So what?
“You need to read up on the concept of Local Thermal Equilibrium and Kirchhoff’s Law.”
OK.

A system is not in local thermodynamic equilibrium if the local kinetic (Maxwellian) temperature is not equal to the Planckian temperature of the radiation field.

Since that is precisely what I am asserting, that the surface is not radiating at its local kinetic temperature, it means I am asserting it is not at LTE. It cannot be, because a significant portion of the absorbed heat is flowing into the atmosphere.

Bart
January 19, 2012 12:26 am

DeWitt Payne says:
January 18, 2012 at 8:07 pm
“The only way the surface can heat to a temperature hotter than a black body is if the emissivity for long wavelength radiation is less than the absorptivity for short wavelength radiation.”
Kirchoff’s law states that blackbody irradiance is the most you can get for a given temperature.
J = (J/sigma)^(1/4)
It is a lower bound for temperature given radiance.

Bart
January 19, 2012 12:29 am

Duh… I used an inequality sign, and it turned into an html tag. Should have been
J .LTE. sigma*T^4
implies
T .GTE. (J/sigma)^(1/4)

Bill Illis
January 19, 2012 2:36 am

DeWitt Payne says:
January 18, 2012 at 8:07 pm
“The only way the surface can heat to a temperature hotter than a black body is if the emissivity for long wavelength radiation is less than the absorptivity for short wavelength radiation.”
—-
The surface temperature of every body that has an atmosphere that we know about in the universe, is hotter than its blackbody temperature.

beng
January 19, 2012 5:07 am

****
Joel Shore says:
January 18, 2012 at 11:47 am
You are right that when there is convection, the lapse rate is going to be close to the adiabatic lapse rate but you have the cause-and-effect partly backwards: Lapse rates less steep than the adiabatic lapse rate (such as in the stratosphere) are stable and hence convection is suppressed. Lapse rates steeper than the adiabatic lapse rate are unstable to convection and hence convection occurs, transporting heat upward until the lapse rate is equal to the adiabatic lapse rate.
*****
Thanks Joel (I do appreciate your comments). I have to think more about this. It’s not just me — after reading this blog & others, seems quite a few obviously smart people aren’t sure about this either. I agree any real, surface-heated world should develop some lapse-rate in their atmospheres. As you say, sorting out cause & effect seems to be the problem.

January 19, 2012 5:18 am

This is interesting…I didn’t know O2 was magnetic…
By 1891, [James] Dewar could produce liquid oxygen in large quantities, and also showed that it and liquid ozone were strongly attracted by a magnet.
–This Month in Physics History, APS News, January 2012

Joe Postma
January 19, 2012 7:17 am

Bart has a fantastic set of posts here. Excellent scientific analysis, and I need to review it a bit more.
I’d like to point out yet another oddity of standard GHE theory. There’s supposed to be back-radiation from IR-emitting molecules, predominantly CO2, that cause 33C of additional heating. That’s the basic GHE theory.
So tell me then: on a spectrometer plot taken from the surface of the Earth and pointing upwards, or even from one taken from above the Earth and looking down – where is the emission line?
If CO2 is radiating all this energy and by definition this emission has to be spectral, then where is the huge & incredibly bright emission line introducing an additional 150 W/m2 into the surface, and that which should also be exiting the TOA? The ENTIRE output spectrum of the Earth only comes out to 240 W/m2, so this additional 150 W/m2 from spectral emission must be huge! Where is it?
It doesn’t exist.
Instead, where it could exist, is a huge hole in the spectrum, a LACK of energy power. It HAS TO EMIT to be said to cause heating by radiative emission in the first place. Yet where it could emit, it doesn’t exist. And at the bottom of the notch where CO2 should be emitting to add all this extra power, is a smooth blackbody curve corresponding to something like -80C. There is a very small emission peak right at 15um, but it’s barely worth noting.
So fine, let’s pretend to go with the standard theory of GHG back-emission. And so I’ll ask: where is the emission?
Pointing an IR sensor at the sky and it telling you the temperature, converted to some power units, is simply taking a temperature reading! It has NO relevance to the huge emission line we should see from CO2 & GHG’s causing all this heating. That’s a fraudulent interpretation of the measurement! The IR sensor is measuring a rough black-body that has LACK of emission flux at GHG wavelengths.
It might just be that simple. Fine: radiation causes heating, we all know that. So show me the radiation from CO2 then. Oh it doesn’t exist? Well then what the heck…non-existent, non-observable spectral emission from GHG’s causes heating. Wonderful.
Now someone might try to back-track and say that the downward IR gets all absorbed by the time it reaches the ground, and because it is all absorbed this is why it is causing heating. But wait, it can’t very well be said to be heating the surface then, can it, if it never GETS to the surface. And additionally, you can’t hide at the bottom of the atmosphere anyway – the TOA is free to space and there’s NO reason we shouldn’t see the 150 W/m2 of GHG spectral emission there. But it’s not there either, is it; except for a tiny little pip at 15um with maybe a couple of Watts in it.
Someone might also try to back-track and say that because all the 15um radiation is absorbed and you don’t see it, that’s why it causes heating. But that’s still inconsistent with the spectral reading at the TOA – it should still be seen at the TOA – and it also implies that LACK of emitted radiation is what causes heating. But that’s what non-GHG’s implicitly do in the first place – not radiate and therefore trap heat – and the whole GHG theory is based on the idea that GHG’s spectrally radiate.
This whole theory is shot; full of holes. With this OP and the comments here in it, and the other one by Robert Brown and the comments in that one, it is clear to anyone reading that GHG Theory is dead. I still think my treatises give a good explanation of why it is dead.
http://www.tech-know.eu/uploads/Understanding_the_Atmosphere_Effect.pdf
http://principia-scientific.org/publications/The_Model_Atmosphere.pdf
http://principia-scientific.org/publications/Copernicus_Meets_the_Greenhouse_Effect.pdf
Read only the last of those links if you want – it is a very short paper with a succinct summary of the paradigm shift.
Of course, I give thanks to the book that started it all:
http://www.amazon.com/Slaying-Sky-Dragon-Greenhouse-ebook/dp/B004DNWJN6

DeWitt Payne
January 19, 2012 8:19 am

Joe Postma,
I suggest you get a textbook on atmospheric radiative transfer and study it so you can make an intelligent argument. Your post above clearly shows you have no idea about how the atmospheric greenhouse and molecular absorption and emission of radiation in the atmosphere works. The fact that there is a valley in the emission spectrum to space at the CO2 band is exactly why CO2 is a greenhouse gas. Grant Petty, A First Course in Atmospheric Radiation, 2nd edition, Sundog Publishing would be a good choice. Or, for a start, read Chapter 5 in Caballero’s Lecture Notes on Physical Meteorology: http://maths.ucd.ie/met/msc/PhysMet/PhysMetLectNotes.pdf
In the spectrum from the surface looking upward, there is no line because the emission cannot exceed the intensity of a black body at the same temperature. In fact, the emission is calculated by multiplying the emissivity, which cannot exceed 1, at a wavelength by the Planck function for that wavelength and temperature. The emission spectrum in the CO2 band closely matches a blackbody at the surface temperature because the emissivity/absorptivity is almost exactly one for center of the CO2 band.

DeWitt Payne
January 19, 2012 8:44 am

Bart,
The kinetic temperature of a gas is not a field emanating from a point source. It’s a function of the local RMS velocity and mass of the molecules. There is no reason whatsoever for that to decline to zero as r increases, at least until the number density of molecules is so low that their mean free path is effectively infinite. And only then if the molecule velocity exceeds the escape velocity. As I pointed out, the kinetic temperature of the actual atmosphere at an altitude of 100 km is ~200K for a surface temperature of 288K. Below 100 K, the temperature varies such that in the stratosphere the temperature is higher than 200K. At 50km altitude for the 1976 US standard atmosphere, the temperature is 270K, not far from the surface temperature of 288K.
If a gas molecule at high altitude were actually in an orbit that didn’t intersect the surface, it’s velocity would be at least an order of magnitude greater than the RMS kinetic velocity at 255 K. That would imply a temperature of ~30,000K. But of course, they don’t have to be in a non-intersecting orbit, and the vast majority aren’t. They stay approximately where they are because they are continually bouncing off other molecules.

Tim Folkerts
January 19, 2012 8:58 am

I don’t have time to address ALL the issues that keep coming up, but let me make one correction to what I said earlier about lapse rates.
IN PRINCIPLE, in a perfectly transparent atmosphere that cannot radiate or absorb EM radiation (and also is not being hit by a solar wind or other energy inputs) that sits around a planet with a perfectly uniform temperature, the equilibrium temperature profile of the atmosphere will be uniform at the temperature of the surface. There will be NO lapse rate.
If the atmosphere DID get a temperature gradient, thermal conduction would slowly bring the temperature profile back to a uniform temperature throughout.
IN PRACTICE, having a lapse rate would be the expected situation. Small amounts of convection would set up a lapse rate. As long as there is convection (which would always happen on a rotating planet that goes from cooler nights to warmer days day), the atmosphere will get some sort of lapse rate.
Furthermore, the power flow through the atmosphere when the temperature gradient is as large as 10 K/km would be less than 1 mW/m^2. So all that is needed to maintain a lapse rate is an energy flow out of the top of the atmosphere of around 1 mW/m^2. Even the tiniest amounts of GHGs or dust would be enough to do that. It is possible that even non-GHGs would be able to radiate that amount. In any case, the actual radiation by GHGs from high in the stratosphere is way more than 1,000x higher than needed to maintain a lapse rate.

Joel Shore
January 19, 2012 9:12 am

Joe Postma says:

If CO2 is radiating all this energy and by definition this emission has to be spectral, then where is the huge & incredibly bright emission line introducing an additional 150 W/m2 into the surface, and that which should also be exiting the TOA?

In addition to what DeWitt Payne said, I should just add that I assume the 150 W/m^2 that you are referring to is a measure of the total radiative forcing due to the greenhouse effect. That being the case, it is not all due to CO2…In fact, the largest portion is due to water vapor and clouds.

Joe Postma
January 19, 2012 9:14 am

Some people agree that there’s no spectral emission from GHG’s. Thank you.

Joel Shore
January 19, 2012 9:25 am

@Tim Folkerts:
I think your statement is a bit backwards: Convection does not set up a lapse rate. Convection only occurs when the lapse rate exceeds the adiabatic lapse rate and it is what drives a lapse rate exceeding the adiabatic lapse rate back down to the adiabatic lapse rate.
If the lapse rate is already less than the adiabatic lapse rate, the atmosphere is stable and convection is actually suppressed.

David
January 19, 2012 9:31 am

A planet (earth, ocean atmosphere) can only cool by radiating to space.
Non GHGs store conductive specific heat, but cannot radiate it to space. (Insulation)
They can however conduct specific heat to radiating GHGs.
Adding more GHGs to an atmosphere accelerates the loss of conducted heat to space, while it slows the loss of radiated heat from the surface. (cooling and warming)

David
January 19, 2012 9:41 am

Tim Folkerts says:
January 19, 2012 at 8:58 am
Tim, as long as there is insolation to the surface and conducted heat to the atmosphere,(even sans convection as in a thought experiment) there would be a thermal gradient based on the mass per m2.

Joe Postma
January 19, 2012 9:45 am

I can’t figure out how to subscribe to this thread without posting. So I am posting 🙂 Happy Thursday.

Tim Folkerts
January 19, 2012 9:49 am

Joe Postma says: “Some people agree that there’s no spectral emission from GHG’s. Thank you.”
And some people are wrong.
GHGs emit very clear bands of IR radiation (due to rotations & vibrations). In addition, ALL gases (including GHGs) emit radiation when electrons jump to/from higher energy orbitals (which is typically visible or UV or near IR, but which does not occur to any measurable extent at room temperature).
People who don’t understand these simple ideas shouldn’t even begin to try explaining the greenhouse effect to others.

DeWitt Payne
January 19, 2012 9:51 am

Bart,
I think I see the problem. Your solution applies to the radiative temperature, which indeed decreases with r. But the specification is that the atmosphere is perfectly transparent, which means it’s not coupled to the radiative temperature at all. In an atmosphere that isn’t perfectly transparent, the radiative field doesn’t decouple from the atmosphere until the atmosphere becomes optically thin, which is well above the surface.
Before I retired, one of the things I did was argon plasma optical emission spectrometry for elemental analysis. The analytical plasma isn’t at LTE because it’s optically thin and a lot of gas and aerosol is being continually pumped through it. The reason that it’s known not to be in LTE is because there are a variety of ways to measure the temperature of the plasma. There’s the electron number density (it is a plasma), the excitation temperature, the brightness temperature and the gas kinetic temperature. If all those temperatures are the same, LTE can be said to exist. But they aren’t and it doesn’t. But you try to arrange things so you can get as close as possible to LTE in the observation zone because the analysis is more robust to changes that occur when different samples are introduced. The usual way to measure how close you were to LTE was to measure the excitation temperature. That was done by measuring the emission intensities of two lines of the same element with different excitation energies, an intense Mg atomic line and an intense Mg singly ionized line. Since the plasma is optically thin, the intensity is directly proportional to the number density of excited atoms. The number density of excited atoms can be used to calculate an excitation temperature based on the Maxwell-Boltzmann distribution. The emission ratio for a constant temperature can then be calculated. But the observed ratio is always less than the calculated ratio. You adjusted the sample flow, viewing height and RF power to maximize the ratio to achieve what was called in the trade, near LTE conditions.
In the atmosphere, you can do something similar by measuring the emission intensity of a molecule. If the emission intensity matches the calculated emission intensity for the gas kinetic temperature, then LTE must exist. You can do the same thing for a surface except what you look at is the shape of the emission spectrum. That way emissivity, unless it varies strongly with wavelength, doesn’t matter. The best IR thermometers use this technique. The emission intensity is measured at two wavelengths and the ratio calculated. The temperature determined by that ratio from the Planck function is independent of emissivity as long as the emissivity is the same at both wavelengths. Emission spectra from surfaces does, in fact, reasonably closely match the shape of the Planck function for the kinetic temperature as measured by a contact thermometer. The variation in emissivity determined by deviation from the Planck curve is being used in the Diviner moon mission to map the geology of the lunar surface as the emissivity spectrum varies in known ways for different minerals.
Another point. If the atmosphere and surface weren’t at LTE, remote satellite sensing of temperature couldn’t work.

Phil.
January 19, 2012 9:53 am

Joe Postma says:
January 19, 2012 at 7:17 am
Bart has a fantastic set of posts here. Excellent scientific analysis, and I need to review it a bit more.
I’d like to point out yet another oddity of standard GHE theory. There’s supposed to be back-radiation from IR-emitting molecules, predominantly CO2, that cause 33C of additional heating. That’s the basic GHE theory.
So tell me then: on a spectrometer plot taken from the surface of the Earth and pointing upwards, or even from one taken from above the Earth and looking down – where is the emission line?

This is embarrassingly bad, your lack of knowledge about spectra in atmospheres is abysmal. Take the advice that I and others have given you about references to read, by scientists who know what they’re talking about, and come back here with some humility. The earth’s atmosphere is not interstellar gas, the physics is different because of the density and the relative rates of collisions to emission rates. A hint to help you, the spectrum of the CO2 band at it’s center has no near surface emissions, that was absorbed and transferred as heat to the rest of the atmosphere. The non-zero signal you see at the band center is the emission from cold CO2 much higher in the atmosphere where the collision frequency is low enough that emission directly to space predominates. Astrophysicists may need to understand many different fields as you claimed before, but you are one whose knowledge in the field of spectroscopy outside interstellar gases is non-existant, I suggest you do what you told me you were going to do last week and talk to one of your professors who understands this area and get educated.

DeWitt Payne
January 19, 2012 9:57 am

Joel Shore,

If the lapse rate is already less than the adiabatic lapse rate, the atmosphere is stable and convection is actually suppressed.

That’s true if the surface is isothermal. But it’s not. The meridional temperature gradient forces circulation because of the pressure gradient force. That circulation forms a heat engine that drives what might other wise be an non-radiative isothermal atmosphere back towards the adiabatic rate. For a non-transparent atmosphere, radiation to space cools the atmosphere and also forces the lapse rate towards the adiabatic rate.

DeWitt Payne
January 19, 2012 9:58 am

Joe Postma,

Some people agree that there’s no spectral emission from GHG’s. Thank you.

Those people would be wrong.

Phil.
January 19, 2012 9:59 am

Ken Coffman says:
January 19, 2012 at 5:18 am
This is interesting…I didn’t know O2 was magnetic…
By 1891, [James] Dewar could produce liquid oxygen in large quantities, and also showed that it and liquid ozone were strongly attracted by a magnet.
–This Month in Physics History, APS News, January 2012</em.
It's paramagnetic because of the unpaired electrons, this is high school chemistry. Oxygen sensors use this to measure the concentration of O2 because N2 is diamagnetic.

Tim Folkerts
January 19, 2012 10:06 am

Joel, you are right that I should have said that differently about the lapse rate & convection. The cooling at the top of the atmosphere should have been the main point. There is on the order of 100 W/m^2 of cooling from the TOA. Conduction itself can only provide on the order of 0.001 W/m^2, so conduction will never be able to erase the lapse rate.
(And I might argue that where the downdrafts are from convection, there the convection would help create the temperature gradient. The cool air high in the atmosphere that descends will warm, and so there would be a lapse rate in those areas from convection. In the areas where the air is rising, then the convection would indeed help erase excessive lapses rates. But this is really just a side issue to the point that a lapse rate WILL get set up in any normal sort of atmosphere, even if the “ideal” case is to have no lapse rate.)

Tim Folkerts
January 19, 2012 10:18 am

David says: “Tim, as long as there is insolation to the surface and conducted heat to the atmosphere,(even sans convection as in a thought experiment) there would be a thermal gradient based on the mass per m2.”
I would simplify this even further. “There is conducted heat if and only if there is a thermal gradient.”
The “mass per m2” could affect the rate of conduction, so in that sense the conduction is “based on” mass per m^2. In my book, the cooling of the upper atmosphere (IR radiation from GHGs) would be the “cause” of conduction of heat and the thermal gradient.
(But as I just argued in a different post, conduction is typically orders of magnitude smaller than convection, evaporation and IR radiation, and so it typically plays an inconsequential role in the dynamics of the troposphere.)

Joe Postma
January 19, 2012 10:20 am

================================
DeWitt Payne says:
January 19, 2012 at 9:58 am
Those people would be wrong.
DeWitt Payne says:
January 19, 2012 at 8:19 am
The fact that there is a valley in the emission spectrum to space at the CO2 band is exactly why CO2 is a greenhouse gas.
================================
Yet the spectral emission from CO2 is supposed to also be why CO2 is a heating GHG. So the presence and lack of CO2 emission is why CO2 is a GHG. Got it, thanks. The spectral emission from CO2 which does and doesn’t exist and why CO2 is a heating and a cooling gas. Thanks.

Spector
January 19, 2012 10:21 am

RE: Bart: (January 17, 2012 at 6:22 pm)
Ref: [The problem with the gravitational heating theory is that it does not provide any mechanism to continuously extract or reflect an average 156 W/m² from a hypothetical average 396 W/m² radiant energy flow of a typical GHG warmed surface so that only the allowed average 240 W/m² actually escapes to outer space.]
“What that means is not that the GHGs heated the surface above what it otherwise would have been. Rather, it means it is maintaining it that much cooler below what it otherwise would have been.”
The Earth is only receiving enough energy from the sun to emit a global average energy flow of about 240 W/m². In that case, the fourth root of the flat area weighted average of the fourth powers of the absolute temperatures around the earth would be on the order of 255 degrees K.
According to the Trenberth diagram, the Earth is actually warm enough to radiate around 396 W/m² from the surface. But the Earth cannot afford to let that radiation get out to space and still maintain a balanced energy budget. Greenhouse gases in the atmosphere solve this problem by continuously capturing 156 W/m² from that flow so that the energy budget remains balanced with our higher surface temperatures. If there were no greenhouse gases in the atmosphere, then there would be nothing to stop the escape of excess radiant energy that the Earth could not afford to lose.
The real problem here is not the greenhouse theory; I think it is the purported hypersensitivity of the atmosphere to the self-masking, narrow band effect of carbon dioxide, by treating it in the public mind as a uniform darkening agent.

Joe Postma
January 19, 2012 10:22 am

=======================================
Tim Folkerts says:
January 19, 2012 at 9:49 am
GHGs emit very clear bands of IR radiation (due to rotations & vibrations). In addition, ALL gases (including GHGs) emit radiation when electrons jump to/from higher energy orbitals (which is typically visible or UV or near IR, but which does not occur to any measurable extent at room temperature).
People who don’t understand these simple ideas shouldn’t even begin to try explaining the greenhouse effect to others.
=======================================
Show me then. The little bit that you can identify of CO2 emission is a mere couple of Watts.
The GHE is described and exposed here:
http://principia-scientific.org/publications/The_Model_Atmosphere.pdf

Joel Shore
January 19, 2012 10:29 am

Grant Petty, A First Course in Atmospheric Radiation, 2nd edition, Sundog Publishing would be a good choice. Or, for a start, read Chapter 5 in Caballero’s Lecture Notes on Physical Meteorology: http://maths.ucd.ie/met/msc/PhysMet/PhysMetLectNotes.pdf

See, in particular, Figure 5.17 on p. 131 which directly shows that what Joe Postma is spreading here are falsehoods.

Joe Postma
January 19, 2012 10:35 am

======================================
Phil. says:
January 19, 2012 at 9:53 am
This is embarrassingly bad, your lack of knowledge about spectra in atmospheres is abysmal. Take the advice that I and others have given you about references to read, by scientists who know what they’re talking about, and come back here with some humility. The earth’s atmosphere is not interstellar gas, the physics is different because of the density and the relative rates of collisions to emission rates. A hint to help you, the spectrum of the CO2 band at it’s center has no near surface emissions, that was absorbed and transferred as heat to the rest of the atmosphere. The non-zero signal you see at the band center is the emission from cold CO2 much higher in the atmosphere where the collision frequency is low enough that emission directly to space predominates. Astrophysicists may need to understand many different fields as you claimed before, but you are one whose knowledge in the field of spectroscopy outside interstellar gases is non-existant, I suggest you do what you told me you were going to do last week and talk to one of your professors who understands this area and get educated.
======================================
Well I know that you all want to backtrack and will take the opportunity to make this a personal assault instead of a scientific one. With every failed logic that’s newly exposed the alarmists will retaliate with innuendo.
So you agree that you can’t see GHG emission near the surface. And you also can’t see it from space. Thank you. So non-existent spectral emission from GHG causes heating. Got it.
Indeed, it is exactly the collision frequency that describes the processes going on. Being collision dominated means the GHG’s don’t have time to radiate. Therefore they can’t re-heat the surface. But that’s what you needed them to do. Indeed, we SEE that they don’t radiate in the spectra. It also means their internal vibratory state is already activated by collision, not by radiation. Most likely outcome is that GHG’s either do nothing, or they cause cooling. I mean we already know that the thermal mass of the air+ ground column keeps the surface from getting as hot as it could.

Bart
January 19, 2012 10:53 am

DeWitt Payne says:
January 19, 2012 at 8:44 am
“There is no reason whatsoever for that to decline to zero as r increases, at least until the number density of molecules is so low that their mean free path is effectively infinite.”
Really, DeWitt. Is it so hard for you to believe that a region near the heat source is hotter than a region far from it? You are grasping at straws.
“As I pointed out, the kinetic temperature of the actual atmosphere at an altitude of 100 km is ~200K for a surface temperature of 288K.”
Please stop talking about the “actual atmosphere” of the Earth. We’re not talking about the Earth. The hypothetical is, what happens on a planet without GHGs? There is no reason whatsoever that it should be anything like your everyday experience of the Earth.
“That would imply a temperature of ~30,000K.”
Orbital velocity is not counted in temperature because the particle is simply following a geodesic in space. Certainly, our satellites in orbit do not radiate like blackbodies at 30,000K!
“But of course, they don’t have to be in a non-intersecting orbit, and the vast majority aren’t.”
At low Earth altitudes where the atmosphere acts as a fluid, this is indeed true. There, the average vector velocity of air molecules is the velocity needed to keep up with the spin of the Earth, which is not terribly large. When you get up into the rarefied regime, it’s a little more haphazard, but your statement is still generally true.
For the Earth and its atmosphere.
But, there’s still a limit to how much the temperature can increase, and your objection of a 2nd law violation is invalid.
Tim Folkerts says:
January 19, 2012 at 8:58 am
“…the equilibrium temperature profile of the atmosphere will be uniform at the temperature of the surface.”
Quite impossible.
“There will be NO lapse rate.”
There will be no adiabatic lapse rate. Such a lapse rate requires a heat sink in the upper atmosphere. Without that heat sink, you do not get a linear lapse rate. You get a convex, monotonically decreasing function of altitude.

Bart
January 19, 2012 10:59 am

DeWitt Payne says:
January 19, 2012 at 9:51 am
“Your solution applies to the radiative temperature”
It applies to conducted temperature. The Laplacian is the expression for conducted heat glow. It results from Fourier’s law and conservation of energy
“If the atmosphere and surface weren’t at LTE, remote satellite sensing of temperature couldn’t work.”
On Earth. There’s not going to be much of a remote sensing industry on our hypothetical non-GHG planet.

Bart
January 19, 2012 11:12 am

Bart says:
January 19, 2012 at 10:53 am
“Without that heat sink, you do not get a linear lapse rate. You get a convex, monotonically decreasing function of altitude.”
In fact, if you look at a plot of temperature versus altitude for the Earth, you find that it is linear in the troposphere and the mesosphere. This suggests massive heat sinks at the top of those regions.
The stratosphere and the thermosphere show the transition to measured decay of temperature which would be expected when there is no proximate outlet for heat to radiate away.
Coincidence?

Bart
January 19, 2012 11:25 am

I will not be able to post much more today. I want to ask those who have learned the standard greenhouse model to try thinking outside the box. You will find that there is no steady state difference between the standard description, and what I am offering.
Thus, you have to start asking for evidence which would resolve which description fits with all the facts. I have offered the main differentiator: the temperature must decrease to zero at infinite radius. Hence, the heat equation says that there will be a gradient downhill in the (upward) radial direction. There can, thus, be no equilibrium until enough heat is dissipated by radiation.
It’s as plain as the noses on your faces: IR absorbing gasses in the Earth’s atmosphere cool the surface, and prevent runaway temperatures.
“Everything not forbidden is compulsory,” said Murray Gell-Mann. There is nothing forbidden in my prescription.

Bart
January 19, 2012 11:29 am

Bart says:
January 19, 2012 at 10:59 am
DeWitt Payne says:
January 19, 2012 at 9:51 am
Erratum:
“The Laplacian is the expression for conducted heat flow…”

Bart
January 19, 2012 11:34 am

Joe Postma says:
January 19, 2012 at 10:35 am
“So you agree that you can’t see GHG emission near the surface.”
With all due respect to Joe, and without taking sides, I want to decouple this line of argument from my own. It is my contention that, in steady state, my description of the process results in an identical situation to the standard greenhouse theory. The heat sinks will be radiating, the backradiation will cancel with radiation from the surface, and everything balances out in exactly the same way, satisfying the SB relationships in the equilibrium condition.

DeWitt Payne
January 19, 2012 11:43 am

Bart,
My graduate work was in electrochemistry, so I know something about using Laplace transforms to solve the diffusion equation for spherical (mercury drop electrode, e.g.) as well as cartesian coordinates. When solving a semi-infinite diffusion problem, there is no requirement for the concentration to go to zero at infinity. If it did and the concentration was not zero at the electrode surface, there would be diffusion away from the electrode until the concentration at the surface was also zero. A concentration gradient can only be stable if there’s a continuous flux into and out of a volume. For an isothermal atmosphere there is no flux out and the volume isn’t infinite. Admittedly, it would take a very long time to reach equilibrium because diffusion is slow. For radiation emitted by the surface, though, there is flux in and out of any volume element above the surface and the total volume is effectively infinite. If the energy lost by radiation isn’t replaced somehow, the surface cools. But again, the problem is defined as energy into the surface from short wavelength radiation equal energy out of the surface from long wavelength radiation.
The boundary conditions for the radiation problem is a flux of 240 W/m² at r = 6378 km and a flux of zero as r increases without limit. The flux will then drop as 1/r². That would make the flux at 100 km above the surface equal to (6378/6478)^2 * 240 = 232.65W/m² so Teff would drop from 255K to 253.1K. And of course there is no accumulation of energy at the TOA because the emitted photons just keep going on (transparent atmosphere).

Joe Postma
January 19, 2012 11:45 am

=====================================
Bart says:
January 19, 2012 at 11:34 am
With all due respect to Joe, and without taking sides, I want to decouple this line of argument from my own. It is my contention that, in steady state, my description of the process results in an identical situation to the standard greenhouse theory. The heat sinks will be radiating, the backradiation will cancel with radiation from the surface, and everything balances out in exactly the same way, satisfying the SB relationships in the equilibrium condition.
=====================================
Oh yes I agree with that. I didn’t assume my criticism and your analysis were coupled 🙂
Cheers.

Tim Folkerts
January 19, 2012 11:56 am

Joe,
It is easy to see the radiation of GHGs both from below the atmosphere and from above the atmosphere. For example: http://www.skepticalscience.com/images/infrared_spectrum.jpg. (This is only above MOST of the atmosphere, but the point still stands.
GHG’s “take bites out of the spectrum” as seen from space, and “create bites in the spectrum” as seen from the ground. I’m not sure how the evidence could be any clearer than this. Nothing that Phil said implied that you couldn’t see the IR radiation. The fact that you are having trouble understanding how the “bites” as seen from space warm the planet means you are still missing some fundamental understanding of the atmospheric greenhouse effect.

DeWitt Payne
January 19, 2012 12:04 pm

That should be: For a transparent atmosphere… , not: For an isothermal atmosphere…
Bart,

The stratosphere and the thermosphere show the transition to measured decay of temperature which would be expected when there is no proximate outlet for heat to radiate away.

Oh, really. Not according to this: http://teachertech.rice.edu/Participants/louviere/Images/profile.gif
In the Ionosphere, the temperature can reach 4,000K at altitudes above 500km ( http://www.agu.org/pubs/current/si/links/2009JA014485.pdf ). Greenhouse gases (other than ozone) do make the stratosphere cooler than it would be otherwise because the stratosphere is optically thin in the thermal IR range and most greenhouse gases do not absorb significantly in the UV. But the troposphere is optically thick for much of the wavelength range so the reduced emission at higher altitude makes the lower atmosphere warmer than if they weren’t present, not cooler. So the stratosphere has been getting a double whammy: a reduction in ozone causing less heating, and an increase in CO2 causing more cooling.

Phil.
January 19, 2012 12:06 pm

ZP says:
January 17, 2012 at 9:18 pm
Willis Eschenbach says:
January 17, 2012 at 5:35 pm
“Come back with the absorption lines for argon in the normal IR range.
I say it absorbs none. Zero. It only absorbs or emits in one way, through electron transitions, and that takes a whole bunch of energy to force one of those, and it doesn’t happen in the IR. Because argon is a monoatomic gas, the atoms of argon have no mechanical way to absorb or emit energy through bending or stretching. There is no physical way for the atoms to absorb or emit energy, an atom of argon is a little ball and the energy simple can’t interact with it except in high-energy electron transitions.”
Willis, you are correct that argon cannot have bending or stretching modes of vibration since it is a monoatomic gas. It can only have electronic transitions; the energy of which depends on the orbitals that the electrons are jumping between. However, you are overestimating the energy associated with most transitions, especially those occurring between n > 3 and n = 3. This overestimation leads you to the incorrect conclusion that argon does not absorb/emit IR radiation.

Your over-estimation of the fraction of Argon atoms in the third electronic level under the Earth’s atmospheric conditions leads you to this incorrect conclusion (and perhaps also Anna’s error). Look up the energy of the third excited state and do a Maxwell-Boltzmann distribution calculation at T=300K you’ll find that it’s essentially all in the ground electronic state which does not absorb in the IR.

Editor
January 19, 2012 12:30 pm

The replies to Joe Postma have been too complicated. Surely, the issue is simple.
1 – Sunlight passes through the atmosphere and reaches the surface, heating it.
2 – Consequently, radiation at certain wavelengths is emitted from the surface.
3 – A subset of that radiation – let us call it “wavelength set A” – is substantially absorbed by GHGs, chiefly CO2 and H2O.
4 – Those GHGs then re-radiate at various wavelengths – let us call it “Wavelength set B” – in all directions.
5 – Some (about half) of B reaches the surface.
6 – We should observe less of set A at TOA than we would expect from steps 1 and 2 above on their own.
7 – We should observe more of set B at the surface than we would expect from the sun alone.
So the question that resolves the issue is simply “do we observe less of set A at TOA and more of set B at the surface?“.
NB. There is no assumption that A and B are different sets, and if sunlight contains some A then allowance has to be made for some absorption on the way in.
The same logic can be applied to today’s changing planet, to confrim that the GHG part of AGW is correct. I have assumed that it is, at least wrt CO2, but it would be nice to see it verified this way.

Phil.
January 19, 2012 12:36 pm

George E. Smith; says:
January 17, 2012 at 2:47 pm
“”””” Leonard Weinstein says:
January 17, 2012 at 8:27 am
Anna & others that think a non-greenhouse gas emits thermal radiation:
Yes there is some radiation even from non-greenhouse gases, but it is so small compared to the ground radiation and greenhouse gas radiation that it can be considered zero for the analysis.
“”””” Anna & others “””””
AKA; Anna and George E. Smith
Well the competing postulates are:-
#1 Stated repeatedly in all sorts of well known Textbooks on Physics. “EVERY object at a Temperature above zero Kelvins, emits “Thermal Radiation”, that being electromagnetic radiation that is emitted solely as a consequence of the Temperature of that object”.
#2 Stated almost solely by “Climate Scientists” in relation to earth’s atmosphere:- “EVERY object EXCEPT GASES at a Temperature above zero Kelvins, emits “Thermal Radiation”, that being electromagnetic radiation that is emitted solely as a consequence of the Temperature of that object”.

This is untrue George, no Physical Chemist would agree with your second postulate, it has nothing to do with Climate science. Homonuclear diatomics do not absorb IR, extremely short lived collisional states, i.e N4, can emit in a continuum but for the purposes of an energy balance this is zero!
Unrelated to the above, but also often cited, with relatively little dispute:- “Some gases having more than two atoms per molecule CAN and DO emit and absorb Electromagnetic Radiation often in the Long Wavelength Infra-red region. These emissions/absorptions are ALWAYS at specific frequencies or bands of frequencies, that correspond to various electron energy levels that are peculiar to each such so-called greenhouse gas species, and are NOT continuum spectra characterized ONLY by the Temperature of the material.”
Not electronic energy levels but rotational and vibrational energy levels, i.e. the nuclei move.
Now when I went to school, EVERY meant EVERY; sans NOTHING if you will.
Gases either emit and absorb thermal EM radiation or they do not. The spectral envelope of such EM radiation is given by the spectrum for a black body at the Same Temperature.
And as Anna said (in effect) nothing is nothing,, not something small.
The radiations from GHGs are a consequence of quantum mechanics, and the electronic structure of the molecule.
Black body Thermal radiation is NOT a consequence of quantum mechanics, it is a resultof ordinary classical physics, and requires no knowledge of any structure of the “particles” that emit and absorb such radiation.
Phil’s very low probability of ionising Argon at 15.8 eV from atmospheric thermal energies, is not challenged; but then you don’t need to ionise argon to get it to emit or absorb radiation, unless it is the specific atomic lines that you want to excite, not thermal.

Anna chose to say that collisional activation in the atmosphere would cause ionization but that this would be ‘quite rare’, not I, I pointed out that this had a probability of 10^-139, a vanishingly small probability of occurring in the Earth’s atmosphere once if you waited for the lifetime of the universe! That is zero by anyone’s definition, Anna made an undefensible comment and was called on it (I notice no mention was made of that gaffe in Anna’s subsequent post).
Even if the probability of the first excited state was the case the chance of that occurring is negligibly small under atmospheric conditions. There’s a reason the spectrum linked by Anna was the result of an electronic discharge!
Well I prefer to have nothing be nothing; not something small.
Sometimes extremely small is the same as nothing, Anna’s example is a case in point, as is the IR spectrum of N2 which consists of about 100 lines 10^10 times weaker than the thousands of lines from CO2 and H2O
Nobody I know of has said that thermal emission from the atmosphere is the major driver of earth’s climate; but either it is there or it is not. Can’t have it both ways. I’m in the same boat as Anna.
Not a good boat to be George in as Anna’s errors show.

Editor
January 19, 2012 12:38 pm

Tim Folkerts – your comment answers the questions. We do as expected see less A at TOA and more B at the surface (I am happy to assume that the graphs do indeed show A and B).

Phil.
January 19, 2012 12:45 pm

Bart says:
January 17, 2012 at 3:29 pm
So, let me update my elevator one last time:
A) Radiation enters the system
B) it is absorbed by the surface, which fluoresces in the IR
C) heat builds up at the interface between the surface and the atmosphere until its apparent temperature suggests an exceedance of the incoming radiance based on the SB relationship
D) in fact, though, the distribution of surface energy states is non-Planckian and/or not omnidirectional so the entire integrated spatially integrated energy flux is actually not as great as expected
E) the heat from the interface conducts through the atmosphere, establishing a temperature gradient as must exist in a spherically distributed atmosphere
F) the transferred heat accumulates in the atmosphere until:
1) highly energetic emissions are stimulated, which balances the energy fluxes all around in the same way GHGs would in the standard “greenhouse” theory
OR
2) the heat accumulates until the atmosphere achieves escape velocity and vanishes.

This is where you introduce the fundamental error in your analysis.
what it should say is that ‘the heat accumulates in the atmosphere until the layer of gas nearest the surface reaches the surface temperature at which point heat transfer ceases due to ΔT=0. There can be no more heat transfer to the atmosphere unless it cools down and ΔT again exceeds 0. This is the fundamental flaw in all the ‘hotter and hotter’ arguments (it’s independent of the profile with altitude).
(G) if greenhouse gasses which can radiate in the IR are added to the atmosphere, it will result in a net cooler atmosphere than otherwise would be the case, because there is now a lower energy outlet for heat to escape.
Not true if the collision rate is much higher than the rate of emission as it is near the Earth’s surface, this is one of Postma’s errors.

January 19, 2012 12:46 pm

Spector says: “Greenhouse gases in the atmosphere solve this problem by continuously capturing 156 W/m² from that flow so that the energy budget remains balanced with our higher surface temperatures.”
Using Avogadro number you contend that the 156 W/m^2 are produced by 113800 molecules of CO2. That is the number of CO2 in one square meter at 380 ppm.
But the N2 and O2 (that also absorb heat or they would not be gases) do no “capturing” of heat.

Joe Postma
January 19, 2012 12:47 pm

Ah, now that’s a much more useful plot than the one Shore posted, which didn’t show what these do. Thanks a lot! So that’s good, you do see B-B emission in the CO2 band. We all know what the AGHE is supposed to be; I highlighted the various problems with it here:
http://www.tech-know.eu/uploads/Understanding_the_Atmosphere_Effect.pdf
http://principia-scientific.org/publications/The_Model_Atmosphere.pdf
http://principia-scientific.org/publications/Copernicus_Meets_the_Greenhouse_Effect.pdf
So the plots show that indeed, the atmosphere radiates a bunch of energy by itself, and the ground-surface does the majority. A lot of the emission in the atmosphere comes from GHG’s, as I would have expected from astrophysical theory, so at least we’re back on track here, after my journey a bit out of bounds 😉 That emission from the atmosphere indicates cooling, given that non-GHG’s in the atmosphere can’t cool by radiation. I think Bart must be correct when he says that the atmosphere will heat up until it radiates enough to cool and equilibrate, which is the function of GHG’s.

Phil.
January 19, 2012 12:50 pm

Bart says:
January 19, 2012 at 11:25 am
“Everything not forbidden is compulsory,” said Murray Gell-Mann. There is nothing forbidden in my prescription.

There is in your elevator description, you propose counter gradient conduction at the surface, that is forbidden!

Tim Folkerts
January 19, 2012 1:13 pm

Bart says: “It’s as plain as the noses on your faces: IR absorbing gasses in the Earth’s atmosphere cool the surface, and prevent runaway temperatures.”
My only reply is to look at the actual satellite data I posted. http://www.skepticalscience.com/images/infrared_spectrum.jpg
* With no GHG’s there would be no “bites” as seen from space
* With no bites, the earth would be radiating more
* With more outgoing radiation, the earth would cool (specifically the surface where we live would have to cool).
>> Therefore, the GHGs are warming the surface.
THAT is the simplest explanation of the greenhouse effect.

Bart
January 19, 2012 1:22 pm

DeWitt Payne says:
January 19, 2012 at 11:43 am
“A concentration gradient can only be stable if there’s a continuous flux into and out of a volume.”
DeWitt – I am precisely arguing that the situation is unstable.
DeWitt Payne says:
January 19, 2012 at 12:04 pm
“Not according to this:”
Here’s a better, non-cartoon graph. Figure 3.1.

Tim Folkerts
January 19, 2012 1:26 pm

mkelly makes two (maybe three or four) mistakes: “Using Avogadro number you contend that the 156 W/m^2 are produced by 113800 molecules of CO2. That is the number of CO2 in one square meter at 380 ppm. ”
1) There is no way to say how many molecules are in a square meter. You need to have a volume, not an area.
2) Near the surface anyway, a cubic meter of air is just over 1 kg. At ~ 30 g/mole, that would be ~ 30 moles of gas, or ~ 2E25 molecules. The number of CO2 molecules would be well over 1E21, not 1E5
3) The energy does not need to be absorbed in an altitude of 1 m. Pretty much everyone uses a number of at least 100m to absorb IR, so now we are up to 1E 23 molecules. If you really want the “number in 1 square meter”, the only logical interpretation would be in the entire column of air, which would be ~ 100 times more, or ~ 1E25 molecules.
Suddenly his objections seem much less valid.
(all numbers are intended to be orders of magnitude)

Bart
January 19, 2012 1:44 pm

“Greenhouse gases (other than ozone) do make the stratosphere cooler than it would be otherwise because the stratosphere is optically thin in the thermal IR range and most greenhouse gases do not absorb significantly in the UV. But the troposphere is optically thick for much of the wavelength range so the reduced emission at higher altitude makes the lower atmosphere warmer than if they weren’t present, not cooler. So the stratosphere has been getting a double whammy: a reduction in ozone causing less heating, and an increase in CO2 causing more cooling.”
Don’t you see, DeWitt? You are confirming exactly what I have been saying: the steady state result is the same no matter how you interpret it.
DeWitt Payne says:
January 19, 2012 at 12:04 pm
“In the Ionosphere, the temperature can reach 4,000K at altitudes above 500km”
There, you are dealing with an entirely different state of matter. So, you have to specify the boundary conditions and evaluate an entirely new PDE.

Bart
January 19, 2012 1:52 pm

Look at it this way, DeWitt. Starting from some temperature at the base, in any finite time, the temperature is going to be less out farther than it is nearer, and it is going to go to zero at infinity. So, there must be a gradient over any finite time.
What is the form of that gradient? Well, through separation of variables, the temperature function is the product of a time varying part, and a spatial varying part. The spatial varying part is a function of spatial coordinates only, so it does not change with time. Therefore, since for any finite time, the spatial solution is of the form A + f(r), where A is a constant and f(r) is a monotonically decreasing function of r, and A must be zero at that time, then it must always be zero, and f(r) must go to zero as r approaches infinity.
QED.

ZP
January 19, 2012 2:02 pm

Phil. says:
January 19, 2012 at 12:06 pm
Your over-estimation of the fraction of Argon atoms in the third electronic level under the Earth’s atmospheric conditions leads you to this incorrect conclusion (and perhaps also Anna’s error). Look up the energy of the third excited state and do a Maxwell-Boltzmann distribution calculation at T=300K you’ll find that it’s essentially all in the ground electronic state which does not absorb in the IR.

Argon is in the third period, therefore the ground state electron configuration is [Ne] 3s2 3p6. The valence electrons are going to be in the third electronic level unless excited.

jjthom
January 19, 2012 4:39 pm

Mike Jonas says: January 19, 2012 at 12:30 pm

6 – We should observe less of set A at TOA than we would expect from steps 1 and 2 above on their own.
7 – We should observe more of set B at the surface than we would expect from the sun alone.
=========
go here
http://www.patarnott.com/atms749/
in the drop down box (presentation by chapter…)
choose chapt 6 thermal emission
slide 9 in the power point presentation gives satellite downward looking spectrum and ground based upward looking spectrum
You will note that where there is a hole in one there is a peak in the other.
This mans website contains a wealth of information – well worth a trawl!

Spector
January 19, 2012 6:01 pm

RE: mkelly: (January 19, 2012 at 12:46 pm)
“Spector says: ‘Greenhouse gases in the atmosphere solve this problem by continuously capturing 156 W/m² from that flow so that the energy budget remains balanced with our higher surface temperatures.’
“Using Avogadro number you contend that the 156 W/m^2 are produced by 113800 molecules of CO2. That is the number of CO2 in one square meter at 380 ppm.
“But the N2 and O2 (that also absorb heat or they would not be gases) do no ‘capturing’ of heat.”

My comment only applies to radiant energy. There is no reference to Avogadro’s number. If the surface of the Earth is so warm that the average radiant power emitted by that surface must be 396 W/m² according to Trenberth and the Stefan-Boltzmann formula, then the atmosphere *must* extract and return an average power of 156 W/m² from that *radiated* surface power before it gets out to space because the Earth as a whole is only receiving enough power from the sun to emit a nominal average of 240 W/m². It is as simple as that. The Earth cannot afford to continuously emit an average of 396 W/m² to outer space and remain as warm as it is. A greenhouse-gas free, transparent atmosphere cannot, by definition, prevent the escape of this radiation from the surface.
Power in watts is energy flow in joules per second. Neglecting the miniscule amount of geothermal power being generated, the principle of conservation of energy requires that the average power emitted by the Earth be the same as the average power being received in a steady state equilibrium condition. All other internal heat transfer issues are irrelevant.
It appears that water vapor may be twice as effective as carbon dioxide in this role.

Peter Offenhartz
January 19, 2012 6:05 pm

Ah, Willis Eschenbach, you are a breath of fresh air! Three cheers for the First Law of Thermodynamics! And Three cheers for the Elevator Speech. May I suggest an addition to The Speech? Many of your (ignorant) critics seem to think that all substances, including nitrogen and oxygen, can radiate in the infrared, that the laws of blackbody radiation apply without regard to the nature of the radiating substance. So some addition along the lines of “at any given wavelength, only substances that absorb radiation can radiate” might help.
Human ignorance and stupidity always amaze me. You have put a stake through the heart of some of it. Thank you!

Tim Folkerts
January 19, 2012 6:57 pm

Bart,
If you impose boundary conditions that T = 0 @ infinity, then I agree with your conclusions.
But for the non-radiating atmosphere, the condition is more like “no energy can flow past the top of the atmosphere”. There is no requirement that the edge of the atmosphere be 0 K. Furthermore, the system is not infinite. The condition is that the TOA cannot lose any energy outward and must approach the temperature of the material closer in. This will eventually lead to equilibration at the surface temperature.
As to the “it will take infinitely long”, well that is true of any such problem. It would take infinitely long for my coffee to cool to the temperature of the room. But there will be a characteristic time describes the system. For a cup of coffee, it might be 10 minutes. For the atmosphere I suspect it is more like a few months. But in any case, after you have waited 10x that time or 20x that time, then the change will be complete for all practical purposes. So after a couple hours I consider the coffee to be room temp. After a few years or decades, the atmosphere would be uniform for all practical purposes.
Now if the conditions are change even slightly (for example a small amount of GHGs are present at the TOA), then the boundary conditions are completely different. Then the boundary emits energy as a function of temp, leading to a definite lapse rate.

Bart
January 19, 2012 7:02 pm

Spector says:
January 19, 2012 at 6:01 pm
“…then the atmosphere *must* extract and return an average power of 156 W/m² from that *radiated* surface power…”
In the steady state. There’s a lot of water vapor in the air, so it can do this, and mkelly has neglected that and other IR emitters in the atmosphere. But, I wanted to take the opportunity to reiterate and reinforce the fact that SB and its variants are purely applicable only in steady state, and loosely applicable only in situations close to a steady state.

Bart
January 19, 2012 8:14 pm

Tim Folkerts says:
January 19, 2012 at 6:57 pm
“If you impose boundary conditions that T = 0 @ infinity, then I agree with your conclusions.”
Hey, it’s a start.
“There is no requirement that the edge of the atmosphere be 0 K.”
If it is anything less than the surface temperature, then there is a downhill temperature gradient into the atmosphere, and energy will accumulate.
“The condition is that the TOA cannot lose…”
What is the TOA? How do you define it?
In any case, the heat equation is assuming a continuum. That the continuum assumption breaks down at some point does not invalidate the whole equation. The equation holds for certain in the region where the continuum assumption is reasonable.
“…any energy outward and must approach the temperature of the material closer in.”
You are begging the question. You are assuming there is an equilibrium without establishing that there actually is one.
I agree it will approach the temperature of the material closer in, but not at the same time. By the time it reaches that temperature, the material closer in will have grown hotter. The exact proportions, from one radial distance to the next, are specified by the spatial part of the solution to the PDE.
“This will eventually lead to equilibration at the surface temperature.”
You present no evidence but your say-so, and are implicitly claiming the heat equation does not hold. That is an extraordinary claim, and requires extraordinary evidence.
“After a few years or decades, the atmosphere would be uniform for all practical purposes.”
You’re not getting it. The spatial part of the solution to the PDE does not change with time. The temperature distribution at any given time is precisely equal to a time modulated amplitude function times the spatial distribution function. It remains conformally identical for all time. Only the scale changes.
Perhaps you have taken classes where you looked at string vibrations? These phenomena are also described by separable partial differential equations. At a particular mode of vibration, the displacement of the string is a constant function, a so-called “mode shape“, which is modulated by a sinusoid in time. The precise shape, the nodes, the endpoints… these do not change, but the shape collapses and reemerges in its mirror image back and forth as the string vibrates sinusoidally in time.
This is the common behavior of systems described by separable PDEs. In this case, the monotonically declining temperature profile is analogous to the mode shape of a string. It can only be magnified or diminished (actually, only magnified forward in time in this case) by the amplitude function, which is a function of time only.
So, if the “mode shape” of this thing is a constant independent of radius, that means the entire spherical shell has to heat uniformly. It starts at zero everywhere. It is at 1K everywhere at the same time. It is at 100K everywhere at the same time. This is physically impossible based solely on the fact that the rate of heat conductance is finite.
“Then the boundary emits energy as a function of temp, leading to a definite lapse rate.”
A definite adiabatic lapse rate, which is exponentially related to pressure, hence linear in altitude. But, that is not the only type of lapse rate which can exist.

January 22, 2012 3:13 pm

Jelbring makes this crucial and erroneous assumption: “Equilibrium atmospheric conditions have been reached meaning that the average total energy of atmospheric molecules is constant”. This is how he mistakenly gets his permanent lapse rate in the absence of a continuing flow of energy. However, that’s not what thermal equilibrium means; it means that the temperature is uniform throughout. An isothermal atmosphere has zero lapse rate.

Scott
January 24, 2012 7:31 pm

This thread now has enough comments that I skimmed through a great deal of them. As a few others have said above: Please accept my apologies for the repetition if someone else made essentially the same post.
One of the biggest points of contention to Eschenbach’s original posting is that the introduced non-GHG atmosphere will result in conduction/convection heat transfer. While true that a non-GHG atmosphere will not radiate LWIR (leaving the black body to be doing most of that), it is not true that no radiation of the atmosphere will result. However (and it’s a big however), the amount of heat absorbed into the non-GHG atmosphere and thus experiencing conduction/convection and radiation will be trivial relative to the black body. For the non-mathematical proof-of-concept hold your hand over a stove that is turned on and realize that the heat transferred is mostly the GHGs and it is still enough to not cook your hand inches from the stove. You can feel the radiative effects to the side of the heating element.
Given that, Eschenbach’s theoretical model should, in my estimation yield a slightly smaller surface temperature than S-B would predict because a very small portion of the energy going into the system would be taken by the conduction/convection/radiation of the atmosphere (though, again, a very small amount of heat that would not be significant).
Thus, the original premise of Eschenbach that a higher surface temperature would be the result from N&Z theories would violate the laws of the thermodynamics is true. If anything, a non-GHG atmosphere would result in a net cooler surface temperature even though any reasonable thermometer would be unlikely to be able to measure the difference.
There was also discussion of an atmosphere equalizing equatorial and polar temperatures with some thermal inversion thrown in, but that does not address the theoretical construct Eschenbach gave of a black body heated by a bunch of mini-stars all around it. In such a case, there would be no equatorial or polar regions in terms of heat. The only movement of the gas would be from convection in Eschenbach’s construct and that would not be much.

February 12, 2012 3:16 am

Tallbloke,

“No Willis, before we can move on to a discussion of the science, you need to acknowledge that Hans Jelbring defined his model planet as one which does not radiate to space”

I’ve been reading Jelbring’s post at your site among other things tonight. That is of course where I paused — and keep going back.
How can one just arbitrarily decide the planet doesn’t radiate to space, even for a thought experiment? I see a thought experiment as one where you think about things that might actually happen in an idealized case, not where you just choose whatever you want to happen in an idealized case.
Is there something I’m missing here?

February 12, 2012 4:25 am

“It neither receives solar radiation nor emits infrared radiation into space.”

I can see how it doesn’t receive solar radiation, but I fail to understand how it can arbitrarily said to not emit any radiation converted via convection.

Paul
February 21, 2012 11:22 am

Willis Eschenbach writes:
“I hold it can be proven that there is no possible mechanism involving gravity and the atmosphere that can raise the temperature of a planet with a transparent GHG-free atmosphere above the theoretical S-B temperature.
And because the atmosphere is transparent, this means that the planet is radiating to space more energy than it receives. This is an obvious violation of conservation of energy, so any theories proposing such a warming must be incorrect.”
here Willis is confusing energy with radiation.
The earth cannot emit all energy by radiation if there is an atmosphere, which gets energy by contact.
Its the same thing with CO2 which cannot radiate in deep parts of the Atmosphere.
Paul

Paul
February 21, 2012 1:48 pm

Thank you for direct answering, Willis, and excuse my bad english,
you said:
“Once the system is at equilibrium, therefore, there is no net flow between the surface and the atmosphere. As a result, as I said, the surface must be losing energy solely by radiation.”
I cannot accept any “equilibrium”, which does not exist in reality, for you cannot measure but only calculate theoretically.
Especially between an atmosphere, that is totally transparent and the earth, which is nearly a black body in infrared radiation, there will never be an “equilibrium”. But there will be always, always a difference in temperature which drives energy flux from earth to atmosphere by day and vice versa at night from atmosphere to earth, when the earth has cooled by radiation.
Consequently without GHG it will be hotter than with, as you can see on earth in the deserts (nearly without H2O) compared with the humide aequatorial region despite more radiation from sun. So what you can measure in satellites, there is always MUCH more radiation coming from the sun, than the back radiation from earth.
Yes of course, earth only can loose energy to space by radiation, but temperature will be higher than S-B-T because of the atmosphere by day and the non-linear dependance between temperature and radiation.
I hope you can agree 🙂
Paul

Paul
February 21, 2012 1:57 pm

once again in other words.
” the surface must be losing energy solely by radiation.”
of course this is correct, but the temperature on ground level must be higher than S-B-T
Paul

Paul
February 21, 2012 2:53 pm

“But if the temperature at ground level must be higher than S-B temperature, then the surface is radiating more than it receives, a violation of Conservation of Energy. Which was the point of my proof.”
Only if you insist, that there is no other energy transfer than radiation you can use S-B temperature for radiative energy-loss. That means, that you equate a planet with atmosphere to a planet without atmosphere, thats surely wrong, as we can see (measure) on the moon.
I still think, temperature must be higher with atmosphere, higher than “S-B temperature”.
By the way, I think you FIRST have to measure the radiation and not the temperature to calculate the “equilibrium” to space.
Energy content of earth´s surface is not only temperature but many other forms of energy, especially kinetic energy. I like to compare this with a big clock, which needs energy source to be kept moving on. So I think that permanently energy must be “used” to something else than only back radiation, otherwise the clock would stop going.
Biological “life” too is a state far outside the thermodynamic equilibrium.
I know, physicists very much like the equilibrium, and they postulated heat death.
Something that sounds very logical, but something, that cannot be found in the universe.
Paul

Paul
February 21, 2012 3:56 pm

“If the atmosphere is say argon, it is transparent, and argon is not a GHG. I have postulated just such an atmosphere in my thought experiment.”
of course,
but an Atmosphere with Argon counts energetically.
It must be distinguished from a planet without any atmosphere!,
this has nothing to do with GHG!

Paul
February 21, 2012 4:09 pm

Excuse me, I don´t want to be unpolite,
Earth’s surface has got a certain Amount of Energy from sun, that leads to a defined temperature. The Surface cannot use this temperature twice. S-B formula means ONLY radiation. So there is no energy left for heat transfer by conduction (and convection) to the Atmosphere. If you still want to have an “equilibrium” to space, you then need a higher temperature of the Surface than S-B-T. Once more, look at the Sahara.
I must sleep now, sorry
best wishes
Paul